SlideShare a Scribd company logo
1 of 257
Download to read offline
ȼȺɋɂɅɖ ɄɊȺȼɑɍɄ
ɆȺɊȱə ɉȱȾɊɍɑɇȺ
ȽȺɅɂɇȺ əɇɑȿɇɄɈ
Ⱥ Ʌ Ƚ ȿ Ȼ Ɋ Ⱥ
ɉɿɞɪɭɱɧɢɤ ɞɥɹ 8 ɤɥɚɫɭ
ɡɚɝɚɥɶɧɨɨɫɜɿɬɧɿɯ ɧɚɜɱɚɥɶɧɢɯ ɡɚɤɥɚɞɿɜ
Ɍɟɪɧɨɩɿɥɶ
ȼɢɞɚɜɧɢɰɬɜɨ «ɉɿɞɪɭɱɧɢɤɢ ɿ ɩɨɫɿɛɧɢɤɢ»
2016
ɍȾɄ 51(075.3)
ȻȻɄ 22.1ɹ723
Ʉ 77
ȿɤɫɩɟɪɬɢ, ɹɤɿ ɡɞɿɣɫɧɢɥɢ ɟɤɫɩɟɪɬɢɡɭ ɞɚɧɨɝɨ ɩɿɞɪɭɱɧɢɤɚ ɩɿɞ ɱɚɫ ɩɪɨɜɟɞɟɧɧɹ ɤɨɧ-
ɤɭɪɫɧɨɝɨ ɜɿɞɛɨɪɭ ɩɪɨɟɤɬɿɜ ɩɿɞɪɭɱɧɢɤɿɜ ɞɥɹ ɭɱɧɿɜ 8 ɤɥɚɫɭ ɡɚɝɚɥɶɧɨɨɫɜɿɬɧɿɯ ɧɚɜɱɚɥɶɧɢɯ
ɡɚɤɥɚɞɿɜ ɿ ɡɪɨɛɢɥɢ ɜɢɫɧɨɜɨɤ ɩɪɨ ɞɨɰɿɥɶɧɿɫɬɶ ɧɚɞɚɧɧɹ ɩɿɞɪɭɱɧɢɤɭ ɝɪɢɮɚ «Ɋɟɤɨɦɟɧɞɨ-
ɜɚɧɨ Ɇɿɧɿɫɬɟɪɫɬɜɨɦ ɨɫɜɿɬɢ ɿ ɧɚɭɤɢ ɍɤɪɚʀɧɢ»:
Ʌɟɜɢɰɶɤɚ ȱ. Ɇ., ɦɟɬɨɞɢɫɬ ɜɿɞɞɿɥɭ ɦɟɬɨɞɢɱɧɨʀ ɪɨɛɨɬɢ Ʉɍ «Ȼɚɲɬɚɧɫɶɤɢɣ ɪɚɣɨɧ-
ɧɢɣ ɫɟɪɜɿɫɧɢɣ ɰɟɧɬɪ ɩɨ ɨɛɫɥɭɝɨɜɭɜɚɧɧɸ ɡɚɤɥɚɞɿɜ ɨɫɜɿɬɢ» Ȼɚɲɬɚɧɫɶɤɨʀ ɪɚɣɨɧɧɨʀ ɪɚɞɢ
Ɇɢɤɨɥɚʀɜɫɶɤɨʀ ɨɛɥɚɫɬɿ;
ɉɨɝɨɪɿɥɹɤ Ɉ. Ɉ., ɞɨɰɟɧɬ ɤɚɮɟɞɪɢ ɬɟɨɪɿʀ ɣɦɨɜɿɪɧɨɫɬɟɣ ɿ ɦɚɬɟɦɚɬɢɱɧɨɝɨ ɚɧɚɥɿɡɭ
ȾȼɇɁ «ɍɠɝɨɪɨɞɫɶɤɢɣ ɧɚɰɿɨɧɚɥɶɧɢɣ ɭɧɿɜɟɪɫɢɬɟɬ», ɤɚɧɞɢɞɚɬ ɮɿɡɢɤɨ-ɦɚɬɟɦɚɬɢɱɧɢɯ ɧɚɭɤ;
Ɋɭɞɟɧɤɨ ȼ. Ɉ., ɭɱɢɬɟɥɶ Ɇɚɪ’ɹɧɿɜɫɶɤɨʀ ɡɚɝɚɥɶɧɨɨɫɜɿɬɧɶɨʀ ɲɤɨɥɢ ȱ–ȱȱȱ ɫɬɭɩɟɧɿɜ
Ɇɚɥɨɜɢɫɤɿɜɫɶɤɨʀ ɪɚɣɨɧɧɨʀ ɪɚɞɢ Ʉɿɪɨɜɨɝɪɚɞɫɶɤɨʀ ɨɛɥɚɫɬɿ, ɭɱɢɬɟɥɶ-ɦɟɬɨɞɢɫɬ, ɡɚɫɥɭɠɟ-
ɧɢɣ ɭɱɢɬɟɥɶ ɍɤɪɚʀɧɢ.
Ɋɟɤɨɦɟɧɞɨɜɚɧɨ
Ɇɿɧɿɫɬɟɪɫɬɜɨɦ ɨɫɜɿɬɢ ɿ ɧɚɭɤɢ ɍɤɪɚʀɧɢ
(ɧɚɤɚɡ ɆɈɇ ɍɤɪɚʀɧɢ ɜɿɞ 10.05.2016 ɪ. ʋ 491)
ȼɢɞɚɧɨ ɡɚ ɪɚɯɭɧɨɤ ɞɟɪɠɚɜɧɢɯ ɤɨɲɬɿɜ.
ɉɪɨɞɚɠ ɡɚɛɨɪɨɧɟɧɨ
Ʉɪɚɜɱɭɤ ȼ.
Ʉ 77 Ⱥɥɝɟɛɪɚ : ɩɿɞɪɭɱ. ɞɥɹ 8 ɤɥ. ɡɚɝɚɥɶɧɨɨɫɜɿɬ. ɧɚɜɱ. ɡɚɤɥ. /
ȼ. Ʉɪɚɜɱɭɤ, Ɇ. ɉɿɞɪɭɱɧɚ, Ƚ. əɧɱɟɧɤɨ. — Ɍɟɪɧɨɩɿɥɶ : ɉɿɞɪɭɱ-
ɧɢɤɢ ɿ ɩɨɫɿɛɧɢɤɢ, 2016. — 256 ɫ.
ISBN 978-966-07-3003-8
ɍȾɄ 51(075.3)
ȻȻɄ 22.1ɹ723
ISBN 978-966-07-3003-8 © Ʉɪɚɜɱɭɤ ȼ., ɉɿɞɪɭɱɧɚ Ɇ., əɧɱɟɧɤɨ Ƚ., 2016
© ȼɢɞɚɜɧɢɰɬɜɨ «ɉɿɞɪɭɱɧɢɤɢ ɿ ɩɨɫɿɛɧɢɤɢ»,
ɨɪɢɝɿɧɚɥ-ɦɚɤɟɬ, 2106
Ȍǻǥ DzǾȁǵǥ!
Ʉɿɥɶɤɚ ɫɥɿɜ ɩɪɨ ɨɫɨɛɥɢɜɨɫɬɿ ɜɢɞɚɧɧɹ.
Ɇɚɬɟɪɿɚɥ ɩɿɞɪɭɱɧɢɤɚ ɩɨɞɿɥɟɧɨ ɧɚ ɬɪɢ ɩɚɪɚɝɪɚɮɢ, ɚ ɩɚɪɚɝɪɚ-
ɮɢ — ɧɚ ɩɭɧɤɬɢ.
Ʉɨɠɧɢɣ ɩɭɧɤɬ ɪɨɡɩɨɱɢɧɚɽɬɶɫɹ ɜɢɤɥɚɞɨɦ ɬɟɨɪɟɬɢɱɧɨɝɨ ɦɚɬɟɪɿ-
ɚɥɭ. Ⱦɟɹɤɿ ɩɭɧɤɬɢ ɦɿɫɬɹɬɶ ɞɨɞɚɬɤɨɜɢɣ ɦɚɬɟɪɿɚɥ ɩɿɞ ɪɭɛɪɢɤɨɸ «Ⱦɥɹ
ɬɢɯ, ɯɬɨ ɯɨɱɟ ɡɧɚɬɢ ɛɿɥɶɲɟ».
Ⱦɚɥɿ ɣɞɟ ɪɭɛɪɢɤɚ «ɉɪɢɤɥɚɞɢ ɪɨɡɜ’ɹɡɚɧɧɹ ɜɩɪɚɜ». ɐɟ ɩɿɞɤɚɡɤɚ.
ȼɨɧɚ ɞɨɩɨɦɨɠɟ ɜɚɦ ɨɡɧɚɣɨɦɢɬɢɫɹ ɡ ɨɫɧɨɜɧɢɦɢ ɜɢɞɚɦɢ ɜɩɪɚɜ, ɫɩɨ-
ɫɨɛɚɦɢ ʀɯ ɪɨɡɜ’ɹɡɭɜɚɧɧɹ ɬɚ ɧɚɜɱɢɬɶ ɩɪɚɜɢɥɶɧɨ ɡɚɩɢɫɭɜɚɬɢ ɪɨɡɜ’ɹ-
ɡɚɧɧɹ. ɉɨɱɚɬɨɤ ɬɚ ɡɚɤɿɧɱɟɧɧɹ ɪɨɡɜ’ɹɡɚɧɧɹ ɤɨɠɧɨʀ ɜɩɪɚɜɢ ɩɨɡɧɚɱɟɧɨ
ɤɪɭɠɟɱɤɨɦ «Ɣ».
ɍ ɤɨɠɧɨɦɭ ɩɭɧɤɬɿ ɫɢɫɬɟɦɭ ɜɩɪɚɜ ɩɨɞɿɥɟɧɨ ɧɚ ɬɪɢ ɪɿɜɧɿ ɫɤɥɚɞ-
ɧɨɫɬɿ.
ɋɩɨɱɚɬɤɭ ɜɚɪɬɨ ɪɨɡɜ’ɹɡɭɜɚɬɢ ɭɫɧɿ ɜɩɪɚɜɢ ɿ ɩɪɨɫɬɿɲɿ ɡɚɞɚɱɿ (ɪɿ-
ɜɟɧɶ Ⱥ), ɚ ɩɨɬɿɦ ɩɟɪɟɣɬɢ ɞɨ ɫɤɥɚɞɧɿɲɢɯ (ɪɿɜɟɧɶ Ȼ). Ɂɚɞɚɱɿ ɪɿɜɧɹ
ȼ — ɞɥɹ ɧɚɣɤɦɿɬɥɢɜɿɲɢɯ, ɬɢɯ, ɯɬɨ ɯɨɱɟ ɜɦɿɬɢ ɬɚ ɡɧɚɬɢ ɛɿɥɶɲɟ ɿ ɦɚ-
ɬɢ ɧɚɣɜɢɳɿ ɨɰɿɧɤɢ. Ⱦɥɹ ɞɟɹɤɢɯ ɡɚɞɚɱ ɰɶɨɝɨ ɪɿɜɧɹ ɧɚɜɟɞɟɧɨ ɪɨɡɜ’ɹ-
ɡɚɧɧɹ.
Ⱦɥɹ ɫɚɦɨɫɬɿɣɧɨʀ ɪɨɛɨɬɢ ɜɞɨɦɚ ɪɟɤɨɦɟɧɞɨɜɚɧɨ ɡɚɞɚɱɿ, ɧɨɦɟɪɢ
ɹɤɢɯ ɜɢɞɿɥɟɧɨ ɤɨɥɶɨɪɨɦ (ɧɚɩɪɢɤɥɚɞ, 255).
Ɋɭɛɪɢɤɚ «ȼɩɪɚɜɢ ɞɥɹ ɩɨɜɬɨɪɟɧɧɹ» ɩɪɢɡɧɚɱɟɧɚ ɞɥɹ ɩɟɪɿɨɞɢɱɧɨ-
ɝɨ ɩɨɜɬɨɪɟɧɧɹ ɨɫɧɨɜɧɢɯ ɜɢɞɿɜ ɜɩɪɚɜ ɬɚ ɩɿɞɝɨɬɨɜɤɢ ɞɨ ɜɢɜɱɟɧɧɹ ɧɨ-
ɜɨɝɨ ɬɟɨɪɟɬɢɱɧɨɝɨ ɦɚɬɟɪɿɚɥɭ.
ɇɚɫɬɭɩɧɚ ɪɭɛɪɢɤɚ «ɉɨɦɿɪɤɭɣɬɟ» ɩɨɜ’ɹɡɚɧɚ ɡ ɨɫɨɛɥɢɜɢɦ ɚɫɩɟɤ-
ɬɨɦ ɦɚɬɟɦɚɬɢɱɧɨʀ ɩɿɞɝɨɬɨɜɤɢ. Ɉɫɧɨɜɧɢɦ ɞɥɹ ɪɨɡɜ’ɹɡɚɧɧɹ ɡɚɞɚɱ ɰɿɽʀ
ɪɭɛɪɢɤɢ ɽ ɜɦɿɧɧɹ ɜɢɯɨɞɢɬɢ ɡ ɧɟɫɬɚɧɞɚɪɬɧɢɯ ɫɢɬɭɚɰɿɣ. Ɋɨɡɜ’ɹ-
ɡɭɜɚɧɧɹ ɬɚɤɢɯ ɡɚɞɚɱ ɪɨɡɜɢɜɚɽ ɝɧɭɱɤɿɫɬɶ ɿ ɤɪɢɬɢɱɧɿɫɬɶ ɦɢɫɥɟɧɧɹ, ɚ
ɰɟ ɞɨɩɨɦɨɠɟ ɜɚɦ ɭ ɦɚɣɛɭɬɧɶɨɦɭ, ɧɟɡɚɥɟɠɧɨ ɜɿɞ ɬɨɝɨ, ɹɤɭ ɩɪɨɮɟɫɿɸ
ɜɢ ɨɛɟɪɟɬɟ.
ɇɚɩɪɢɤɿɧɰɿ ɤɨɠɧɨɝɨ ɩɚɪɚɝɪɚɮɚ ɭɦɿɳɟɧɨ ɡɚɩɢɬɚɧɧɹ ɬɚ ɜɩɪɚɜɢ
ɞɥɹ ɩɨɜɬɨɪɟɧɧɹ, ɡɚɜɞɚɧɧɹ ɞɥɹ ɫɚɦɨɩɟɪɟɜɿɪɤɢ ɱɨɬɢɪɶɨɯ ɪɿɜɧɿɜ ɫɤɥɚɞ-
ɧɨɫɬɿ.
ɍ ɤɿɧɰɿ ɩɿɞɪɭɱɧɢɤɚ ɩɨɞɚɧɨ ɜɩɪɚɜɢ ɞɥɹ ɩɨɜɬɨɪɟɧɧɹ ɦɚɬɟɪɿɚɥɭ ɡɚ
ɭɜɟɫɶ ɤɭɪɫ ɚɥɝɟɛɪɢ 8 ɤɥɚɫɭ, ɡɚɞɚɱɿ ɩɿɞɜɢɳɟɧɨʀ ɫɤɥɚɞɧɨɫɬɿ, ɞɨɜɿɞɤɨ-
ɜɢɣ ɦɚɬɟɪɿɚɥ.
ɓɢɪɨ ɛɚɠɚɽɦɨ ɭɫɩɿɯɭ!
6 § 1. Ɋɚɰɿɨɧɚɥɶɧɿ ɜɢɪɚɡɢ
1. ɐɿɥɿ, ɞɪɨɛɨɜɿ ɬɚ ɪɚɰɿɨɧɚɥɶɧɿ ɜɢɪɚɡɢ. ɍ ɫɶɨɦɨɦɭ ɤɥɚɫɿ ɦɢ ɜɢɜɱɚɥɢ
ɰɿɥɿ ɜɢɪɚɡɢ. ɉɪɢɤɥɚɞɚɦɢ ɬɚɤɢɯ ɜɢɪɚɡɿɜ ɽ:
ɚ + b; 3ɚ2
; 2ɯ(ɯ – ɭ)2
; ;
3
c ɚ : 4; b; 3.
ɉɪɢɝɚɞɚɣɦɨ: ɰɿɥɿ ɜɢɪɚɡɢ ɦɨɠɭɬɶ ɦɿɫɬɢɬɢ ɞɿʀ ɞɨɞɚɜɚɧɧɹ, ɜɿɞɧɿɦɚɧɧɹ,
ɦɧɨɠɟɧɧɹ, ɩɿɞɧɟɫɟɧɧɹ ɞɨ ɫɬɟɩɟɧɹ, ɚ ɬɚɤɨɠ ɞɿɸ ɞɿɥɟɧɧɹ, ɚɥɟ ɬɿɥɶɤɢ ɧɚ ɱɢɫɥɨ,
ɜɿɞɦɿɧɧɟ ɜɿɞ ɧɭɥɹ.
Ʉɨɠɧɢɣ ɰɿɥɢɣ ɜɢɪɚɡ ɦɨɠɧɚ ɡɚɩɢɫɚɬɢ ɭ ɜɢɝɥɹɞɿ ɦɧɨɝɨɱɥɟɧɚ. ɇɚɩɪɢɤɥɚɞ,
2ɯ(ɯ – ɭ)2
= 2ɯ(ɯ2
– 2ɯɭ + ɭ2
) = 2ɯ3
– 4ɯ2
ɭ + 2ɯɭ2
.
Ɋɨɡɝɥɹɧɟɦɨ ɜɢɪɚɡɢ
5
1,
1
y
y
+
+ 2 2
17 ,ab
a b−
3ɚ : b, 2
( ) .xx y
x y
− −
+
ɐɿ ɜɢɪɚɡɢ ɜɿɞɪɿɡɧɹɸɬɶɫɹ ɜɿɞ ɰɿɥɢɯ ɜɢɪɚɡɿɜ ɬɢɦ, ɳɨ ɦɿɫɬɹɬɶ ɞɿɸ ɞɿɥɟɧɧɹ ɧɚ ɜɢ-
ɪɚɡ ɡɿ ɡɦɿɧɧɨɸ. Ɍɚɤɿ ɜɢɪɚɡɢ ɧɚɡɢɜɚɸɬɶ ɞɪɨɛɨɜɢɦɢ ɪɚɰɿɨɧɚɥɶɧɢɦɢ ɜɢɪɚɡɚɦɢ.
ɐɿɥɿ ɣ ɞɪɨɛɨɜɿ ɪɚɰɿɨɧɚɥɶɧɿ ɜɢɪɚɡɢ ɧɚɡɢɜɚɸɬɶ ɪɚɰɿɨɧɚɥɶɧɢɦɢ ɜɢɪɚɡɚɦɢ.
3
4b a+
ɐɿɥɢɣ ɜɢɪɚɡ
34 a
b
+
Ⱦɪɨɛɨɜɢɣ ɜɢɪɚɡ
Ɋɚɰɿɨɧɚɥɶɧɿ ɜɢɪɚɡɢ
Ɋɨɡɝɥɹɧɟɦɨ ɪɚɰɿɨɧɚɥɶɧɿ ɜɢɪɚɡɢ ,ab
a b+
2,3
,
( 2)x y +
5
7
a . ȼɨɧɢ ɽ ɱɚɫɬɤɚɦɢ
ɞɜɨɯ ɜɢɪɚɡɿɜ, ɞɨ ɬɨɝɨ ɠ, ɞɿɸ ɞɿɥɟɧɧɹ ɡɚɩɢɫɚɧɨ ɡɚ ɞɨɩɨɦɨɝɨɸ ɪɢɫɤɢ ɞɪɨɛɭ. Ɍɚɤɿ
ɜɢɪɚɡɢ ɧɚɡɢɜɚɸɬɶ ɞɪɨɛɚɦɢ.
əɤɳɨ ɦɚɽɦɨ ɞɪɿɛ ,A
B
ɞɟ A ɿ B — ɞɟɹɤɿ ɱɢɫɥɨɜɿ ɜɢɪɚɡɢ ɚɛɨ ɜɢɪɚɡɢ ɡɿ ɡɦɿɧɧɢ-
ɦɢ, ɬɨ ɜɢɪɚɡ A ɧɚɡɢɜɚɸɬɶ ɱɢɫɟɥɶɧɢɤɨɦ ɞɪɨɛɭ, ɚ ɜɢɪɚɡ B — ɡɧɚɦɟɧɧɢɤɨɦ.
Ɉɬɠɟ, ab
a b+
— ɞɪɿɛ ɿɡ ɱɢɫɟɥɶɧɢɤɨɦ ab ɿ ɡɧɚɦɟɧɧɢɤɨɦ a + b.
1. Ɋɚɰɿɨɧɚɥɶɧɿ ɜɢɪɚɡɢ. Ɋɚɰɿɨɧɚɥɶɧɿ ɞɪɨɛɢ 7
Ⱦɪɿɛ ,A
B
ɭ ɹɤɨɦɭ A ɿ B — ɦɧɨɝɨɱɥɟɧɢ, ɧɚɡɢɜɚɸɬɶ ɪɚɰɿɨɧɚɥɶɧɢɦ ɞɪɨɛɨɦ.
ɇɚɩɪɢɤɥɚɞ, 4 ,
3x +
,a b
a b
+
− 2 2
,
x y
x xy y
+
+ +
,
x
a 3
b
— ɪɚɰɿɨɧɚɥɶɧɿ ɞɪɨɛɢ.
2. Ⱦɨɩɭɫɬɢɦɿ ɡɧɚɱɟɧɧɹ ɡɦɿɧɧɢɯ. Ɋɨɡɝɥɹɧɟɦɨ ɞɪɨɛɨɜɢɣ ɜɢɪɚɡ 5 .
2a −
əɤɳɨ ɚ = 3, ɬɨ ɡɧɚɱɟɧɧɹ ɰɶɨɝɨ ɜɢɪɚɡɭ ɞɨɪɿɜɧɸɽ: 5
3 2−
= 5
1
= 5;
ɹɤɳɨ ɚ = –6, ɬɨ ɡɧɚɱɟɧɧɹ ɜɢɪɚɡɭ ɞɨɪɿɜɧɸɽ: 5
6 2− −
= 5.
8
−
Ɂɧɚɱɟɧɧɹ ɜɢɪɚɡɭ 5
2a −
ɦɨɠɧɚ ɡɧɚɣɬɢ ɞɥɹ ɛɭɞɶ-ɹɤɨɝɨ ɡɧɚɱɟɧɧɹ ɚ, ɤɪɿɦ ɚ = 2.
əɤɳɨ ɚ = 2, ɬɨ ɡɧɚɦɟɧɧɢɤ ɚ – 2 ɞɨɪɿɜɧɸɽ ɧɭɥɸ, ɚ ɧɚ ɧɭɥɶ ɞɿɥɢɬɢ ɧɟ ɦɨɠɧɚ. Ʉɚɠɭɬɶ:
ɹɤɳɨ ɚ ≠ 2, ɬɨ ɜɢɪɚɡ 5
2a −
ɦɚɽ ɡɦɿɫɬ, ɚ ɹɤɳɨ ɚ = 2, ɬɨ ɜɢɪɚɡ ɧɟ ɦɚɽ ɡɦɿɫɬɭ. Ɂɧɚɱɟɧɧɹ
ɡɦɿɧɧɢɯ, ɞɥɹ ɹɤɢɯ ɜɢɪɚɡ ɦɚɽ ɡɦɿɫɬ, ɧɚɡɢɜɚɸɬɶ ɞɨɩɭɫɬɢɦɢɦɢ ɡɧɚɱɟɧɧɹɦɢ ɡɦɿɧ-
ɧɢɯ.
Ɉɡɧɚɱɟɧɧɹ
Ⱦɨɩɭɫɬɢɦɢɦɢ ɡɧɚɱɟɧɧɹɦɢ ɡɦɿɧɧɢɯ ɜɢɪɚɡɭ ɧɚɡɢɜɚɸɬɶ ɬɚɤɿ
ʀɯ ɡɧɚɱɟɧɧɹ, ɞɥɹ ɹɤɢɯ ɜɢɪɚɡ ɦɚɽ ɡɦɿɫɬ.
Ɍɚɤ, ɞɥɹ ɜɢɪɚɡɭ 5
2a −
ɞɨɩɭɫɬɢɦɢɦɢ ɡɧɚɱɟɧɧɹɦɢ ɡɦɿɧɧɨʀ ɽ ɜɫɿ ɡɧɚɱɟɧɧɹ
ɚ, ɤɪɿɦ ɚ = 2.
Ⱦɨɩɭɫɬɢɦɢɦɢ ɡɧɚɱɟɧɧɹɦɢ ɡɦɿɧɧɢɯ ɛɭɞɶ-ɹɤɨɝɨ ɰɿɥɨɝɨ ɜɢɪɚɡɭ ɽ ɜɫɿ ɡɧɚɱɟɧ-
ɧɹ ɡɦɿɧɧɢɯ. Ⱦɨɩɭɫɬɢɦɢɦɢ ɡɧɚɱɟɧɧɹɦɢ ɡɦɿɧɧɢɯ ɞɪɨɛɨɜɨɝɨ ɪɚɰɿɨɧɚɥɶɧɨɝɨ ɜɢɪɚ-
ɡɭ ɽ ɜɫɿ ɡɧɚɱɟɧɧɹ ɡɦɿɧɧɢɯ, ɤɪɿɦ ɬɢɯ, ɞɥɹ ɹɤɢɯ ɞɨɪɿɜɧɸɽ ɧɭɥɸ ɡɧɚɦɟɧɧɢɤ ɯɨɱɚ ɛ
ɨɞɧɨɝɨ ɡ ɞɪɨɛɿɜ, ɳɨ ɜɯɨɞɹɬɶ ɞɨ ɞɚɧɨɝɨ ɜɢɪɚɡɭ.
3. Ɍɨɬɨɠɧɨ ɪɿɜɧɿ ɜɢɪɚɡɢ. Ɍɨɬɨɠɧɨɫɬɿ. Ɋɨɡɝɥɹɧɟɦɨ ɰɿɥɢɣ ɜɢɪɚɡ
x2
+ x(2 – x). Ɉɫɤɿɥɶɤɢ
ɯ2
+ ɯ(2 – ɯ) = ɯ2
+ 2ɯ – ɯ2
= 2ɯ,
ɬɨ ɞɥɹ ɛɭɞɶ-ɹɤɨɝɨ ɡɧɚɱɟɧɧɹ ɡɦɿɧɧɨʀ x ɜɿɞɩɨɜɿɞɧɿ ɡɧɚɱɟɧɧɹ ɜɢɪɚɡɿɜ ɯ2
+ ɯ(2 – ɯ)
ɿ 2x ɞɨɪɿɜɧɸɸɬɶ ɨɞɧɟ ɨɞɧɨɦɭ. Ɍɚɤɿ ɰɿɥɿ ɜɢɪɚɡɢ ɦɢ ɧɚɡɢɜɚɥɢ ɬɨɬɨɠɧɨ ɪɿɜɧɢɦɢ.
Ⱥ ɹɤɿ ɞɜɚ ɧɟ ɰɿɥɿ ɜɢɪɚɡɢ ɜɜɚɠɚɸɬɶ ɬɨɬɨɠɧɨ ɪɿɜɧɢɦɢ?
8 § 1. Ɋɚɰɿɨɧɚɥɶɧɿ ɜɢɪɚɡɢ
Ɋɨɡɝɥɹɧɟɦɨ ɞɪɨɛɨɜɿ ɜɢɪɚɡɢ
2
(2 )
1
x x x
x
+ −
−
ɿ
2
1
x
x −
. Ⱦɨɩɭɫɬɢɦɢɦɢ ɡɧɚɱɟɧɧɹ-
ɦɢ ɡɦɿɧɧɨʀ ɨɛɨɯ ɜɢɪɚɡɿɜ ɽ ɜɫɿ ɡɧɚɱɟɧɧɹ x, ɤɪɿɦ ɯ = 1. ɐɿ ɜɢɪɚɡɢ ɦɚɸɬɶ ɨɞɧɚɤɨɜɿ
ɡɧɚɦɟɧɧɢɤɢ ɣ ɬɨɬɨɠɧɨ ɪɿɜɧɿ ɱɢɫɟɥɶɧɢɤɢ. Ɍɨɦɭ ɞɥɹ ɤɨɠɧɨɝɨ ɞɨɩɭɫɬɢɦɨɝɨ
ɡɧɚɱɟɧɧɹ ɯ ɜɿɞɩɨɜɿɞɧɿ ɡɧɚɱɟɧɧɹ ɜɢɪɚɡɿɜ ɞɨɪɿɜɧɸɸɬɶ ɨɞɧɟ ɨɞɧɨɦɭ. Ɍɚɤɿ ɜɢɪɚɡɢ
ɧɚɡɢɜɚɸɬɶ ɬɨɬɨɠɧɨ ɪɿɜɧɢɦɢ.
Ɉɡɧɚɱɟɧɧɹ
Ⱦɜɚ ɜɢɪɚɡɢ ɧɚɡɢɜɚɸɬɶ ɬɨɬɨɠɧɨ ɪɿɜɧɢɦɢ, ɹɤɳɨ ɞɥɹ ɛɭɞɶ-
ɹɤɢɯ ɞɨɩɭɫɬɢɦɢɯ ɞɥɹ ɧɢɯ ɡɧɚɱɟɧɶ ɡɦɿɧɧɢɯ ɜɿɞɩɨɜɿɞɧɿ ɡɧɚ-
ɱɟɧɧɹ ɜɢɪɚɡɿɜ ɞɨɪɿɜɧɸɸɬɶ ɨɞɧɟ ɨɞɧɨɦɭ.
əɤɳɨ ɞɜɚ ɬɨɬɨɠɧɨ ɪɿɜɧɿ ɜɢɪɚɡɢ
2
(2 )
1
x x x
x
+ −
−
ɬɚ 2
1
x
x −
ɫɩɨɥɭɱɢɬɢ ɡɧɚɤɨɦ
«=», ɬɨ ɨɞɟɪɠɢɦɨ ɪɿɜɧɿɫɬɶ
2
(2 )
1
x x x
x
+ −
−
= 2
1
x
x −
, ɹɤɚ ɽ ɩɪɚɜɢɥɶɧɨɸ ɞɥɹ ɜɫɿɯ
ɞɨɩɭɫɬɢɦɢɯ ɡɧɚɱɟɧɶ ɯ. Ɍɚɤɭ ɪɿɜɧɿɫɬɶ ɧɚɡɢɜɚɸɬɶ ɬɨɬɨɠɧɿɫɬɸ.
Ɉɡɧɚɱɟɧɧɹ
Ɋɿɜɧɿɫɬɶ, ɹɤɚ ɽ ɩɪɚɜɢɥɶɧɨɸ ɞɥɹ ɜɫɿɯ ɞɨɩɭɫɬɢɦɢɯ ɡɧɚɱɟɧɶ
ɡɦɿɧɧɢɯ, ɳɨ ɜɯɨɞɹɬɶ ɞɨ ɧɟʀ, ɧɚɡɢɜɚɸɬɶ ɬɨɬɨɠɧɿɫɬɸ.
ɇɚɩɪɢɤɥɚɞ,
2
2 2 ,
3 3 9
ab a a b
a b ab
⋅ =
⋅ 2 2
( )( )
xy xy
x y x yx y
=
− +−
— ɬɨɬɨɠɧɨɫɬɿ.
Ɂɚɦɿɧɭ ɨɞɧɨɝɨ ɜɢɪɚɡɭ ɬɨɬɨɠɧɨ ɪɿɜɧɢɦ ɣɨɦɭ ɜɢɪɚɡɨɦ ɧɚɡɢɜɚɸɬɶ ɬɨ-
ɬɨɠɧɢɦ ɩɟɪɟɬɜɨɪɟɧɧɹɦ ɜɢɪɚɡɭ.
ȼɩɪɚɜɚ 1. Ɂɧɚɣɬɢ ɡɧɚɱɟɧɧɹ ɜɢɪɚɡɭ 28
3
x
x
+
+
, ɹɤɳɨ ɯ = 4; ɯ = 1
3
.
Ɣ əɤɳɨ ɯ = 4, ɬɨ 28
3
x
x
+
+
= 284
4 3
+
+
= 284
7
+ = 4 + 4 = 8.
əɤɳɨ ɯ = 1
3
, ɬɨ 28
3
x
x
+
+
= 1 28
3 1 3
3
+
+
= 1 28
3 10
3
+ = 1 328
3 10
+ ⋅ =
= 1 42
3 5
+ = 1 28
3 5
+ = 118 .
15
Ɣ
1. Ɋɚɰɿɨɧɚɥɶɧɿ ɜɢɪɚɡɢ. Ɋɚɰɿɨɧɚɥɶɧɿ ɞɪɨɛɢ 9
ȼɩɪɚɜɚ 2. Ɂɧɚɣɬɢ ɡɧɚɱɟɧɧɹ ɜɢɪɚɡɭ
2
( )( )a b a b b
a b
− + +
+
, ɹɤɳɨ:
ɚ) a = 8; b = 32; ɛ) a = 0,6; b = –0,6.
Ɣ ɋɩɪɨɫɬɢɦɨ ɞɚɧɢɣ ɜɢɪɚɡ:
2
( )( )a b a b b
a b
− + +
+
=
2 2 2
a b b
a b
− +
+
=
2
.a
a b+
ɚ) əɤɳɨ a = 8; b = 32, ɬɨ
2
a
a b+
=
2
8
8 32+
= 64
40
= 1,6.
ɛ) əɤɳɨ a = 0,6; b = –0,6, ɬɨ
22
0,6 0,36
0,6 0,6 0
a
a b
= =
+ −
— ɧɟ ɦɚɽ ɡɦɿɫɬɭ. Ɣ
ȼɩɪɚɜɚ 3. ȼɤɚɡɚɬɢ ɞɨɩɭɫɬɢɦɿ ɡɧɚɱɟɧɧɹ ɡɦɿɧɧɨʀ ɭ ɜɢɪɚɡɿ:
ɚ)
4
;
3
y
y
y
+
+
−
ɛ) 2
2 1 ;a
a a
−
+
ɜ) 2
4 .
8
x
x
+
+
Ɣ ɚ) Ⱦɨɩɭɫɬɢɦɢɦɢ ɽ ɜɫɿ ɡɧɚɱɟɧɧɹ ɭ, ɤɪɿɦ ɭ = 3.
ɛ) Ɂɧɚɣɞɟɦɨ ɡɧɚɱɟɧɧɹ ɚ, ɞɥɹ ɹɤɢɯ ɡɧɚɦɟɧɧɢɤ ɞɪɨɛɭ ɞɨɪɿɜɧɸɽ ɧɭɥɸ:
ɚ2
+ ɚ = 0; ɚ(ɚ + 1) = 0; ɚ = 0 ɚɛɨ ɚ + 1 = 0; ɚ = 0 ɚɛɨ ɚ = –1.
Ⱦɨɩɭɫɬɢɦɢɦɢ ɽ ɜɫɿ ɡɧɚɱɟɧɧɹ ɚ, ɤɪɿɦ ɚ = 0 ɿ ɚ = –1.
ɜ) Ⱦɥɹ ɛɭɞɶ-ɹɤɨɝɨ ɡɧɚɱɟɧɧɹ ɯ ɡɧɚɱɟɧɧɹ ɡɧɚɦɟɧɧɢɤɚ ɯ2
+ 8 ɧɟ ɦɟɧɲɟ ɧɿɠ
8, ɚ ɬɨɦɭ ɧɟ ɞɨɪɿɜɧɸɽ ɧɭɥɸ. Ɉɬɠɟ, ɞɨɩɭɫɬɢɦɢɦɢ ɽ ɜɫɿ ɡɧɚɱɟɧɧɹ ɯ. Ɣ
1. əɤɿ ɡ ɜɢɪɚɡɿɜ ɽ ɰɿɥɢɦɢ ɜɢɪɚɡɚɦɢ? ɞɪɨɛɨɜɢɦɢ? əɤɿ ɡ ɜɢɪɚɡɿɜ ɽ ɞɪɨɛɚɦɢ?
ɪɚɰɿɨɧɚɥɶɧɢɦɢ ɞɪɨɛɚɦɢ?
ɚ) ;a b
a b
+
−
ɛ) 2
;
3
x x+ ɜ) 24 ;x
x
−
ɝ) ( )31 ;
2
b a+ ⋅ ɞ) 5 ;
( 1)x y +
ɟ) .
5
xy x+
2. Ⱦɥɹ ɹɤɢɯ ɡɧɚɱɟɧɶ ɡɦɿɧɧɨʀ ɜɢɪɚɡ ɧɟ ɦɚɽ ɡɦɿɫɬɭ? ɇɚɡɜɿɬɶ ɞɨɩɭɫɬɢɦɿ
ɡɧɚɱɟɧɧɹ ɡɦɿɧɧɨʀ ɭ ɜɢɪɚɡɿ:
ɚ) 8;
c
ɛ) 9 ;
1
x
x
−
−
ɜ) 4 .
( 2)
b
b b
+
−
10 § 1. Ɋɚɰɿɨɧɚɥɶɧɿ ɜɢɪɚɡɢ
3. Ɂɧɚɣɞɿɬɶ ɡɧɚɱɟɧɧɹ ɜɢɪɚɡɭ 10
a
, ɹɤɳɨ a = 10; a = –1; a = 2.
4. əɤɿ ɡ ɪɿɜɧɨɫɬɟɣ ɽ ɬɨɬɨɠɧɨɫɬɹɦɢ?
ɚ) 3 4 ;
1 1
a a a
a a
+ =
− −
ɛ) 3 3 ;
1 1
a a a
a a
⋅ =
− −
ɜ) 2
( )
b b
a a b a ab
=
+ +
.
Ɂɧɚɣɞɿɬɶ ɡɧɚɱɟɧɧɹ ɜɢɪɚɡɭ:
5. ɚ)
2
5
x
x
−
+
, ɹɤɳɨ ɯ = 0; ɯ = 5; ɯ = –3;
ɛ) 2ab
a b−
, ɹɤɳɨ ɚ = 4, b = 2; ɚ = –4, b = 6.
6. ɚ)
2
( )
4
y
y
−
−
, ɹɤɳɨ ɭ = 0; ɭ = 6; ɭ = –1; ɛ) 2
2
b c
b c
+
−
, ɹɤɳɨ b = 3, c = 4.
Ɂɚɩɨɜɧɿɬɶ ɬɚɛɥɢɰɸ:
7.
ɯ –2 –1 0 1 1,5 2
1
x
x +
8.
ɚ –4 –1 0 1 2 2,5
3
2a −
ɍɤɚɠɿɬɶ ɞɨɩɭɫɬɢɦɿ ɡɧɚɱɟɧɧɹ ɡɦɿɧɧɨʀ ɭ ɜɢɪɚɡɿ:
9. ɚ)
2
6 1;
2
x
x
+
−
ɛ) 6 1 ;
( 3)
a
a a
+
−
ɜ) 1 ;
1
b
b b
+
+
ɝ) 2
11 .
2
x
x +
10. ɚ)
3
1
;
3
y
y
−
+
ɛ) 5 1 ;
2 2x x
−
−
ɜ) ;
( 1)( 1)
m
m m− +
ɝ) 2
1 .
1
a
a
+
+
11. Ⱥɜɬɨɦɨɛɿɥɶ ɩɪɨʀɯɚɜ 195 ɤɦ ɡɚ t ɝɨɞ. Ɂɚɩɢɲɿɬɶ ɭ ɜɢɝɥɹɞɿ ɜɢɪɚɡɭ ɲɜɢɞ-
ɤɿɫɬɶ ɚɜɬɨɦɨɛɿɥɹ. Ɂɧɚɣɞɿɬɶ ɡɧɚɱɟɧɧɹ ɰɶɨɝɨ ɜɢɪɚɡɭ, ɹɤɳɨ t = 3.
12. Ɉɩɟɪɚɬɨɪ ɧɚɛɪɚɜ 45 ɫɬɨɪɿɧɨɤ ɬɟɤɫɬɭ ɡɚ k ɝɨɞ. Ɂɚɩɢɲɿɬɶ ɭ ɜɢɝɥɹɞɿ ɜɢɪɚɡɭ
ɤɿɥɶɤɿɫɬɶ ɫɬɨɪɿɧɨɤ, ɹɤɿ ɨɩɟɪɚɬɨɪ ɧɚɛɢɪɚɜ ɡɚ 1 ɝɨɞ. Ɂɧɚɣɞɿɬɶ ɡɧɚɱɟɧɧɹ
ɰɶɨɝɨ ɜɢɪɚɡɭ, ɹɤɳɨ k = 9.
1. Ɋɚɰɿɨɧɚɥɶɧɿ ɜɢɪɚɡɢ. Ɋɚɰɿɨɧɚɥɶɧɿ ɞɪɨɛɢ 11
Ⱦɥɹ ɹɤɢɯ ɡɧɚɱɟɧɶ ɡɦɿɧɧɨʀ ɜɢɪɚɡ ɧɟ ɦɚɽ ɡɦɿɫɬɭ?
13. ɚ) 2
4 1;
4
x
x
+
−
ɛ) 2
8 ;
5
a
a a−
ɜ) 2
5
( 6)
y
y
−
−
.
14. ɚ) 2
3 ;
7
x
x x−
ɛ) 2
2 7 ;
9
z
z
+
−
ɜ) 2
1 2
( 2)
b
b b
−
+
.
Ɂɧɚɣɞɿɬɶ ɞɨɩɭɫɬɢɦɿ ɡɧɚɱɟɧɧɹ ɡɦɿɧɧɨʀ ɭ ɜɢɪɚɡɿ:
15. ɚ) 2
7 ;
4 1
b b
b
+
−
ɛ) 2
3 ;
4 ( 2)
k
k− +
ɜ) 2
6 .
12
m m
mm m
+
−+
16. ɚ) 2
5 ;
4 9
c
c−
ɛ) 2
3 2 ;
(3 ) 9
−
+ −
n
n
ɜ) 2
5 1.
4
a a
aa
++
−
Ɂɧɚɣɞɿɬɶ ɡɧɚɱɟɧɧɹ ɜɢɪɚɡɭ:
17. 2 3
3 1
a
a
−
+
, ɹɤɳɨ ɚ = –0,2; ɚ = 2
3
; ɚ = 13
6
.
18. 2
5 3
x
x
−
−
, ɹɤɳɨ ɯ = 0,7; ɯ = 3
7
; ɯ = 11
5
.
19. Ɂɧɚɣɞɿɬɶ ɡɧɚɱɟɧɧɹ ɜɢɪɚɡɭ
2 2
2x xy y
y
− +
, ɹɤɳɨ:
ɚ) ɯ = 44; ɭ = 4; ɛ) ɯ = 46; ɭ = 46; ɜ) ɯ = 1,25; ɭ = 0,25.
20. Ɂɧɚɣɞɿɬɶ ɡɧɚɱɟɧɧɹ ɜɢɪɚɡɭ
(1 ) (1 )
4
m n n m
n
− + +
, ɹɤɳɨ:
ɚ) m = 67; n = –67; ɛ) m = 16,75; n = 0,25.
21. Ⱦɨ ɦɚɝɚɡɢɧɭ ɡɚɜɟɡɥɢ 15 ɥ ɜɢɧɨɝɪɚɞɧɨɝɨ ɫɨɤɭ ɜ ɦɚɥɢɯ ɭɩɚɤɨɜɤɚɯ ɿ 25 ɥ —
ɭ ɜɟɥɢɤɢɯ. ɋɤɿɥɶɤɢ ɜɫɶɨɝɨ ɭɩɚɤɨɜɨɤ ɫɨɤɭ ɡɚɜɟɡɥɢ ɞɨ ɦɚɝɚɡɢɧɭ, ɹɤɳɨ ɤɨ-
ɠɧɚ ɦɚɥɚ ɭɩɚɤɨɜɤɚ ɦɿɫɬɢɬɶ ɚ ɥ ɫɨɤɭ, ɚ ɤɨɠɧɚ ɜɟɥɢɤɚ — b ɥ?
22. ɉɟɪɲɢɣ ɪɨɛɿɬɧɢɤ ɜɢɤɥɚɜ ɩɥɢɬɤɨɸ 48 ɦ2
ɞɨɪɿɠɤɢ ɡɚ n ɝɨɞ, ɚ ɞɪɭɝɢɣ — 64 ɦ2
ɡɚ m ɝɨɞ. ɋɤɿɥɶɤɢ ɤɜɚɞɪɚɬɧɢɯ ɦɟɬɪɿɜ ɞɨɪɿɠɤɢ ɜɢɤɥɚɞɚɥɢ ɡɚ 1 ɝɨɞ ɨɛɢɞɜɚ
ɪɨɛɿɬɧɢɤɢ ɪɚɡɨɦ?
23. Ʉɚɬɟɪ ɩɪɨɣɲɨɜ 25 ɤɦ ɡɚ ɬɟɱɿɽɸ ɪɿɱɤɢ ɿ 20 ɤɦ ɩɪɨɬɢ ɬɟɱɿʀ. Ɂɧɚɣɞɿɬɶ ɱɚɫ
ɪɭɯɭ ɤɚɬɟɪɚ, ɹɤɳɨ ɣɨɝɨ ɲɜɢɞɤɿɫɬɶ ɭ ɫɬɨɹɱɿɣ ɜɨɞɿ ɞɨɪɿɜɧɸɽ v ɤɦ/ɝɨɞ, ɚ
ɲɜɢɞɤɿɫɬɶ ɬɟɱɿʀ ɪɿɱɤɢ — u ɤɦ/ɝɨɞ.
12 § 1. Ɋɚɰɿɨɧɚɥɶɧɿ ɜɢɪɚɡɢ
24. Ɂɧɚɣɞɿɬɶ ɞɨɩɭɫɬɢɦɿ ɡɧɚɱɟɧɧɹ ɡɦɿɧɧɨʀ ɭ ɜɢɪɚɡɿ:
ɚ) 11 1;
3
x
x
−
−
ɛ)
3
;
y
y y−
ɜ) 2
;
2
m
m m−
ɝ) 3 .
1 1
a
a
+
− +
25. Ⱦɨɜɟɞɿɬɶ, ɳɨ ɜɢɪɚɡ 2 2
4 4 9
x y
x y x y
+
+ − − +
ɦɚɽ ɡɦɿɫɬ ɞɥɹ ɛɭɞɶ-ɹɤɢɯ ɡɧɚɱɟɧɶ
ɡɦɿɧɧɢɯ.
26. ɉɨʀɡɞ ɦɚɜ ɩɨɞɨɥɚɬɢ ɲɥɹɯ ɡɚɜɞɨɜɠɤɢ 250 ɤɦ, ɪɭɯɚɸɱɢɫɶ ɡɿ ɲɜɢɞɤɿɫɬɸ
ɚ ɤɦ/ɝɨɞ. Ⱥɥɟ ɱɟɪɟɡ 2 ɝɨɞ ɩɿɫɥɹ ɩɨɱɚɬɤɭ ɪɭɯɭ ɣɨɝɨ ɧɚ ɩɟɜɧɢɣ ɱɚɫ
ɡɚɬɪɢɦɚɥɢ. ɓɨɛ ɩɪɢɛɭɬɢ ɞɨ ɦɿɫɰɹ ɩɪɢɡɧɚɱɟɧɧɹ ɜɱɚɫɧɨ, ɩɨʀɡɞ ɩɿɫɥɹ
ɡɚɬɪɢɦɤɢ ɡɛɿɥɶɲɢɜ ɲɜɢɞɤɿɫɬɶ ɧɚ 25 ɤɦ/ɝɨɞ. ɇɚ ɹɤɢɣ ɱɚɫ ɡɚɬɪɢɦɚɥɢ
ɩɨʀɡɞ?
27. Ɋɨɡɤɥɚɞɿɬɶ ɧɚ ɦɧɨɠɧɢɤɢ:
ɚ) ab2
– ac2
; ɛ) ɯ3
+ 8;
ɜ) xy + 8x + 9ɭ + 72; ɝ) a2
– 4b2
+ a + 2b.
28. ɉɨɪɿɜɧɹɣɬɟ ɞɪɨɛɢ: 7
9
ɿ 20 ;
27
11
18
ɿ 17 ;
24
7
15
ɿ 9 .
25
29. ɋɤɨɪɨɬɿɬɶ ɞɪɨɛɢ: 18 ;
48
56 ;
98
96 ;
123
175 ;
325
77 .
121
30. ɒɤɨɥɿ ɩɨɬɪɿɛɧɨ ɡɚɤɭɩɢɬɢ ɩɚɪɬɢ. ɉɟɪɲɚ ɮɿɪɦɚ ɩɪɨɩɨɧɭɽ ɤɭɩɢɬɢ ɩɚɪɬɢ
ɩɨ 975 ɝɪɧ ɡɚ ɤɨɠɧɭ ɿ 4% ɜɚɪɬɨɫɬɿ ɭɫɿɯ ɤɭɩɥɟɧɢɯ ɩɚɪɬ ɡɚ ɞɨɫɬɚɜɤɭ, ɚ
ɞɪɭɝɚ — ɩɨ 1010 ɝɪɧ ɡɚ ɤɨɠɧɭ ɿ ɛɟɡɤɨɲɬɨɜɧɭ ɞɨɫɬɚɜɤɭ. ɍ ɹɤɿɣ ɮɿɪɦɿ
ɜɢɝɿɞɧɿɲɟ ɤɭɩɭɜɚɬɢ ɩɚɪɬɢ?
31. ɇɚ ɱɚɪɿɜɧɿɣ ɹɛɥɭɧɿ ɪɨɫɬɭɬɶ 55 ɹɛɥɭɤ. Ɂ ɧɟʀ ɞɨɡɜɨɥɹɸɬɶ ɡɪɢɜɚɬɢ 2, 3, 6
ɚɛɨ 9 ɹɛɥɭɤ. Ɂɚɦɿɫɬɶ ɧɢɯ ɧɚ ɹɛɥɭɧɿ ɜɿɞɪɚɡɭ ɜɢɪɨɫɬɚɸɬɶ ɜɿɞɩɨɜɿɞɧɨ 4, 5, 2
ɚɛɨ 7 ɧɨɜɢɯ ɹɛɥɭɤ. ɑɢ ɦɨɠɧɚ ɡ ɹɛɥɭɧɿ ɡɿɪɜɚɬɢ ɜɫɿ ɹɛɥɭɤɚ, ɹɤɳɨ ɩɿɫɥɹ
ɡɪɢɜɚɧɧɹ ɨɫɬɚɧɧɶɨɝɨ ɜɨɧɢ ɛɿɥɶɲɟ ɧɟ ɜɢɪɨɫɬɚɸɬɶ?
2. Ɉɫɧɨɜɧɚ ɜɥɚɫɬɢɜɿɫɬɶ ɞɪɨɛɭ 13
1. Ɉɫɧɨɜɧɚ ɜɥɚɫɬɢɜɿɫɬɶ ɞɪɨɛɭ. ɉɪɢɝɚɞɚɣɦɨ ɨɫɧɨɜɧɭ ɜɥɚɫɬɢɜɿɫɬɶ ɡɜɢ-
ɱɚɣɧɢɯ ɞɪɨɛɿɜ: ɹɤɳɨ ɱɢɫɟɥɶɧɢɤ ɿ ɡɧɚɦɟɧɧɢɤ ɞɪɨɛɭ ɩɨɦɧɨɠɢɬɢ ɚɛɨ ɩɨɞɿɥɢɬɢ
ɧɚ ɬɟ ɫɚɦɟ ɧɚɬɭɪɚɥɶɧɟ ɱɢɫɥɨ, ɬɨ ɨɬɪɢɦɚɽɦɨ ɞɪɿɛ, ɹɤɢɣ ɞɨɪɿɜɧɸɽ ɞɚɧɨɦɭ. Ɉɬ-
ɠɟ, ɹɤɳɨ a, b ɿ k — ɧɚɬɭɪɚɥɶɧɿ ɱɢɫɥɚ, ɬɨ a ak
b bk
= ɿ .ak a
bk b
=
Ⱥɧɚɥɨɝɿɱɧɚ ɜɥɚɫɬɢɜɿɫɬɶ ɫɩɪɚɜɟɞɥɢɜɚ ɞɥɹ ɛɭɞɶ-ɹɤɢɯ ɞɪɨɛɿɜ. Ⱥ ɫɚɦɟ:
Ⱦɥɹ ɛɭɞɶ-ɹɤɢɯ ɡɧɚɱɟɧɶ ɚ, b ɿ k, ɞɟ b ≠ 0 ɿ k ≠ 0, ɜɢɤɨɧɭɸɬɶɫɹ ɪɿɜɧɨɫɬɿ
;
a ak
b bk
= .
ak a
bk b
=
Ⱦɚɧɿ ɪɿɜɧɨɫɬɿ ɽ ɬɨɬɨɠɧɨɫɬɹɦɢ ɿ ɜɢɪɚɠɚɸɬɶ ɨɫɧɨɜɧɭ ɜɥɚɫɬɢɜɿɫɬɶ ɞɪɨɛɭ,
ɹɤɭ ɦɨɠɧɚ ɫɮɨɪɦɭɥɸɜɚɬɢ ɬɚɤ:
əɤɳɨ ɱɢɫɟɥɶɧɢɤ ɿ ɡɧɚɦɟɧɧɢɤ ɞɪɨɛɭ ɩɨɦɧɨɠɢɬɢ ɚɛɨ ɩɨɞɿɥɢɬɢ ɧɚ
ɜɢɪɚɡ, ɧɟ ɬɨɬɨɠɧɨ ɪɿɜɧɢɣ ɧɭɥɸ, ɬɨ ɨɞɟɪɠɢɦɨ ɞɪɿɛ, ɬɨɬɨɠɧɨ ɪɿɜɧɢɣ
ɞɚɧɨɦɭ.
2. ɋɤɨɪɨɱɟɧɧɹ ɞɪɨɛɿɜ. Ɂɚ ɞɨɩɨɦɨɝɨɸ ɬɨɬɨɠɧɨɫɬɿ ak a
bk b
= ɞɪɿɛ ak
bk
ɦɨɠ-
ɧɚ ɡɚɦɿɧɢɬɢ ɞɪɨɛɨɦ ,a
b
ɬɨɛɬɨ ɞɪɿɛ ak
bk
ɦɨɠɧɚ ɫɤɨɪɨɬɢɬɢ ɧɚ ɫɩɿɥɶɧɢɣ ɦɧɨɠ-
ɧɢɤ k ɱɢɫɟɥɶɧɢɤɚ ɿ ɡɧɚɦɟɧɧɢɤɚ. ɇɚɩɪɢɤɥɚɞ,
2
4 2 2 2 ;
2 2
n n n n
mn m n m
⋅= =
⋅
2 2
2
( )( )
.
( )
a b a ba b a b
a a b aa ab
+ −− +
= =
−−
Ɋɿɜɧɨɫɬɿ
2
4 2
2
n n
mn m
= ɿ
2 2
2
a b a b
aa ab
− +
=
−
ɽ ɬɨɬɨɠɧɨɫɬɹɦɢ, ɬɨɛɬɨ ɜɨɧɢ ɽ ɩɪɚ-
ɜɢɥɶɧɢɦɢ ɞɥɹ ɜɫɿɯ ɞɨɩɭɫɬɢɦɢɯ ɡɧɚɱɟɧɶ ɡɦɿɧɧɢɯ (ɩɟɪɲɚ — ɞɥɹ ɜɫɿɯ ɡɧɚɱɟɧɶ m
ɿ n, ɞɟ m ≠ 0, n ≠ 0; ɞɪɭɝɚ — ɞɥɹ ɜɫɿɯ ɡɧɚɱɟɧɶ ɚ ɿ b, ɞɟ ɚ ≠ 0, a ≠ b).
ɓɨɛ ɫɤɨɪɨɬɢɬɢ ɞɪɿɛ, ɩɨɬɪɿɛɧɨ:
1) ɜɢɞɿɥɢɬɢ ɫɩɿɥɶɧɢɣ ɦɧɨɠɧɢɤ ɱɢɫɟɥɶɧɢɤɚ ɿ ɡɧɚɦɟɧɧɢɤɚ ɞɪɨɛɭ;
2) ɜɢɤɨɧɚɬɢ ɫɤɨɪɨɱɟɧɧɹ ɧɚ ɫɩɿɥɶɧɢɣ ɦɧɨɠɧɢɤ.
14 § 1. Ɋɚɰɿɨɧɚɥɶɧɿ ɜɢɪɚɡɢ
3. Ɂɜɟɞɟɧɧɹ ɞɪɨɛɿɜ ɞɨ ɫɩɿɥɶɧɨɝɨ ɡɧɚɦɟɧɧɢɤɚ. Ɂɚ ɞɨɩɨɦɨɝɨɸ ɬɨɬɨɠɧɨɫɬɿ
a ak
b bk
= ɞɪɿɛ a
b
ɦɨɠɧɚ ɡɜɨɞɢɬɢ ɞɨ ɧɨɜɨɝɨ ɡɧɚɦɟɧɧɢɤɚ. ɇɚɩɪɢɤɥɚɞ,
2
3 3 2 6
2 2
x x x x
y y x xy
⋅= =
⋅
— ɡɜɟɥɢ ɞɪɿɛ 3x
y
ɞɨ ɡɧɚɦɟɧɧɢɤɚ 2ɯɭ.
Ȼɭɞɶ-ɹɤɿ ɞɪɨɛɢ ɡ ɪɿɡɧɢɦɢ ɡɧɚɦɟɧɧɢɤɚɦɢ, ɹɤ ɿ ɡɜɢɱɚɣɧɿ ɞɪɨɛɢ, ɦɨɠɧɚ ɡɜɟɫ-
ɬɢ ɞɨ ɫɩɿɥɶɧɨɝɨ ɡɧɚɦɟɧɧɢɤɚ. Ɋɨɡɝɥɹɧɟɦɨ ɩɪɢɤɥɚɞɢ.
ɉɪɢɤɥɚɞ 1. Ɂɜɟɫɬɢ ɞɨ ɫɩɿɥɶɧɨɝɨ ɡɧɚɦɟɧɧɢɤɚ ɞɪɨɛɢ 2
3
y
x
ɬɚ 4 .
y
Ɣ ɋɩɿɥɶɧɢɦ ɡɧɚɦɟɧɧɢɤɨɦ ɞɚɧɢɯ ɞɪɨɛɿɜ ɽ ɞɨɛɭɬɨɤ ʀɯɧɿɯ ɡɧɚɦɟɧɧɢɤɿɜ,
ɬɨɛɬɨ 3ɯ2
ɭ. Ⱦɨɞɚɬɤɨɜɢɦ ɦɧɨɠɧɢɤɨɦ ɞɥɹ ɩɟɪɲɨɝɨ ɞɪɨɛɭ ɽ ɭ, ɞɥɹ ɞɪɭɝɨɝɨ —
3ɯ2
. Ɍɨɞɿ:
2
2 2 2
;
3 3 3
y y y y
x x y x y
⋅
= =
⋅
2 2
2 2
4 4 3 12 .
3 3
x x
y y x x y
⋅= =
⋅
Ɣ
ɉɪɢɤɥɚɞ 2. Ɂɜɟɫɬɢ ɞɨ ɫɩɿɥɶɧɨɝɨ ɡɧɚɦɟɧɧɢɤɚ ɞɪɨɛɢ 3
5
8a b
ɿ 2 2
7 .
12a c
Ɣ Ɂɧɚɦɟɧɧɢɤɢ ɨɛɨɯ ɞɪɨɛɿɜ ɽ ɨɞɧɨɱɥɟɧɚɦɢ, ɬɨɦɭ ɫɩɿɥɶɧɢɣ ɡɧɚɦɟɧɧɢɤ
ɲɭɤɚɬɢɦɟɦɨ ɭ ɜɢɝɥɹɞɿ ɨɞɧɨɱɥɟɧɚ, ɞɨ ɬɨɝɨ ɠ ɹɤɨɦɨɝɚ ɦɟɧɲɨɝɨ ɫɬɟɩɟɧɹ. Ɂɚ ɤɨ-
ɟɮɿɰɿɽɧɬ ɰɶɨɝɨ ɨɞɧɨɱɥɟɧɚ ɜɿɡɶɦɟɦɨ ɧɚɣɦɟɧɲɟ ɫɩɿɥɶɧɟ ɤɪɚɬɧɟ ɤɨɟɮɿɰɿɽɧɬɿɜ
ɡɧɚɦɟɧɧɢɤɿɜ ɞɚɧɢɯ ɞɪɨɛɿɜ, ɬɨɛɬɨ 24, ɚ ɤɨɠɧɭ ɡɦɿɧɧɭ ɜɿɡɶɦɟɦɨ ɡ ɧɚɣɛɿɥɶɲɢɦ
ɩɨɤɚɡɧɢɤɨɦ, ɡ ɹɤɢɦ ɜɨɧɚ ɜɯɨɞɢɬɶ ɭ ɡɧɚɦɟɧɧɢɤɢ ɞɪɨɛɿɜ, ɬɨɛɬɨ ɜɿɡɶɦɟɦɨ a3
, b ɿ
c2
. Ɍɨɞɿ ɫɩɿɥɶɧɢɦ ɡɧɚɦɟɧɧɢɤɨɦ ɛɭɞɟ 24a3
bc2
. Ⱦɨɞɚɬɤɨɜɢɦ ɦɧɨɠɧɢɤɨɦ ɞɥɹ
ɩɟɪɲɨɝɨ ɞɪɨɛɭ ɽ 3ɫ2
, ɛɨ 24a3
bc2
= 8a3
b ⋅ 3c2
; ɞɥɹ ɞɪɭɝɨɝɨ — 2ab, ɛɨ 24a3
bc2
=
= 12a2
c2
⋅ 2ab. Ɇɚɬɢɦɟɦɨ:
2 2
3 3 2 3 2
5 5 3 15 ;
8 24 24
c c
a b a bc a bc
⋅= = 2 2 3 2 3 2
7 7 2 14 .
12 24 24
ab ab
a c a bc a bc
⋅= = Ɣ
ɓɨɛ ɡɜɟɫɬɢ ɞɨ ɩɪɨɫɬɿɲɨɝɨ ɫɩɿɥɶɧɨɝɨ ɡɧɚɦɟɧɧɢɤɚ ɞɪɨɛɢ, ɡɧɚɦɟɧɧɢɤɚɦɢ
ɹɤɢɯ ɽ ɨɞɧɨɱɥɟɧɢ, ɩɨɬɪɿɛɧɨ:
1) ɡɧɚɣɬɢ ɧɚɣɦɟɧɲɟ ɫɩɿɥɶɧɟ ɤɪɚɬɧɟ (ɇɋɄ) ɤɨɟɮɿɰɿɽɧɬɿɜ ɡɧɚɦɟɧɧɢɤɿɜ;
2) ɭɬɜɨɪɢɬɢ ɫɩɿɥɶɧɢɣ ɡɧɚɦɟɧɧɢɤ ɭ ɜɢɝɥɹɞɿ ɞɨɛɭɬɤɭ ɇɋɄ ɿ ɫɬɟɩɟɧɿɜ ɡɦɿɧ-
ɧɢɯ ɡ ɧɚɣɛɿɥɶɲɢɦ ɩɨɤɚɡɧɢɤɨɦ, ɡ ɹɤɢɦ ɜɨɧɢ ɜɯɨɞɹɬɶ ɞɨ ɡɧɚɦɟɧɧɢɤɿɜ;
2. Ɉɫɧɨɜɧɚ ɜɥɚɫɬɢɜɿɫɬɶ ɞɪɨɛɭ 15
3) ɩɨɦɧɨɠɢɬɢ ɱɢɫɟɥɶɧɢɤ ɿ ɡɧɚɦɟɧɧɢɤ ɤɨɠɧɨɝɨ ɞɪɨɛɭ ɧɚ ɜɿɞɩɨɜɿɞɧɢɣ ɞɨ-
ɞɚɬɤɨɜɢɣ ɦɧɨɠɧɢɤ. (ɓɨɛ ɡɧɚɣɬɢ ɞɨɞɚɬɤɨɜɢɣ ɦɧɨɠɧɢɤ ɞɥɹ ɞɪɨɛɭ, ɩɨɬɪɿ-
ɛɧɨ ɡɚɩɢɫɚɬɢ ɫɩɿɥɶɧɢɣ ɡɧɚɦɟɧɧɢɤ ɭ ɜɢɝɥɹɞɿ ɞɨɛɭɬɤɭ ɞɜɨɯ ɨɞɧɨɱɥɟɧɿɜ,
ɨɞɧɢɦ ɡ ɹɤɢɯ ɽ ɡɧɚɦɟɧɧɢɤ ɞɚɧɨɝɨ ɞɪɨɛɭ. Ɍɨɞɿ ɞɪɭɝɢɣ ɨɞɧɨɱɥɟɧ ɛɭɞɟ ɞɨ-
ɞɚɬɤɨɜɢɦ ɦɧɨɠɧɢɤɨɦ.)
ɉɪɢɤɥɚɞ 3. Ɂɜɟɫɬɢ ɞɨ ɫɩɿɥɶɧɨɝɨ ɡɧɚɦɟɧɧɢɤɚ ɞɪɨɛɢ 2
3
a ab−
ɿ 2
2 .
a ab+
Ɣ Ɋɨɡɤɥɚɞɟɦɨ ɧɚ ɦɧɨɠɧɢɤɢ ɡɧɚɦɟɧɧɢɤ ɤɨɠɧɨɝɨ ɞɪɨɛɭ:
2
3 3 ;
( )a a ba ab
=
−− 2
2 2 .
( )a a ba ab
=
++
ɋɩɿɥɶɧɢɦ ɡɧɚɦɟɧɧɢɤɨɦ ɞɪɨɛɿɜ ɽ ɞɨɛɭɬɨɤ ɚ(ɚ – b)(ɚ + b) = ɚ(ɚ2
– b2
). Ⱦɨɞɚɬ-
ɤɨɜɢɦ ɦɧɨɠɧɢɤɨɦ ɞɥɹ ɩɟɪɲɨɝɨ ɞɪɨɛɭ ɽ ɜɢɪɚɡ ɚ + b, ɞɥɹ ɞɪɭɝɨɝɨ — ɜɢɪɚɡ ɚ – b.
ɉɨɦɧɨɠɢɜɲɢ ɱɢɫɟɥɶɧɢɤ ɿ ɡɧɚɦɟɧɧɢɤ ɤɨɠɧɨɝɨ ɞɪɨɛɭ ɧɚ ɜɿɞɩɨɜɿɞɧɢɣ ɞɨɞɚɬɤɨ-
ɜɢɣ ɦɧɨɠɧɢɤ, ɨɞɟɪɠɢɦɨ:
2 2 2
3( )3 ;
( )
a b
a ab a a b
+
=
− − 2 2 2
2( )2 .
( )
a b
a ab a a b
−
=
+ −
Ɣ
ɓɨɛ ɡɜɟɫɬɢ ɞɨ ɩɪɨɫɬɿɲɨɝɨ ɫɩɿɥɶɧɨɝɨ ɡɧɚɦɟɧɧɢɤɚ ɞɪɨɛɢ, ɡɧɚɦɟɧɧɢɤɚɦɢ
ɹɤɢɯ ɽ ɦɧɨɝɨɱɥɟɧɢ, ɩɨɬɪɿɛɧɨ:
1) ɪɨɡɤɥɚɫɬɢ ɧɚ ɦɧɨɠɧɢɤɢ ɡɧɚɦɟɧɧɢɤ ɤɨɠɧɨɝɨ ɞɪɨɛɭ;
2) ɭɬɜɨɪɢɬɢ ɫɩɿɥɶɧɢɣ ɡɧɚɦɟɧɧɢɤ ɭ ɜɢɝɥɹɞɿ ɞɨɛɭɬɤɭ ɨɞɟɪɠɚɧɢɯ ɦɧɨɠɧɢ-
ɤɿɜ ɡ ɧɚɣɛɿɥɶɲɢɦ ɩɨɤɚɡɧɢɤɨɦ, ɡ ɹɤɢɦ ɜɨɧɢ ɜɯɨɞɹɬɶ ɞɨ ɡɧɚɦɟɧɧɢɤɿɜ;
3) ɩɨɦɧɨɠɢɬɢ ɱɢɫɟɥɶɧɢɤ ɿ ɡɧɚɦɟɧɧɢɤ ɤɨɠɧɨɝɨ ɞɪɨɛɭ ɧɚ ɜɿɞɩɨɜɿɞɧɢɣ ɞɨ-
ɞɚɬɤɨɜɢɣ ɦɧɨɠɧɢɤ.
4. Ɂɦɿɧɚ ɡɧɚɤɚ ɱɢɫɟɥɶɧɢɤɚ ɚɛɨ ɡɧɚɦɟɧɧɢɤɚ ɞɪɨɛɭ. Ɋɨɡɝɥɹɧɟɦɨ ɩɪɚɜɢ-
ɥɶɧɭ ɱɢɫɥɨɜɭ ɪɿɜɧɿɫɬɶ 1 1 .
2 2
− =− Ȳʀ ɦɨɠɧɚ ɩɪɨɤɨɦɟɧɬɭɜɚɬɢ ɬɚɤ: ɹɤɳɨ ɡɦɿɧɢɬɢ
ɡɧɚɤ ɭ ɱɢɫɟɥɶɧɢɤɭ ɞɪɨɛɭ ɿ ɡɧɚɤ ɩɟɪɟɞ ɞɪɨɛɨɦ, ɬɨ ɨɞɟɪɠɢɦɨ ɞɪɿɛ, ɹɤɢɣ ɞɨɪɿɜ-
ɧɸɽ ɞɚɧɨɦɭ.
16 § 1. Ɋɚɰɿɨɧɚɥɶɧɿ ɜɢɪɚɡɢ
ɍ ɬɚɤɢɣ ɠɟ ɫɩɨɫɿɛ ɡɦɿɧɸɸɬɶ ɡɧɚɤ ɱɢɫɟɥɶɧɢɤɚ ɚɛɨ ɡɧɚɦɟɧɧɢɤɚ ɛɭɞɶ-
ɹɤɨɝɨ ɞɪɨɛɭ, ɜɢɤɨɪɢɫɬɨɜɭɸɱɢ ɬɨɬɨɠɧɨɫɬɿ:
,a a
b b
− =− .a a
b b
=−
−
əɤɳɨ ɡɦɿɧɢɬɢ ɡɧɚɤ ɭ ɱɢɫɟɥɶɧɢɤɭ ɚɛɨ ɡɧɚɦɟɧɧɢɤɭ ɞɪɨɛɭ ɿ ɡɧɚɤ ɩɟɪɟɞ ɞɪɨ-
ɛɨɦ, ɬɨ ɨɞɟɪɠɢɦɨ ɞɪɿɛ, ɬɨɬɨɠɧɨ ɪɿɜɧɢɣ ɞɚɧɨɦɭ.
Ⱦɨɜɟɞɟɦɨ ɨɫɧɨɜɧɭ ɜɥɚɫɬɢɜɿɫɬɶ ɞɪɨɛɿɜ. ɉɨɤɚɠɟɦɨ, ɳɨ ɪɿɜɧɿɫɬɶ a ak
b bk
= ɽ ɬɨɬɨɠ-
ɧɿɫɬɸ, ɬɨɛɬɨ ɳɨ ɜɨɧɚ ɜɢɤɨɧɭɽɬɶɫɹ ɞɥɹ ɛɭɞɶ-ɹɤɢɯ ɡɧɚɱɟɧɶ ɚ, b ɿ k, ɞɟ b ≠ 0 ɿ k ≠ 0.
ɇɟɯɚɣ .a m
b
= Ɂɚ ɨɡɧɚɱɟɧɧɹɦ ɱɚɫɬɤɢ ɦɚɽɦɨ: ɚ = bm. ɉɨɦɧɨɠɢɜɲɢ ɨɛɢɞɜɿ ɱɚɫɬɢ-
ɧɢ ɨɞɟɪɠɚɧɨʀ ɪɿɜɧɨɫɬɿ ɧɚ k, ɦɚɬɢɦɟɦɨ ɩɪɚɜɢɥɶɧɭ ɪɿɜɧɿɫɬɶ ɚk = (bm)k ɚɛɨ ɚk = (bk)m.
Ɉɫɤɿɥɶɤɢ b ≠ 0 ɿ k ≠ 0, ɬɨ bk ≠ 0. ɍ ɬɚɤɨɦɭ ɜɢɩɚɞɤɭ ɡ ɪɿɜɧɨɫɬɿ ɚk = (bk)m ɡɧɨɜɭ ɠ ɬɚɤɢ
ɡɚ ɨɡɧɚɱɟɧɧɹɦ ɱɚɫɬɤɢ ɦɚɽɦɨ: .akm
bk
= Ɉɬɠɟ, .a akm
b bk
= =
ȼɩɪɚɜɚ 1. ȼɢɞɿɥɢɬɢ ɫɩɿɥɶɧɢɣ ɦɧɨɠɧɢɤ ɱɢɫɟɥɶɧɢɤɚ ɬɚ ɡɧɚɦɟɧɧɢɤɚ ɞɪɨɛɭ ɣ
ɫɤɨɪɨɬɢɬɢ ɞɪɿɛ:
ɚ) 12 ;
8
a
ab
ɛ)
3
2 2
18
.
6
xy
x y
−
−
Ɣ ɚ) 12
8
a
ab
= 4 3
4 2
a
a b
⋅
⋅
= 3 .
2b
ɛ)
3
2 2
18
6
xy
x y
−
−
=
2
2
6 3
6
xy y
xy x
− ⋅
− ⋅
=
3
.
y
x
Ɣ
ȼɩɪɚɜɚ 2. ɋɤɨɪɨɬɢɬɢ ɞɪɿɛ:
ɚ) 2 2
10 5 ;
4
b a
a b
−
−
ɛ)
2 2
3 3
.
x xy y
x y
+ +
−
Ɣ ɚ) 2 2
10 5
4
b a
a b
−
−
=
5( 2 )
( 2 )( 2 )
a b
a b a b
− −
− +
= 5
2a b
−
+
= 5 .
2a b
−
+
2. Ɉɫɧɨɜɧɚ ɜɥɚɫɬɢɜɿɫɬɶ ɞɪɨɛɭ 17
ɛ)
2 2
3 3
x xy y
x y
+ +
−
=
2 2
2 2
( )( )
x xy y
x y x xy y
+ +
− + +
= 1 .
x y−
Ɣ
ȼɩɪɚɜɚ 3. Ɂɜɟɫɬɢ ɞɪɿɛ 3
7
a
b
ɞɨ ɡɧɚɦɟɧɧɢɤɚ 42ɚ2
b.
Ɣ Ɉɫɤɿɥɶɤɢ 42ɚ2
b = 7b ⋅ 6ɚ2
, ɬɨ, ɩɨɦɧɨɠɢɜɲɢ ɱɢɫɟɥɶɧɢɤ ɿ ɡɧɚɦɟɧɧɢɤ ɞɚ-
ɧɨɝɨ ɞɪɨɛɭ ɧɚ 6ɚ2
, ɦɚɬɢɦɟɦɨ: 3
7
a
b
=
2
2
3 6
7 6
a a
b a
⋅
⋅
=
3
2
18 .
42
a
a b
Ɣ
ȼɩɪɚɜɚ 4. Ɂɜɟɫɬɢ ɞɨ ɫɩɿɥɶɧɨɝɨ ɡɧɚɦɟɧɧɢɤɚ ɞɪɨɛɢ 2 2
3
m n−
ɿ 9 .
n m−
• 2 2
3 3 ;
( )( )m n m nm n
=
− +−
9 9 .
n m m n
= −
− −
ɋɩɿɥɶɧɢɦ ɡɧɚɦɟɧɧɢɤɨɦ
ɞɪɨɛɿɜ ɽ ɞɨɛɭɬɨɤ (m – n)(m + n). Ⱦɨɞɚɬɤɨɜɢɦ ɦɧɨɠɧɢɤɨɦ ɞɥɹ ɩɟɪɲɨɝɨ ɞɪɨɛɭ ɽ
1, ɞɥɹ ɞɪɭɝɨɝɨ — m + n. Ɍɨɦɭ ɩɟɪɲɢɣ ɞɪɿɛ ɡɚɥɢɲɚɽɦɨ ɛɟɡ ɡɦɿɧɢ, ɚ ɞɥɹ ɞɪɭɝɨ-
ɝɨ ɞɪɨɛɭ ɦɚɬɢɦɟɦɨ: 2 2
9( ) 9( )9 .
( )( )
m n m n
n m m n m n m n
+ +
= − = −
− − + −
•
32. ɋɤɨɪɨɬɿɬɶ ɞɪɿɛ:
ɚ) 5 ;
15
x
y
ɛ) ;
4
ab
b
ɜ)
( 2)
;
( 2)
m n
n n
−
−
ɝ)
2
3
18 .a
a
33. Ɂɜɟɞɿɬɶ ɞɪɿɛ:
ɚ) 11
b
ɞɨ ɡɧɚɦɟɧɧɢɤɚ b2
; ɛ) 3
2
x
y
ɞɨ ɡɧɚɦɟɧɧɢɤɚ 4ɯɭ.
34. Ɂɜɟɞɿɬɶ ɞɨ ɫɩɿɥɶɧɨɝɨ ɡɧɚɦɟɧɧɢɤɚ ɞɪɨɛɢ:
ɚ) a
b
ɿ
3
b ; ɛ) 1
mn
ɿ 2
1
n
.
35. Ɂɦɿɧɿɬɶ ɡɧɚɤ ɡɧɚɦɟɧɧɢɤɚ ɞɪɨɛɭ:
ɚ) 2
( )x y− −
; ɛ) 2
x y−
.
18 § 1. Ɋɚɰɿɨɧɚɥɶɧɿ ɜɢɪɚɡɢ
36. ȼɢɞɿɥɿɬɶ ɫɩɿɥɶɧɢɣ ɦɧɨɠɧɢɤ ɱɢɫɟɥɶɧɢɤɚ ɬɚ ɡɧɚɦɟɧɧɢɤɚ ɞɪɨɛɭ ɣ
ɫɤɨɪɨɬɿɬɶ ɞɪɿɛ:
ɚ) 3 ;
5
x
x
ɛ) 4 ;
6
a
a
ɜ) 9 ;
6
ab
b
ɝ)
2
2
10
.
15
x y
xy
−
ɋɤɨɪɨɬɿɬɶ ɞɪɿɛ:
37. ɚ)
2 2
2 3
28
;
35
x y
x y
ɛ)
2 2
24 ;
36
b c
bc
ɜ)
2
2 2
15 ;
40
mn
m n
− ɝ)
2 4
4 3
8 .
12
k m
k m−
38. ɚ)
2 2
3
18 ;
12
c n
n
ɛ)
2
36
;
28
xy
xy
ɜ)
2
2 3
40 ;
24
ab
a b−
ɝ)
3
3
14 .
42
ac
bc
−
−
39. ɚ)
( )
;
a m n
m n
−
−
ɛ)
( )
;
3 ( )
b c d
b c d
+
+
ɜ) 5 ;
15 20
k
k +
ɝ)
2
.m mn
mn
−
40. ɚ)
( )
;
( )
ab a b
c a b
+
+
ɛ)
( 2 )
;
( )
m x y
m x y
−
−
ɜ) 3 9 ;
3
x
x
−
−
ɝ)
7
.
5
xy
xy y−
41. ɉɨɞɚɣɬɟ ɱɚɫɬɤɭ ɭ ɜɢɝɥɹɞɿ ɞɪɨɛɭ ɣ ɫɤɨɪɨɬɿɬɶ ɞɪɿɛ:
ɚ) 10ɚ2
b2
: (5ɚ3
b); ɛ) 24m2
n : (–6mn); ɜ) (–28ab3
) : (–21b4
).
Ɂɧɚɣɞɿɬɶ ɡɧɚɱɟɧɧɹ ɜɢɪɚɡɭ:
42. ɚ)
3
2 2
20
4
a b
a b
, ɹɤɳɨ ɚ = 48; b = 16; ɚ = –4,2; b = 2,1.
ɛ) 15ɯ2
ɭ3
: (30ɯɭ2
), ɹɤɳɨ ɯ = 300; ɭ = 0,06.
43. ɚ)
3
2 2
18
2
bc
b c
, ɹɤɳɨ b = 3; c = 4,5; b = –1,4, c = 2,8;
ɛ) 64m2
n4
: (16mn2
), ɹɤɳɨ m = 0,25; n = 25.
ɋɤɨɪɨɬɿɬɶ ɞɪɿɛ:
44. ɚ) 6 3 ;
8 4
a b
a b
−
−
ɛ)
2
2
12 16 ;
3 4
a a
a a
−
−
ɜ)
2
2
;
xy x y
xy xy
+
−
ɝ)
2 2
;a b
a b
−
−
ɞ)
2
9 ;
7 21
a
a
−
+
ɟ) 2
10 20 ;
7 28
x
x
−
−
ɽ) 2
4 8
;
4 4
y
y y
−
− +
ɠ)
2
2
6 9.
9
x x
x
+ +
−
2. Ɉɫɧɨɜɧɚ ɜɥɚɫɬɢɜɿɫɬɶ ɞɪɨɛɭ 19
45. ɚ)
9 6
;
15 10
x y
x y
−
−
ɛ)
2
2
2 ;
2
c c
c c
+
−
ɜ) 2
10 10
;
x y
xy y
−
−
ɝ)
2
2
;ab a
ab b
+
+
ɞ)
2 2
;m n
n m
−
+
ɟ) 2 2
5 5
;
x y
x y
+
−
ɽ)
2
2 1;
3 3
m m
m
+ +
+
ɠ)
2
2
25 .
10 25
a
a a
−
− +
46. Ɂɜɟɞɿɬɶ ɞɪɿɛ:
ɚ)
4
k
p
ɞɨ ɡɧɚɦɟɧɧɢɤɚ: 12p; 16p2
; ɛ) 2
5
2a
ɞɨ ɡɧɚɦɟɧɧɢɤɚ: 4ɚ4
; 10ɚ2
b.
47. Ɂɜɟɞɿɬɶ ɞɪɿɛ 4
3xy
ɞɨ ɡɧɚɦɟɧɧɢɤɚ: 15xy; 3xy2
; 9ɯ3
y.
Ɂɜɟɞɿɬɶ ɞɨ ɫɩɿɥɶɧɨɝɨ ɡɧɚɦɟɧɧɢɤɚ ɞɪɨɛɢ:
48. ɚ) x
y
ɿ 2 ;
x
ɛ) m
ab
ɿ 4 ;
b
ɜ) 2
d
a
ɿ 3
1 ;
a
ɝ) 3
2c
ɿ 2
9 ;
c
ɞ) 1
3c
ɿ 2 ;
5c
ɟ) 3
8a
ɿ 1 ;
12a
ɽ) 2
18
x
a
ɿ 4
;
27
y
a
ɠ) 5
6ab
ɿ 5 ;
4b
ɡ) 2
3
p
a
ɿ .
6
q
ab
49. ɚ) 3
2a
ɿ 2 ;
b
ɛ) 7
8a
ɿ 5 ;
a
ɜ)
2
k
b
ɿ ;
3
n
b
ɝ) 8
15ab
ɿ 7 ;
20ab
ɞ) 2
5
6a
ɿ 7 ;
18a
ɟ) 3
3
4y
ɿ 2
7 .
20y
50. ɚ) 5
1a +
ɿ 4 ;
2a +
ɛ) 3
2( 1)a −
ɿ 2 ;
3( 1)a −
ɜ) 1
ab b+
ɿ 1 .
1a +
51. ɚ) 1
3c +
ɿ 2 ;
1c −
ɛ) 3
8( 2)b +
ɿ 1 ;
4( 2)b +
ɜ) 8
xy x−
ɿ 7 .
1y −
ɋɤɨɪɨɬɿɬɶ ɞɪɿɛ:
52. ɚ)
2
2 2
6 9 ;
4 9
ab b
a b
−
−
ɛ)
2 2
2 2
4 25 ;
4 20 25
c x
c cx x
−
+ +
ɜ)
3 3
2 2
2 8
;
2
x y xy
xy x y
−
−
20 § 1. Ɋɚɰɿɨɧɚɥɶɧɿ ɜɢɪɚɡɢ
ɝ)
3
8;
2
x
x
+
+
ɞ)
2
3
3 9 ;
27
z z
z
+ +
−
ɟ)
6
2
1
;
1
y
y
−
−
ɽ) ;
ax cx ay cy
cx cy
+ − −
−
ɠ)
2 2
2
2 ;b ab a
a ab ax bx
+ +
+ − −
ɡ) 2
8 4 .
2 2
a b
ab b ad bd
+
+ − −
53. ɚ) 2 2
14 63 ;
4 81
b c
b c
−
−
ɛ) 2
3 12 ;
8 16
kn n
k k
−
− +
ɜ)
2
2 3
6 2 ;
9
mn m
mn m
+
−
ɝ) 3
15 5 ;
27
c
c
−
−
ɞ)
2 2
2
;
x y
xy x y y
−
− + −
ɟ)
2
2
.a ac bc ab
a b abc
+ + +
+
Ɂɧɚɣɞɿɬɶ ɡɧɚɱɟɧɧɹ ɜɢɪɚɡɭ:
54. ɚ)
2
2
3 9
9
a a
a
+
−
, ɹɤɳɨ ɚ = 4; ɚ = 1
3
− ;
ɛ)
2 2
2
( )
m n m n
m n
− − +
−
, ɹɤɳɨ m = 9,51; n = –0,49.
55. ɚ)
2
4
5 10
x
x
−
+
, ɹɤɳɨ ɯ = –1; ɯ = 2
9
; ɛ)
2
2 2
( )
,
a b
a b ab
+
+
ɹɤɳɨ a = 2,5; b = 4.
Ⱦɨɜɟɞɿɬɶ ɬɨɬɨɠɧɿɫɬɶ:
56. ɚ)
2 2
2 2
12 36 6 ;
636
a ab b a b
a ba b
+ + +=
−−
ɛ)
4
2
2
8 2 4.
2
m m m m
m m
− = + +
−
57. ɚ)
2 2
2 2
2
;
4( )4 4
x xy y x y
x yx y
− + −
=
+−
ɛ)
3
2
8 1 2 1.
4 2 1
x x
x x
+ = +
− +
58. Ɂɜɟɞɿɬɶ ɞɪɿɛ:
ɚ) 7
x y+
ɞɨ ɡɧɚɦɟɧɧɢɤɚ ɯ2
+ ɯɭ; ɛ) 2
x y+
ɞɨ ɡɧɚɦɟɧɧɢɤɚ ɯ2
+ 2ɯɭ + ɭ2
;
ɜ) c
a b−
ɞɨ ɡɧɚɦɟɧɧɢɤɚ ɚ2
– b2
; ɝ) n
m n−
ɞɨ ɡɧɚɦɟɧɧɢɤɚ m3
– n3
.
59. Ɂɜɟɞɿɬɶ ɞɪɿɛ:
ɚ) 2a
x y+
ɞɨ ɡɧɚɦɟɧɧɢɤɚ ɯ2
– ɭ2
; ɛ) 1
a c+
ɞɨ ɡɧɚɦɟɧɧɢɤɚ ɚ3
+ ɫ3
.
Ɂɜɟɞɿɬɶ ɞɨ ɫɩɿɥɶɧɨɝɨ ɡɧɚɦɟɧɧɢɤɚ ɞɪɨɛɢ:
60. ɚ) 3
9
14a b
ɿ 2
5 ;
21ab
ɛ) 3 3
1
18x y
ɿ 4
1 ;
27xy
ɜ) 2
9
a
m n
ɿ 5 3
.
15
b
m n
2. Ɉɫɧɨɜɧɚ ɜɥɚɫɬɢɜɿɫɬɶ ɞɪɨɛɭ 21
61. ɚ) 3 3
8
9x y
ɿ 5
5 ;
24xy
ɛ) 3 2
16
a
m n
ɿ 4
;
24
b
m n
ɜ) 4 2
15
c
x y
ɿ 2
2 .
25x y
62. ɚ) 2
3
x xy+
ɿ 2
2 ;
xy y+
ɛ) 2 2
x
x y−
ɿ ;
y
x y+
ɜ) 2 2
2
m
m mn n+ +
ɿ ;n
m n+
ɝ) 2
4 1
c
c −
ɿ
2
;
1 2
c
c−
ɞ) 3
1
1 x−
ɿ 2 ;
1x −
ɟ) 3
8
y
y −
ɿ 2
2 .
2 4y y+ +
63. ɚ) 2 2
x
x x y+
ɿ 2
;
y
y y+
ɛ) 2
4 4
m
m m− +
ɿ 3 ;
2 m−
ɜ) 2 2
4
a
a b−
ɿ .
2
b
b a−
64. ɋɤɨɪɨɬɿɬɶ ɞɪɿɛ, ɞɟ n — ɧɚɬɭɪɚɥɶɧɟ ɱɢɫɥɨ:
ɚ)
2
13824 ;
15552
n
n
x
x
+
ɛ) 2
2045 ;
1755
n
n
x
x
ɜ)
2 2
2 2
3 2
;
2
x xy y
x xy y
+ +
− −
ɝ)
3 2
2
2 2
.
2
y y y
y y
+ − −
+ −
65. ɉɨɛɭɞɭɣɬɟ ɝɪɚɮɿɤ ɮɭɧɤɰɿʀ, ɡɚɞɚɧɨʀ ɮɨɪɦɭɥɨɸ:
ɚ)
2
1;
1
xy
x
−=
−
ɛ) .
x
y
x
=
66. Ⱦɨɜɟɞɿɬɶ ɬɨɬɨɠɧɿɫɬɶ:
ɚ)
2
2 2
3 5 15 10 25 ;
3 3 5 5
ab a b a a
ab a b b a ab a b
+ + + + +=
+ + + + + +
ɛ)
2 3 2 3 3 2 3 2
.
2 3 4 6 3 2 6 4
xy y x xy y x
xz z x xz z x
+ + + + + +
=
+ + + + + +
67. Ɉɛɱɢɫɥɿɬɶ:
ɚ) 4 2 ;
9 9
+ ɛ) 5 2;
7 7
− ɜ) 1 53 2 ;
6 6
+ ɝ) 1 54 1 .
8 8
−
22 § 1. Ɋɚɰɿɨɧɚɥɶɧɿ ɜɢɪɚɡɢ
68. Ɋɨɡɜ’ɹɠɿɬɶ ɪɿɜɧɹɧɧɹ:
ɚ) 5 1 3 5;
7 9 7 9
x x− = + ɛ) ( )( ) ( )
2
1 1 1 1 .
2 2 2 2
x x x− + − − =
69. ɋɶɨɝɨɞɧɿ ɜ ɦɚɝɚɡɢɧɿ 2 ɤɝ ɩɨɦɿɞɨɪɿɜ ɿ 3 ɤɝ ɨɝɿɪɤɿɜ ɤɨɲɬɭɸɬɶ 28 ɝɪɧ.
Ɍɢɠɞɟɧɶ ɬɨɦɭ, ɤɨɥɢ ɩɨɦɿɞɨɪɢ ɣ ɨɝɿɪɤɢ ɛɭɥɢ ɞɨɪɨɠɱɢɦɢ ɧɚ 25 %, 1 ɤɝ
ɩɨɦɿɞɨɪɿɜ ɿ 2 ɤɝ ɨɝɿɪɤɿɜ ɤɨɲɬɭɜɚɥɢ 20 ɝɪɧ. ɋɤɿɥɶɤɢ ɤɨɲɬɭɸɬɶ ɫɶɨɝɨɞɧɿ
1 ɤɝ ɩɨɦɿɞɨɪɿɜ ɿ ɫɤɿɥɶɤɢ 1 ɤɝ ɨɝɿɪɤɿɜ?
70. ȯ ɞɜɚ ɫɩɥɚɜɢ ɦɿɞɿ ɣ ɰɢɧɤɭ. ȼ ɨɞɢɧ ɫɩɥɚɜ ɦɿɞɶ ɿ ɰɢɧɤ ɜɯɨɞɹɬɶ ɭ
ɜɿɞɧɨɲɟɧɧɿ 5 : 2, ɚ ɜ ɿɧɲɢɣ — ɭ ɜɿɞɧɨɲɟɧɧɿ 3 : 4. ɋɤɿɥɶɤɢ ɩɨɬɪɿɛɧɨ ɜɡɹɬɢ
ɤɿɥɨɝɪɚɦɿɜ ɤɨɠɧɨɝɨ ɫɩɥɚɜɭ, ɳɨɛ ɨɞɟɪɠɚɬɢ 28 ɤɝ ɧɨɜɨɝɨ ɡ ɨɞɧɚɤɨɜɢɦ
ɭɦɿɫɬɨɦ ɦɿɞɿ ɣ ɰɢɧɤɭ?
71. ɍ ɫɶɨɦɢɯ ɿ ɜɨɫɶɦɢɯ ɤɥɚɫɚɯ ɲɤɨɥɢ ɪɚɡɨɦ ɧɚɜɱɚɸɬɶɫɹ 180 ɭɱɧɿɜ. Ʉɨɠɧɢɣ
ɜɨɫɶɦɢɤɥɚɫɧɢɤ ɞɪɭɠɢɬɶ ɿɡ 7 ɫɟɦɢɤɥɚɫɧɢɤɚɦɢ, ɚ ɤɨɠɧɢɣ ɫɟɦɢɤɥɚɫɧɢɤ —
ɿɡ 8 ɜɨɫɶɦɢɤɥɚɫɧɢɤɚɦɢ. ɋɤɿɥɶɤɢ ɜɫɶɨɝɨ ɜɨɫɶɦɢɤɥɚɫɧɢɤɿɜ ɽ ɭ ɲɤɨɥɿ?
1. Ⱦɨɞɚɜɚɧɧɹ ɞɪɨɛɿɜ ɡ ɨɞɧɚɤɨɜɢɦɢ ɡɧɚɦɟɧɧɢɤɚɦɢ. Ⱦɪɨɛɢ ɡ ɨɞɧɚɤɨɜɢɦɢ
ɡɧɚɦɟɧɧɢɤɚɦɢ ɞɨɞɚɸɬɶ ɬɚɤ ɫɚɦɨ, ɹɤ ɿ ɡɜɢɱɚɣɧɿ ɞɪɨɛɢ ɡ ɨɞɧɚɤɨɜɢɦɢ ɡɧɚɦɟɧɧɢ-
ɤɚɦɢ, ɬɨɛɬɨ ɞɨɞɚɸɬɶ ʀɯɧɿ ɱɢɫɟɥɶɧɢɤɢ, ɚ ɡɧɚɦɟɧɧɢɤ ɡɚɥɢɲɚɸɬɶ ɬɨɣ ɫɚɦɢɣ:
.a c a c
b b b
++ = (1)
Ɋɿɜɧɿɫɬɶ (1) ɽ ɬɨɬɨɠɧɿɫɬɸ, ɬɨɛɬɨ ɜɨɧɚ ɽ ɩɪɚɜɢɥɶɧɨɸ ɞɥɹ ɛɭɞɶ-ɹɤɢɯ ɡɧɚ-
ɱɟɧɶ ɚ, b ɿ ɫ, ɞɟ b ≠ 0.
Ɂ ɬɨɬɨɠɧɨɫɬɿ (1) ɜɢɩɥɢɜɚɽ ɬɚɤɟ ɩɪɚɜɢɥɨ ɞɨɞɚɜɚɧɧɹ ɞɪɨɛɿɜ ɡ ɨɞɧɚɤɨɜɢɦɢ
ɡɧɚɦɟɧɧɢɤɚɦɢ:
ɓɨɛ ɞɨɞɚɬɢ ɞɪɨɛɢ ɡ ɨɞɧɚɤɨɜɢɦɢ ɡɧɚɦɟɧɧɢɤɚɦɢ, ɩɨɬɪɿɛɧɨ ɞɨɞɚɬɢ
ʀɯɧɿ ɱɢɫɟɥɶɧɢɤɢ, ɚ ɡɧɚɦɟɧɧɢɤ ɡɚɥɢɲɢɬɢ ɬɨɣ ɫɚɦɢɣ.
3. Ⱦɨɞɚɜɚɧɧɹ ɿ ɜɿɞɧɿɦɚɧɧɹ ɞɪɨɛɿɜ ɡ ɨɞɧɚɤɨɜɢɦɢ ɡɧɚɦɟɧɧɢɤɚɦɢ 23
2. ȼɿɞɧɿɦɚɧɧɹ ɞɪɨɛɿɜ ɡ ɨɞɧɚɤɨɜɢɦɢ ɡɧɚɦɟɧɧɢɤɚɦɢ. ȼɿɞɧɿɦɚɧɧɹ ɞɪɨɛɿɜ
ɡ ɨɞɧɚɤɨɜɢɦɢ ɡɧɚɦɟɧɧɢɤɚɦɢ ɜɢɤɨɧɭɸɬɶ ɧɚ ɨɫɧɨɜɿ ɬɨɬɨɠɧɨɫɬɿ
.
a c a c
b b b
−
− = (2)
Ɂ ɬɨɬɨɠɧɨɫɬɿ (2) ɜɢɩɥɢɜɚɽ ɩɪɚɜɢɥɨ ɜɿɞɧɿɦɚɧɧɹ ɞɪɨɛɿɜ ɡ ɨɞɧɚɤɨɜɢɦɢ ɡɧɚ-
ɦɟɧɧɢɤɚɦɢ:
ɓɨɛ ɜɿɞɧɹɬɢ ɞɪɨɛɢ ɡ ɨɞɧɚɤɨɜɢɦɢ ɡɧɚɦɟɧɧɢɤɚɦɢ, ɩɨɬɪɿɛɧɨ ɜɿɞ ɱɢɫɟ-
ɥɶɧɢɤɚ ɡɦɟɧɲɭɜɚɧɨɝɨ ɜɿɞɧɹɬɢ ɱɢɫɟɥɶɧɢɤ ɜɿɞ’ɽɦɧɢɤɚ, ɚ ɡɧɚɦɟɧɧɢɤ
ɡɚɥɢɲɢɬɢ ɬɨɣ ɫɚɦɢɣ.
3. Ɂɚɩɢɫ ɞɪɨɛɭ ɭ ɜɢɝɥɹɞɿ ɫɭɦɢ ɚɛɨ ɪɿɡɧɢɰɿ ɞɪɨɛɿɜ. ɍ ɤɨɠɧɿɣ ɡ ɬɨɬɨɠɧɨ-
ɫɬɟɣ (1) ɿ (2) ɩɟɪɟɫɬɚɜɢɦɨ ɦɿɫɰɹɦɢ ɥɿɜɭ ɿ ɩɪɚɜɭ ɱɚɫɬɢɧɢ:
;a c a c
b b b
+ = + .a c a c
b b b
− = −
Ɉɞɟɪɠɚɧɿ ɬɨɬɨɠɧɨɫɬɿ ɦɨɠɧɚ ɜɢɤɨɪɢɫɬɨɜɭɜɚɬɢ, ɹɤɳɨ ɩɨɬɪɿɛɧɨ ɡɚɩɢɫɚɬɢ
ɞɪɿɛ ɭ ɜɢɝɥɹɞɿ ɫɭɦɢ ɚɛɨ ɪɿɡɧɢɰɿ ɞɪɨɛɿɜ.
ȼɩɪɚɜɚ 1. Ⱦɨɞɚɬɢ ɞɪɨɛɢ:
ɚ) 7 5 ;
a a
+ ɛ) 2 1 1 3 ;a b
ab ab ab
+ −+ + ɜ)
5 2
.
2 2
x y x y
x y x y
− −
+
− −
Ɣ ɚ) 7 5
a a
+ = 7 5
a
+ = 12.
a
ɛ) 2 1 1 3a b
ab ab ab
+ −+ + = 2 1 1 3a b
ab
+ + − + = 2 3.a b
ab
+ +
ɜ)
5 2
2 2
x y x y
x y x y
− −
+
− −
=
5 2
2
x y x y
x y
− + −
−
=
6 3
2
x y
x y
−
−
=
3(2 )
2
x y
x y
−
−
= 3. Ɣ
ȼɩɪɚɜɚ 2. ȼɿɞɧɹɬɢ ɞɪɨɛɢ:
ɚ) 2 2
4 2 3 ;
2 3 2 3
n n
n n n n
+−
− −
ɛ) 2 3 .a a
x y y x
−
− −
Ɣ ɚ) 2 2
4 2 3
2 3 2 3
n n
n n n n
+−
− −
= 2
4 (2 3)
2 3
n n
n n
− +
−
= 2 3
(2 3)
n
n n
−
−
= 1 .
n
24 § 1. Ɋɚɰɿɨɧɚɥɶɧɿ ɜɢɪɚɡɢ
ɛ) Ɂɦɿɧɢɜɲɢ ɡɧɚɤ ɡɧɚɦɟɧɧɢɤɚ ɞɪɭɝɨɝɨ ɞɪɨɛɭ, ɦɚɬɢɦɟɦɨ:
2 3a a
x y y x
−
− −
= 2 3
( )
a a
x y x y
−
− − −
= 2 3a a
x y x y
+
− −
= 2 3a a
x y
+
−
= 5 .a
x y−
Ɣ
ȼɩɪɚɜɚ 3. Ɂɚɩɢɫɚɬɢ ɞɪɿɛ ɭ ɜɢɝɥɹɞɿ ɫɭɦɢ ɚɛɨ ɪɿɡɧɢɰɿ ɰɿɥɨɝɨ ɱɢɫɥɚ ɿ ɞɪɨɛɭ:
ɚ) 3 5;a
a
+ ɛ) 2 2 1;a b
a b
+ +
+
ɜ) 3 1.
1
n
n
+
+
Ɣ ɚ) 3 5 3 5 53 .a a
a a a a
+ = + = +
ɛ)
2( )2 2 1 1 12 .
a ba b
a b a b a b a b
++ + = + = +
+ + + +
ɜ)
3( 1) 2 3( 1)3 1 3 3 2 2 23 .
1 1 1 1 1 1
n nn n
n n n n n n
+ − ++ + −= = = − = −
+ + + + + +
Ɣ
72. Ɂɧɚɣɞɿɬɶ ɫɭɦɭ ɞɪɨɛɿɜ:
ɚ) ;
4 4
a b+ ɛ) 9 3 ;
11 11
b b+ ɜ) 3 ;a a
x x
+ ɝ) .x a a
d d
+ +
73. Ɂɧɚɣɞɿɬɶ ɪɿɡɧɢɰɸ ɞɪɨɛɿɜ:
ɚ) ;
7 7
yx − ɛ) 8 3 ;
9 9
n n− ɜ) ;
a y y
x x
+
− ɝ)
2
.
x y x
c c
+
−
ȼɢɤɨɧɚɣɬɟ ɞɨɞɚɜɚɧɧɹ (ɜɿɞɧɿɦɚɧɧɹ) ɞɪɨɛɿɜ:
74. ɚ) 2 3 ;b b
a a
+ ɛ) 5 3 7 1;
3 1 3 1
n n
n n
+ −+
+ +
ɜ) 2 3 4 3 ;a a
xy xy xy
− + +
ɝ) 6 3 ;
5 5
a a
p p
− ɞ) 3 3 ;
9 9
a a
a a
+ −− ɟ) 9 2.
7 7
b
b b
+−
− −
75. ɚ) 3 5 2 7 ;
5 5
n n
n n
+ −+ ɛ) 2 1 1 2 ;
1 1
x x
x x
+ −+
− −
ɜ) 2 2 ;
2 2 2
n n n
m m m
+ −+ +
ɝ) 3 ;c c
b b
+ − ɞ) 7 3 3 2 ;
4 1 4 1
b b
b b
− −−
− −
ɟ) 4 1 4 5 .
2 2 2
a a
a a a
− ++ −
3. Ⱦɨɞɚɜɚɧɧɹ ɿ ɜɿɞɧɿɦɚɧɧɹ ɞɪɨɛɿɜ ɡ ɨɞɧɚɤɨɜɢɦɢ ɡɧɚɦɟɧɧɢɤɚɦɢ 25
ɋɩɪɨɫɬɿɬɶ ɜɢɪɚɡ:
76. ɚ) 7 2 5 2 ;
4 1 4 1
x x
x x
− ++
+ +
ɛ) 3 3 ;a b a b
a b a b
− −−
+ +
ɜ) 3 5 1;
3 1 3 1
a a
a a
− ++
− −
ɝ)
6 12 ;
2 2
p
p p
−
− −
ɞ) 5 5 ;a b
a b b a
+ ++
− −
ɟ) 4 3 4 .
2 2
x x
x y y x
+ −−
− −
77. ɚ) 6 1 4 ;
1 1
m m
m m
− +−
− −
ɛ)
3 7 5
;
x y x y
x y x y
+ +
+
+ +
ɜ) 8 8 ;m n
m n n m
+
− −
ɝ) 9 9.
3 3
c c
c c
− +−
− −
Ɂɧɚɣɞɿɬɶ ɡɧɚɱɟɧɧɹ ɜɢɪɚɡɭ:
78. ɚ) 3 3 1
4 4
b b
b b
− −+ , ɹɤɳɨ b = –3; ɛ)
2 2
7 1 1
3 3
a a
a a
− −− , ɹɤɳɨ ɚ = 0,28.
79. ɚ) 2 2
3 3a a
a a
− ++ , ɹɤɳɨ ɚ = 5; ɛ) 4 5 2 1
2 2
c c
c c
+ −− , ɹɤɳɨ ɫ = 0,4.
ɋɩɪɨɫɬɿɬɶ ɜɢɪɚɡ:
80. ɚ)
2
2 1 ;
1 1
c c
c c
− −
− −
ɛ)
5 3
;
1 1
a a
a a
−
− −
ɜ)
2
2 2
9 6 ;
9 9
b b
b b
+ +
− −
ɝ)
4 2 3
2 4 2 4
12 4 ;
3 3
a a b
a b b a b b
−
− −
ɞ)
2
4 5 1 2 ;
2 1 2 1 2 1
x x x
x x x
− −+ −
+ + +
ɟ) 2 2 2 2 2 2
2 3
.
xy x xy y y
x y x y x y
− −
− −
− − −
81. ɚ)
2 2
5 14 10
;
2 2
y y
y y
− −
−
− −
ɛ)
2 2
3 3
2
;
( ) ( )
x y xy
x y x y
+
−
− −
ɜ)
2
3 3
1 ;
1 1
a a
a a
+ +
− −
ɝ)
2 4
2 2
18 2 ;
3 3
a b
a b a b
−
− −
ɞ)
2 2
21 3 ;
3 3 3
n n
n n n
−− +
− − −
ɟ) 2 2 2
8 2 3 .
(5 ) (5 ) (5 )
a b a b a b
a b a b a b
− − +− −
+ + +
26 § 1. Ɋɚɰɿɨɧɚɥɶɧɿ ɜɢɪɚɡɢ
Ⱦɨɜɟɞɿɬɶ ɬɨɬɨɠɧɿɫɬɶ:
82. ɚ)
2 2
( ) ( )
4;
a b a b
ab ab
+ −
− = ɛ)
3 2
2 2 2
4 4 .
( 2) ( 2) ( 2)
x x x x
x x x
− + =
− − −
83. ɚ)
2 2
2 2 2 2
( ) ( )
2;
m n m n
m n m n
+ −
+ =
+ +
ɛ)
2
2 2 2 2
4 2 2 .
24 4
a ab a
a ba b a b
− =
+− −
ɉɨɞɚɣɬɟ ɞɪɿɛ ɭ ɜɢɝɥɹɞɿ ɫɭɦɢ ɚɛɨ ɪɿɡɧɢɰɿ ɰɿɥɨɝɨ ɱɢɫɥɚ ɿ ɞɪɨɛɭ:
84. ɚ) 3 8 ;
2
x
x
+ ɛ) 5 5 2 ;m n
m n
− +
−
ɜ)
4 5
;
2
y
y
+
+
ɝ)
2
.
x y
x y
−
+
85. ɚ) 14 5;
7
b + ɛ) 3 3 ;b c a
b c
+ −
+
ɜ) 6 1;
2 1
c
c
+
+
ɝ)
4 3
.
x y
x y
−
−
86. ɋɩɪɨɫɬɿɬɶ ɜɢɪɚɡ:
ɚ)
2 4
2 2
1 ;
1 1
x x
x x x x
+ +
+ + + +
ɛ)
4 4 2 2
3 3 3 3
.a x a x
a x a x
+ +
+ +
ȼɤɚɡɿɜɤɚ. ɚ) Ɇɧɨɝɨɱɥɟɧ ɯ4
+ ɯ2
+ 1 ɦɨɠɧɚ ɪɨɡɤɥɚɫɬɢ ɧɚ ɦɧɨɠɧɢɤɢ, ɡɚɩɢɫɚɜɲɢ
ɣɨɝɨ ɭ ɜɢɝɥɹɞɿ (ɯ4
+ 2ɯ2
+ 1) – ɯ2
.
87. Ɉɛɱɢɫɥɿɬɶ:
ɚ) 5 3;
6 8
+ ɛ) 3 4 ;
14 21
− ɜ) 2 5 7 ;
9 6 12
− + ɝ) 1 2 57 4 .
3 11 6
⋅ −
88. ɉɨɞɚɣɬɟ ɨɞɧɨɱɥɟɧ 24ɚ7
b8
ɭ ɜɢɝɥɹɞɿ ɞɨɛɭɬɤɭ ɞɜɨɯ ɨɞɧɨɱɥɟɧɿɜ, ɨɞɧɢɦ ɡ
ɹɤɢɯ ɽ:
ɚ) 6ɚ4
b7
; ɛ) 4ɚ2
b5
; ɜ) 24ɚ7
b; ɝ) 8ɚb7
.
89. Ɂ Ʉɢɽɜɚ ɿ ɑɟɪɤɚɫ, ɜɿɞɫɬɚɧɶ ɦɿɠ ɹɤɢɦɢ ɞɨɪɿɜɧɸɽ 190 ɤɦ, ɨɞɧɨɱɚɫɧɨ ɧɚɡɭ-
ɫɬɪɿɱ ɨɞɢɧ ɨɞɧɨɦɭ ɜɢʀɯɚɥɢ ɚɜɬɨɛɭɫ ɬɚ ɥɟɝɤɨɜɢɣ ɚɜɬɨɦɨɛɿɥɶ ɿ ɡɭɫɬɪɿɥɢɫɹ
ɱɟɪɟɡ 1 ɝɨɞ 15 ɯɜ. Ɂɧɚɣɞɿɬɶ ɲɜɢɞɤɿɫɬɶ ɚɜɬɨɦɨɛɿɥɹ, ɹɤɳɨ ɜɨɧɚ ɜ 11
9
ɪɚɡɭ
ɛɿɥɶɲɚ ɜɿɞ ɲɜɢɞɤɨɫɬɿ ɚɜɬɨɛɭɫɚ.
4. Ⱦɨɞɚɜɚɧɧɹ ɿ ɜɿɞɧɿɦɚɧɧɹ ɞɪɨɛɿɜ ɡ ɪɿɡɧɢɦɢ ɡɧɚɦɟɧɧɢɤɚɦɢ 27
90. ȼɿɞɩɨɜɿɞɧɨ ɞɨ ɧɨɪɦ ɚɝɪɨɬɟɯɧɿɤɢ ɡɟɪɧɨ ɩɨɬɪɿɛɧɨ ɡɚɫɢɩɚɬɢ ɧɚ ɬɪɢɜɚɥɟ
ɡɛɟɪɿɝɚɧɧɹ ɡɚ ɜɨɥɨɝɨɫɬɿ 14% (ɤɨɧɞɢɰɿɣɧɢɣ ɫɬɚɧ). Ɂɿɛɪɚɧɟ ɡɟɪɧɨ, ɦɚɫɚ
ɹɤɨɝɨ ɞɨɪɿɜɧɸɽ 43 ɬ, ɦɚɽ ɜɨɥɨɝɿɫɬɶ 24%. ɇɚ ɫɤɿɥɶɤɢ ɬɨɧɧ ɡɦɟɧɲɢɬɶɫɹ
ɦɚɫɚ ɰɶɨɝɨ ɡɟɪɧɚ ɩɪɢ ɞɨɜɟɞɟɧɧɿ ɣɨɝɨ ɞɨ ɤɨɧɞɢɰɿɣɧɨɝɨ ɫɬɚɧɭ?
91. ɇɚ ɤɥɿɬɱɚɫɬɨɦɭ ɩɚɩɟɪɿ ɫɢɞɹɬɶ 100 ɩɚɜɭɤɿɜ, ɩɨ ɨɞɧɨɦɭ ɭ ɤɥɿɬɢɧɰɿ. ɉɚɜɭɤɢ
ɜɨɪɨɝɭɸɬɶ, ɹɤɳɨ ɤɥɿɬɢɧɤɢ, ɭ ɹɤɢɯ ɜɨɧɢ ɫɢɞɹɬɶ, ɦɚɸɬɶ ɯɨɱɚ ɛ ɨɞɧɭ
ɫɩɿɥɶɧɭ ɜɟɪɲɢɧɭ. Ⱦɨɜɟɞɿɬɶ, ɳɨ ɫɟɪɟɞ ɧɢɯ ɡɚɜɠɞɢ ɦɨɠɧɚ ɜɢɛɪɚɬɢ ɧɟ
ɦɟɧɲɟ ɧɿɠ 25 ɩɚɜɭɤɿɜ, ɛɭɞɶ-ɹɤɿ ɞɜɚ ɡ ɹɤɢɯ ɧɟ ɜɨɪɨɝɭɸɬɶ.
ɓɨɛ ɞɨɞɚɬɢ ɚɛɨ ɜɿɞɧɹɬɢ ɡɜɢɱɚɣɧɿ ɞɪɨɛɢ ɡ ɪɿɡɧɢɦɢ ɡɧɚɦɟɧɧɢɤɚɦɢ, ɩɨɬɪɿ-
ɛɧɨ ɡɜɟɫɬɢ ɞɪɨɛɢ ɞɨ ɫɩɿɥɶɧɨɝɨ ɡɧɚɦɟɧɧɢɤɚ ɿ ɞɨɞɚɬɢ ɚɛɨ ɜɿɞɧɹɬɢ ɨɞɟɪɠɚɧɿ
ɞɪɨɛɢ ɡ ɨɞɧɚɤɨɜɢɦɢ ɡɧɚɦɟɧɧɢɤɚɦɢ.
ɍ ɬɚɤɢɣ ɠɟ ɫɩɨɫɿɛ ɞɨɞɚɸɬɶ ɿ ɜɿɞɧɿɦɚɸɬɶ ɛɭɞɶ-ɹɤɿ ɞɪɨɛɢ ɡ ɪɿɡɧɢɦɢ ɡɧɚ-
ɦɟɧɧɢɤɚɦɢ.
ɇɟɯɚɣ ɩɨɬɪɿɛɧɨ ɞɨɞɚɬɢ ɞɪɨɛɢ a
b
ɿ ,c
d
ɹɤɿ ɦɚɸɬɶ ɪɿɡɧɿ ɡɧɚɦɟɧɧɢɤɢ. Ɂɜɟ-
ɞɟɦɨ ɰɿ ɞɪɨɛɢ ɞɨ ɫɩɿɥɶɧɨɝɨ ɡɧɚɦɟɧɧɢɤɚ bd. Ⱦɥɹ ɰɶɨɝɨ ɱɢɫɟɥɶɧɢɤ ɿ ɡɧɚɦɟɧɧɢɤ
ɩɟɪɲɨɝɨ ɞɪɨɛɭ ɩɨɦɧɨɠɢɦɨ ɧɚ d, ɚ ɞɪɭɝɨɝɨ ɞɪɨɛɭ — ɧɚ b. Ɉɞɟɪɠɢɦɨ:
;=a ad
b bd
.c bc
d bd
=
Ɂɧɚɸɱɢ, ɹɤ ɞɨɞɚɬɢ ɞɪɨɛɢ ɡ ɨɞɧɚɤɨɜɢɦɢ ɡɧɚɦɟɧɧɢɤɚɦɢ, ɦɚɬɢɦɟɦɨ:
.++ = + =a c ad bc ad bc
b d bd bd bd
Ɉɬɠɟ,
.a c ad bc
b d bd
++ =
ȼɿɞɧɿɦɚɸɬɶ ɞɪɨɛɢ ɡ ɪɿɡɧɢɦɢ ɡɧɚɦɟɧɧɢɤɚɦɢ ɚɧɚɥɨɝɿɱɧɨ, ɚ ɫɚɦɟ:
.a c ad bc
b d bd
−− =
28 § 1. Ɋɚɰɿɨɧɚɥɶɧɿ ɜɢɪɚɡɢ
ɓɨɛ ɞɨɞɚɬɢ ɚɛɨ ɜɿɞɧɹɬɢ ɞɪɨɛɢ ɡ ɪɿɡɧɢɦɢ ɡɧɚɦɟɧɧɢɤɚɦɢ, ɩɨɬɪɿɛɧɨ:
1) ɡɜɟɫɬɢ ɞɪɨɛɢ ɞɨ ɫɩɿɥɶɧɨɝɨ ɡɧɚɦɟɧɧɢɤɚ;
2) ɞɨɞɚɬɢ ɚɛɨ ɜɿɞɧɹɬɢ ɨɞɟɪɠɚɧɿ ɞɪɨɛɢ ɡ ɨɞɧɚɤɨɜɢɦɢ ɡɧɚɦɟɧɧɢɤɚɦɢ.
ȼɩɪɚɜɚ 1. ȼɢɤɨɧɚɬɢ ɞɨɞɚɜɚɧɧɹ (ɜɿɞɧɿɦɚɧɧɹ) ɞɪɨɛɿɜ:
ɚ) 5 4 ;b c
ac b
+ ɛ) 2 2 3
4 7 ;
9 12x y xy
+ ɜ) 2 2
2 2 .
xy y x xy
−
− −
• ɚ) ɋɩɿɥɶɧɢɦ ɡɧɚɦɟɧɧɢɤɨɦ ɞɪɨɛɿɜ ɽ ɞɨɛɭɬɨɤ ʀɯɧɿɯ ɡɧɚɦɟɧɧɢɤɿɜ. Ɍɨɦɭ
ɞɨɞɚɬɤɨɜɢɣ ɦɧɨɠɧɢɤ ɞɥɹ ɩɟɪɲɨɝɨ ɞɪɨɛɭ — b, ɚ ɞɥɹ ɞɪɭɝɨɝɨ — ɚɫ.
5b
ac
4c
b
+
b ac
=
2 2
5 4 .b ac
abc
+
ɛ) ɋɩɿɥɶɧɢɦ ɡɧɚɦɟɧɧɢɤɨɦ ɞɪɨɛɿɜ ɽ 36ɯ2
ɭ3
. Ⱦɨɞɚɬɤɨɜɢɦ ɦɧɨɠɧɢɤɨɦ ɞɥɹ
ɩɟɪɲɨɝɨ ɞɪɨɛɭ ɽ 4ɭ, ɞɥɹ ɞɪɭɝɨɝɨ — 3ɯ.
4 7+
4y 3x
9x y 12xy32 2
= 2 3
16 21
.
36
y x
x y
+
ɜ) Ɋɨɡɤɥɚɜɲɢ ɧɚ ɦɧɨɠɧɢɤɢ ɡɧɚɦɟɧɧɢɤɢ ɞɪɨɛɿɜ, ɦɚɬɢɦɟɦɨ:
2 2
2 2
xy y x xy
−
− −
=
2
y x y( )
2
x( )x y
x y
=
2 2
( )
x y
xy x y
−
−
=
2( )
( )
x y
xy x y
−
−
= 2 .
xy
•
ȼɩɪɚɜɚ 2. ɉɨɞɚɬɢ ɭ ɜɢɝɥɹɞɿ ɞɪɨɛɭ ɜɢɪɚɡ
2
23 .
1
mm
m
+− +
−
• ȼɢɪɚɡ ɬ – 3 ɡɚɩɢɲɟɦɨ ɭ ɜɢɝɥɹɞɿ ɞɪɨɛɭ 3.
1
m − Ɍɨɞɿ:
2
23
1
mm
m
+− +
−
= m 3
1
2 +
1
m
m
1 m
+
2
=
2
( 3)(1 ) 2
1
m m m
m
− − + +
−
=
=
2 2
3 3 2
1
m m m m
m
− − + + +
−
= 4 1.
1
m
m
−
−
•
4. Ⱦɨɞɚɜɚɧɧɹ ɿ ɜɿɞɧɿɦɚɧɧɹ ɞɪɨɛɿɜ ɡ ɪɿɡɧɢɦɢ ɡɧɚɦɟɧɧɢɤɚɦɢ 29
ȼɩɪɚɜɚ 3. Ⱦɨɜɟɫɬɢ ɬɨɬɨɠɧɿɫɬɶ
2
1 .
( )
a b b
a b a a a b
+ − =
+ +
• ɉɟɪɟɬɜɨɪɢɦɨ ɥɿɜɭ ɱɚɫɬɢɧɭ ɪɿɜɧɨɫɬɿ:
1a b
a b a
+ −
+
= a
ba+
b
a
1
1
a a b( + )a a b+
+ =
2
( ) ( )
( )
a b a b a a b
a a b
+ + − +
+
=
=
2 2 2
( )
a ab b a ab
a a b
+ + − −
+
=
2
.
( )+
b
a a b
ɒɥɹɯɨɦ ɬɨɬɨɠɧɢɯ ɩɟɪɟɬɜɨɪɟɧɶ ɥɿɜɭ ɱɚɫɬɢɧɭ ɪɿɜɧɨɫɬɿ ɡɜɟɥɢ ɞɨ ɩɪɚɜɨʀ
ɱɚɫɬɢɧɢ. Ɍɨɦɭ ɰɹ ɪɿɜɧɿɫɬɶ ɽ ɬɨɬɨɠɧɿɫɬɸ. •
ɉɪɢɦɿɬɤɚ. ɇɚɝɚɞɚɽɦɨ, ɳɨ ɞɥɹ ɞɨɜɟɞɟɧɧɹ ɬɨɬɨɠɧɨɫɬɟɣ ɨɞɧɭ ɱɚɫɬɢɧɭ
ɬɨɬɨɠɧɨɫɬɿ ɡɜɨɞɹɬɶ ɞɨ ɿɧɲɨʀ ɱɚɫɬɢɧɢ ɚɛɨ ɨɛɢɞɜɿ ɱɚɫɬɢɧɢ ɡɜɨɞɹɬɶ ɞɨ ɬɨɝɨ
ɫɚɦɨɝɨ ɜɢɪɚɡɭ, ɚɛɨ ɭɬɜɨɪɸɸɬɶ ɪɿɡɧɢɰɸ ɥɿɜɨʀ ɬɚ ɩɪɚɜɨʀ ɱɚɫɬɢɧ ɿ ɞɨɜɨɞɹɬɶ,
ɳɨ ɜɨɧɚ ɞɨɪɿɜɧɸɽ ɧɭɥɸ.
ȼɢɤɨɧɚɣɬɟ ɞɨɞɚɜɚɧɧɹ (ɜɿɞɧɿɦɚɧɧɹ) ɞɪɨɛɿɜ:
92. ɚ) ;a m
c n
− ɛ) ;
3 12
a b+ ɜ) 5 3 ;
4 5
a b
x x
−
ɝ) 7 ;
9 6
c c
y y
− ɞ) 5 7 ;
12 18
b b
x x
+ ɟ) 4 6 .
15 25
b a
a b
−
93. ɚ) ;
6 18
c ad+ ɛ) 3 2 ;
5 3
k k
a a
+ ɜ) 5 .
24 36
n n
x x
−
94. ɚ) 2 3 ;
4
a
a
+ − ɛ)
10 37 3 ;
yx
x y
−+ + ɜ) 2 2 ;a b a b
b a
+ −−
ɝ) 2 2 ;
1 1z z
+
− +
ɞ) ;a a
a c c
−
+
ɟ)
1 2
.
2 1
y y
y y
− +
−
− +
95. ɚ) 2 1 1;
3
m
m
− + ɛ) 4 2 3 2 ;a b
a b
− ++ ɜ)
2 2
;
x y y x
x y
− −
−
ɝ) 2 1 ;
2k k
+
+
ɞ) 2 3 ;
2 1 3 2
a a
a a
−
+ +
ɟ) 1 .
1
x x
x x
+ −
−
30 § 1. Ɋɚɰɿɨɧɚɥɶɧɿ ɜɢɪɚɡɢ
ɉɨɞɚɣɬɟ ɭ ɜɢɝɥɹɞɿ ɞɪɨɛɭ:
96. ɚ) 22 ;
2 1n
+
−
ɛ)
3 2
3 ;
x y
x
−
− ɜ)
2
.
2
y
y
y
−
−
97. ɚ)
5 2
2;
1
y
y
−
+
+
ɛ) 21 ;
2 3
x
x
−
+
ɜ)
2
2 3 3 .
1
c c
c
+ −
−
ɋɩɪɨɫɬɿɬɶ ɜɢɪɚɡ:
98. ɚ) 4 1 5 6 ;
8 12
x x
x x
− −+ ɛ) 6 18;
6 18
a a
a a
− +− ɜ) 5 4 ;
2( ) 3( )
a a
a b a b
−
+ +
ɝ)
2
2 2 ;a b a b
a ab
+ −+ ɞ) 2
1 5;c
c c
−− ɟ) 2 2
;m n m n
m n mn
+ −+
ɽ) 2
1 2 3;
3 6
y x
xy x
− ++ ɠ)
77 ;
( )
y
x x x y
−
+
ɡ) 3 3 .
( )
a
a b b a b
+
+ +
99. ɚ) 3 1 2 1;
6 4
b b
b b
− −− ɛ) 4 4 ;
5( ) 3( )
b c b c
b c b c
+ −+
− −
ɜ)
3 1;
3 3
x y x
xy x
+ −+
ɝ) 2
2 1 ;a
aa
+ − ɞ) 2
2 ;
2
a b a
abab
+ −+ ɟ) 2 1 2 .
( 1) 1
m
m m m
+ −
− −
100. ɚ) 3 3 ;
2 2 4 4
a b a b
a b a b
− ++
− −
ɛ) 1 3 ;
3 12 2 8
x x
x x
− −−
− −
ɜ) 2
4 2 ;
2 cc c
+
−
ɝ) 2
3 2 3 ;
3 3
b
b b b
−−
+ +
ɞ)
2
2
;
2 4
k k
k k
−
− −
ɟ) 2 2
4 4 .
2
m n
m nm mn n
− +
++ +
101. ɚ) 5 ;
2 2
m n m n
m n m n
+ −+
− −
ɛ) 2 3 ;
3 3 5 5
−
+ +
a a
a a
ɜ) 2
1 2 ;
1
n
n n n
−+
− −
ɝ) 2
15 3 ;
5 aa a
−
+
ɞ) 2
16 4 ;
416 xx
+
+−
ɟ) 2 2
1 .
2
x
x yx xy y
−
−− +
Ⱦɨɜɟɞɿɬɶ ɬɨɬɨɠɧɿɫɬɶ:
102. ɚ) 2 2
1 1 2 ;m
m n m n m n
+ =
− + −
ɛ) 2 2
1 1 .
11 1
b
bb b
− =
+− −
103. ɚ) 2
3 3 9 ;
3 3a a a a
− =
− −
ɛ) 2 2
1 .
( ) ( )
a b
a ba b a b
− =
−− −
4. Ⱦɨɞɚɜɚɧɧɹ ɿ ɜɿɞɧɿɦɚɧɧɹ ɞɪɨɛɿɜ ɡ ɪɿɡɧɢɦɢ ɡɧɚɦɟɧɧɢɤɚɦɢ 31
ɉɟɪɟɬɜɨɪɿɬɶ ɭ ɞɪɿɛ ɜɢɪɚɡ:
104. ɚ) 2 2 2
1 1 ;a bc
a bc ab c
+ −+ ɛ) 4 2 3 3
3 14 1 ;
12 9
yx
x y x y
−+ −
ɜ) 1 1 ;c d
cm cn dm dn
+ +−
+ +
ɝ) ;
12 12 18 18
x x
x y x y
+
− +
ɞ) ;
4 4 4 4
a b a b
a b b a
− ++
+ −
ɟ) 2 2 2
1 1 .
( )b a a b
−
− −
105. ɚ) 3 2 2 3
2 1 3 1 ;
16 24
a b
a b a b
+ +− ɛ) 7 5 ;
ax ay by bx
−
− −
ɜ) 2 2 2
;b b
a b a ab
−
− +
ɝ) 2
1 .
2 2( )
x y
x yx y
−
+
++
ɋɩɪɨɫɬɿɬɶ ɜɢɪɚɡ:
106. ɚ)
2
2
5 3 1 ;
3 1 6 2
c c
c c c
+ −−
+ +
ɛ)
2
2
;b b
y bby y
+
−−
ɜ) 2
;
6 9 4 9
n n
n n
+
+ −
ɝ) 2
5 2 1 ;
10 225
x
xx
− −
−−
ɞ) 2 2
2 8 ;
2 4
a
a a a
− +
+ −
ɟ) 2 2
2 8 ;
8 2 4
a b a b
a ab b ab
+ ++
− −
ɽ) 2
5 15 ;
2 12 12 36m m m
−
+ + +
ɠ) 2 2 2
4 ;
9 24 16 3 4
x x
x xa a xa a
+
− + −
ɡ) 3 2
9 3 ;
27 3 9
a
a a a
−+
+ − +
ɢ)
2
3
2 2 4 .
2 8
x x
x x
+−
− −
107. ɚ) 2
1 10 ;
10
− +−
−
a a
a a a
ɛ) 2 2
1 1 ;b a
ab b ab a
+ ++
− −
ɜ) 2
9 9 ;
2 6 9
x
x x
−
+ −
ɝ) 2
5 20 ;
4 32 64b b
+
− −
ɞ) 2
5 45 ;
9 18 81
+
− − +
a a
a a a
ɟ) 2 2 2
;
4 12 9 4 6
y y
x xy y x xy
−
+ + +
ɽ) 3 2
1 ;
1 1
x
x x x
−
− + +
ɠ) 3
1 2 .
2 8
a
a a
+
+ +
32 § 1. Ɋɚɰɿɨɧɚɥɶɧɿ ɜɢɪɚɡɢ
ɉɨɞɚɣɬɟ ɭ ɜɢɝɥɹɞɿ ɞɪɨɛɭ:
108. ɚ) 5 6 1 1;
8 4 2
a a a
a a a
+ − ++ − ɛ) 1 1 ;
x y
xy x y
−
− −
ɜ) 2 2
;a b a b a b
aba b
+ + −− − ɝ) 2
2 2 5 ;m n
mn n m
− − +
ɞ) 2
3 3 ;
2 2
a b b
ab b a
+ − + ɟ) 1 2 1;
2 1
x x
x x
− −− +
+ +
ɽ)
2 2
;
x y
x y
x y
+
+ −
+
ɠ)
4
2
2
1 1.
1
aa
a
+− +
−
109. ɚ) 3 1 1;
3 3
a
a a
+ + − ɛ) 2 2 2 2
1 1 ;a b
a b ab a b
+− −
ɜ)
2
2
1 1 ;nk
k kn
−− − ɝ) 2
1 3 1 ;
3 3
x x
xy yx
− −− +
ɞ)
2
( )
;
m n
m n
m n
−
− +
+
ɟ) 2 11 .
1 1
x
x x
− + −
− +
ɋɩɪɨɫɬɿɬɶ ɜɢɪɚɡ:
110. ɚ) 2
2 4 1 ;
5 25 5 2525a aa
− −
− +−
ɛ) 2 2
2 3 3 1 .
2 4 4 4
x x
x x x x
+ +− +
+ − − +
111. ɚ) 2 2
1 1 3 ;
2 2 4
a
a b b a a b
− −
+ − −
ɛ) 2
3 1 2 .
3 36 9
m
m mm m
− + −
− ++ +
Ɂɧɚɣɞɿɬɶ ɡɧɚɱɟɧɧɹ ɜɢɪɚɡɭ:
112. ɚ) 2
2 2 1
2 2
x x
x xx x
− −− +
+ +
, ɹɤɳɨ ɯ = 3 ;
11
−
ɛ)
2
2 2
2 3 3a a a ab
a b a b a b
+− −
− + −
, ɹɤɳɨ ɚ = –1,5; b = 11,5.
113. ɚ)
2
2
3 2 3
3 3
a a
a a a a
− −
− −
, ɹɤɳɨ a = 2 ;
17
ɛ)
2
2 2
2 6 4
3 2 9 4
x y xy y
x y x y
+ +
−
− −
, ɹɤɳɨ ɯ = 4,2; ɭ = 1,3.
4. Ⱦɨɞɚɜɚɧɧɹ ɿ ɜɿɞɧɿɦɚɧɧɹ ɞɪɨɛɿɜ ɡ ɪɿɡɧɢɦɢ ɡɧɚɦɟɧɧɢɤɚɦɢ 33
Ⱦɨɜɟɞɿɬɶ ɬɨɬɨɠɧɿɫɬɶ:
114. ɚ) 2 2 2
2(2 1)2 ;
2 1 1 ( 1)( 1)
xx x
x x x x x
++= −
+ + − − +
ɛ) 2 2 2
3 1 5 3 3 1 .
1 2 2 2 ( 1)
b b
bb b b b b
− ++ − =
− + −
115. ɚ)
2
2 2 2 2
;
2 ( )
n m n m
n mn m n mn n n m
+= +
− + − −
ɛ) 2 2
1 3 4 15 21 .
3 3 9 9
a
a a a a
−− − =
+ − − −
116. Ɂɧɚɣɞɿɬɶ ɬɚɤɿ ɱɢɫɥɚ ɚ ɿ b, ɳɨɛ ɪɿɜɧɿɫɬɶ ɜɢɤɨɧɭɜɚɥɚɫɹ ɞɥɹ ɜɫɿɯ ɞɨɩɭɫɬɢ-
ɦɢɯ ɡɧɚɱɟɧɶ ɯ:
ɚ) 1
( 1)( 2)x x+ +
= ;
1 2
a b
x x
+
+ +
ɛ) 2 1
( 3)( 4)
x
x x
−
− +
= .
3 4
a b
x x
+
− +
117. ɋɩɪɨɫɬɿɬɶ ɜɢɪɚɡ:
ɚ) 1 1 1 ;
( )( ) ( )( ) ( )( )a b a c b c b a c a c b
+ +
− − − − − −
ɛ) 2 4 8
1 1 2 4 8 ;
1 1 1 1 1x x x x x
+ + + +
− + + + +
ɜ)
3 7
2 4 8
1 1 2 4 8 ;
1 1 1 1 1
x x x
x x x x x
− − − −
− + + + +
ɝ) 1 1 1 1 ;
( 1) ( 1)( 2) ( 2)( 3) ( 3)( 4)a a a a a a a a
+ + +
+ + + + + + +
ɞ)
2 2 3
2 3 2 3 2 4
1 1 2 .
1 1 1 1
b b b b b b
b b b b b b b b
+ − − −+ + +
− − + − + + + −
118. Ⱦɨɜɟɞɿɬɶ ɬɨɬɨɠɧɿɫɬɶ:
ɚ)
( )( ) ( )( ) ( )( )
a b b c c a
b c c a c a a b a b b c
+ + ++ +
− − − − − −
= 0;
ɛ)
( )( ) ( )( ) ( )( )
b c c a a b
a b a c b c b a c a c b
− − −+ +
− − − − − −
= 2 2 2 .
a b b c c a
+ +
− − −
2* ȼ. Ʉɪɚɜɱɭɤ. Ⱥɥɝɟɛɪɚ. 8 ɤɥ.
34 § 1. Ɋɚɰɿɨɧɚɥɶɧɿ ɜɢɪɚɡɢ
119. Ɉɛɱɢɫɥɿɬɶ:
ɚ) 9 24 5 ;
16 25 27
⋅ ⋅ ɛ) 4 2 16 8: : ;
9 27 17 51
−
ɜ) ( )5 10,4 8 5 0,8 5: 2 ;
8 2
+ ⋅ − ⋅ − ɝ) ( )7 2 21 8 8,9 2,6 : 34 .
8 3 5
§ ·⋅ − − ⋅¨ ¸
© ¹
120. Ɋɨɡɜ’ɹɠɿɬɶ ɫɢɫɬɟɦɭ ɪɿɜɧɹɧɶ:
ɚ)
2 3 12;
2 4;
x y
x y
+ =­
®
− =¯
ɛ)
3 5 22;
7 2 24.
x y
x y
− =­
®
+ =¯
121. ɉɪɨɬɹɝɨɦ ɪɨɤɭ ɜɤɥɚɞɧɢɤ ɡɧɹɜ ɡɿ ɫɜɨɝɨ ɪɚɯɭɧɤɭ 3
5
ɭɫɿɯ ɝɪɨɲɟɣ ɿ ɧɟ ɪɨɛɢɜ
ɧɨɜɢɯ ɜɧɟɫɤɿɜ. ɍ ɤɿɧɰɿ ɪɨɤɭ ɛɚɧɤ ɧɚɪɚɯɭɜɚɜ 12% ɪɿɱɧɢɯ, ɿ ɧɚ ɪɚɯɭɧɤɭ
ɜɤɥɚɞɧɢɤɚ ɫɬɚɥɨ 896 ɝɪɧ. ɋɤɿɥɶɤɢ ɝɪɨɲɟɣ ɛɭɥɨ ɧɚ ɪɚɯɭɧɤɭ ɜɤɥɚɞɧɢɤɚ ɧɚ
ɩɨɱɚɬɤɭ ɪɨɤɭ?
122. Ʉɨɦɩ’ɸɬɟɪɧɢɣ ɤɥɭɛ ɩɥɚɧɭɽ ɩɪɚɰɸɜɚɬɢ 9 ɝɨɞ ɧɚ ɞɟɧɶ ɣ ɨɛɫɥɭɝɨɜɭɜɚɬɢ
ɤɨɠɧɨɝɨ ɱɥɟɧɚ ɤɥɭɛɭ ɳɨɞɟɧɧɨ ɡɚ ɨɤɪɟɦɢɦ ɤɨɦɩ’ɸɬɟɪɨɦ ɩɪɨɬɹɝɨɦ 1,5 ɝɨɞ.
əɤɭ ɧɚɣɦɟɧɲɭ ɤɿɥɶɤɿɫɬɶ ɤɨɦɩ’ɸɬɟɪɿɜ ɞɥɹ ɰɶɨɝɨ ɩɨɬɪɿɛɧɨ, ɹɤɳɨ ɤɿɥɶɤɿɫɬɶ
ɱɥɟɧɿɜ ɤɥɭɛɭ ɞɨɪɿɜɧɸɽ 50?
123. ɍ ɤɥɚɫɿ ɧɚɜɱɚɸɬɶɫɹ 29 ɭɱɧɿɜ. ȼɿɞɨɦɨ, ɳɨ ɫɟɪɟɞ ɛɭɞɶ-ɹɤɢɯ ɬɪɶɨɯ ɡ ɧɢɯ ɽ
ɩɪɢɧɚɣɦɧɿ ɞɜɨɽ ɞɪɭɡɿɜ. Ⱦɨɜɟɞɿɬɶ, ɳɨ ɭ ɤɥɚɫɿ ɡɧɚɣɞɟɬɶɫɹ ɭɱɟɧɶ, ɹɤɢɣ ɦɚɽ
ɧɟ ɦɟɧɲɟ ɧɿɠ 14 ɞɪɭɡɿɜ.
Ɂɚɜɞɚɧɧɹ ɞɥɹ ɫɚɦɨɩɟɪɟɜɿɪɤɢ ʋ 1 35
Ɂɚɜɞɚɧɧɹ ɞɥɹ ɫɚɦɨɩɟɪɟɜɿɪɤɢ ʋ 1
Ɋɿɜɟɧɶ 1
1. ɑɨɦɭ ɞɨɪɿɜɧɸɽ ɡɧɚɱɟɧɧɹ ɜɢɪɚɡɭ 2 1
5
x
x
−
−
, ɹɤɳɨ ɯ = –4?
ɚ) 1; ɛ) –1; ɜ) 7 ;
9
ɝ) ɧɟ ɿɫɧɭɽ.
2. Ⱦɥɹ ɹɤɢɯ ɡɧɚɱɟɧɶ ɡɦɿɧɧɨʀ ɧɟ ɦɚɽ ɡɦɿɫɬɭ ɜɢɪɚɡ 8 ?
2 5
x
x
+
−
ɚ) ɯ = 0; ɛ) ɯ = 2; ɜ) ɯ = 2,5; ɝ) ɯ = 5.
3. ɋɤɨɪɨɬɿɬɶ ɞɪɿɛ
2
3
18 .
3
a
a
ɚ) 18;
a
ɛ) 3
1 ;
6a
ɜ) 6 ;
a
ɝ) 3
6 .
a
4. Ɂɜɟɞɿɬɶ ɞɪɿɛ 3
b
ɞɨ ɡɧɚɦɟɧɧɢɤɚ b2
.
ɚ) 2
3 ;
b
ɛ) 2
3 ;b
b
ɜ)
2
2
3 ;b
b
ɝ)
2
3 .b
b
5. Ⱦɨɞɚɣɬɟ ɞɪɨɛɢ: 2 2
3 1 5 8
.
y y
y y
− −
+
ɚ) 2
4 11
;
y
y
+
ɛ) 2
4 5
;
2
y
y
−
ɜ) 4 – 5ɭ; ɝ) 2
4 5
.
y
y
−
6. ɋɩɪɨɫɬɿɬɶ ɜɢɪɚɡ .
2
a b a b
a a
+ −−
ɚ) ;
2
a b
a
+ ɛ) ;b
a
ɜ) 3 ;
2
a b
a
+ ɝ) 3 .
2
a b
a
+
36 § 1. Ɋɚɰɿɨɧɚɥɶɧɿ ɜɢɪɚɡɢ
Ɋɿɜɟɧɶ 2
7. ɍɫɬɚɧɨɜɿɬɶ ɜɿɞɩɨɜɿɞɧɿɫɬɶ ɦɿɠ ɜɢɪɚɡɚɦɢ (1–4) ɬɚ ɡɧɚɱɟɧɧɹɦɢ ɡɦɿɧɧɨʀ
(Ⱥ–Ⱦ), ɞɥɹ ɹɤɢɯ ɜɢɪɚɡ ɧɟ ɦɚɽ ɡɦɿɫɬɭ.
1) 3 ;
2 3
a
a
−
−
Ⱥ) 0; –1;
2)
3
2
4 5;c
c
+ Ȼ) 3;
3) 2
11 ;
2 10
x
x x−
ȼ) 1,5;
4) 2 1 1.
1 2
z z
z z
− +−
+
Ƚ) 0; 5;
Ⱦ) 0.
8. ɋɤɨɪɨɬɿɬɶ ɞɪɿɛ:
ɚ)
3 2
4
27 ;
36
a b
a b
ɛ) 5 10 .
3 6
a b
a b
−
−
9. Ɂɧɚɣɞɿɬɶ ɡɧɚɱɟɧɧɹ ɜɢɪɚɡɭ
2
2 6
3
a a
a
−
−
, ɹɤɳɨ ɚ = 1,7.
10. ɉɨɞɚɣɬɟ ɭ ɜɢɝɥɹɞɿ ɞɪɨɛɭ:
ɚ) 3 2
35 ;
yx
a b ab
− ɛ) .
4 4 7 7
a b
x y x y
+
+ +
11. ɋɩɪɨɫɬɿɬɶ ɜɢɪɚɡ:
ɚ)
2
2 ;
x y
x y
+
+
−
ɛ) 2 2
8 16 .−
− −
n
m n m n
Ɋɿɜɟɧɶ 3
12. Ɂɧɚɣɞɿɬɶ ɞɨɩɭɫɬɢɦɿ ɡɧɚɱɟɧɧɹ ɡɦɿɧɧɨʀ ɭ ɜɢɪɚɡɿ 2
14 .
( 2) 4− −
k
k
13. ɋɤɨɪɨɬɿɬɶ ɞɪɿɛ:
ɚ)
2 5
3
196
;
35
x y
x y
ɛ)
3 2
2 3 2
3 .
9
a a
ab a b
−
−
14. ɉɨɞɚɣɬɟ ɭ ɜɢɝɥɹɞɿ ɞɪɨɛɭ:
ɚ) 2 2
1 1 ;a a
a a a a
+ −−
− +
ɛ) 2
.
( )
mn n n
m n m nm n
+ +
− ++
Ɂɚɜɞɚɧɧɹ ɞɥɹ ɫɚɦɨɩɟɪɟɜɿɪɤɢ ʋ 1 37
15. Ɂɧɚɣɞɿɬɶ ɡɧɚɱɟɧɧɹ ɜɢɪɚɡɭ 2 2
1 1
m mn mn n
−
− −
, ɹɤɳɨ ɬ = 0,7; ɩ = 1.
3
16. ɋɩɪɨɫɬɿɬɶ ɜɢɪɚɡ:
ɚ) 2 ;aba b
a b
+ −
+
ɛ) 2 2 2
.
2
b a b
a ab b b ab
+−
− + −
Ɋɿɜɟɧɶ 4
17. Ⱦɥɹ ɹɤɢɯ ɡɧɚɱɟɧɶ ɡɦɿɧɧɨʀ ɧɟ ɦɚɽ ɡɦɿɫɬɭ ɜɢɪɚɡ?
ɚ) 2
15 ;
2 15a a+ −
ɛ) 3
| 7 | | |− +x x
.
18. ɋɤɨɪɨɬɿɬɶ ɞɪɿɛ:
ɚ)
3 2
10 4 40 ;
10
x x x
x
− − +
−
ɛ)
2 2
16 64.
8
x x a
x a
− − +
+ −
19. ɉɨɞɚɣɬɟ ɭ ɜɢɝɥɹɞɿ ɞɪɨɛɭ:
ɚ) 2 2
5 1 6 1 ;
5 5 6 12 6
x x
x x x x
− −−
+ + +
ɛ)
2 2
2 2
2 4 .
2 2 4 4
a b a b
a b a b a ab b
++ −
− + + +
20. ɋɩɪɨɫɬɿɬɶ ɜɢɪɚɡ
2
2 3
1 1 4 .
2 2 4 8 2
a a
a a a a a
++ − +
− + − −
21. Ⱦɨɜɟɞɿɬɶ ɬɨɬɨɠɧɿɫɬɶ 2 4 8
1 1 2 4 8 .
1 1 1 1 1
+ + + =
− + + + −x x x x x
38 § 1. Ɋɚɰɿɨɧɚɥɶɧɿ ɜɢɪɚɡɢ
1. Ɇɧɨɠɟɧɧɹ ɞɪɨɛɿɜ. Ʉɨɥɢ ɦɧɨɠɚɬɶ ɡɜɢɱɚɣɧɿ ɞɪɨɛɢ, ɬɨ ɨɤɪɟɦɨ ɦɧɨɠɚɬɶ
ʀɯɧɿ ɱɢɫɟɥɶɧɢɤɢ ɬɚ ɡɧɚɦɟɧɧɢɤɢ ɿ ɩɟɪɲɢɣ ɞɨɛɭɬɨɤ ɡɚɩɢɫɭɸɬɶ ɱɢɫɟɥɶɧɢɤɨɦ
ɞɪɨɛɭ, ɚ ɞɪɭɝɢɣ — ɡɧɚɦɟɧɧɢɤɨɦ. ɇɚɩɪɢɤɥɚɞ, 3 5 3 5 15 .
4 7 4 7 28
⋅⋅ = =
⋅
Ɍɚɤ ɫɚɦɨ ɦɧɨɠɚɬɶ ɛɭɞɶ-ɹɤɿ ɞɪɨɛɢ a
b
ɿ c
d
:
.a c ac
b d bd
⋅ = (1)
Ɋɿɜɧɿɫɬɶ (1) ɽ ɬɨɬɨɠɧɿɫɬɸ, ɬɨɛɬɨ ɜɨɧɚ ɽ ɩɪɚɜɢɥɶɧɨɸ ɞɥɹ ɛɭɞɶ-ɹɤɢɯ ɡɧɚ-
ɱɟɧɶ ɚ, b, c ɿ d, ɞɟ b ≠ 0 ɿ d ≠ 0.
Ɂ ɬɨɬɨɠɧɨɫɬɿ (1) ɜɢɩɥɢɜɚɽ ɩɪɚɜɢɥɨ ɦɧɨɠɟɧɧɹ ɞɪɨɛɿɜ:
ɓɨɛ ɩɨɦɧɨɠɢɬɢ ɞɪɿɛ ɧɚ ɞɪɿɛ, ɩɨɬɪɿɛɧɨ ɩɟɪɟɦɧɨɠɢɬɢ ɨɤɪɟɦɨ ʀɯɧɿ
ɱɢɫɟɥɶɧɢɤɢ ɬɚ ɡɧɚɦɟɧɧɢɤɢ ɿ ɩɟɪɲɢɣ ɞɨɛɭɬɨɤ ɡɚɩɢɫɚɬɢ ɱɢɫɟɥɶɧɢɤɨɦ, ɚ
ɞɪɭɝɢɣ — ɡɧɚɦɟɧɧɢɤɨɦ ɞɪɨɛɭ.
ɐɟ ɩɪɚɜɢɥɨ ɩɨɲɢɪɸɽɬɶɫɹ ɧɚ ɜɢɩɚɞɨɤ ɦɧɨɠɟɧɧɹ ɬɪɶɨɯ ɿ ɛɿɥɶɲɟ ɞɪɨɛɿɜ.
2. ɉɿɞɧɟɫɟɧɧɹ ɞɪɨɛɭ ɞɨ ɫɬɟɩɟɧɹ. ȼɢɤɨɪɢɫɬɨɜɭɸɱɢ ɩɪɚɜɢɥɨ ɦɧɨɠɟɧɧɹ
ɞɪɨɛɿɜ, ɩɿɞɧɟɫɟɦɨ ɞɪɿɛ ɞɨ n-ɝɨ ɫɬɟɩɟɧɹ:
( )
P
N
ɪɚɡɿɜ
ɪɚɡɿɜɪɚɡɿɜ
...... .
...
n
n n
n
nn
a a a a aa a a
b b b b bb b b
= ⋅ ⋅ ⋅ = =
 

Ɉɬɠɟ,
( ) .
n n
n
a a
b b
= (2)
Ɂ ɬɨɬɨɠɧɨɫɬɿ (2) ɜɢɩɥɢɜɚɽ ɩɪɚɜɢɥɨ ɩɿɞɧɟɫɟɧɧɹ ɞɪɨɛɭ ɞɨ ɫɬɟɩɟɧɹ:
ɓɨɛ ɩɿɞɧɟɫɬɢ ɞɪɿɛ ɞɨ ɫɬɟɩɟɧɹ, ɩɨɬɪɿɛɧɨ ɩɿɞɧɟɫɬɢ ɞɨ ɰɶɨɝɨ ɫɬɟɩɟɧɹ
ɱɢɫɟɥɶɧɢɤ ɬɚ ɡɧɚɦɟɧɧɢɤ ɿ ɩɟɪɲɢɣ ɪɟɡɭɥɶɬɚɬ ɡɚɩɢɫɚɬɢ ɱɢɫɟɥɶɧɢ-
ɤɨɦ, ɚ ɞɪɭɝɢɣ — ɡɧɚɦɟɧɧɢɤɨɦ ɞɪɨɛɭ.
5. Ɇɧɨɠɟɧɧɹ ɞɪɨɛɿɜ. ɉɿɞɧɟɫɟɧɧɹ ɞɪɨɛɭ ɞɨ ɫɬɟɩɟɧɹ 39
ȼɩɪɚɜɚ 1. ȼɢɤɨɧɚɬɢ ɦɧɨɠɟɧɧɹ:
ɚ)
4 3
2 3
6 ;
8
a b c
c a
⋅ ɛ)
2
2 2 2
.ab b b
a a b
+ ⋅
−
• ɚ)
4 3
2 3
6
8
a b c
c a
⋅ =
4 3
2 3
6
8
a b c
c a
⋅
⋅
= 3 ;
4
abc
ɛ)
2
2 2 2
ab b b
a a b
+ ⋅
−
= 2
( )
( )( )
b a b b
a a b a b
+ ⋅
⋅ − +
=
2
2
( )
b
a a b
⋅
−
•
ȼɩɪɚɜɚ 2. ɉɨɦɧɨɠɢɬɢ ɞɪɿɛ 3
3
x
x
+
−
ɧɚ ɦɧɨɝɨɱɥɟɧ ɯ – 3.
• Ɂɚɩɢɫɚɜɲɢ ɦɧɨɝɨɱɥɟɧ ɯ – 3 ɭ ɜɢɝɥɹɞɿ ɞɪɨɛɭ 3
1
x − , ɦɚɬɢɦɟɦɨ:
3 ( 3)
3
+ ⋅ −
−
x x
x
= 3 3
3 1
x x
x
+ −⋅
−
=
( 3) ( 3)
( 3) 1
x x
x
+ ⋅ −
− ⋅
= 3x + .
ɋɤɨɪɨɱɟɧɢɣ ɡɚɩɢɫ: 3 ( 3)
3
+ ⋅ −
−
x x
x
=
( 3)( 3)
3
x x
x
+ −
−
= 3x + .•
ȼɩɪɚɜɚ 3. ɉɿɞɧɟɫɬɢ ɞɨ ɤɜɚɞɪɚɬɚ ɞɪɿɛ
3
2
2
5
a b
m n
− .
•
23
2
2
5
a b
m n
§ ·
−¨ ¸
© ¹
=
23
2
2
5
§ ·
¨ ¸
© ¹
a b
m n
=
3 2
2 2
(2 )
(5 )
a b
m n
=
2 3 2 2
2 2 2 2
2 ( )
5 ( )
a b
m n
⋅ ⋅
⋅ ⋅
=
6 2
4 2
4 .
25
a b
m n
ɋɤɨɪɨɱɟɧɢɣ ɡɚɩɢɫ:
23
2
2
5
a b
m n
§ ·
−¨ ¸
© ¹
=
6 2
4 2
4 .
25
a b
m n
•
124. ȼɢɤɨɧɚɣɬɟ ɦɧɨɠɟɧɧɹ:
ɚ) ;a m
b n
⋅ ɛ) 5 2 ;
3 5
b
a
⋅ ɜ) 2 ;m
k
⋅ ɝ) 3 1 .
4
x
x
⋅
40 § 1. Ɋɚɰɿɨɧɚɥɶɧɿ ɜɢɪɚɡɢ
125. ɉɿɞɧɟɫɿɬɶ ɞɨ ɫɬɟɩɟɧɹ:
ɚ) ( )
2
;a
c
ɛ) ( )
2
2 ;
3
a
c
ɜ)
42
;a
c
§ ·
¨ ¸
© ¹
ɝ)
32
3
3 .a
c
§ ·
−¨ ¸
© ¹
ȼɢɤɨɧɚɣɬɟ ɦɧɨɠɟɧɧɹ:
126. ɚ) 4 5 ;
3 16
b
a
⋅ ɛ) 3 2 ;
5 9
k
k
⋅ ɜ)
2
8 1 ;
11
b
b
⋅ ɝ)
4
2
14 ;
7
§ ·
⋅ −¨ ¸
© ¹
y
y
ɞ) 2 4
25 ;
5
c d
d c
⋅ ɟ)
3
2
1012 ;
5
yx
y x
⋅ ɽ) 3
34 ;
2
ax
x
⋅ ɠ) ( ) 25 3 .
6
m n m
n
− ⋅
127. ɚ)
4
2
3 2 ;
94
b
b
⋅ ɛ) 5
6 21;
7
k
k
⋅ ɜ)
3 2
6 ;
9
§ ·
⋅ −¨ ¸
© ¹
x a
a x
ɝ) 2
5 2 .
4
a ay
y
⋅
ɉɨɞɚɣɬɟ ɭ ɜɢɝɥɹɞɿ ɞɪɨɛɭ:
128. ɚ)
2 2
3 5
25 10 ;
4 15
a b
b a
⋅ ɛ) 3
4
5 3 ;
9
a ab
b
− ⋅ ɜ) 2
5
17 .
34
§ ·
− ⋅ −¨ ¸
© ¹
y
x y
x
129. ɚ)
22
2 3
272 ;
9 4
yx
y x
⋅ ɛ) 2
3
2 ( 10 );
5
a ab
b
− ⋅ − ɜ) 2
3
312 .
16
§ ·⋅ −¨ ¸
© ¹
m
mn
ȼɢɤɨɧɚɣɬɟ ɦɧɨɠɟɧɧɹ:
130. ɚ)
3
2 2
;
( )
x y a b
a x y
−
⋅
−
ɛ)
33
( )
;
m nx
m n x
+
⋅
+
ɜ)
2
3
3 3
;
x y b
x yb
+
⋅
+
ɝ) 2
2 1 7 ;
2 17
x x
xx x
− −⋅
−−
ɞ)
2
9 2 ;
2 3
m m
m m
− +⋅
+ −
ɟ)
2
2
4 4 4 .
4 4
a a a
a a
− + +⋅
+ −
131. ɚ)
4
2
;
ya b
a by
+ ⋅
+
ɛ)
4
3 2
;
( )
x y a
a x y
+
⋅
+
ɜ) 2
;ab ac k
b ck
+ ⋅
+
ɝ)
2
2 1 1;
1 1
b b x
x b
− + −⋅
− −
ɞ)
2 2
16
;
4
y b
ab y
−
⋅
−
ɟ)
2 5
3 2
2 1 .
1
c c c
c c
+ + ⋅
−
ɉɿɞɧɟɫɿɬɶ ɞɨ ɫɬɟɩɟɧɹ:
132. ɚ)
22
2 ;x
y
§ ·
¨ ¸
© ¹
ɛ)
43
2 ;
3
a
b
§ ·
−¨ ¸
© ¹
ɜ)
32 3
;
5
n k
m
§ ·
−¨ ¸
© ¹
ɝ)
32 4
3
3 .
4
a b
c
§ ·
¨ ¸
© ¹
5. Ɇɧɨɠɟɧɧɹ ɞɪɨɛɿɜ. ɉɿɞɧɟɫɟɧɧɹ ɞɪɨɛɭ ɞɨ ɫɬɟɩɟɧɹ 41
133. ɚ)
2
2
3 ;m
n
§ ·
¨ ¸
© ¹
ɛ)
3
2
2
2 ;x
y z
§ ·
¨ ¸
© ¹
ɜ)
33
2
3 ;
5
a b
c
§ ·
−¨ ¸
© ¹
ɝ)
23 4
3
9
.
5
x y
a
§ ·
−¨ ¸
© ¹
ȼɢɤɨɧɚɣɬɟ ɦɧɨɠɟɧɧɹ:
134. ɚ)
23
3 2 2
512 ;
18
x ya b
x a a
⋅ ⋅ ɛ)
2
3 2
3 2 4 ;
4 3 3
m n n
n m m
§ ·⋅ − ⋅¨ ¸
© ¹
ɜ) 2 3
5 ;
10
bx byab
xy y a
+
⋅
+
ɝ)
2
2 3 2 3
;
xy a a
a a x y
+⋅
+
ɞ)
2
2
18 4 2 ;
272
a x
axx x
−⋅
−
ɟ)
2
2
3 2 .
9 6
a b ab a
a bab b
+ +⋅
++
135. ɚ)
2 3 2
3 2
6 ;
3 4
ab m mn
mn a ab
⋅ ⋅ ɛ)
2 3
2
5 5
;
15 15
x y x x
x yx x
+ +⋅
++
ɜ) 2 3
6 316 .
2 20
ymn
y y m n
+
⋅
+
ɋɩɪɨɫɬɿɬɶ ɜɢɪɚɡ:
136. ɚ) 2 2
3 1 1 ;
1 9 1
a a
a a
− +⋅
− −
ɛ)
22 2
;
9 9
x xyb a
x y a b
−− ⋅
− −
ɜ)
2
2 2
4
;
( ) 32
xy x xy
x y x
+
⋅
+
ɝ)
2 2 2
5
;
10
y a y
a y y
−§ ·
⋅¨ ¸−© ¹
ɞ)
2 2 2
(2 1)
;
7 7 4 2
x a b
b a x
+ −⋅
− +
ɟ)
2 2
3 2 2
4 4 2 ;
( ) 2 2
m n m mn n
m n m n
− + +⋅
+ −
ɽ)
2 23 3
2 2
;
2 2
x ym n
y x m mn n
−− ⋅
− + +
ɠ)
2 2
3 3
4 3 3 .
2
c a a b
a ca b
− +⋅
++
137. ɚ) 2 2
5 5 ;
2
x ya b
a bx xy y
−− ⋅
−− +
ɛ)
2
2 2
( )3 3 ;
5 5
a ba b
a b b a
+− ⋅
+ −
ɜ)
2 2 2
2
(4 )
;
12 3
x x y
xxy y
− −
⋅
−+
ɝ)
2 2 2
3 3
1
;
3 32 2
y a ab b
ya b
− − +⋅
−+
ɞ) ( )
2
2
15 ;
52 2
a a b
aab b
+⋅
+
ɟ)
2 2
3 2
1 1 .
1 2 1
a x x
x a a
− + +⋅
− + +
42 § 1. Ɋɚɰɿɨɧɚɥɶɧɿ ɜɢɪɚɡɢ
Ɂɧɚɣɞɿɬɶ ɡɧɚɱɟɧɧɹ ɜɢɪɚɡɭ:
138. ɚ)
22 5
3
2 3 98 ,
7 94
a b a
b a
§ ·⋅ − ⋅¨ ¸
© ¹
ɹɤɳɨ ɚ = –1,25; b = 8;
ɛ)
2 3 2
2 2
8 16 3 12
16 4
x x x x
x x x
+ + −⋅
− +
, ɹɤɳɨ ɯ = –1; ɯ = 0,8; ɯ = 24 .
3
139. ɚ)
3 9
2 4 2
16 3 ,
2 3
yx
y x y
§ · § ·
⋅ ⋅ −¨ ¸ ¨ ¸
© ¹ © ¹
ɹɤɳɨ ɯ = 1 ;
7
ɭ = 0,5;
ɛ)
3 3
3 2
27 4
0,2 3 9
a a
a a a
+ ⋅
− +
, ɹɤɳɨ ɚ = –4; ɚ = 5; ɚ = 1 .
4
−
140. ɋɩɪɨɫɬɿɬɶ ɜɢɪɚɡ:
ɚ)
2 2
2 2
2 2
;
2 2
x xy xz yz x xy x y
x xy xz yz x xy x y
+ + + − − +
⋅
− + − + − −
ɛ)
23 34 2 2
2 2 2
2 .
2
a a b b a b
a ab b a b
§ ·§ ·− + −§ ·⋅¨ ¸¨ ¸¨ ¸
− + −© ¹© ¹ © ¹
141. Ɂɧɚɣɞɿɬɶ ɡɧɚɱɟɧɧɹ ɜɢɪɚɡɭ
21
1
16 4 1616
64 4 64
n n
n
n n
+
+
§ ·+ ⋅⋅¨ ¸
− ⋅© ¹
, ɹɤɳɨ n = 74; n = 1000.
142. Ⱦɨɜɟɞɿɬɶ ɬɨɬɨɠɧɿɫɬɶ
2
2
( 2) 2 2
2( 2) 2
x a x a x a x
x a xx a x a
+ + + − +
⋅ =
+ −− + +
.
143. Ɂɧɚɣɞɿɬɶ ɱɢɫɥɚ, ɨɛɟɪɧɟɧɿ ɞɨ ɞɚɧɢɯ: 2 ;
7
4; 51 ;
6
0,2; 1,6.
144. Ɉɛɱɢɫɥɿɬɶ:
ɚ) 18 4: ;
25 15
ɛ) 2 14 : 42 ;
3 6
− ɜ) 1 20,125:3 1 : 7.
8 5
−
6. Ⱦɿɥɟɧɧɹ ɞɪɨɛɿɜ 43
145. ɍ ɩɟɪɲɨɦɭ ɪɟɡɟɪɜɭɚɪɿ ɛɭɥɨ 480 ɥ ɜɨɞɢ, ɚ ɜ ɞɪɭɝɨɦɭ — 282 ɥ. Ɂ ɩɟɪɲɨɝɨ
ɪɟɡɟɪɜɭɚɪɚ ɛɟɪɭɬɶ ɳɨɞɟɧɧɨ 25 ɥ ɜɨɞɢ, ɚ ɡ ɞɪɭɝɨɝɨ — 16 ɥ. ɑɟɪɟɡ ɫɤɿɥɶɤɢ
ɞɧɿɜ ɭ ɩɟɪɲɨɦɭ ɪɟɡɟɪɜɭɚɪɿ ɜɨɞɢ ɛɭɞɟ ɭɞɜɿɱɿ ɛɿɥɶɲɟ, ɧɿɠ ɭ ɞɪɭɝɨɦɭ?
146*. ȼɿɞ ɩɪɢɫɬɚɧɿ A ɞɨ ɩɪɢɫɬɚɧɿ B ɡɚ ɬɟɱɿɽɸ ɪɿɱɤɢ ɨɞɧɨɱɚɫɧɨ ɜɿɞɩɥɢɜɥɢ ɤɚɬɟɪ
ɿ ɩɥɿɬ. Ʉɨɥɢ ɱɟɪɟɡ 1,5 ɝɨɞ ɤɚɬɟɪ ɩɪɢɛɭɜ ɞɨ ɩɪɢɫɬɚɧɿ B, ɩɥɨɬɭ ɡɚɥɢɲɚɥɨɫɹ
ɩɪɨɩɥɢɫɬɢ ɞɨ ɰɿɽʀ ɩɪɢɫɬɚɧɿ ɳɟ 27 ɤɦ. ɇɟ ɡɚɬɪɢɦɭɸɱɢɫɶ ɧɚ ɩɪɢɫɬɚɧɿ B,
ɤɚɬɟɪ ɜɢɪɭɲɢɜ ɭ ɡɜɨɪɨɬɧɢɣ ɲɥɹɯ. ɑɟɪɟɡ ɹɤɢɣ ɱɚɫ ɩɿɫɥɹ ɜɿɞɩɪɚɜɤɢ ɜɿɞ
ɩɪɢɫɬɚɧɿ B ɤɚɬɟɪ ɡɭɫɬɪɿɧɟ ɩɥɿɬ? ɑɨɦɭ ɞɨɪɿɜɧɸɽ ɲɜɢɞɤɿɫɬɶ ɤɚɬɟɪɚ ɭ
ɫɬɨɹɱɿɣ ɜɨɞɿ?
147. ɍ ɬɪɢɞɟɜ’ɹɬɨɦɭ ɤɨɪɨɥɿɜɫɬɜɿ ɤɨɠɧɿ ɞɜɚ ɦɿɫɬɚ ɡ’ɽɞɧɚɧɿ ɞɨɪɨɝɨɸ ɡ ɨɞɧɨ-
ɫɬɨɪɨɧɧɿɦ ɪɭɯɨɦ. Ⱦɨɜɟɞɿɬɶ, ɳɨ ɿɫɧɭɽ ɦɿɫɬɨ, ɡ ɹɤɨɝɨ ɜ ɛɭɞɶ-ɹɤɟ ɿɧɲɟ
ɦɿɫɬɨ ɦɨɠɧɚ ɩɪɨʀɯɚɬɢ ɥɢɲɟ ɨɞɧɿɽɸ ɚɛɨ ɞɜɨɦɚ ɞɨɪɨɝɚɦɢ.
Ʉɨɥɢ ɞɿɥɹɬɶ ɡɜɢɱɚɣɧɿ ɞɪɨɛɢ, ɬɨ ɩɟɪɲɢɣ ɞɪɿɛ ɦɧɨɠɚɬɶ ɧɚ ɞɪɿɛ, ɨɛɟɪɧɟɧɢɣ
ɞɨ ɞɪɭɝɨɝɨ. ɇɚɩɪɢɤɥɚɞ, 2 3 2 5 10: .
7 5 7 3 21
= ⋅ =
ɍ ɬɚɤɢɣ ɠɟ ɫɩɨɫɿɛ ɞɿɥɹɬɶ ɛɭɞɶ-ɹɤɿ ɞɪɨɛɢ a
b
ɿ c
d
:
: .a c a d
b d b c
= ⋅
Ɉɫɬɚɧɧɹ ɪɿɜɧɿɫɬɶ ɽ ɬɨɬɨɠɧɿɫɬɸ, ɬɨɛɬɨ ɜɨɧɚ ɽ ɩɪɚɜɢɥɶɧɨɸ ɞɥɹ ɜɫɿɯ ɡɧɚ-
ɱɟɧɶ ɚ, b, c ɿ d, ɞɟ b ≠ 0, ɫ ≠ 0 ɿ d ≠ 0. Ɂ ɰɿɽʀ ɬɨɬɨɠɧɨɫɬɿ ɜɢɩɥɢɜɚɽ ɩɪɚɜɢɥɨ ɞɿ-
ɥɟɧɧɹ ɞɪɨɛɿɜ:
ɓɨɛ ɩɨɞɿɥɢɬɢ ɨɞɢɧ ɞɪɿɛ ɧɚ ɞɪɭɝɢɣ, ɩɨɬɪɿɛɧɨ ɩɟɪɲɢɣ ɞɪɿɛ ɩɨɦɧɨ-
ɠɢɬɢ ɧɚ ɞɪɿɛ, ɨɛɟɪɧɟɧɢɣ ɞɨ ɞɪɭɝɨɝɨ.
ɇɚɩɪɢɤɥɚɞ, 2 2 2
2 2 2 2 2 4: .
2
a a a b a b
b a bb b b a
⋅= ⋅ = =
⋅
44 § 1. Ɋɚɰɿɨɧɚɥɶɧɿ ɜɢɪɚɡɢ
ȼɩɪɚɜɚ 1. ȼɢɤɨɧɚɬɢ ɞɿɥɟɧɧɹ:
ɚ)
2 3
3
15 : ;
714
a a
cc
ɛ) 2 2
3: ;
1
ab b
a a a− −
ɜ)
2 2
4
: (2 ).
2
x y
x y
x
−
−
• ɚ)
2 3
3
15 :
714
a a
cc
=
2
3 3
15 7
14
a c
c a
⋅ =
2
3 3
15 7
14
a c
c a
⋅
⋅
= 2
15 .
2c a
ɛ) 2 2
3:
1
ab b
a a a− −
=
( 1)
( 1)( 1) 3
a aab
a a b
−
⋅
− +
=
( 1)
( 1)( 1) 3
ab a a
a a b
⋅ −
− + ⋅
=
2
3( 1)
a
a
⋅
+
ɜ)
2 2
4
:(2 )
2
x y
x y
x
−
− =
(2 )(2 ) 1
2 2
x y x y
x x y
− +
⋅
−
=
(2 )(2 )
2 (2 )
x y x y
x x y
− +
−
=
2
2
x y
x
+
⋅ •
148. ȼɢɤɨɧɚɣɬɟ ɞɿɥɟɧɧɹ:
ɚ) : ;x m
y n
ɛ) 1 1: ;
a b
ɜ) : 2;
4
a ɝ) 33: .
x
ȼɢɤɨɧɚɣɬɟ ɞɿɥɟɧɧɹ:
149. ɚ) 2: ;
9 3
a a ɛ) 6 2: ;
5 15
ab b
c
ɜ) 2 1: ;x
x
ɝ) 9 :3;
d
ɞ) 3
2
3819 : ;
5
nn
p
ɟ) ( )
3
33 : 11 ;
12
c c
m
ɽ)
3
5 3
: ;
2 4
c c
a a
ɠ)
2 3
3 4
12 3: .
25 5
ab b
x x
150. ɚ) 4 3
3 1: ;
10 5
x
y y
ɛ)
3
4
2
1827 : ;
7
aa
b
ɜ)
2
2 3
6 3: ;
5
x x
y y
ɝ) ( )
2
2
3
5 : 10 .
7
mn n
k
6. Ⱦɿɥɟɧɧɹ ɞɪɨɛɿɜ 45
151. ɚ)
3 4
4
18 6: ;
155
a a
bb
−
−
ɛ)
3 4
2 3
9 36: ;
20 5
b b
n n
− ɜ)
2 3
2 28
14 : .
5
x y
xy
z
§ ·
−¨ ¸
© ¹
152. ɚ)
2 2
2 3
8 4: ;
9 3
x x
y y
− ɛ)
3 4
2
15 9: ;
82
m m
nn
− ɜ)
3 2 4
6 3
: .
5 10
xy x y§ ·
− −¨ ¸
© ¹
ɋɩɪɨɫɬɿɬɶ ɜɢɪɚɡ:
153. ɚ) 4 2
6 6 : ;a b a b
c c
+ + ɛ)
2
3
: ;mn n m n
aa
− − ɜ)
2 2
2 3
: ;c d c d
k k
− +
ɝ)
6 4
2
: ;
416
x x
xx −−
ɞ)
3
2
: ;
36 9
b b
bb b −− +
ɟ)
2 2
4 4 4
: .
1 1
y y y
y y
− + −
+ +
154. ɚ)
3
: ;x x
ab ac b c+ +
ɛ)
2
4 3
4 2: ;a a
c c
− − ɜ)
2
5
1: ;m n mn m
yy
+ +
ɝ)
2
2
25 5: ;k k
k k
− + ɞ)
2 2
2
: ;
10 25
x xy y x y− + −
ɟ) 2 2
2 2: .
2 1 1
a a
a a a
− −
+ + −
ȼɢɤɨɧɚɣɬɟ ɞɿɥɟɧɧɹ:
155. ɚ)
3 2
3 4
5 : ;
4 8
x x x
a a
− ɛ)
2 2
3 2 2
1 : ;b b b
ac a c
− + ɜ)
2
2
5 10 2: ;
( )
a a a
a ba b
− −
−−
ɝ)
2
2
3 3 : (2 2);
1
x x
x
− +
+
ɞ)
2 2
2 2
18 24: ;
1 (1 )
ab a b
x x− −
ɟ)
2 2
2 : .a ab b a b
a b ac bc
+ + +
− −
156. ɚ)
3 2 2
3 3
7 : ;
11 22
c c c
ab a b
− ɛ)
2 2
2
4 2: ;
1
x x x
xx x
− −
++
ɜ)
2
2 2
( ) 6 6: .
1
m n m n
m m m
− −
+ −
ɋɩɪɨɫɬɿɬɶ ɜɢɪɚɡ:
157. ɚ)
32 6 2
2
36 3: ;
25 5
a x ax
b b
§ ·
−¨ ¸
© ¹
ɛ) ( )
24 2
3
2 10: ;
3 9
m m
n n
§ ·
− ¨ ¸
© ¹
46 § 1. Ɋɚɰɿɨɧɚɥɶɧɿ ɜɢɪɚɡɢ
ɜ)
2 2
2
5 15: ;
7 142
a b a a
b bcc
§ ·
⋅¨ ¸
© ¹
ɝ)
33 4
4
39 3: : .
5 45
yx x b
y xy
§ ·
¨ ¸
© ¹
158. ɚ)
52 3 7
5: ;
2 64
mn m n§ ·
−¨ ¸
© ¹
ɛ)
3 22 3
3 9: ;
2 4
a b a
c c
§ · § ·
¨ ¸ ¨ ¸
© ¹ © ¹
ɜ)
2 2
3 4
3 65: : ;
65 5
xy x yz
yz z
§ ·
¨ ¸
© ¹
ɝ)
3 2 2 2
4 2 2
27 2 3: .
4 3 4
a b a c b
c b ac
§ ·
⋅¨ ¸
© ¹
159. ɚ) 2 2 2 2
3 6 7 14: ;
2
a b a b
a b a ab b
+ +
− − +
ɛ)
2 2
2 2
4 4 1 1 4: ;
3 3
c c c
x y x y
+ + −
− −
ɜ)
2 2
3 3 2 2
4: ;mn m n
m n m mn n+ − +
ɝ) ( )
2 2
2 2
3 : ;a a ab
a b a b
+
− −
ɞ)
2 2
3 2 2
2 2 2
: ;
( ) 2
x y x xy y
x y x xy y
− − +
+ + +
ɟ)
3 2
2 2
27 3 9: .
2 4 1
a a a
a a a
− + +
− −
160. ɚ)
2 2 2
2 2 3
( )
: ;
a b a b
ab b ab b
− −
+ +
ɛ)
2
2 2
2 2( 2 )
: ;
3 3
+ +
− −
x y x y
x y x y
ɜ) 2 3
2 4 2: ;
1 1
c d ac ad
b b b
+ +
+ + −
ɝ)
2 2
2 3
: .
4 2 8
ab ac c b
a a a
− −
− + +
Ɂɧɚɣɞɿɬɶ ɡɧɚɱɟɧɧɹ ɜɢɪɚɡɭ:
161. ɚ)
4 22 3
3
2 : ,
7 49
a a
b b
§ · § ·
−¨ ¸ ¨ ¸
© ¹ © ¹
ɹɤɳɨ ɚ = –0,25; b = 4;
ɛ)
2 2
2
4 4 1 2:
10 54 1
m m m m
mm
+ + +
−−
, ɹɤɳɨ m = –5; m = 0,5; m = 1 .
15
162. ( )
2 2
2 2
2:
2 4
a a ab
a b a b
−
+ −
, ɹɤɳɨ ɚ = –3, b = 4; ɚ = 78, b = 11.
Ⱦɨɜɟɞɿɬɶ ɬɨɬɨɠɧɿɫɬɶ:
163. ɚ)
2 2
2
2 ( )5 5: ;
4 58
b a ba b a b
ab ab
−− + =
ɛ) ( )
2 2
2
1 : .
( )
m m m n m n
m n m n m n m n
− +− − =−
+ − + −
6. Ⱦɿɥɟɧɧɹ ɞɪɨɛɿɜ 47
164. ɚ) ( )
2 2
16 63 12 : ;
2 4
x y xx y
x x y
−
− =
+
ɛ)
3 2
2 2
1 1 1 2 2: : .
2 42 2 1
a a a aa
aa a a
− − + −− =
+ −
165. ɋɩɪɨɫɬɿɬɶ ɜɢɪɚɡ:
ɚ)
22
6 4 2 2
0,25 0,5
: ;
1 1
x x
x x x x
− −§ ·
¨ ¸
− − + −© ¹
ɛ)
2 2
2 2
6 5 4 5: .a a a a
a ab a b a ab a b
+ + + −
− + − − − +
166. Ⱦɨɜɟɞɿɬɶ, ɳɨ ɜɢɪɚɡ
2 2
2 2
3 2 3
:
22
x y x xy y
x yx xy y
+ + −
++ −
ɧɚɛɭɜɚɽ ɥɢɲɟ ɞɨɞɚɬɧɢɯ
ɡɧɚɱɟɧɶ.
167. ɇɚ ɩɪɢɱɚɥɿ Ⱥ ɫɬɨɹɬɶ ɩɿɞɧɿɦɚɥɶɧɿ ɤɪɚɧɢ ʋ 1 ɿ ʋ 2, ɚ ɧɚ ɩɪɢɱɚɥɿ Ȼ — ɩɿɞ-
ɧɿɦɚɥɶɧɿ ɤɪɚɧɢ ʋ 3 ɿ ʋ 4. Ɂɚ ɞɨɩɨɦɨɝɨɸ ɤɪɚɧɚ ʋ 1 ɦɨɠɧɚ ɪɨɡɜɚɧ-
ɬɚɠɢɬɢ ɛɚɪɠɭ ɧɚ 3 ɝɨɞ, 2 ɝɨɞ ɿ 1 ɝɨɞ ɲɜɢɞɲɟ, ɧɿɠ ɡɚ ɞɨɩɨɦɨɝɨɸ ɜɿɞ-
ɩɨɜɿɞɧɨ ɤɪɚɧɿɜ ʋ 2, ʋ 3 ɿ ʋ 4. ɇɚ ɹɤɨɦɭ ɩɪɢɱɚɥɿ ɡɚ ɞɨɩɨɦɨɝɨɸ ɨɛɨɯ ɣɨ-
ɝɨ ɤɪɚɧɿɜ ɦɨɠɧɚ ɲɜɢɞɲɟ ɪɨɡɜɚɧɬɚɠɢɬɢ ɛɚɪɠɭ?
168. Ɋɨɡɜ’ɹɠɿɬɶ ɪɿɜɧɹɧɧɹ:
ɚ) 3(ɯ + 4) = 4(ɯ + 3); ɛ) 2ɯ(ɯ – 1) + ɯ(ɯ – 2) = 3ɯ2
– 2.
169. Ɏɭɧɤɰɿɹ ɡɚɞɚɧɚ ɮɨɪɦɭɥɨɸ ɭ = 5ɯ – 8.
ɚ) Ɂɧɚɣɞɿɬɶ ɡɧɚɱɟɧɧɹ ɮɭɧɤɰɿʀ, ɹɤɳɨ ɯ = –1; ɯ = 3.
ɛ) Ɂɧɚɣɞɿɬɶ ɡɧɚɱɟɧɧɹ ɚɪɝɭɦɟɧɬɭ, ɹɤɳɨ ɭ = –3; ɭ = 6.
170. Ɏɭɬɛɨɥɤɚ ɤɨɲɬɭɽ n ɝɪɧ. əɤɳɨ ɤɭɩɭɜɚɬɢ ɞɜɿ ɮɭɬɛɨɥɤɢ, ɬɨ ɦɚɝɚɡɢɧ ɧɚ
ɞɪɭɝɭ ɮɭɬɛɨɥɤɭ ɞɚɽ ɡɧɢɠɤɭ 30%. ɋɤɿɥɶɤɢ ɝɪɢɜɟɧɶ ɞɨɜɟɞɟɬɶɫɹ ɡɚɩɥɚɬɢɬɢ,
ɹɤɳɨ ɤɭɩɭɜɚɬɢ ɞɜɿ ɮɭɬɛɨɥɤɢ?
171. ȯ ɫɬɚɥɶ ɞɜɨɯ ɫɨɪɬɿɜ ɡ ɭɦɿɫɬɨɦ ɧɿɤɟɥɸ 10% ɿ 40%. ɋɤɿɥɶɤɢ ɬɨɧɧ ɫɬɚɥɿ
ɤɨɠɧɨɝɨ ɫɨɪɬɭ ɩɨɬɪɿɛɧɨ ɜɡɹɬɢ, ɳɨɛ ɩɿɫɥɹ ɩɟɪɟɩɥɚɜɤɢ ɨɞɟɪɠɚɬɢ 12 ɬ ɫɬɚ-
ɥɿ, ɹɤɚ ɦɿɫɬɢɥɚ ɛ 30% ɧɿɤɟɥɸ?
48 § 1. Ɋɚɰɿɨɧɚɥɶɧɿ ɜɢɪɚɡɢ
172. ɍ ɤɥɿɬɢɧɤɚɯ ɬɚɛɥɢɰɿ ɪɨɡɦɿɪɭ 3 × 3 ɡɚɩɢɫɚɧɨ ɰɿɥɿ ɱɢɫɥɚ ɬɚɤ, ɳɨ ɛɭɞɶ-ɹɤɿ
ɞɜɚ ɱɢɫɥɚ, ɡɚɩɢɫɚɧɿ ɭ ɫɭɫɿɞɧɿɯ ɩɨ ɫɬɨɪɨɧɿ ɤɥɿɬɢɧɤɚɯ, ɜɿɞɪɿɡɧɹɸɬɶɫɹ ɧɟ
ɛɿɥɶɲɟ ɧɿɠ ɧɚ 1. Ⱦɨɜɟɞɿɬɶ, ɳɨ ɿɫɧɭɸɬɶ ɞɜɿ ɤɥɿɬɢɧɤɢ, ɭ ɹɤɢɯ ɡɚɩɢɫɚɧɨ ɬɟ
ɫɚɦɟ ɱɢɫɥɨ.
ɍ ɤɭɪɫɿ ɚɥɝɟɛɪɢ ɧɚɦ ɭɠɟ ɬɪɚɩɥɹɥɨɫɹ ɱɢɦɚɥɨ ɡɚɜɞɚɧɶ, ɞɥɹ ɪɨɡɜ’ɹɡɚɧɧɹ
ɹɤɢɯ ɩɨɬɪɿɛɧɨ ɛɭɥɨ ɩɟɪɟɬɜɨɪɸɜɚɬɢ ɬɨɣ ɱɢ ɿɧɲɢɣ ɜɢɪɚɡ. Ɂɨɤɪɟɦɚ, ɩɟɪɟɬɜɨɪɟɧ-
ɧɹ ɰɿɥɢɯ ɪɚɰɿɨɧɚɥɶɧɢɯ ɜɢɪɚɡɿɜ ɦɢ ɜɢɤɨɪɢɫɬɨɜɭɜɚɥɢ ɞɥɹ ɪɨɡɜ’ɹɡɭɜɚɧɧɹ ɪɿɜɧɹɧɶ,
ɞɨɜɟɞɟɧɧɹ ɬɨɬɨɠɧɨɫɬɟɣ, ɡɧɚɯɨɞɠɟɧɧɹ ɡɧɚɱɟɧɶ ɜɢɪɚɡɿɜ. Ɋɨɡɝɥɹɧɟɦɨ ɞɟɹɤɿ ɡɚ-
ɞɚɱɿ, ɩɨɜ’ɹɡɚɧɿ ɡ ɬɨɬɨɠɧɢɦɢ ɩɟɪɟɬɜɨɪɟɧɧɹɦɢ ɞɪɨɛɨɜɢɯ ɪɚɰɿɨɧɚɥɶɧɢɯ ɜɢɪɚɡɿɜ.
ɉɪɢɤɥɚɞ 1. ɋɩɪɨɫɬɢɬɢ ɜɢɪɚɡ ( )
2
2
31 : 1 .
1 1
a a
a a
§ ·
+ −¨ ¸+ −© ¹
• ɋɩɨɱɚɬɤɭ ɩɨɞɚɦɨ ɜɢɪɚɡɢ ɜ ɤɨɠɧɿɣ ɞɭɠɰɿ ɭ ɜɢɝɥɹɞɿ ɞɪɨɛɿɜ, ɚ ɩɨɬɿɦ
ɡɧɚɣɞɟɦɨ ʀɯ ɱɚɫɬɤɭ:
1) 1
1
a
a
+
+
= 1
1
a a
a
+ +
+
= 2 1;
1
a
a
+
+
2)
2
2
31
1
a
a
−
−
=
2 2
2
1 3
1
a a
a
− −
−
=
2
2
1 4 ;
1
a
a
−
−
3)
2
2
2 1 1 4:
1 1
a a
a a
+ −
+ −
=
2
2
2 1 1
1 1 4
a a
a a
+ −⋅
+ −
=
(2 1) (1 )(1 )
( 1) (1 2 )(1 2 )
a a a
a a a
+ ⋅ − +
+ ⋅ − +
= 1 .
1 2
−
−
a
a
ɉɪɨɜɟɞɟɧɿ ɩɟɪɟɬɜɨɪɟɧɧɹ ɦɨɠɧɚ ɡɚɩɢɫɭɜɚɬɢ ɜ ɪɹɞɨɤ:
( )
2
2
31 : 1
1 1
a a
a a
§ ·
+ −¨ ¸+ −© ¹
=
2 2
2
1 1 3:
1 1
a a a a
a a
+ + − −
+ −
=
2
2
2 1 1 4:
1 1
a a
a a
+ −
+ −
=
=
2
2
2 1 1
1 1 4
a a
a a
+ −⋅
+ −
=
(2 1) (1 )(1 )
( 1) (1 2 )(1 2 )
a a a
a a a
+ ⋅ − +
+ ⋅ − +
= 1 .
1 2
−
−
a
a
•
7. Ɍɨɬɨɠɧɿ ɩɟɪɟɬɜɨɪɟɧɧɹ ɪɚɰɿɨɧɚɥɶɧɢɯ ɜɢɪɚɡɿɜ 49
Ɋɚɰɿɨɧɚɥɶɧɢɣ ɜɢɪɚɡ ɭ ɩɪɢɤɥɚɞɿ 1 ɦɢ ɡɜɟɥɢ ɞɨ ɪɚɰɿɨɧɚɥɶɧɨɝɨ ɞɪɨɛɭ
1 .
1 2
−
−
a
a
ȼɡɚɝɚɥɿ, ɛɭɞɶ-ɹɤɢɣ ɪɚɰɿɨɧɚɥɶɧɢɣ ɜɢɪɚɡ ɦɨɠɧɚ ɩɨɞɚɬɢ ɭ ɜɢɝɥɹɞɿ ɪɚɰɿɨ-
ɧɚɥɶɧɨɝɨ ɞɪɨɛɭ.
ɉɪɢɤɥɚɞ 2. Ⱦɨɜɟɫɬɢ, ɳɨ ɞɥɹ ɜɫɿɯ ɞɨɩɭɫɬɢɦɢɯ ɡɧɚɱɟɧɶ ɡɦɿɧɧɢɯ ɜɢɪɚɡ
2
2 2
2 2 3
:
2
xy y x y x y
x y x yx y
− + +
+
+ +−
ɧɚɛɭɜɚɽ ɬɨɝɨ ɫɚɦɨɝɨ ɡɧɚɱɟɧɧɹ.
• ɋɩɪɨɫɬɢɦɨ ɞɚɧɢɣ ɜɢɪɚɡ:
2
2 2
2 2 3
:
2
xy y x y x y
x y x yx y
− + +
+
+ +−
=
( ) 2 3
( )( ) 2 2
y x y x y x y
x y x y x y x y
− + +
⋅ +
− + + +
=
=
2 3
2 2
y x y
x y x y
+
+
+ +
=
2 4
2
x y
x y
+
+
=
2( 2 )
2
x y
x y
+
+
= 2.
Ɉɬɠɟ, ɞɥɹ ɜɫɿɯ ɞɨɩɭɫɬɢɦɢɯ ɡɧɚɱɟɧɶ ɡɦɿɧɧɢɯ ɡɧɚɱɟɧɧɹ ɜɢɪɚɡɭ ɞɨɪɿɜɧɸɽ
ɬɨɦɭ ɫɚɦɨɦɭ ɱɢɫɥɭ (ɱɢɫɥɭ 2). •
ɉɪɢɤɥɚɞ 3. Ⱦɨɜɟɫɬɢ ɬɨɬɨɠɧɿɫɬɶ
1 1
.
1 1
b aa b
b a
a b
+
+=
−−
• ɋɩɪɨɫɬɢɦɨ ɥɿɜɭ ɱɚɫɬɢɧɭ ɪɿɜɧɨɫɬɿ:
1 1
1 1
a b
a b
+
−
= ( ) ( )1 1 1 1:
a b a b
+ − = :b a b a
ab ab
+ − = b a ab
ab b a
+ ⋅
−
= .b a
b a
+
−
ɒɥɹɯɨɦ ɬɨɬɨɠɧɢɯ ɩɟɪɟɬɜɨɪɟɧɶ ɥɿɜɭ ɱɚɫɬɢɧɭ ɪɿɜɧɨɫɬɿ ɡɜɟɥɢ ɞɨ ɩɪɚɜɨʀ
ɱɚɫɬɢɧɢ. Ɍɨɦɭ ɰɹ ɪɿɜɧɿɫɬɶ ɽ ɬɨɬɨɠɧɿɫɬɸ. •
ɋɩɪɨɫɬɿɬɶ ɜɢɪɚɡ:
173. ɚ) ( )
2
1 11 : ;
3
−+ a
a
ɛ) ( )1 1 5: ;
5 5 5a a a
−
+ − +
50 § 1. Ɋɚɰɿɨɧɚɥɶɧɿ ɜɢɪɚɡɢ
ɜ)
2
2
49 1 1 ;
7
a
a aa
− ⋅ −
+
ɝ) ( )2 1 6: ;
2 2 1 2
b
b b b
−
− − −
ɞ)
4 3
2
: ;
2 8 48 16
a a a
a aa a
−
− −− +
ɟ) .
x yx x
x y x y xy
+§ ·
− ⋅¨ ¸− +© ¹
174. ɚ) ( ) 2
1 41 ;
2 1b b b
− ⋅
− +
ɛ)
2
5 100 1 ;
5 10
a a
a a a
− −− ⋅
− −
ɜ)
2 2
2 9 3 ;
3 2 4
x x x
x x
+ ⋅ −
+ −
ɝ) ( )4 2 8: .
2 1 2
c
c c c
−
+ + +
175. ɋɩɪɨɫɬɿɬɶ ɜɢɪɚɡ
2 2
12 6
2 1 3
x x x
x x
+ ⋅ −
+ +
ɿ ɡɧɚɣɞɿɬɶ ɣɨɝɨ ɡɧɚɱɟɧɧɹ, ɹɤɳɨ ɯ = 6.
176. ɋɩɪɨɫɬɿɬɶ ɜɢɪɚɡ ( ) 9:
9 9 9
c c c
c c c
−
− + +
ɿ ɡɧɚɣɞɿɬɶ ɣɨɝɨ ɡɧɚɱɟɧɧɹ, ɹɤɳɨ ɫ = 11.
Ⱦɨɜɟɞɿɬɶ ɬɨɬɨɠɧɿɫɬɶ:
177. ɚ) 2
: 1;
9 81 918 81
a a a
a aa a
+ =
+ ++ +
ɛ)
6 4 2
2
2 1: 1.
3 3 12 1
b b b b
b bb b
+− = +
− −− +
178. ɚ) 4 1 1: 2 2 ;x x y
x y x y x y
§ ·
+ = −¨ ¸+ − +© ¹
ɛ)
2
2
5 10 3 15 5.
2
a a a
a aa
+ +⋅ − =
+
ɋɩɪɨɫɬɿɬɶ ɜɢɪɚɡ:
179. ɚ)
3 2
2 2 2 2 2
;
( )
§ ·−− ⋅ −¨ ¸− + − −© ¹
m m mn m n
n m m n m n m n
ɛ)
2
2 2
3 3 ;
3 2 3 3 9
a a a a a
a a a a a
+ +§ ·+ ⋅ −¨ ¸− + − −© ¹
7. Ɍɨɬɨɠɧɿ ɩɟɪɟɬɜɨɪɟɧɧɹ ɪɚɰɿɨɧɚɥɶɧɢɯ ɜɢɪɚɡɿɜ 51
ɜ)
2
2 2 2 2
2 4 2 1: ;
2 24 4 4
a a a
a b b aa ab b a b
§ · § ·− +¨ ¸¨ ¸+ −+ + −© ¹© ¹
ɝ) ( )
2
2 2
( )1 1 2 1 1 : ;
a b
a b a b aba b
+§ ·+ + ⋅ +¨ ¸+© ¹
ɞ) ( )
2
3 2
2 1 1 51 .
11 1
§ ·+ + + +− ⋅ + −¨ ¸ +− + +© ¹
x x x x x
x xx x x
180. ɚ)
22
2 2 2 2
: 1 ;
xy yx x
x yx y x y
− § ·
− −¨ ¸++ − © ¹
ɛ)
2
2 2 2
2 3 3 1 3 2: ;
2 4 4 4 4
x x x
x x x x x
+ + +§ ·+ +¨ ¸+ − − + −© ¹
ɜ) 2 3 3
2 6 13 15 5: ;
2 4 8 2 16
x x x
x x x x
− − −§ ·−¨ ¸
− + + +© ¹
ɝ)
4 2
3 2 2
49 3 27 40 .
427 3 9 16
a a a a
aa a a a
+ + −§ ·− ⋅ +¨ ¸ ++ − + −© ¹
181. ɚ)
1
;
11
a
b
ab
+
+
ɛ)
96
;
31
x
x
x
− +
−
ɜ)
1 1
;
1 1
m n m n
m n m n
+
+ −
−
+ −
ɝ)
2
2
.
1
m a
ma
m a
a m
+
+ −
182. ɚ) ;
1 1
a b
b a
a b
−
−
ɛ)
1 1
1 1.
1 1
1 1
c c
c c
−
− +
+
− +
183. Ⱦɨɜɟɞɿɬɶ, ɳɨ ɞɥɹ ɜɫɿɯ ɞɨɩɭɫɬɢɦɢɯ ɡɧɚɱɟɧɶ ɡɦɿɧɧɢɯ ɡɧɚɱɟɧɧɹ ɜɢɪɚɡɭ
a b a a b a
a b b a b b
+ ++ − ⋅
− −
ɞɨɪɿɜɧɸɽ ɬɨɦɭ ɫɚɦɨɦɭ ɱɢɫɥɭ.
184. Ⱦɨɜɟɞɿɬɶ, ɳɨ ɡɧɚɱɟɧɧɹ ɜɢɪɚɡɭ
2 2 2 2
2 2
:m n m nm n mn
m n m n
§ · § ·
− −¨ ¸ ¨ ¸+ +© ¹ © ¹
ɽ ɞɨɞɚɬɧɢɦ
ɱɢɫɥɨɦ ɞɥɹ ɛɭɞɶ-ɹɤɢɯ ɞɨɩɭɫɬɢɦɢɯ ɡɧɚɱɟɧɶ ɡɦɿɧɧɢɯ.
52 § 1. Ɋɚɰɿɨɧɚɥɶɧɿ ɜɢɪɚɡɢ
185. Ⱦɨɜɟɞɿɬɶ, ɳɨ ɞɥɹ ɜɫɿɯ ɞɨɩɭɫɬɢɦɢɯ ɡɧɚɱɟɧɶ ɡɦɿɧɧɢɯ ɡɧɚɱɟɧɧɹ ɜɢɪɚɡɭ
( )
2 2
2 2
1 1 4: 1 a b
a b a b a b
§ ·−+ −¨ ¸− + −© ¹
ɧɟ ɡɚɥɟɠɚɬɶ ɜɿɞ ɡɧɚɱɟɧɶ b.
Ⱦɨɜɟɞɿɬɶ ɬɨɬɨɠɧɿɫɬɶ:
186. ɚ)
2 2 2 2
1 · : ;m n m n m n m
mn m n n m m m n
§ ·§ ·− −− − =¨ ¸¨ ¸+ +© ¹© ¹
ɛ)
2 22
2 2 2 2
( 2)2 1 3 1· ;
3 4 4
a aa a a
a a a a a a
§ ·+ −− + −− =¨ ¸
− − −© ¹
ɜ) ( )2 2
2 1: 2 .b a b a
a b a b a ba ab b ab
§ ·− + − + =¨ ¸+ ++ +© ¹
187. ɚ) ( )3 2
1 3 3 2 1 1;
1 11 1
aa
a aa a a
−§ ·− + ⋅ − =¨ ¸+ ++ − +© ¹
ɛ) 2 2 2 2
: .
2
a a a a a n
a n a n a na n an a n
−§ · § ·− − =¨ ¸ ¨ ¸+ − ++ + −© ¹ © ¹
188. ɋɩɪɨɫɬɿɬɶ ɜɢɪɚɡ:
ɚ) 2 2 3
1 3 3: : ;
9 3 39 3 9 27
b a a a
b a aba a a b
−§ ·−¨ ¸− − +− + −© ¹
ɛ)
2
2 2
10 3 211 2 .
53 ( 5 )
y xy y
x yx xy x y
§ · § ·−
+ ⋅ ⋅ +¨ ¸ ¨ ¸−− −© ¹ © ¹
189. Ɂɧɚɣɞɿɬɶ ɡɧɚɱɟɧɧɹ ɜɢɪɚɡɭ:
ɚ) 2
2
1 ,x
x
+ ɹɤɳɨ 1 2,5;x
x
+ = ɛ) 2
2
1 ,
4
x
x
+ ɹɤɳɨ 1 0,5.
2
x
x
− = −
190. Ⱦɨɜɟɞɿɬɶ ɬɨɬɨɠɧɿɫɬɶ:
ɚ) 2 2 3 2
( 3)3 4 1 1 ;
1 12 1 1 3 3 2 1
x xx
x xx x x x x x x
+§ ·−§ ·+ ⋅ + =¨ ¸ ¨ ¸+ −+ + − + − − +© ¹ © ¹
7. Ɍɨɬɨɠɧɿ ɩɟɪɟɬɜɨɪɟɧɧɹ ɪɚɰɿɨɧɚɥɶɧɢɯ ɜɢɪɚɡɿɜ 53
ɛ) ( )
22
2 3 2
2 1 3 21 .
2 312 4 3 6 5
a a a a a
a aa a a a a
+ − − −− = ⋅ −
− −− − + − +
191. ɉɨɞɚɣɬɟ ɭ ɜɢɝɥɹɞɿ ɪɚɰɿɨɧɚɥɶɧɨɝɨ ɞɪɨɛɭ: 1 .
11
11
1 x
+
−
−
192. Ɋɨɡɜ’ɹɠɿɬɶ ɪɿɜɧɹɧɧɹ:
ɚ) (ɯ – 1)(ɯ2
+ ɯ + 1) – ɯ3
– ɯ2
= 2ɯ; ɛ) (ɯ + 2)2
– 4 = 0;
ɜ) 4 1;
2 3
x x +− = ɝ)
3 3
6.
5 4
− +
+ =
y y
193. Ⱦɥɹ ɹɤɢɯ ɡɧɚɱɟɧɶ ɚ ɪɿɜɧɹɧɧɹ (ɚ2
– 3ɚ)ɯ = 2ɚ – 6 ɧɟ ɦɚɽ ɤɨɪɟɧɿɜ? ɦɚɽ ɨɞɢɧ
ɤɨɪɿɧɶ?
194. Ɂ ɩɭɧɤɬɭ A ɞɨ ɩɭɧɤɬɭ B, ɜɿɞɫɬɚɧɶ ɦɿɠ ɹɤɢɦɢ ɞɨɪɿɜɧɸɽ 360 ɤɦ, ɜɢɣɲɨɜ
ɬɨɜɚɪɧɢɣ ɩɨʀɡɞ ɿ ɪɭɯɚɜɫɹ ɡɿ ɲɜɢɞɤɿɫɬɸ 50 ɤɦ/ɝɨɞ. ɑɟɪɟɡ 40 ɯɜ ɧɚɡɭɫɬɪɿɱ
ɣɨɦɭ ɡ ɩɭɧɤɬɭ B ɜɢɣɲɨɜ ɩɚɫɚɠɢɪɫɶɤɢɣ ɩɨʀɡɞ ɿ ɪɭɯɚɜɫɹ ɡɿ ɲɜɢɞɤɿɫɬɸ
90 ɤɦ/ɝɨɞ. ɇɚ ɹɤɿɣ ɜɿɞɫɬɚɧɿ ɜɿɞ ɩɭɧɤɬɭ A ɩɨʀɡɞɢ ɡɭɫɬɪɿɥɢɫɹ?
195*. ɉɨ ɤɪɭɝɨɜɿɣ ɞɨɪɿɠɰɿ ɜɟɥɨɬɪɟɤɭ ʀɞɭɬɶ ɞɜɚ ɜɟɥɨɫɢɩɟɞɢɫɬɢ ɡɿ ɫɬɚɥɢɦɢ
ɲɜɢɞɤɨɫɬɹɦɢ. Ʉɨɥɢ ɜɨɧɢ ʀɞɭɬɶ ɭ ɩɪɨɬɢɥɟɠɧɢɯ ɧɚɩɪɹɦɤɚɯ, ɬɨ ɡɭɫɬɪɿɱɚ-
ɸɬɶɫɹ ɱɟɪɟɡ ɤɨɠɧɿ 10 ɫ; ɤɨɥɢ ɠ ʀɞɭɬɶ ɜ ɨɞɧɨɦɭ ɧɚɩɪɹɦɤɭ, ɬɨ ɨɞɢɧ ɧɚɡɞɨ-
ɝɚɧɹɽ ɿɧɲɨɝɨ ɱɟɪɟɡ ɤɨɠɧɿ 100 ɫ. əɤɚ ɲɜɢɞɤɿɫɬɶ ɤɨɠɧɨɝɨ ɜɟɥɨɫɢɩɟɞɢɫɬɚ,
ɹɤɳɨ ɞɨɜɠɢɧɚ ɞɨɪɿɠɤɢ ɞɨɪɿɜɧɸɽ 200 ɦ?
196. ɇɚ ɞɨɲɰɿ ɧɚɩɢɫɚɧɿ ɱɢɫɥɚ 1, 2, 3, …, 25. Ⱦɨɡɜɨɥɹɽɬɶɫɹ ɫɬɟɪɬɢ ɛɭɞɶ-ɹɤɿ ɞɜɚ
ɱɢɫɥɚ ɿ ɧɚɩɢɫɚɬɢ ʀɯ ɞɨɛɭɬɨɤ. ɉɨɜɬɨɪɢɜɲɢ ɬɚɤɭ ɨɩɟɪɚɰɿɸ 24 ɪɚɡɢ, ɨɞɟɪ-
ɠɢɦɨ ɨɞɧɟ ɱɢɫɥɨ. Ⱦɨɜɟɞɿɬɶ, ɳɨ ɰɟ ɱɢɫɥɨ ɞɿɥɢɬɶɫɹ ɧɚ 1 000 000.
54 § 1. Ɋɚɰɿɨɧɚɥɶɧɿ ɜɢɪɚɡɢ
1. ɐɿɥɿ ɬɚ ɞɪɨɛɨɜɿ ɪɚɰɿɨɧɚɥɶɧɿ ɪɿɜɧɹɧɧɹ. Ɋɨɡɝɥɹɧɟɦɨ ɪɿɜɧɹɧɧɹ:
2(ɯ – 7) = 3ɯ – 9; 6 4;
9
x
x
=
−
5 3
1 4
.
x x
=
− −
Ʌɿɜɚ ɿ ɩɪɚɜɚ ɱɚɫɬɢɧɢ ɤɨɠɧɨɝɨ ɡ ɰɢɯ ɪɿɜɧɹɧɶ ɽ ɪɚɰɿɨɧɚɥɶɧɢɦɢ ɜɢɪɚɡɚɦɢ.
Ɍɚɤɿ ɪɿɜɧɹɧɧɹ ɧɚɡɢɜɚɸɬɶ ɪɚɰɿɨɧɚɥɶɧɢɦɢ ɪɿɜɧɹɧɧɹɦɢ.
Ɉɡɧɚɱɟɧɧɹ
Ɋɿɜɧɹɧɧɹ, ɥɿɜɚ ɿ ɩɪɚɜɚ ɱɚɫɬɢɧɢ ɹɤɢɯ ɽ ɪɚɰɿɨɧɚɥɶɧɢɦɢ ɜɢ-
ɪɚɡɚɦɢ, ɧɚɡɢɜɚɸɬɶ ɪɚɰɿɨɧɚɥɶɧɢɦɢ ɪɿɜɧɹɧɧɹɦɢ.
Ɋɚɰɿɨɧɚɥɶɧɿ ɪɿɜɧɹɧɧɹ ɩɨɞɿɥɹɸɬɶ ɧɚ ɰɿɥɿ ɣ ɞɪɨɛɨɜɿ. əɤɳɨ ɨɛɢɞɜɿ ɱɚɫɬɢɧɢ
ɪɚɰɿɨɧɚɥɶɧɨɝɨ ɪɿɜɧɹɧɧɹ ɽ ɰɿɥɢɦɢ ɜɢɪɚɡɚɦɢ, ɬɨ ɬɚɤɟ ɪɿɜɧɹɧɧɹ ɧɚɡɢɜɚɸɬɶ ɰɿɥɢɦ
ɪɚɰɿɨɧɚɥɶɧɢɦ ɪɿɜɧɹɧɧɹɦ. Ɋɚɰɿɨɧɚɥɶɧɟ ɪɿɜɧɹɧɧɹ, ɭ ɹɤɨɝɨ ɯɨɱɚ ɛ ɨɞɧɚ ɱɚɫɬɢɧɚ ɽ
ɞɪɨɛɨɜɢɦ ɜɢɪɚɡɨɦ, ɧɚɡɢɜɚɸɬɶ ɞɪɨɛɨɜɢɦ ɪɚɰɿɨɧɚɥɶɧɢɦ ɪɿɜɧɹɧɧɹɦ.
2(ɯ – 7) = 3ɯ – 9 — ɰɿɥɟ ɪɚɰɿɨɧɚɥɶɧɟ ɪɿɜɧɹɧɧɹ;
3( 2) 2( 1)
5 3
y y− +
= — ɰɿɥɟ ɪɚɰɿɨɧɚɥɶɧɟ ɪɿɜɧɹɧɧɹ;
6 4
9
x
x
=
−
— ɞɪɨɛɨɜɟ ɪɚɰɿɨɧɚɥɶɧɟ ɪɿɜɧɹɧɧɹ;
5 3
1 4x x
=
− −
— ɞɪɨɛɨɜɟ ɪɚɰɿɨɧɚɥɶɧɟ ɪɿɜɧɹɧɧɹ.
2. Ɋɨɡɜ’ɹɡɭɜɚɧɧɹ ɞɪɨɛɨɜɢɯ ɪɚɰɿɨɧɚɥɶɧɢɯ ɪɿɜɧɹɧɶ ɧɚ ɨɫɧɨɜɿ ɭɦɨɜɢ
ɪɿɜɧɨɫɬɿ ɞɪɨɛɭ ɧɭɥɸ. ɉɪɢɝɚɞɚɣɦɨ: ɞɪɿɛ ɞɨɪɿɜɧɸɽ ɧɭɥɸ ɬɨɞɿ ɣ ɬɿɥɶɤɢ ɬɨɞɿ,
ɤɨɥɢ ɣɨɝɨ ɱɢɫɟɥɶɧɢɤ ɞɨɪɿɜɧɸɽ ɧɭɥɸ, ɚ ɡɧɚɦɟɧɧɢɤ ɜɿɞɦɿɧɧɢɣ ɜɿɞ ɧɭɥɹ.
0=a
b
ɬɨɞɿ ɣ ɬɿɥɶɤɢ ɬɨɞɿ, ɤɨɥɢ ɚ = 0 ɿ b ≠ 0.
Ⱦɚɧɟ ɬɜɟɪɞɠɟɧɧɹ ɦɨɠɧɚ ɜɢɤɨɪɢɫɬɨɜɭɜɚɬɢ ɞɥɹ ɪɨɡɜ’ɹɡɭɜɚɧɧɹ ɞɪɨɛɨɜɢɯ
ɪɚɰɿɨɧɚɥɶɧɢɯ ɪɿɜɧɹɧɶ. Ɋɨɡɝɥɹɧɟɦɨ ɩɪɢɤɥɚɞɢ.
ɉɪɢɤɥɚɞ 1. Ɋɨɡɜ’ɹɡɚɬɢ ɪɿɜɧɹɧɧɹ
2
2 0.
2
x x
x
− =
−
• ȼɢɤɨɪɢɫɬɚɽɦɨ ɭɦɨɜɭ, ɡɚ ɹɤɨʀ ɞɪɿɛ ɞɨɪɿɜɧɸɽ ɧɭɥɸ. ɉɪɢɪɿɜɧɹɽɦɨ ɱɢ-
ɫɟɥɶɧɢɤ ɞɪɨɛɭ ɞɨ ɧɭɥɹ:
ɯ2
– 2ɯ = 0; ɯ(ɯ – 2) = 0; ɯ = 0 ɚɛɨ ɯ = 2.
8. Ɋɚɰɿɨɧɚɥɶɧɿ ɪɿɜɧɹɧɧɹ 55
ɉɟɪɟɜɿɪɢɦɨ, ɱɢ ɞɥɹ ɡɧɚɣɞɟɧɢɯ ɡɧɚɱɟɧɶ ɯ ɡɧɚɦɟɧɧɢɤ ɯ – 2 ɜɿɞɦɿɧɧɢɣ ɜɿɞ ɧɭɥɹ.
əɤɳɨ ɯ = 0, ɬɨ ɯ – 2 = 0 – 2 = –2 ≠ 0. Ɍɨɦɭ ɯ = 0 — ɤɨɪɿɧɶ ɪɿɜɧɹɧɧɹ.
əɤɳɨ ɯ = 2, ɬɨ ɯ – 2 = 2 – 2 = 0. Ɍɨɦɭ ɯ = 2 — ɧɟ ɽ ɤɨɪɟɧɟɦ ɪɿɜɧɹɧɧɹ.
ȼɿɞɩɨɜɿɞɶ. 0. •
ɉɪɢɤɥɚɞ 2. Ɋɨɡɜ’ɹɡɚɬɢ ɪɿɜɧɹɧɧɹ 6 4
9
=
−
x .
x
• Ɂɜɟɞɟɦɨ ɞɚɧɟ ɪɿɜɧɹɧɧɹ ɞɨ ɪɿɜɧɹɧɧɹ, ɥɿɜɚ ɱɚɫɬɢɧɚ ɹɤɨɝɨ ɽ ɞɪɨɛɨɦ, ɚ
ɩɪɚɜɚ — ɧɭɥɟɦ:
6 4;
9
x
x
=
−
6 4 0;
9
x
x
− =
−
6 4( 9)
0;
9
x x
x
− −
=
−
2 36 0
9
x .
x
+ =
−
ɉɪɢɪɿɜɧɹɽɦɨ ɱɢɫɟɥɶɧɢɤ ɞɪɨɛɭ 2 36
9
x
x
+
−
ɞɨ ɧɭɥɹ:
2ɯ + 36 = 0; 2ɯ = –36; ɯ = –18.
əɤɳɨ ɯ = –18, ɬɨ ɡɧɚɦɟɧɧɢɤ ɯ – 9 ɞɪɨɛɭ ɜɿɞɦɿɧɧɢɣ ɜɿɞ ɧɭɥɹ. ɋɩɪɚɜɞɿ:
ɯ – 9 = –18 – 9 = –27 ≠ 0.
Ɉɬɠɟ, ɯ = –18 — ɤɨɪɿɧɶ ɞɚɧɨɝɨ ɪɿɜɧɹɧɧɹ.
ȼɿɞɩɨɜɿɞɶ. –18. •
ɓɨɛ ɪɨɡɜ’ɹɡɚɬɢ ɞɪɨɛɨɜɟ ɪɚɰɿɨɧɚɥɶɧɟ ɪɿɜɧɹɧɧɹ ɧɚ ɨɫɧɨɜɿ ɭɦɨɜɢ ɪɿɜɧɨɫɬɿ
ɞɪɨɛɭ ɧɭɥɸ, ɩɨɬɪɿɛɧɨ:
1) ɡɜɟɫɬɢ ɣɨɝɨ ɞɨ ɜɢɝɥɹɞɭ
( )
0,
( )
f x
g x
= ɞɟ f(x) ɿ g(x) — ɰɿɥɿ ɪɚɰɿɨɧɚɥɶɧɿ
ɜɢɪɚɡɢ;
2) ɩɪɢɪɿɜɧɹɬɢ ɞɨ ɧɭɥɹ ɱɢɫɟɥɶɧɢɤ ɞɪɨɛɭ ɣ ɪɨɡɜ’ɹɡɚɬɢ ɨɞɟɪɠɚɧɟ ɰɿɥɟ ɪɚ-
ɰɿɨɧɚɥɶɧɟ ɪɿɜɧɹɧɧɹ f(x) = 0;
3) ɜɢɤɥɸɱɢɬɢ ɡ ɣɨɝɨ ɤɨɪɟɧɿɜ ɬɿ, ɞɥɹ ɹɤɢɯ ɡɧɚɦɟɧɧɢɤ ɞɪɨɛɭ ɞɨɪɿɜɧɸɽ ɧɭɥɸ.
3. Ɋɿɜɧɨɫɢɥɶɧɿɫɬɶ ɪɿɜɧɹɧɶ. Ɋɨɡɜ’ɹɡɭɸɱɢ ɩɪɢɤɥɚɞ 1, ɦɢ ɦɚɥɢ ɥɚɧɰɸɠɨɤ
ɪɿɜɧɹɧɶ
Ɋɿɜɧɹɧɧɹ
2
2 0
2
x x
x
− =
−
ɯ2
– 2ɯ = 0 ɯ(ɯ – 2) = 0
Ʉɨɪɟɧɿ 0 0; 2 0; 2
ɉɟɪɲɟ ɡ ɰɢɯ ɪɿɜɧɹɧɶ ɦɚɽ ɨɞɢɧ ɤɨɪɿɧɶ — ɱɢɫɥɨ 0, ɞɪɭɝɟ ɬɚ ɬɪɟɬɽ ɪɿɜɧɹɧ-
ɧɹ ɦɚɸɬɶ ɞɜɚ ɬɿ ɫɚɦɿ ɤɨɪɟɧɿ — ɱɢɫɥɚ 0 ɿ 2.
56 § 1. Ɋɚɰɿɨɧɚɥɶɧɿ ɜɢɪɚɡɢ
Ɉɡɧɚɱɟɧɧɹ
Ⱦɜɚ ɪɿɜɧɹɧɧɹ, ɹɤɿ ɦɚɸɬɶ ɬɿ ɫɚɦɿ ɤɨɪɟɧɿ, ɧɚɡɢɜɚɸɬɶ ɪɿɜɧɨɫɢ-
ɥɶɧɢɦɢ. Ⱦɜɚ ɪɿɜɧɹɧɧɹ, ɹɤɿ ɧɟ ɦɚɸɬɶ ɤɨɪɟɧɿɜ, ɬɟɠ ɜɜɚɠɚɸɬɶ
ɪɿɜɧɨɫɢɥɶɧɢɦɢ.
Ɉɬɠɟ,
ɪɿɜɧɹɧɧɹ ɯ2
– 2ɯ = 0 ɿ ɯ(ɯ – 2) = 0 ɪɿɜɧɨɫɢɥɶɧɿ;
ɪɿɜɧɹɧɧɹ
2
2 0
2
x x
x
− =
−
ɿ ɯ2
– 2ɯ = 0 ɧɟ ɪɿɜɧɨɫɢɥɶɧɿ.
Ɋɿɜɧɹɧɧɹ ɯ + 6 = ɯ ɿ 0ɯ = 1 ɪɿɜɧɨɫɢɥɶɧɿ, ɛɨ ɤɨɠɧɟ ɡ ɧɢɯ ɧɟ ɦɚɽ ɤɨɪɟɧɿɜ.
Ɉɫɤɿɥɶɤɢ ɪɨɡɜ’ɹɡɭɜɚɧɧɹ ɪɿɜɧɹɧɧɹ
2
2 0
2
x x
x
− =
−
ɡɜɨɞɢɬɶɫɹ ɞɨ ɪɨɡɜ’ɹɡɭɜɚɧ-
ɧɹ ɪɿɜɧɹɧɧɹ ɯ2
– 2ɯ = 0 ɿ ɩɟɪɟɜɿɪɤɢ ɭɦɨɜɢ ɯ – 2 ≠ 0, ɬɨ ɤɚɠɭɬɶ, ɳɨ ɪɿɜɧɹɧɧɹ
2
2 0
2
x x
x
− =
−
ɪɿɜɧɨɫɢɥɶɧɟ ɫɢɫɬɟɦɿ
2
2 0;
2 0.
x x
x
­ − =
®
− ≠¯
Ɋɨɡɜ’ɹɡɤɨɦ ɰɿɽʀ ɫɢɫɬɟɦɢ, ɹɤ ɦɢ
ɜɠɟ ɡ’ɹɫɭɜɚɥɢ, ɽ ɱɢɫɥɨ ɯ = 0.
Ɋɿɜɧɹɧɧɹ
( )
0
( )
f x
g x
= ɪɿɜɧɨɫɢɥɶɧɟ ɫɢɫɬɟɦɿ
( ) 0;
( ) 0.
f x
g x
=­
®
≠¯
ɍ 7 ɤɥɚɫɿ ɦɢ ɪɨɡɝɥɹɞɚɥɢ ɩɟɪɟɬɜɨɪɟɧɧɹ ɪɿɜɧɹɧɶ, ɜɢɤɨɧɭɸɱɢ ɹɤɿ, ɨɞɟɪɠɭ-
ɸɬɶ ɪɿɜɧɹɧɧɹ ɡ ɬɢɦɢ ɫɚɦɢɦɢ ɤɨɪɟɧɹɦɢ. Ɉɬɠɟ, ɰɿ ɩɟɪɟɬɜɨɪɟɧɧɹ ɩɟɪɟɜɨɞɹɬɶ ɪɿ-
ɜɧɹɧɧɹ ɜ ɪɿɜɧɨɫɢɥɶɧɟ ɣɨɦɭ ɪɿɜɧɹɧɧɹ. Ɂ ɧɢɦɢ ɩɨɜ’ɹɡɚɧɿ ɬɚɤɿ ɨɫɧɨɜɧɿ ɜɥɚɫɬɢɜɨ-
ɫɬɿ ɪɿɜɧɹɧɶ:
ȼɥɚɫɬɢɜɿɫɬɶ 1. əɤɳɨ ɜ ɞɟɹɤɿɣ ɱɚɫɬɢɧɿ ɪɿɜɧɹɧɧɹ ɜɢɤɨɧɚɬɢ ɬɨɬɨɠɧɟ ɩɟ-
ɪɟɬɜɨɪɟɧɧɹ, ɹɤɟ ɧɟ ɡɦɿɧɸɽ ɞɨɩɭɫɬɢɦɿ ɡɧɚɱɟɧɧɹ ɡɦɿɧɧɨʀ, ɬɨ ɨɞɟɪɠɢɦɨ ɪɿɜɧɹɧɧɹ,
ɪɿɜɧɨɫɢɥɶɧɟ ɞɚɧɨɦɭ.
ȼɥɚɫɬɢɜɿɫɬɶ 2. əɤɳɨ ɞɟɹɤɢɣ ɞɨɞɚɧɨɤ ɩɟɪɟɧɟɫɬɢ ɡ ɨɞɧɿɽʀ ɱɚɫɬɢɧɢ ɪɿɜ-
ɧɹɧɧɹ ɜ ɿɧɲɭ, ɡɦɿɧɢɜɲɢ ɣɨɝɨ ɡɧɚɤ ɧɚ ɩɪɨɬɢɥɟɠɧɢɣ, ɬɨ ɨɞɟɪɠɢɦɨ ɪɿɜɧɹɧɧɹ,
ɪɿɜɧɨɫɢɥɶɧɟ ɞɚɧɨɦɭ.
ȼɥɚɫɬɢɜɿɫɬɶ 3. əɤɳɨ ɨɛɢɞɜɿ ɱɚɫɬɢɧɢ ɪɿɜɧɹɧɧɹ ɩɨɦɧɨɠɢɬɢ ɚɛɨ ɩɨɞɿɥɢ-
ɬɢ ɬɟ ɫɚɦɟ, ɜɿɞɦɿɧɧɟ ɜɿɞ ɧɭɥɹ ɱɢɫɥɨ, ɬɨ ɨɞɟɪɠɢɦɨ ɪɿɜɧɹɧɧɹ, ɪɿɜɧɨɫɢɥɶɧɟ ɞɚ-
ɧɨɦɭ.
8. Ɋɚɰɿɨɧɚɥɶɧɿ ɪɿɜɧɹɧɧɹ 57
4. Ɇɧɨɠɟɧɧɹ ɨɛɨɯ ɱɚɫɬɢɧ ɪɿɜɧɹɧɧɹ ɧɚ ɜɢɪɚɡ ɡɿ ɡɦɿɧɧɨɸ. Ɋɨɡɝɥɹɧɟɦɨ
ɩɪɢɤɥɚɞ.
ɉɪɢɤɥɚɞ 3. Ɋɨɡɜ’ɹɡɚɬɢ ɪɿɜɧɹɧɧɹ 2
1 1 6 .
3 3 9
y
y y y
+
− =
− + −
• Ɉɫɤɿɥɶɤɢ ɭ2
– 9 = (ɭ – 3)(ɭ + 3), ɬɨ ɫɩɿɥɶɧɢɦ ɡɧɚɦɟɧɧɢɤɨɦ ɭɫɿɯ ɞɪɨɛɿɜ,
ɹɤɿ ɜɯɨɞɹɬɶ ɭ ɪɿɜɧɹɧɧɹ, ɽ (ɭ – 3)(ɭ + 3). ɉɨɦɧɨɠɢɜɲɢ ɨɛɢɞɜɿ ɱɚɫɬɢɧɢ ɪɿɜɧɹɧɧɹ
ɧɚ ɫɩɿɥɶɧɢɣ ɡɧɚɦɟɧɧɢɤ, ɡɚ ɭɦɨɜɢ, ɳɨ (ɭ – 3)(ɭ + 3) ≠ 0, ɦɚɬɢɦɟɦɨ:
2
1 1 6 ( 3)( 3);
3 3 9
y
y y
y y y
+
− = ⋅ − +
− + −
(ɭ + 3)(ɭ + 1) – (ɭ – 3) = 6;
ɭ2
+ ɭ + 3ɭ + 3 – ɭ + 3 – 6 = 0;
ɭ2
+ 3ɭ = 0; ɭ(ɭ + 3) = 0; ɭ = 0 ɚɛɨ ɭ = –3.
əɤɳɨ ɭ = 0, ɬɨ (ɭ – 3)(ɭ + 3) = –3 ⋅ 3 ≠ 0. Ɍɨɦɭ ɭ = 0 — ɤɨɪɿɧɶ ɪɿɜɧɹɧɧɹ.
əɤɳɨ ɭ = –3, ɬɨ (ɭ – 3)(ɭ + 3) = –6 ⋅ 0 = 0. Ɍɨɦɭ ɭ = –3 — ɧɟ ɽ ɤɨɪɟɧɟɦ ɪɿɜ-
ɧɹɧɧɹ.
ȼɿɞɩɨɜɿɞɶ. 0. •
Ɂɜɟɪɧɟɦɨ ɭɜɚɝɭ, ɳɨ ɪɿɜɧɹɧɧɹ 2
1 1 6
3 3 9
y
y y y
+
− =
− + −
ɦɚɽ ɨɞɢɧ ɤɨɪɿɧɶ
ɭ = 0, ɚ ɨɞɟɪɠɚɧɟ ɭ ɪɨɡɜ’ɹɡɚɧɧɿ ɪɿɜɧɹɧɧɹ (ɭ + 3)(ɭ + 1) – (ɭ – 3) = 6 — ɞɜɚ ɤɨɪɟɧɿ
ɭ = 0 ɬɚ ɭ = –3. Ɉɬɠɟ, ɩɨɦɧɨɠɢɜɲɢ ɨɛɢɞɜɿ ɱɚɫɬɢɧɢ ɞɪɨɛɨɜɨɝɨ ɪɿɜɧɹɧɧɹ ɧɚ ɫɩɿ-
ɥɶɧɢɣ ɡɧɚɦɟɧɧɢɤ, ɦɢ ɧɟ ɜɬɪɚɬɢɥɢ ɣɨɝɨ ɤɨɪɿɧɶ, ɩɪɨɬɟ ɨɞɟɪɠɚɥɢ ɫɬɨɪɨɧɧɿɣ
ɳɨɞɨ ɰɶɨɝɨ ɪɿɜɧɹɧɧɹ ɤɨɪɿɧɶ ɭ = –3.
ɉɪɚɜɢɥɶɧɢɦ ɽ ɬɚɤɟ ɬɜɟɪɞɠɟɧɧɹ:
əɤɳɨ ɨɛɢɞɜɿ ɱɚɫɬɢɧɢ ɞɟɹɤɨɝɨ ɪɿɜɧɹɧɧɹ ɩɨɦɧɨɠɢɬɢ ɧɚ ɰɿɥɢɣ ɜɢɪɚɡ ɡɿ
ɡɦɿɧɧɨɸ, ɬɨ ɦɨɠɧɚ ɨɞɟɪɠɚɬɢ ɪɿɜɧɹɧɧɹ, ɧɟ ɪɿɜɧɨɫɢɥɶɧɟ ɞɚɧɨɦɭ. Ɉɞɟɪɠɚɧɟ
ɪɿɜɧɹɧɧɹ ɦɚɽ ɬɚɤɿ ɜɥɚɫɬɢɜɨɫɬɿ: 1) ɣɨɝɨ ɤɨɪɟɧɹɦɢ ɽ ɜɫɿ ɤɨɪɟɧɿ ɞɚɧɨɝɨ ɪɿɜɧɹɧ-
ɧɹ; 2) ɜɨɧɨ ɦɨɠɟ ɦɚɬɢ ɫɬɨɪɨɧɧɿ ɤɨɪɟɧɿ ɳɨɞɨ ɞɚɧɨɝɨ ɪɿɜɧɹɧɧɹ.
ɋɬɨɪɨɧɧɿɦɢ ɤɨɪɟɧɹɦɢ ɦɨɠɭɬɶ ɛɭɬɢ ɡɧɚɱɟɧɧɹ ɡɦɿɧɧɨʀ, ɞɥɹ ɹɤɢɯ ɰɿɥɢɣ
ɜɢɪɚɡ, ɧɚ ɹɤɢɣ ɦɢ ɦɧɨɠɢɦɨ ɨɛɢɞɜɿ ɱɚɫɬɢɧɢ ɪɿɜɧɹɧɧɹ, ɧɚɛɭɜɚɽ ɡɧɚɱɟɧɧɹ 0. ɐɿ
ɫɬɨɪɨɧɧɿ ɤɨɪɟɧɿ ɦɨɠɧɚ ɜɿɞɤɢɧɭɬɢ, ɡɪɨɛɢɜɲɢ ɩɟɪɟɜɿɪɤɭ.
58 § 1. Ɋɚɰɿɨɧɚɥɶɧɿ ɜɢɪɚɡɢ
ɓɨɛ ɪɨɡɜ’ɹɡɚɬɢ ɞɪɨɛɨɜɟ ɪɚɰɿɨɧɚɥɶɧɟ ɪɿɜɧɹɧɧɹ, ɦɨɠɧɚ:
1) ɩɨɦɧɨɠɢɬɢ ɨɛɢɞɜɿ ɱɚɫɬɢɧɢ ɪɿɜɧɹɧɧɹ ɧɚ ɫɩɿɥɶɧɢɣ ɡɧɚɦɟɧɧɢɤ ɞɪɨɛɿɜ, ɹɤɿ
ɜɯɨɞɹɬɶ ɞɨ ɪɿɜɧɹɧɧɹ, ɿ ɡɚɦɿɧɢɬɢ ɣɨɝɨ ɰɿɥɢɦ ɪɚɰɿɨɧɚɥɶɧɢɦ ɪɿɜɧɹɧɧɹɦ;
2) ɪɨɡɜ’ɹɡɚɬɢ ɨɞɟɪɠɚɧɟ ɰɿɥɟ ɪɚɰɿɨɧɚɥɶɧɟ ɪɿɜɧɹɧɧɹ;
3) ɜɢɤɥɸɱɢɬɢ ɡ ɣɨɝɨ ɤɨɪɟɧɿɜ ɬɿ, ɞɥɹ ɹɤɢɯ ɫɩɿɥɶɧɢɣ ɡɧɚɦɟɧɧɢɤ ɞɪɨɛɿɜ ɞɨ-
ɪɿɜɧɸɽ ɧɭɥɸ.
ȼɩɪɚɜɚ 1. Ɋɨɡɜ’ɹɡɚɬɢ ɪɿɜɧɹɧɧɹ
2
2
2 2 1
22
x x .
x xx x
− + =
++
•
2
2 2 1 0;
( 2) 2
x x
x x x x
− + − =
+ +
2 2
2 2 ( 2)
0;
( 2)
x x x
x x
− + + −
=
+
2
0;
( 2)
x x
x x
+ =
+
2
0;
( 2) 0;
x x
x x
­ + =
®
+ ≠¯
( 1) 0;
( 2) 0;
x x
x x
+ =­
®
+ ≠¯
0 ɚɛɨ 1;
0;
2;
x x
x
x
= = −­
°
≠®
° ≠ −¯
x = –1.
ȼɿɞɩɨɜɿɞɶ. –1. •
ȼɩɪɚɜɚ 2. Ɋɨɡɜ’ɹɡɚɬɢ ɪɿɜɧɹɧɧɹ 2
2 2 5.
2 4
x
x x
+=
− −
• Ɋɨɡɝɥɹɞɚɬɢɦɟɦɨ ɪɿɜɧɿɫɬɶ 2
2 2 5
2 4
x
x x
+=
− −
ɹɤ ɩɪɨɩɨɪɰɿɸ. Ɂɚ ɨɫɧɨɜɧɨɸ
ɜɥɚɫɬɢɜɿɫɬɸ ɩɪɨɩɨɪɰɿʀ ɦɚɽɦɨ:
2(ɯ2
– 4) = (ɯ – 2)(2ɯ + 5), ɡɚ ɭɦɨɜɢ, ɳɨ ɯ – 2 ≠ 0 ɿ ɯ2
– 4 ≠ 0.
Ɋɨɡɜ’ɹɠɟɦɨ ɨɞɟɪɠɚɧɟ ɪɿɜɧɹɧɧɹ:
2ɯ2
– 8 = 2ɯ2
+ 5ɯ – 4ɯ – 10; –8 = ɯ – 10; ɯ = 2.
əɤɳɨ ɯ = 2, ɬɨ ɯ – 2 = 2 – 2 = 0, ɬɨɛɬɨ ɞɥɹ ɯ = 2 ɭɦɨɜɚ ɯ – 2 ≠ 0 ɧɟ ɜɢɤɨ-
ɧɭɽɬɶɫɹ. Ɍɨɦɭ ɯ = 2 — ɧɟ ɤɨɪɿɧɶ ɪɿɜɧɹɧɧɹ.
ȼɿɞɩɨɜɿɞɶ. Ʉɨɪɟɧɿɜ ɧɟɦɚɽ. •
ȼɩɪɚɜɚ 3. Ɂ ɦɿɫɬɚ A ɞɨ ɦɿɫɬɚ B, ɜɿɞɫɬɚɧɶ ɦɿɠ ɹɤɢɦɢ ɞɨɪɿɜɧɸɽ 21 ɤɦ, ɜɢʀɯɚɜ
ɜɟɥɨɫɢɩɟɞɢɫɬ, ɚ ɱɟɪɟɡ 20 ɯɜ ɭɫɥɿɞ ɡɚ ɧɢɦ — ɦɨɬɨɰɢɤɥɿɫɬ, ɲɜɢɞɤɿɫɬɶ
ɹɤɨɝɨ ɭɬɪɢɱɿ ɛɿɥɶɲɚ ɜɿɞ ɲɜɢɞɤɨɫɬɿ ɜɟɥɨɫɢɩɟɞɢɫɬɚ. Ɂɧɚɣɬɢ ɲɜɢɞɤɿɫɬɶ
ɜɟɥɨɫɢɩɟɞɢɫɬɚ, ɹɤɳɨ ɜɿɞɨɦɨ, ɳɨ ɦɨɬɨɰɢɤɥɿɫɬ ɩɪɢʀɯɚɜ ɭ ɦɿɫɬɨ B ɧɚ 40 ɯɜ
ɪɚɧɿɲɟ, ɧɿɠ ɜɟɥɨɫɢɩɟɞɢɫɬ.
8. Ɋɚɰɿɨɧɚɥɶɧɿ ɪɿɜɧɹɧɧɹ 59
• ɇɟɯɚɣ ɲɜɢɞɤɿɫɬɶ ɜɟɥɨɫɢɩɟɞɢɫɬɚ ɞɨɪɿɜɧɸɽ ɯ ɤɦ/ɝɨɞ, ɬɨɞɿ ɲɜɢɞɤɿɫɬɶ ɦɨ-
ɬɨɰɢɤɥɿɫɬɚ — 3ɯ ɤɦ/ɝɨɞ. ɒɥɹɯ ɡɚɜɞɨɜɠɤɢ 21 ɤɦ ɜɟɥɨɫɢɩɟɞɢɫɬ ɩɨɞɨɥɚɜ ɡɚ
21
x
ɝɨɞ, ɚ ɦɨɬɨɰɢɤɥɿɫɬ — ɡɚ 21 7
3x x
= (ɝɨɞ). Ɉɫɤɿɥɶɤɢ ɜɟɥɨɫɢɩɟɞɢɫɬ ɛɭɜ ɭ ɞɨɪɨɡɿ
ɧɚ 20 ɯɜ + 40 ɯɜ = 60 ɯɜ = 1 ɝɨɞ ɞɨɜɲɟ, ɧɿɠ ɦɨɬɨɰɢɤɥɿɫɬ, ɬɨ ɦɚɽɦɨ ɪɿɜɧɹɧɧɹ
21 7 1− = .
x x
Ɋɨɡɜ’ɹɠɟɦɨ ɰɟ ɪɿɜɧɹɧɧɹ:
21 7
1 0;
x x
− − =
21 7
0;
x
x
− −
=
14
0;
x
x
−
=
14 0;
0;
x
x
− =­
®
≠¯
ɯ = 14.
ȼɿɞɩɨɜɿɞɶ. 14 ɤɦ/ɝɨɞ. •
197. ɇɚɡɜɿɬɶ ɰɿɥɿ ɪɚɰɿɨɧɚɥɶɧɿ ɪɿɜɧɹɧɧɹ; ɞɪɨɛɨɜɿ ɪɚɰɿɨɧɚɥɶɧɿ ɪɿɜɧɹɧɧɹ:
ɚ) 5 1;
7x
=
−
ɛ) 3( 11) 0;x − = ɜ) 1 3;
3
x− = ɝ) 2
2 0.
x
=
198. ɑɢ ɽ ɱɢɫɥɨ 1 ɤɨɪɟɧɟɦ ɪɿɜɧɹɧɧɹ?
ɚ) 1 0;
5
x
x
− =
−
ɛ) 1 0;
4 4
x
x
− =
−
ɜ)
2
1 0;
1
x
x
− =
+
ɝ) 2 3
1 1
.
x x
=
− +
199. ɑɢ ɪɿɜɧɨɫɢɥɶɧɿ ɪɿɜɧɹɧɧɹ?
ɚ) 4 8
1
x
x
=
−
ɿ 2
1
x ;
x
=
−
ɛ) 1 3
2 1x x
=
+
ɿ 1 3 0;
2 1x x
− =
+
ɜ)
2
( 3)
0
x x
x
+
= ɿ ( 3) 0;x x + = ɝ) 2 4
2 2
x
x x
=
− −
ɿ 2ɯ = 4.
Ɋɨɡɜ’ɹɠɿɬɶ ɪɿɜɧɹɧɧɹ:
200. ɚ) 8 0;
1
x
x
+ =
−
ɛ) 1 0;
8
x
x
− =
+
ɜ) 2
2 8 0.
16
x
x
− =
−
201. ɚ) 2 6 0;
3 3
x x
x x
−+ =
− −
ɛ) 3 1 2 2
1 1
x x ;
x x
+ −=
+ +
ɜ) 5 2 .
6 6
x x
x x
− =
− −
202. ɚ) 2 1 0;x
x
+ + = ɛ) 2;
1
x
x
=
+
ɜ) 10 3.x
x
− =
60 § 1. Ɋɚɰɿɨɧɚɥɶɧɿ ɜɢɪɚɡɢ
203. ɚ) 1 1 1;
3 4x x
+ = ɛ) 1 3 2;
5 4x x
+ = ɜ) 4 1 = 1.
3 2x x
−
204. ɚ) 2 4 0;
4
x
x
+ =
+
ɛ) 2
2 10 0;
25
x
x
− =
−
ɜ) 4 12 ;
2 2
x
x x
=
− −
ɝ) 3 4 6 ;
2 2
x x
x x
− −= ɞ) 1 2 0;
5
x
x
− + =
+
ɟ) 5 10 4;x
x
− =
ɽ) 5 4;
2 1
x
x
− = −
−
ɠ) 2 1 5;
2x x
+ = ɡ) 1 1 = 1.
2 3x x
−
205. ɚ)
2
2 4 0;
2
− =
−
x x
x
ɛ) 2 3 0;
2 2x x
− =
+ −
ɜ) 3 2
1 2 3
x x .
x x
+ =
− +
206. ɚ)
2
3 0;
2 1
x x
x
− =
+
ɛ)
2
0;
2
+ =
+
x x
x
ɜ) 5 3
2 3 1
x x .
x x
− =
+ −
207. əɤɟ ɱɢɫɥɨ ɩɨɬɪɿɛɧɨ ɞɨɞɚɬɢ ɞɨ ɡɧɚɦɟɧɧɢɤɚ ɞɪɨɛɭ 19
41
, ɳɨɛ ɨɞɟɪɠɚɬɢ
ɞɪɿɛ, ɹɤɢɣ ɞɨɪɿɜɧɸɽ 1
3
?
208. əɤɟ ɱɢɫɥɨ ɩɨɬɪɿɛɧɨ ɜɿɞɧɹɬɢ ɜɿɞ ɡɧɚɦɟɧɧɢɤɚ ɞɪɨɛɭ 3
47
, ɳɨɛ ɨɞɟɪɠɚɬɢ
ɞɪɿɛ, ɹɤɢɣ ɞɨɪɿɜɧɸɽ 1
4
?
209. əɤɟ ɬɟ ɫɚɦɟ ɱɢɫɥɨ ɩɨɬɪɿɛɧɨ ɞɨɞɚɬɢ ɞɨ ɱɢɫɟɥɶɧɢɤɚ ɞɪɨɛɭ 1
2
ɣ
ɩɨɦɧɨɠɢɬɢ ɧɚ ɧɶɨɝɨ ɡɧɚɦɟɧɧɢɤ ɰɶɨɝɨ ɞɪɨɛɭ, ɳɨɛ ɨɞɟɪɠɚɬɢ ɞɪɿɛ, ɹɤɢɣ
ɞɨɪɿɜɧɸɽ 2
3
?
210. ɇɚ ɹɤɟ ɬɟ ɫɚɦɟ ɱɢɫɥɨ ɩɨɬɪɿɛɧɨ ɩɨɦɧɨɠɢɬɢ ɱɢɫɟɥɶɧɢɤ ɞɪɨɛɭ 1
5
ɣ ɞɨɞɚɬɢ
ɣɨɝɨ ɞɨ ɡɧɚɦɟɧɧɢɤɚ ɰɶɨɝɨ ɞɪɨɛɭ, ɳɨɛ ɨɞɟɪɠɚɬɢ ɞɪɿɛ, ɹɤɢɣ ɞɨɪɿɜɧɸɽ 1
2
?
211. ɑɢ ɪɿɜɧɨɫɢɥɶɧɿ ɪɿɜɧɹɧɧɹ
(2 3)
0
x x
x
−
= ɬɚ ɯ(2ɯ – 3) = 0? ȼɿɞɩɨɜɿɞɶ ɨɛʉɪɭɧ-
ɬɭɣɬɟ.
212. Ⱦɨɜɟɞɿɬɶ, ɳɨ ɪɿɜɧɹɧɧɹ 2 3
1 1
x
x x
+ =
− −
ɬɚ ɯ + 2 = 3 ɧɟ ɪɿɜɧɨɫɢɥɶɧɿ.
Algebra 8kl kravchyk_2016
Algebra 8kl kravchyk_2016
Algebra 8kl kravchyk_2016
Algebra 8kl kravchyk_2016
Algebra 8kl kravchyk_2016
Algebra 8kl kravchyk_2016
Algebra 8kl kravchyk_2016
Algebra 8kl kravchyk_2016
Algebra 8kl kravchyk_2016
Algebra 8kl kravchyk_2016
Algebra 8kl kravchyk_2016
Algebra 8kl kravchyk_2016
Algebra 8kl kravchyk_2016
Algebra 8kl kravchyk_2016
Algebra 8kl kravchyk_2016
Algebra 8kl kravchyk_2016
Algebra 8kl kravchyk_2016
Algebra 8kl kravchyk_2016
Algebra 8kl kravchyk_2016
Algebra 8kl kravchyk_2016
Algebra 8kl kravchyk_2016
Algebra 8kl kravchyk_2016
Algebra 8kl kravchyk_2016
Algebra 8kl kravchyk_2016
Algebra 8kl kravchyk_2016
Algebra 8kl kravchyk_2016
Algebra 8kl kravchyk_2016
Algebra 8kl kravchyk_2016
Algebra 8kl kravchyk_2016
Algebra 8kl kravchyk_2016
Algebra 8kl kravchyk_2016
Algebra 8kl kravchyk_2016
Algebra 8kl kravchyk_2016
Algebra 8kl kravchyk_2016
Algebra 8kl kravchyk_2016
Algebra 8kl kravchyk_2016
Algebra 8kl kravchyk_2016
Algebra 8kl kravchyk_2016
Algebra 8kl kravchyk_2016
Algebra 8kl kravchyk_2016
Algebra 8kl kravchyk_2016
Algebra 8kl kravchyk_2016
Algebra 8kl kravchyk_2016
Algebra 8kl kravchyk_2016
Algebra 8kl kravchyk_2016
Algebra 8kl kravchyk_2016
Algebra 8kl kravchyk_2016
Algebra 8kl kravchyk_2016
Algebra 8kl kravchyk_2016
Algebra 8kl kravchyk_2016
Algebra 8kl kravchyk_2016
Algebra 8kl kravchyk_2016
Algebra 8kl kravchyk_2016
Algebra 8kl kravchyk_2016
Algebra 8kl kravchyk_2016
Algebra 8kl kravchyk_2016
Algebra 8kl kravchyk_2016
Algebra 8kl kravchyk_2016
Algebra 8kl kravchyk_2016
Algebra 8kl kravchyk_2016
Algebra 8kl kravchyk_2016
Algebra 8kl kravchyk_2016
Algebra 8kl kravchyk_2016
Algebra 8kl kravchyk_2016
Algebra 8kl kravchyk_2016
Algebra 8kl kravchyk_2016
Algebra 8kl kravchyk_2016
Algebra 8kl kravchyk_2016
Algebra 8kl kravchyk_2016
Algebra 8kl kravchyk_2016
Algebra 8kl kravchyk_2016
Algebra 8kl kravchyk_2016
Algebra 8kl kravchyk_2016
Algebra 8kl kravchyk_2016
Algebra 8kl kravchyk_2016
Algebra 8kl kravchyk_2016
Algebra 8kl kravchyk_2016
Algebra 8kl kravchyk_2016
Algebra 8kl kravchyk_2016
Algebra 8kl kravchyk_2016
Algebra 8kl kravchyk_2016
Algebra 8kl kravchyk_2016
Algebra 8kl kravchyk_2016
Algebra 8kl kravchyk_2016
Algebra 8kl kravchyk_2016
Algebra 8kl kravchyk_2016
Algebra 8kl kravchyk_2016
Algebra 8kl kravchyk_2016
Algebra 8kl kravchyk_2016
Algebra 8kl kravchyk_2016
Algebra 8kl kravchyk_2016
Algebra 8kl kravchyk_2016
Algebra 8kl kravchyk_2016
Algebra 8kl kravchyk_2016
Algebra 8kl kravchyk_2016
Algebra 8kl kravchyk_2016
Algebra 8kl kravchyk_2016
Algebra 8kl kravchyk_2016
Algebra 8kl kravchyk_2016
Algebra 8kl kravchyk_2016
Algebra 8kl kravchyk_2016
Algebra 8kl kravchyk_2016
Algebra 8kl kravchyk_2016
Algebra 8kl kravchyk_2016
Algebra 8kl kravchyk_2016
Algebra 8kl kravchyk_2016
Algebra 8kl kravchyk_2016
Algebra 8kl kravchyk_2016
Algebra 8kl kravchyk_2016
Algebra 8kl kravchyk_2016
Algebra 8kl kravchyk_2016
Algebra 8kl kravchyk_2016
Algebra 8kl kravchyk_2016
Algebra 8kl kravchyk_2016
Algebra 8kl kravchyk_2016
Algebra 8kl kravchyk_2016
Algebra 8kl kravchyk_2016
Algebra 8kl kravchyk_2016
Algebra 8kl kravchyk_2016
Algebra 8kl kravchyk_2016
Algebra 8kl kravchyk_2016
Algebra 8kl kravchyk_2016
Algebra 8kl kravchyk_2016
Algebra 8kl kravchyk_2016
Algebra 8kl kravchyk_2016
Algebra 8kl kravchyk_2016
Algebra 8kl kravchyk_2016
Algebra 8kl kravchyk_2016
Algebra 8kl kravchyk_2016
Algebra 8kl kravchyk_2016
Algebra 8kl kravchyk_2016
Algebra 8kl kravchyk_2016
Algebra 8kl kravchyk_2016
Algebra 8kl kravchyk_2016
Algebra 8kl kravchyk_2016
Algebra 8kl kravchyk_2016
Algebra 8kl kravchyk_2016
Algebra 8kl kravchyk_2016
Algebra 8kl kravchyk_2016
Algebra 8kl kravchyk_2016
Algebra 8kl kravchyk_2016
Algebra 8kl kravchyk_2016
Algebra 8kl kravchyk_2016
Algebra 8kl kravchyk_2016
Algebra 8kl kravchyk_2016
Algebra 8kl kravchyk_2016
Algebra 8kl kravchyk_2016
Algebra 8kl kravchyk_2016
Algebra 8kl kravchyk_2016
Algebra 8kl kravchyk_2016
Algebra 8kl kravchyk_2016
Algebra 8kl kravchyk_2016
Algebra 8kl kravchyk_2016
Algebra 8kl kravchyk_2016
Algebra 8kl kravchyk_2016
Algebra 8kl kravchyk_2016
Algebra 8kl kravchyk_2016
Algebra 8kl kravchyk_2016
Algebra 8kl kravchyk_2016
Algebra 8kl kravchyk_2016
Algebra 8kl kravchyk_2016
Algebra 8kl kravchyk_2016
Algebra 8kl kravchyk_2016
Algebra 8kl kravchyk_2016
Algebra 8kl kravchyk_2016
Algebra 8kl kravchyk_2016
Algebra 8kl kravchyk_2016
Algebra 8kl kravchyk_2016
Algebra 8kl kravchyk_2016
Algebra 8kl kravchyk_2016
Algebra 8kl kravchyk_2016
Algebra 8kl kravchyk_2016
Algebra 8kl kravchyk_2016
Algebra 8kl kravchyk_2016
Algebra 8kl kravchyk_2016
Algebra 8kl kravchyk_2016
Algebra 8kl kravchyk_2016
Algebra 8kl kravchyk_2016
Algebra 8kl kravchyk_2016
Algebra 8kl kravchyk_2016
Algebra 8kl kravchyk_2016
Algebra 8kl kravchyk_2016
Algebra 8kl kravchyk_2016
Algebra 8kl kravchyk_2016
Algebra 8kl kravchyk_2016
Algebra 8kl kravchyk_2016
Algebra 8kl kravchyk_2016
Algebra 8kl kravchyk_2016
Algebra 8kl kravchyk_2016
Algebra 8kl kravchyk_2016
Algebra 8kl kravchyk_2016
Algebra 8kl kravchyk_2016
Algebra 8kl kravchyk_2016
Algebra 8kl kravchyk_2016
Algebra 8kl kravchyk_2016
Algebra 8kl kravchyk_2016

More Related Content

More from della street

Ykr yaz 8kl_pentiluk_2016
Ykr yaz 8kl_pentiluk_2016Ykr yaz 8kl_pentiluk_2016
Ykr yaz 8kl_pentiluk_2016della street
 
Rus yaz 8kl_balandina_2016_4rik
Rus yaz 8kl_balandina_2016_4rikRus yaz 8kl_balandina_2016_4rik
Rus yaz 8kl_balandina_2016_4rikdella street
 
Rus yaz 8kl_balandina_2016
Rus yaz 8kl_balandina_2016Rus yaz 8kl_balandina_2016
Rus yaz 8kl_balandina_2016della street
 
Nemez yaz 8kl_sotnik_2016_8rik
Nemez yaz 8kl_sotnik_2016_8rikNemez yaz 8kl_sotnik_2016_8rik
Nemez yaz 8kl_sotnik_2016_8rikdella street
 
Nemez yaz 8kl_sotnik_2016
Nemez yaz 8kl_sotnik_2016Nemez yaz 8kl_sotnik_2016
Nemez yaz 8kl_sotnik_2016della street
 
Ist ykr 8kl_gisem_2016_rus
Ist ykr 8kl_gisem_2016_rusIst ykr 8kl_gisem_2016_rus
Ist ykr 8kl_gisem_2016_rusdella street
 
Ist ykr 8kl_gisem_2016
Ist ykr 8kl_gisem_2016Ist ykr 8kl_gisem_2016
Ist ykr 8kl_gisem_2016della street
 
Geograf 8kl dovgan_2016_rus
Geograf 8kl dovgan_2016_rusGeograf 8kl dovgan_2016_rus
Geograf 8kl dovgan_2016_rusdella street
 
Geograf 8kl dovgan_2016
Geograf 8kl dovgan_2016Geograf 8kl dovgan_2016
Geograf 8kl dovgan_2016della street
 
Biolog 8kl zadorozhn_2016
Biolog 8kl zadorozhn_2016Biolog 8kl zadorozhn_2016
Biolog 8kl zadorozhn_2016della street
 
Ang yaz 8kl_byrenko_2016
Ang yaz 8kl_byrenko_2016Ang yaz 8kl_byrenko_2016
Ang yaz 8kl_byrenko_2016della street
 
Geometr 8kl roganin_2016
Geometr 8kl roganin_2016Geometr 8kl roganin_2016
Geometr 8kl roganin_2016della street
 
Biolog 8kl mizhyk_2016_rus
Biolog 8kl mizhyk_2016_rusBiolog 8kl mizhyk_2016_rus
Biolog 8kl mizhyk_2016_rusdella street
 
Biolog 8kl mizhyk_2016
Biolog 8kl mizhyk_2016Biolog 8kl mizhyk_2016
Biolog 8kl mizhyk_2016della street
 
4 klas matematika_zajika_2015
4 klas matematika_zajika_20154 klas matematika_zajika_2015
4 klas matematika_zajika_2015della street
 
4 klas matematika_listopad_2015_ukr
4 klas matematika_listopad_2015_ukr4 klas matematika_listopad_2015_ukr
4 klas matematika_listopad_2015_ukrdella street
 
7 klas algebra_tarasenkova_2015
7 klas algebra_tarasenkova_20157 klas algebra_tarasenkova_2015
7 klas algebra_tarasenkova_2015della street
 
4 matem skvorz_2_2015
4 matem skvorz_2_20154 matem skvorz_2_2015
4 matem skvorz_2_2015della street
 
4 matem olyan_2015_ua
4 matem olyan_2015_ua4 matem olyan_2015_ua
4 matem olyan_2015_uadella street
 
4 matem shostak_2015
4 matem shostak_20154 matem shostak_2015
4 matem shostak_2015della street
 

More from della street (20)

Ykr yaz 8kl_pentiluk_2016
Ykr yaz 8kl_pentiluk_2016Ykr yaz 8kl_pentiluk_2016
Ykr yaz 8kl_pentiluk_2016
 
Rus yaz 8kl_balandina_2016_4rik
Rus yaz 8kl_balandina_2016_4rikRus yaz 8kl_balandina_2016_4rik
Rus yaz 8kl_balandina_2016_4rik
 
Rus yaz 8kl_balandina_2016
Rus yaz 8kl_balandina_2016Rus yaz 8kl_balandina_2016
Rus yaz 8kl_balandina_2016
 
Nemez yaz 8kl_sotnik_2016_8rik
Nemez yaz 8kl_sotnik_2016_8rikNemez yaz 8kl_sotnik_2016_8rik
Nemez yaz 8kl_sotnik_2016_8rik
 
Nemez yaz 8kl_sotnik_2016
Nemez yaz 8kl_sotnik_2016Nemez yaz 8kl_sotnik_2016
Nemez yaz 8kl_sotnik_2016
 
Ist ykr 8kl_gisem_2016_rus
Ist ykr 8kl_gisem_2016_rusIst ykr 8kl_gisem_2016_rus
Ist ykr 8kl_gisem_2016_rus
 
Ist ykr 8kl_gisem_2016
Ist ykr 8kl_gisem_2016Ist ykr 8kl_gisem_2016
Ist ykr 8kl_gisem_2016
 
Geograf 8kl dovgan_2016_rus
Geograf 8kl dovgan_2016_rusGeograf 8kl dovgan_2016_rus
Geograf 8kl dovgan_2016_rus
 
Geograf 8kl dovgan_2016
Geograf 8kl dovgan_2016Geograf 8kl dovgan_2016
Geograf 8kl dovgan_2016
 
Biolog 8kl zadorozhn_2016
Biolog 8kl zadorozhn_2016Biolog 8kl zadorozhn_2016
Biolog 8kl zadorozhn_2016
 
Ang yaz 8kl_byrenko_2016
Ang yaz 8kl_byrenko_2016Ang yaz 8kl_byrenko_2016
Ang yaz 8kl_byrenko_2016
 
Geometr 8kl roganin_2016
Geometr 8kl roganin_2016Geometr 8kl roganin_2016
Geometr 8kl roganin_2016
 
Biolog 8kl mizhyk_2016_rus
Biolog 8kl mizhyk_2016_rusBiolog 8kl mizhyk_2016_rus
Biolog 8kl mizhyk_2016_rus
 
Biolog 8kl mizhyk_2016
Biolog 8kl mizhyk_2016Biolog 8kl mizhyk_2016
Biolog 8kl mizhyk_2016
 
4 klas matematika_zajika_2015
4 klas matematika_zajika_20154 klas matematika_zajika_2015
4 klas matematika_zajika_2015
 
4 klas matematika_listopad_2015_ukr
4 klas matematika_listopad_2015_ukr4 klas matematika_listopad_2015_ukr
4 klas matematika_listopad_2015_ukr
 
7 klas algebra_tarasenkova_2015
7 klas algebra_tarasenkova_20157 klas algebra_tarasenkova_2015
7 klas algebra_tarasenkova_2015
 
4 matem skvorz_2_2015
4 matem skvorz_2_20154 matem skvorz_2_2015
4 matem skvorz_2_2015
 
4 matem olyan_2015_ua
4 matem olyan_2015_ua4 matem olyan_2015_ua
4 matem olyan_2015_ua
 
4 matem shostak_2015
4 matem shostak_20154 matem shostak_2015
4 matem shostak_2015
 

Algebra 8kl kravchyk_2016

  • 1.
  • 2. ȼȺɋɂɅɖ ɄɊȺȼɑɍɄ ɆȺɊȱə ɉȱȾɊɍɑɇȺ ȽȺɅɂɇȺ əɇɑȿɇɄɈ Ⱥ Ʌ Ƚ ȿ Ȼ Ɋ Ⱥ ɉɿɞɪɭɱɧɢɤ ɞɥɹ 8 ɤɥɚɫɭ ɡɚɝɚɥɶɧɨɨɫɜɿɬɧɿɯ ɧɚɜɱɚɥɶɧɢɯ ɡɚɤɥɚɞɿɜ Ɍɟɪɧɨɩɿɥɶ ȼɢɞɚɜɧɢɰɬɜɨ «ɉɿɞɪɭɱɧɢɤɢ ɿ ɩɨɫɿɛɧɢɤɢ» 2016
  • 3. ɍȾɄ 51(075.3) ȻȻɄ 22.1ɹ723 Ʉ 77 ȿɤɫɩɟɪɬɢ, ɹɤɿ ɡɞɿɣɫɧɢɥɢ ɟɤɫɩɟɪɬɢɡɭ ɞɚɧɨɝɨ ɩɿɞɪɭɱɧɢɤɚ ɩɿɞ ɱɚɫ ɩɪɨɜɟɞɟɧɧɹ ɤɨɧ- ɤɭɪɫɧɨɝɨ ɜɿɞɛɨɪɭ ɩɪɨɟɤɬɿɜ ɩɿɞɪɭɱɧɢɤɿɜ ɞɥɹ ɭɱɧɿɜ 8 ɤɥɚɫɭ ɡɚɝɚɥɶɧɨɨɫɜɿɬɧɿɯ ɧɚɜɱɚɥɶɧɢɯ ɡɚɤɥɚɞɿɜ ɿ ɡɪɨɛɢɥɢ ɜɢɫɧɨɜɨɤ ɩɪɨ ɞɨɰɿɥɶɧɿɫɬɶ ɧɚɞɚɧɧɹ ɩɿɞɪɭɱɧɢɤɭ ɝɪɢɮɚ «Ɋɟɤɨɦɟɧɞɨ- ɜɚɧɨ Ɇɿɧɿɫɬɟɪɫɬɜɨɦ ɨɫɜɿɬɢ ɿ ɧɚɭɤɢ ɍɤɪɚʀɧɢ»: Ʌɟɜɢɰɶɤɚ ȱ. Ɇ., ɦɟɬɨɞɢɫɬ ɜɿɞɞɿɥɭ ɦɟɬɨɞɢɱɧɨʀ ɪɨɛɨɬɢ Ʉɍ «Ȼɚɲɬɚɧɫɶɤɢɣ ɪɚɣɨɧ- ɧɢɣ ɫɟɪɜɿɫɧɢɣ ɰɟɧɬɪ ɩɨ ɨɛɫɥɭɝɨɜɭɜɚɧɧɸ ɡɚɤɥɚɞɿɜ ɨɫɜɿɬɢ» Ȼɚɲɬɚɧɫɶɤɨʀ ɪɚɣɨɧɧɨʀ ɪɚɞɢ Ɇɢɤɨɥɚʀɜɫɶɤɨʀ ɨɛɥɚɫɬɿ; ɉɨɝɨɪɿɥɹɤ Ɉ. Ɉ., ɞɨɰɟɧɬ ɤɚɮɟɞɪɢ ɬɟɨɪɿʀ ɣɦɨɜɿɪɧɨɫɬɟɣ ɿ ɦɚɬɟɦɚɬɢɱɧɨɝɨ ɚɧɚɥɿɡɭ ȾȼɇɁ «ɍɠɝɨɪɨɞɫɶɤɢɣ ɧɚɰɿɨɧɚɥɶɧɢɣ ɭɧɿɜɟɪɫɢɬɟɬ», ɤɚɧɞɢɞɚɬ ɮɿɡɢɤɨ-ɦɚɬɟɦɚɬɢɱɧɢɯ ɧɚɭɤ; Ɋɭɞɟɧɤɨ ȼ. Ɉ., ɭɱɢɬɟɥɶ Ɇɚɪ’ɹɧɿɜɫɶɤɨʀ ɡɚɝɚɥɶɧɨɨɫɜɿɬɧɶɨʀ ɲɤɨɥɢ ȱ–ȱȱȱ ɫɬɭɩɟɧɿɜ Ɇɚɥɨɜɢɫɤɿɜɫɶɤɨʀ ɪɚɣɨɧɧɨʀ ɪɚɞɢ Ʉɿɪɨɜɨɝɪɚɞɫɶɤɨʀ ɨɛɥɚɫɬɿ, ɭɱɢɬɟɥɶ-ɦɟɬɨɞɢɫɬ, ɡɚɫɥɭɠɟ- ɧɢɣ ɭɱɢɬɟɥɶ ɍɤɪɚʀɧɢ. Ɋɟɤɨɦɟɧɞɨɜɚɧɨ Ɇɿɧɿɫɬɟɪɫɬɜɨɦ ɨɫɜɿɬɢ ɿ ɧɚɭɤɢ ɍɤɪɚʀɧɢ (ɧɚɤɚɡ ɆɈɇ ɍɤɪɚʀɧɢ ɜɿɞ 10.05.2016 ɪ. ʋ 491) ȼɢɞɚɧɨ ɡɚ ɪɚɯɭɧɨɤ ɞɟɪɠɚɜɧɢɯ ɤɨɲɬɿɜ. ɉɪɨɞɚɠ ɡɚɛɨɪɨɧɟɧɨ Ʉɪɚɜɱɭɤ ȼ. Ʉ 77 Ⱥɥɝɟɛɪɚ : ɩɿɞɪɭɱ. ɞɥɹ 8 ɤɥ. ɡɚɝɚɥɶɧɨɨɫɜɿɬ. ɧɚɜɱ. ɡɚɤɥ. / ȼ. Ʉɪɚɜɱɭɤ, Ɇ. ɉɿɞɪɭɱɧɚ, Ƚ. əɧɱɟɧɤɨ. — Ɍɟɪɧɨɩɿɥɶ : ɉɿɞɪɭɱ- ɧɢɤɢ ɿ ɩɨɫɿɛɧɢɤɢ, 2016. — 256 ɫ. ISBN 978-966-07-3003-8 ɍȾɄ 51(075.3) ȻȻɄ 22.1ɹ723 ISBN 978-966-07-3003-8 © Ʉɪɚɜɱɭɤ ȼ., ɉɿɞɪɭɱɧɚ Ɇ., əɧɱɟɧɤɨ Ƚ., 2016 © ȼɢɞɚɜɧɢɰɬɜɨ «ɉɿɞɪɭɱɧɢɤɢ ɿ ɩɨɫɿɛɧɢɤɢ», ɨɪɢɝɿɧɚɥ-ɦɚɤɟɬ, 2106
  • 4. Ȍǻǥ DzǾȁǵǥ! Ʉɿɥɶɤɚ ɫɥɿɜ ɩɪɨ ɨɫɨɛɥɢɜɨɫɬɿ ɜɢɞɚɧɧɹ. Ɇɚɬɟɪɿɚɥ ɩɿɞɪɭɱɧɢɤɚ ɩɨɞɿɥɟɧɨ ɧɚ ɬɪɢ ɩɚɪɚɝɪɚɮɢ, ɚ ɩɚɪɚɝɪɚ- ɮɢ — ɧɚ ɩɭɧɤɬɢ. Ʉɨɠɧɢɣ ɩɭɧɤɬ ɪɨɡɩɨɱɢɧɚɽɬɶɫɹ ɜɢɤɥɚɞɨɦ ɬɟɨɪɟɬɢɱɧɨɝɨ ɦɚɬɟɪɿ- ɚɥɭ. Ⱦɟɹɤɿ ɩɭɧɤɬɢ ɦɿɫɬɹɬɶ ɞɨɞɚɬɤɨɜɢɣ ɦɚɬɟɪɿɚɥ ɩɿɞ ɪɭɛɪɢɤɨɸ «Ⱦɥɹ ɬɢɯ, ɯɬɨ ɯɨɱɟ ɡɧɚɬɢ ɛɿɥɶɲɟ». Ⱦɚɥɿ ɣɞɟ ɪɭɛɪɢɤɚ «ɉɪɢɤɥɚɞɢ ɪɨɡɜ’ɹɡɚɧɧɹ ɜɩɪɚɜ». ɐɟ ɩɿɞɤɚɡɤɚ. ȼɨɧɚ ɞɨɩɨɦɨɠɟ ɜɚɦ ɨɡɧɚɣɨɦɢɬɢɫɹ ɡ ɨɫɧɨɜɧɢɦɢ ɜɢɞɚɦɢ ɜɩɪɚɜ, ɫɩɨ- ɫɨɛɚɦɢ ʀɯ ɪɨɡɜ’ɹɡɭɜɚɧɧɹ ɬɚ ɧɚɜɱɢɬɶ ɩɪɚɜɢɥɶɧɨ ɡɚɩɢɫɭɜɚɬɢ ɪɨɡɜ’ɹ- ɡɚɧɧɹ. ɉɨɱɚɬɨɤ ɬɚ ɡɚɤɿɧɱɟɧɧɹ ɪɨɡɜ’ɹɡɚɧɧɹ ɤɨɠɧɨʀ ɜɩɪɚɜɢ ɩɨɡɧɚɱɟɧɨ ɤɪɭɠɟɱɤɨɦ «Ɣ». ɍ ɤɨɠɧɨɦɭ ɩɭɧɤɬɿ ɫɢɫɬɟɦɭ ɜɩɪɚɜ ɩɨɞɿɥɟɧɨ ɧɚ ɬɪɢ ɪɿɜɧɿ ɫɤɥɚɞ- ɧɨɫɬɿ. ɋɩɨɱɚɬɤɭ ɜɚɪɬɨ ɪɨɡɜ’ɹɡɭɜɚɬɢ ɭɫɧɿ ɜɩɪɚɜɢ ɿ ɩɪɨɫɬɿɲɿ ɡɚɞɚɱɿ (ɪɿ- ɜɟɧɶ Ⱥ), ɚ ɩɨɬɿɦ ɩɟɪɟɣɬɢ ɞɨ ɫɤɥɚɞɧɿɲɢɯ (ɪɿɜɟɧɶ Ȼ). Ɂɚɞɚɱɿ ɪɿɜɧɹ
  • 5. ȼ — ɞɥɹ ɧɚɣɤɦɿɬɥɢɜɿɲɢɯ, ɬɢɯ, ɯɬɨ ɯɨɱɟ ɜɦɿɬɢ ɬɚ ɡɧɚɬɢ ɛɿɥɶɲɟ ɿ ɦɚ- ɬɢ ɧɚɣɜɢɳɿ ɨɰɿɧɤɢ. Ⱦɥɹ ɞɟɹɤɢɯ ɡɚɞɚɱ ɰɶɨɝɨ ɪɿɜɧɹ ɧɚɜɟɞɟɧɨ ɪɨɡɜ’ɹ- ɡɚɧɧɹ. Ⱦɥɹ ɫɚɦɨɫɬɿɣɧɨʀ ɪɨɛɨɬɢ ɜɞɨɦɚ ɪɟɤɨɦɟɧɞɨɜɚɧɨ ɡɚɞɚɱɿ, ɧɨɦɟɪɢ ɹɤɢɯ ɜɢɞɿɥɟɧɨ ɤɨɥɶɨɪɨɦ (ɧɚɩɪɢɤɥɚɞ, 255). Ɋɭɛɪɢɤɚ «ȼɩɪɚɜɢ ɞɥɹ ɩɨɜɬɨɪɟɧɧɹ» ɩɪɢɡɧɚɱɟɧɚ ɞɥɹ ɩɟɪɿɨɞɢɱɧɨ- ɝɨ ɩɨɜɬɨɪɟɧɧɹ ɨɫɧɨɜɧɢɯ ɜɢɞɿɜ ɜɩɪɚɜ ɬɚ ɩɿɞɝɨɬɨɜɤɢ ɞɨ ɜɢɜɱɟɧɧɹ ɧɨ- ɜɨɝɨ ɬɟɨɪɟɬɢɱɧɨɝɨ ɦɚɬɟɪɿɚɥɭ. ɇɚɫɬɭɩɧɚ ɪɭɛɪɢɤɚ «ɉɨɦɿɪɤɭɣɬɟ» ɩɨɜ’ɹɡɚɧɚ ɡ ɨɫɨɛɥɢɜɢɦ ɚɫɩɟɤ- ɬɨɦ ɦɚɬɟɦɚɬɢɱɧɨʀ ɩɿɞɝɨɬɨɜɤɢ. Ɉɫɧɨɜɧɢɦ ɞɥɹ ɪɨɡɜ’ɹɡɚɧɧɹ ɡɚɞɚɱ ɰɿɽʀ ɪɭɛɪɢɤɢ ɽ ɜɦɿɧɧɹ ɜɢɯɨɞɢɬɢ ɡ ɧɟɫɬɚɧɞɚɪɬɧɢɯ ɫɢɬɭɚɰɿɣ. Ɋɨɡɜ’ɹ- ɡɭɜɚɧɧɹ ɬɚɤɢɯ ɡɚɞɚɱ ɪɨɡɜɢɜɚɽ ɝɧɭɱɤɿɫɬɶ ɿ ɤɪɢɬɢɱɧɿɫɬɶ ɦɢɫɥɟɧɧɹ, ɚ ɰɟ ɞɨɩɨɦɨɠɟ ɜɚɦ ɭ ɦɚɣɛɭɬɧɶɨɦɭ, ɧɟɡɚɥɟɠɧɨ ɜɿɞ ɬɨɝɨ, ɹɤɭ ɩɪɨɮɟɫɿɸ ɜɢ ɨɛɟɪɟɬɟ. ɇɚɩɪɢɤɿɧɰɿ ɤɨɠɧɨɝɨ ɩɚɪɚɝɪɚɮɚ ɭɦɿɳɟɧɨ ɡɚɩɢɬɚɧɧɹ ɬɚ ɜɩɪɚɜɢ ɞɥɹ ɩɨɜɬɨɪɟɧɧɹ, ɡɚɜɞɚɧɧɹ ɞɥɹ ɫɚɦɨɩɟɪɟɜɿɪɤɢ ɱɨɬɢɪɶɨɯ ɪɿɜɧɿɜ ɫɤɥɚɞ- ɧɨɫɬɿ. ɍ ɤɿɧɰɿ ɩɿɞɪɭɱɧɢɤɚ ɩɨɞɚɧɨ ɜɩɪɚɜɢ ɞɥɹ ɩɨɜɬɨɪɟɧɧɹ ɦɚɬɟɪɿɚɥɭ ɡɚ ɭɜɟɫɶ ɤɭɪɫ ɚɥɝɟɛɪɢ 8 ɤɥɚɫɭ, ɡɚɞɚɱɿ ɩɿɞɜɢɳɟɧɨʀ ɫɤɥɚɞɧɨɫɬɿ, ɞɨɜɿɞɤɨ- ɜɢɣ ɦɚɬɟɪɿɚɥ. ɓɢɪɨ ɛɚɠɚɽɦɨ ɭɫɩɿɯɭ!
  • 6.
  • 7. 6 § 1. Ɋɚɰɿɨɧɚɥɶɧɿ ɜɢɪɚɡɢ 1. ɐɿɥɿ, ɞɪɨɛɨɜɿ ɬɚ ɪɚɰɿɨɧɚɥɶɧɿ ɜɢɪɚɡɢ. ɍ ɫɶɨɦɨɦɭ ɤɥɚɫɿ ɦɢ ɜɢɜɱɚɥɢ ɰɿɥɿ ɜɢɪɚɡɢ. ɉɪɢɤɥɚɞɚɦɢ ɬɚɤɢɯ ɜɢɪɚɡɿɜ ɽ: ɚ + b; 3ɚ2 ; 2ɯ(ɯ – ɭ)2 ; ; 3 c ɚ : 4; b; 3. ɉɪɢɝɚɞɚɣɦɨ: ɰɿɥɿ ɜɢɪɚɡɢ ɦɨɠɭɬɶ ɦɿɫɬɢɬɢ ɞɿʀ ɞɨɞɚɜɚɧɧɹ, ɜɿɞɧɿɦɚɧɧɹ, ɦɧɨɠɟɧɧɹ, ɩɿɞɧɟɫɟɧɧɹ ɞɨ ɫɬɟɩɟɧɹ, ɚ ɬɚɤɨɠ ɞɿɸ ɞɿɥɟɧɧɹ, ɚɥɟ ɬɿɥɶɤɢ ɧɚ ɱɢɫɥɨ, ɜɿɞɦɿɧɧɟ ɜɿɞ ɧɭɥɹ. Ʉɨɠɧɢɣ ɰɿɥɢɣ ɜɢɪɚɡ ɦɨɠɧɚ ɡɚɩɢɫɚɬɢ ɭ ɜɢɝɥɹɞɿ ɦɧɨɝɨɱɥɟɧɚ. ɇɚɩɪɢɤɥɚɞ, 2ɯ(ɯ – ɭ)2 = 2ɯ(ɯ2 – 2ɯɭ + ɭ2 ) = 2ɯ3 – 4ɯ2 ɭ + 2ɯɭ2 . Ɋɨɡɝɥɹɧɟɦɨ ɜɢɪɚɡɢ 5 1, 1 y y + + 2 2 17 ,ab a b− 3ɚ : b, 2 ( ) .xx y x y − − + ɐɿ ɜɢɪɚɡɢ ɜɿɞɪɿɡɧɹɸɬɶɫɹ ɜɿɞ ɰɿɥɢɯ ɜɢɪɚɡɿɜ ɬɢɦ, ɳɨ ɦɿɫɬɹɬɶ ɞɿɸ ɞɿɥɟɧɧɹ ɧɚ ɜɢ- ɪɚɡ ɡɿ ɡɦɿɧɧɨɸ. Ɍɚɤɿ ɜɢɪɚɡɢ ɧɚɡɢɜɚɸɬɶ ɞɪɨɛɨɜɢɦɢ ɪɚɰɿɨɧɚɥɶɧɢɦɢ ɜɢɪɚɡɚɦɢ. ɐɿɥɿ ɣ ɞɪɨɛɨɜɿ ɪɚɰɿɨɧɚɥɶɧɿ ɜɢɪɚɡɢ ɧɚɡɢɜɚɸɬɶ ɪɚɰɿɨɧɚɥɶɧɢɦɢ ɜɢɪɚɡɚɦɢ. 3 4b a+ ɐɿɥɢɣ ɜɢɪɚɡ 34 a b + Ⱦɪɨɛɨɜɢɣ ɜɢɪɚɡ Ɋɚɰɿɨɧɚɥɶɧɿ ɜɢɪɚɡɢ Ɋɨɡɝɥɹɧɟɦɨ ɪɚɰɿɨɧɚɥɶɧɿ ɜɢɪɚɡɢ ,ab a b+ 2,3 , ( 2)x y + 5 7 a . ȼɨɧɢ ɽ ɱɚɫɬɤɚɦɢ ɞɜɨɯ ɜɢɪɚɡɿɜ, ɞɨ ɬɨɝɨ ɠ, ɞɿɸ ɞɿɥɟɧɧɹ ɡɚɩɢɫɚɧɨ ɡɚ ɞɨɩɨɦɨɝɨɸ ɪɢɫɤɢ ɞɪɨɛɭ. Ɍɚɤɿ ɜɢɪɚɡɢ ɧɚɡɢɜɚɸɬɶ ɞɪɨɛɚɦɢ. əɤɳɨ ɦɚɽɦɨ ɞɪɿɛ ,A B ɞɟ A ɿ B — ɞɟɹɤɿ ɱɢɫɥɨɜɿ ɜɢɪɚɡɢ ɚɛɨ ɜɢɪɚɡɢ ɡɿ ɡɦɿɧɧɢ- ɦɢ, ɬɨ ɜɢɪɚɡ A ɧɚɡɢɜɚɸɬɶ ɱɢɫɟɥɶɧɢɤɨɦ ɞɪɨɛɭ, ɚ ɜɢɪɚɡ B — ɡɧɚɦɟɧɧɢɤɨɦ. Ɉɬɠɟ, ab a b+ — ɞɪɿɛ ɿɡ ɱɢɫɟɥɶɧɢɤɨɦ ab ɿ ɡɧɚɦɟɧɧɢɤɨɦ a + b.
  • 8. 1. Ɋɚɰɿɨɧɚɥɶɧɿ ɜɢɪɚɡɢ. Ɋɚɰɿɨɧɚɥɶɧɿ ɞɪɨɛɢ 7 Ⱦɪɿɛ ,A B ɭ ɹɤɨɦɭ A ɿ B — ɦɧɨɝɨɱɥɟɧɢ, ɧɚɡɢɜɚɸɬɶ ɪɚɰɿɨɧɚɥɶɧɢɦ ɞɪɨɛɨɦ. ɇɚɩɪɢɤɥɚɞ, 4 , 3x + ,a b a b + − 2 2 , x y x xy y + + + , x a 3 b — ɪɚɰɿɨɧɚɥɶɧɿ ɞɪɨɛɢ. 2. Ⱦɨɩɭɫɬɢɦɿ ɡɧɚɱɟɧɧɹ ɡɦɿɧɧɢɯ. Ɋɨɡɝɥɹɧɟɦɨ ɞɪɨɛɨɜɢɣ ɜɢɪɚɡ 5 . 2a − əɤɳɨ ɚ = 3, ɬɨ ɡɧɚɱɟɧɧɹ ɰɶɨɝɨ ɜɢɪɚɡɭ ɞɨɪɿɜɧɸɽ: 5 3 2− = 5 1 = 5; ɹɤɳɨ ɚ = –6, ɬɨ ɡɧɚɱɟɧɧɹ ɜɢɪɚɡɭ ɞɨɪɿɜɧɸɽ: 5 6 2− − = 5. 8 − Ɂɧɚɱɟɧɧɹ ɜɢɪɚɡɭ 5 2a − ɦɨɠɧɚ ɡɧɚɣɬɢ ɞɥɹ ɛɭɞɶ-ɹɤɨɝɨ ɡɧɚɱɟɧɧɹ ɚ, ɤɪɿɦ ɚ = 2. əɤɳɨ ɚ = 2, ɬɨ ɡɧɚɦɟɧɧɢɤ ɚ – 2 ɞɨɪɿɜɧɸɽ ɧɭɥɸ, ɚ ɧɚ ɧɭɥɶ ɞɿɥɢɬɢ ɧɟ ɦɨɠɧɚ. Ʉɚɠɭɬɶ: ɹɤɳɨ ɚ ≠ 2, ɬɨ ɜɢɪɚɡ 5 2a − ɦɚɽ ɡɦɿɫɬ, ɚ ɹɤɳɨ ɚ = 2, ɬɨ ɜɢɪɚɡ ɧɟ ɦɚɽ ɡɦɿɫɬɭ. Ɂɧɚɱɟɧɧɹ ɡɦɿɧɧɢɯ, ɞɥɹ ɹɤɢɯ ɜɢɪɚɡ ɦɚɽ ɡɦɿɫɬ, ɧɚɡɢɜɚɸɬɶ ɞɨɩɭɫɬɢɦɢɦɢ ɡɧɚɱɟɧɧɹɦɢ ɡɦɿɧ- ɧɢɯ. Ɉɡɧɚɱɟɧɧɹ Ⱦɨɩɭɫɬɢɦɢɦɢ ɡɧɚɱɟɧɧɹɦɢ ɡɦɿɧɧɢɯ ɜɢɪɚɡɭ ɧɚɡɢɜɚɸɬɶ ɬɚɤɿ ʀɯ ɡɧɚɱɟɧɧɹ, ɞɥɹ ɹɤɢɯ ɜɢɪɚɡ ɦɚɽ ɡɦɿɫɬ. Ɍɚɤ, ɞɥɹ ɜɢɪɚɡɭ 5 2a − ɞɨɩɭɫɬɢɦɢɦɢ ɡɧɚɱɟɧɧɹɦɢ ɡɦɿɧɧɨʀ ɽ ɜɫɿ ɡɧɚɱɟɧɧɹ ɚ, ɤɪɿɦ ɚ = 2. Ⱦɨɩɭɫɬɢɦɢɦɢ ɡɧɚɱɟɧɧɹɦɢ ɡɦɿɧɧɢɯ ɛɭɞɶ-ɹɤɨɝɨ ɰɿɥɨɝɨ ɜɢɪɚɡɭ ɽ ɜɫɿ ɡɧɚɱɟɧ- ɧɹ ɡɦɿɧɧɢɯ. Ⱦɨɩɭɫɬɢɦɢɦɢ ɡɧɚɱɟɧɧɹɦɢ ɡɦɿɧɧɢɯ ɞɪɨɛɨɜɨɝɨ ɪɚɰɿɨɧɚɥɶɧɨɝɨ ɜɢɪɚ- ɡɭ ɽ ɜɫɿ ɡɧɚɱɟɧɧɹ ɡɦɿɧɧɢɯ, ɤɪɿɦ ɬɢɯ, ɞɥɹ ɹɤɢɯ ɞɨɪɿɜɧɸɽ ɧɭɥɸ ɡɧɚɦɟɧɧɢɤ ɯɨɱɚ ɛ ɨɞɧɨɝɨ ɡ ɞɪɨɛɿɜ, ɳɨ ɜɯɨɞɹɬɶ ɞɨ ɞɚɧɨɝɨ ɜɢɪɚɡɭ. 3. Ɍɨɬɨɠɧɨ ɪɿɜɧɿ ɜɢɪɚɡɢ. Ɍɨɬɨɠɧɨɫɬɿ. Ɋɨɡɝɥɹɧɟɦɨ ɰɿɥɢɣ ɜɢɪɚɡ x2 + x(2 – x). Ɉɫɤɿɥɶɤɢ ɯ2 + ɯ(2 – ɯ) = ɯ2 + 2ɯ – ɯ2 = 2ɯ, ɬɨ ɞɥɹ ɛɭɞɶ-ɹɤɨɝɨ ɡɧɚɱɟɧɧɹ ɡɦɿɧɧɨʀ x ɜɿɞɩɨɜɿɞɧɿ ɡɧɚɱɟɧɧɹ ɜɢɪɚɡɿɜ ɯ2 + ɯ(2 – ɯ) ɿ 2x ɞɨɪɿɜɧɸɸɬɶ ɨɞɧɟ ɨɞɧɨɦɭ. Ɍɚɤɿ ɰɿɥɿ ɜɢɪɚɡɢ ɦɢ ɧɚɡɢɜɚɥɢ ɬɨɬɨɠɧɨ ɪɿɜɧɢɦɢ. Ⱥ ɹɤɿ ɞɜɚ ɧɟ ɰɿɥɿ ɜɢɪɚɡɢ ɜɜɚɠɚɸɬɶ ɬɨɬɨɠɧɨ ɪɿɜɧɢɦɢ?
  • 9. 8 § 1. Ɋɚɰɿɨɧɚɥɶɧɿ ɜɢɪɚɡɢ Ɋɨɡɝɥɹɧɟɦɨ ɞɪɨɛɨɜɿ ɜɢɪɚɡɢ 2 (2 ) 1 x x x x + − − ɿ 2 1 x x − . Ⱦɨɩɭɫɬɢɦɢɦɢ ɡɧɚɱɟɧɧɹ- ɦɢ ɡɦɿɧɧɨʀ ɨɛɨɯ ɜɢɪɚɡɿɜ ɽ ɜɫɿ ɡɧɚɱɟɧɧɹ x, ɤɪɿɦ ɯ = 1. ɐɿ ɜɢɪɚɡɢ ɦɚɸɬɶ ɨɞɧɚɤɨɜɿ ɡɧɚɦɟɧɧɢɤɢ ɣ ɬɨɬɨɠɧɨ ɪɿɜɧɿ ɱɢɫɟɥɶɧɢɤɢ. Ɍɨɦɭ ɞɥɹ ɤɨɠɧɨɝɨ ɞɨɩɭɫɬɢɦɨɝɨ ɡɧɚɱɟɧɧɹ ɯ ɜɿɞɩɨɜɿɞɧɿ ɡɧɚɱɟɧɧɹ ɜɢɪɚɡɿɜ ɞɨɪɿɜɧɸɸɬɶ ɨɞɧɟ ɨɞɧɨɦɭ. Ɍɚɤɿ ɜɢɪɚɡɢ ɧɚɡɢɜɚɸɬɶ ɬɨɬɨɠɧɨ ɪɿɜɧɢɦɢ. Ɉɡɧɚɱɟɧɧɹ Ⱦɜɚ ɜɢɪɚɡɢ ɧɚɡɢɜɚɸɬɶ ɬɨɬɨɠɧɨ ɪɿɜɧɢɦɢ, ɹɤɳɨ ɞɥɹ ɛɭɞɶ- ɹɤɢɯ ɞɨɩɭɫɬɢɦɢɯ ɞɥɹ ɧɢɯ ɡɧɚɱɟɧɶ ɡɦɿɧɧɢɯ ɜɿɞɩɨɜɿɞɧɿ ɡɧɚ- ɱɟɧɧɹ ɜɢɪɚɡɿɜ ɞɨɪɿɜɧɸɸɬɶ ɨɞɧɟ ɨɞɧɨɦɭ. əɤɳɨ ɞɜɚ ɬɨɬɨɠɧɨ ɪɿɜɧɿ ɜɢɪɚɡɢ 2 (2 ) 1 x x x x + − − ɬɚ 2 1 x x − ɫɩɨɥɭɱɢɬɢ ɡɧɚɤɨɦ «=», ɬɨ ɨɞɟɪɠɢɦɨ ɪɿɜɧɿɫɬɶ 2 (2 ) 1 x x x x + − − = 2 1 x x − , ɹɤɚ ɽ ɩɪɚɜɢɥɶɧɨɸ ɞɥɹ ɜɫɿɯ ɞɨɩɭɫɬɢɦɢɯ ɡɧɚɱɟɧɶ ɯ. Ɍɚɤɭ ɪɿɜɧɿɫɬɶ ɧɚɡɢɜɚɸɬɶ ɬɨɬɨɠɧɿɫɬɸ. Ɉɡɧɚɱɟɧɧɹ Ɋɿɜɧɿɫɬɶ, ɹɤɚ ɽ ɩɪɚɜɢɥɶɧɨɸ ɞɥɹ ɜɫɿɯ ɞɨɩɭɫɬɢɦɢɯ ɡɧɚɱɟɧɶ ɡɦɿɧɧɢɯ, ɳɨ ɜɯɨɞɹɬɶ ɞɨ ɧɟʀ, ɧɚɡɢɜɚɸɬɶ ɬɨɬɨɠɧɿɫɬɸ. ɇɚɩɪɢɤɥɚɞ, 2 2 2 , 3 3 9 ab a a b a b ab ⋅ = ⋅ 2 2 ( )( ) xy xy x y x yx y = − +− — ɬɨɬɨɠɧɨɫɬɿ. Ɂɚɦɿɧɭ ɨɞɧɨɝɨ ɜɢɪɚɡɭ ɬɨɬɨɠɧɨ ɪɿɜɧɢɦ ɣɨɦɭ ɜɢɪɚɡɨɦ ɧɚɡɢɜɚɸɬɶ ɬɨ- ɬɨɠɧɢɦ ɩɟɪɟɬɜɨɪɟɧɧɹɦ ɜɢɪɚɡɭ. ȼɩɪɚɜɚ 1. Ɂɧɚɣɬɢ ɡɧɚɱɟɧɧɹ ɜɢɪɚɡɭ 28 3 x x + + , ɹɤɳɨ ɯ = 4; ɯ = 1 3 . Ɣ əɤɳɨ ɯ = 4, ɬɨ 28 3 x x + + = 284 4 3 + + = 284 7 + = 4 + 4 = 8. əɤɳɨ ɯ = 1 3 , ɬɨ 28 3 x x + + = 1 28 3 1 3 3 + + = 1 28 3 10 3 + = 1 328 3 10 + ⋅ = = 1 42 3 5 + = 1 28 3 5 + = 118 . 15 Ɣ
  • 10. 1. Ɋɚɰɿɨɧɚɥɶɧɿ ɜɢɪɚɡɢ. Ɋɚɰɿɨɧɚɥɶɧɿ ɞɪɨɛɢ 9 ȼɩɪɚɜɚ 2. Ɂɧɚɣɬɢ ɡɧɚɱɟɧɧɹ ɜɢɪɚɡɭ 2 ( )( )a b a b b a b − + + + , ɹɤɳɨ: ɚ) a = 8; b = 32; ɛ) a = 0,6; b = –0,6. Ɣ ɋɩɪɨɫɬɢɦɨ ɞɚɧɢɣ ɜɢɪɚɡ: 2 ( )( )a b a b b a b − + + + = 2 2 2 a b b a b − + + = 2 .a a b+ ɚ) əɤɳɨ a = 8; b = 32, ɬɨ 2 a a b+ = 2 8 8 32+ = 64 40 = 1,6. ɛ) əɤɳɨ a = 0,6; b = –0,6, ɬɨ 22 0,6 0,36 0,6 0,6 0 a a b = = + − — ɧɟ ɦɚɽ ɡɦɿɫɬɭ. Ɣ ȼɩɪɚɜɚ 3. ȼɤɚɡɚɬɢ ɞɨɩɭɫɬɢɦɿ ɡɧɚɱɟɧɧɹ ɡɦɿɧɧɨʀ ɭ ɜɢɪɚɡɿ: ɚ) 4 ; 3 y y y + + − ɛ) 2 2 1 ;a a a − + ɜ) 2 4 . 8 x x + + Ɣ ɚ) Ⱦɨɩɭɫɬɢɦɢɦɢ ɽ ɜɫɿ ɡɧɚɱɟɧɧɹ ɭ, ɤɪɿɦ ɭ = 3. ɛ) Ɂɧɚɣɞɟɦɨ ɡɧɚɱɟɧɧɹ ɚ, ɞɥɹ ɹɤɢɯ ɡɧɚɦɟɧɧɢɤ ɞɪɨɛɭ ɞɨɪɿɜɧɸɽ ɧɭɥɸ: ɚ2 + ɚ = 0; ɚ(ɚ + 1) = 0; ɚ = 0 ɚɛɨ ɚ + 1 = 0; ɚ = 0 ɚɛɨ ɚ = –1. Ⱦɨɩɭɫɬɢɦɢɦɢ ɽ ɜɫɿ ɡɧɚɱɟɧɧɹ ɚ, ɤɪɿɦ ɚ = 0 ɿ ɚ = –1. ɜ) Ⱦɥɹ ɛɭɞɶ-ɹɤɨɝɨ ɡɧɚɱɟɧɧɹ ɯ ɡɧɚɱɟɧɧɹ ɡɧɚɦɟɧɧɢɤɚ ɯ2 + 8 ɧɟ ɦɟɧɲɟ ɧɿɠ 8, ɚ ɬɨɦɭ ɧɟ ɞɨɪɿɜɧɸɽ ɧɭɥɸ. Ɉɬɠɟ, ɞɨɩɭɫɬɢɦɢɦɢ ɽ ɜɫɿ ɡɧɚɱɟɧɧɹ ɯ. Ɣ 1. əɤɿ ɡ ɜɢɪɚɡɿɜ ɽ ɰɿɥɢɦɢ ɜɢɪɚɡɚɦɢ? ɞɪɨɛɨɜɢɦɢ? əɤɿ ɡ ɜɢɪɚɡɿɜ ɽ ɞɪɨɛɚɦɢ? ɪɚɰɿɨɧɚɥɶɧɢɦɢ ɞɪɨɛɚɦɢ? ɚ) ;a b a b + − ɛ) 2 ; 3 x x+ ɜ) 24 ;x x − ɝ) ( )31 ; 2 b a+ ⋅ ɞ) 5 ; ( 1)x y + ɟ) . 5 xy x+ 2. Ⱦɥɹ ɹɤɢɯ ɡɧɚɱɟɧɶ ɡɦɿɧɧɨʀ ɜɢɪɚɡ ɧɟ ɦɚɽ ɡɦɿɫɬɭ? ɇɚɡɜɿɬɶ ɞɨɩɭɫɬɢɦɿ ɡɧɚɱɟɧɧɹ ɡɦɿɧɧɨʀ ɭ ɜɢɪɚɡɿ: ɚ) 8; c ɛ) 9 ; 1 x x − − ɜ) 4 . ( 2) b b b + −
  • 11. 10 § 1. Ɋɚɰɿɨɧɚɥɶɧɿ ɜɢɪɚɡɢ 3. Ɂɧɚɣɞɿɬɶ ɡɧɚɱɟɧɧɹ ɜɢɪɚɡɭ 10 a , ɹɤɳɨ a = 10; a = –1; a = 2. 4. əɤɿ ɡ ɪɿɜɧɨɫɬɟɣ ɽ ɬɨɬɨɠɧɨɫɬɹɦɢ? ɚ) 3 4 ; 1 1 a a a a a + = − − ɛ) 3 3 ; 1 1 a a a a a ⋅ = − − ɜ) 2 ( ) b b a a b a ab = + + . Ɂɧɚɣɞɿɬɶ ɡɧɚɱɟɧɧɹ ɜɢɪɚɡɭ: 5. ɚ) 2 5 x x − + , ɹɤɳɨ ɯ = 0; ɯ = 5; ɯ = –3; ɛ) 2ab a b− , ɹɤɳɨ ɚ = 4, b = 2; ɚ = –4, b = 6. 6. ɚ) 2 ( ) 4 y y − − , ɹɤɳɨ ɭ = 0; ɭ = 6; ɭ = –1; ɛ) 2 2 b c b c + − , ɹɤɳɨ b = 3, c = 4. Ɂɚɩɨɜɧɿɬɶ ɬɚɛɥɢɰɸ: 7. ɯ –2 –1 0 1 1,5 2 1 x x + 8. ɚ –4 –1 0 1 2 2,5 3 2a − ɍɤɚɠɿɬɶ ɞɨɩɭɫɬɢɦɿ ɡɧɚɱɟɧɧɹ ɡɦɿɧɧɨʀ ɭ ɜɢɪɚɡɿ: 9. ɚ) 2 6 1; 2 x x + − ɛ) 6 1 ; ( 3) a a a + − ɜ) 1 ; 1 b b b + + ɝ) 2 11 . 2 x x + 10. ɚ) 3 1 ; 3 y y − + ɛ) 5 1 ; 2 2x x − − ɜ) ; ( 1)( 1) m m m− + ɝ) 2 1 . 1 a a + + 11. Ⱥɜɬɨɦɨɛɿɥɶ ɩɪɨʀɯɚɜ 195 ɤɦ ɡɚ t ɝɨɞ. Ɂɚɩɢɲɿɬɶ ɭ ɜɢɝɥɹɞɿ ɜɢɪɚɡɭ ɲɜɢɞ- ɤɿɫɬɶ ɚɜɬɨɦɨɛɿɥɹ. Ɂɧɚɣɞɿɬɶ ɡɧɚɱɟɧɧɹ ɰɶɨɝɨ ɜɢɪɚɡɭ, ɹɤɳɨ t = 3. 12. Ɉɩɟɪɚɬɨɪ ɧɚɛɪɚɜ 45 ɫɬɨɪɿɧɨɤ ɬɟɤɫɬɭ ɡɚ k ɝɨɞ. Ɂɚɩɢɲɿɬɶ ɭ ɜɢɝɥɹɞɿ ɜɢɪɚɡɭ ɤɿɥɶɤɿɫɬɶ ɫɬɨɪɿɧɨɤ, ɹɤɿ ɨɩɟɪɚɬɨɪ ɧɚɛɢɪɚɜ ɡɚ 1 ɝɨɞ. Ɂɧɚɣɞɿɬɶ ɡɧɚɱɟɧɧɹ ɰɶɨɝɨ ɜɢɪɚɡɭ, ɹɤɳɨ k = 9.
  • 12. 1. Ɋɚɰɿɨɧɚɥɶɧɿ ɜɢɪɚɡɢ. Ɋɚɰɿɨɧɚɥɶɧɿ ɞɪɨɛɢ 11 Ⱦɥɹ ɹɤɢɯ ɡɧɚɱɟɧɶ ɡɦɿɧɧɨʀ ɜɢɪɚɡ ɧɟ ɦɚɽ ɡɦɿɫɬɭ? 13. ɚ) 2 4 1; 4 x x + − ɛ) 2 8 ; 5 a a a− ɜ) 2 5 ( 6) y y − − . 14. ɚ) 2 3 ; 7 x x x− ɛ) 2 2 7 ; 9 z z + − ɜ) 2 1 2 ( 2) b b b − + . Ɂɧɚɣɞɿɬɶ ɞɨɩɭɫɬɢɦɿ ɡɧɚɱɟɧɧɹ ɡɦɿɧɧɨʀ ɭ ɜɢɪɚɡɿ: 15. ɚ) 2 7 ; 4 1 b b b + − ɛ) 2 3 ; 4 ( 2) k k− + ɜ) 2 6 . 12 m m mm m + −+ 16. ɚ) 2 5 ; 4 9 c c− ɛ) 2 3 2 ; (3 ) 9 − + − n n ɜ) 2 5 1. 4 a a aa ++ − Ɂɧɚɣɞɿɬɶ ɡɧɚɱɟɧɧɹ ɜɢɪɚɡɭ: 17. 2 3 3 1 a a − + , ɹɤɳɨ ɚ = –0,2; ɚ = 2 3 ; ɚ = 13 6 . 18. 2 5 3 x x − − , ɹɤɳɨ ɯ = 0,7; ɯ = 3 7 ; ɯ = 11 5 . 19. Ɂɧɚɣɞɿɬɶ ɡɧɚɱɟɧɧɹ ɜɢɪɚɡɭ 2 2 2x xy y y − + , ɹɤɳɨ: ɚ) ɯ = 44; ɭ = 4; ɛ) ɯ = 46; ɭ = 46; ɜ) ɯ = 1,25; ɭ = 0,25. 20. Ɂɧɚɣɞɿɬɶ ɡɧɚɱɟɧɧɹ ɜɢɪɚɡɭ (1 ) (1 ) 4 m n n m n − + + , ɹɤɳɨ: ɚ) m = 67; n = –67; ɛ) m = 16,75; n = 0,25. 21. Ⱦɨ ɦɚɝɚɡɢɧɭ ɡɚɜɟɡɥɢ 15 ɥ ɜɢɧɨɝɪɚɞɧɨɝɨ ɫɨɤɭ ɜ ɦɚɥɢɯ ɭɩɚɤɨɜɤɚɯ ɿ 25 ɥ — ɭ ɜɟɥɢɤɢɯ. ɋɤɿɥɶɤɢ ɜɫɶɨɝɨ ɭɩɚɤɨɜɨɤ ɫɨɤɭ ɡɚɜɟɡɥɢ ɞɨ ɦɚɝɚɡɢɧɭ, ɹɤɳɨ ɤɨ- ɠɧɚ ɦɚɥɚ ɭɩɚɤɨɜɤɚ ɦɿɫɬɢɬɶ ɚ ɥ ɫɨɤɭ, ɚ ɤɨɠɧɚ ɜɟɥɢɤɚ — b ɥ? 22. ɉɟɪɲɢɣ ɪɨɛɿɬɧɢɤ ɜɢɤɥɚɜ ɩɥɢɬɤɨɸ 48 ɦ2 ɞɨɪɿɠɤɢ ɡɚ n ɝɨɞ, ɚ ɞɪɭɝɢɣ — 64 ɦ2 ɡɚ m ɝɨɞ. ɋɤɿɥɶɤɢ ɤɜɚɞɪɚɬɧɢɯ ɦɟɬɪɿɜ ɞɨɪɿɠɤɢ ɜɢɤɥɚɞɚɥɢ ɡɚ 1 ɝɨɞ ɨɛɢɞɜɚ ɪɨɛɿɬɧɢɤɢ ɪɚɡɨɦ? 23. Ʉɚɬɟɪ ɩɪɨɣɲɨɜ 25 ɤɦ ɡɚ ɬɟɱɿɽɸ ɪɿɱɤɢ ɿ 20 ɤɦ ɩɪɨɬɢ ɬɟɱɿʀ. Ɂɧɚɣɞɿɬɶ ɱɚɫ ɪɭɯɭ ɤɚɬɟɪɚ, ɹɤɳɨ ɣɨɝɨ ɲɜɢɞɤɿɫɬɶ ɭ ɫɬɨɹɱɿɣ ɜɨɞɿ ɞɨɪɿɜɧɸɽ v ɤɦ/ɝɨɞ, ɚ ɲɜɢɞɤɿɫɬɶ ɬɟɱɿʀ ɪɿɱɤɢ — u ɤɦ/ɝɨɞ.
  • 13. 12 § 1. Ɋɚɰɿɨɧɚɥɶɧɿ ɜɢɪɚɡɢ 24. Ɂɧɚɣɞɿɬɶ ɞɨɩɭɫɬɢɦɿ ɡɧɚɱɟɧɧɹ ɡɦɿɧɧɨʀ ɭ ɜɢɪɚɡɿ: ɚ) 11 1; 3 x x − − ɛ) 3 ; y y y− ɜ) 2 ; 2 m m m− ɝ) 3 . 1 1 a a + − + 25. Ⱦɨɜɟɞɿɬɶ, ɳɨ ɜɢɪɚɡ 2 2 4 4 9 x y x y x y + + − − + ɦɚɽ ɡɦɿɫɬ ɞɥɹ ɛɭɞɶ-ɹɤɢɯ ɡɧɚɱɟɧɶ ɡɦɿɧɧɢɯ. 26. ɉɨʀɡɞ ɦɚɜ ɩɨɞɨɥɚɬɢ ɲɥɹɯ ɡɚɜɞɨɜɠɤɢ 250 ɤɦ, ɪɭɯɚɸɱɢɫɶ ɡɿ ɲɜɢɞɤɿɫɬɸ ɚ ɤɦ/ɝɨɞ. Ⱥɥɟ ɱɟɪɟɡ 2 ɝɨɞ ɩɿɫɥɹ ɩɨɱɚɬɤɭ ɪɭɯɭ ɣɨɝɨ ɧɚ ɩɟɜɧɢɣ ɱɚɫ ɡɚɬɪɢɦɚɥɢ. ɓɨɛ ɩɪɢɛɭɬɢ ɞɨ ɦɿɫɰɹ ɩɪɢɡɧɚɱɟɧɧɹ ɜɱɚɫɧɨ, ɩɨʀɡɞ ɩɿɫɥɹ ɡɚɬɪɢɦɤɢ ɡɛɿɥɶɲɢɜ ɲɜɢɞɤɿɫɬɶ ɧɚ 25 ɤɦ/ɝɨɞ. ɇɚ ɹɤɢɣ ɱɚɫ ɡɚɬɪɢɦɚɥɢ ɩɨʀɡɞ? 27. Ɋɨɡɤɥɚɞɿɬɶ ɧɚ ɦɧɨɠɧɢɤɢ: ɚ) ab2 – ac2 ; ɛ) ɯ3 + 8; ɜ) xy + 8x + 9ɭ + 72; ɝ) a2 – 4b2 + a + 2b. 28. ɉɨɪɿɜɧɹɣɬɟ ɞɪɨɛɢ: 7 9 ɿ 20 ; 27 11 18 ɿ 17 ; 24 7 15 ɿ 9 . 25 29. ɋɤɨɪɨɬɿɬɶ ɞɪɨɛɢ: 18 ; 48 56 ; 98 96 ; 123 175 ; 325 77 . 121 30. ɒɤɨɥɿ ɩɨɬɪɿɛɧɨ ɡɚɤɭɩɢɬɢ ɩɚɪɬɢ. ɉɟɪɲɚ ɮɿɪɦɚ ɩɪɨɩɨɧɭɽ ɤɭɩɢɬɢ ɩɚɪɬɢ ɩɨ 975 ɝɪɧ ɡɚ ɤɨɠɧɭ ɿ 4% ɜɚɪɬɨɫɬɿ ɭɫɿɯ ɤɭɩɥɟɧɢɯ ɩɚɪɬ ɡɚ ɞɨɫɬɚɜɤɭ, ɚ ɞɪɭɝɚ — ɩɨ 1010 ɝɪɧ ɡɚ ɤɨɠɧɭ ɿ ɛɟɡɤɨɲɬɨɜɧɭ ɞɨɫɬɚɜɤɭ. ɍ ɹɤɿɣ ɮɿɪɦɿ ɜɢɝɿɞɧɿɲɟ ɤɭɩɭɜɚɬɢ ɩɚɪɬɢ? 31. ɇɚ ɱɚɪɿɜɧɿɣ ɹɛɥɭɧɿ ɪɨɫɬɭɬɶ 55 ɹɛɥɭɤ. Ɂ ɧɟʀ ɞɨɡɜɨɥɹɸɬɶ ɡɪɢɜɚɬɢ 2, 3, 6 ɚɛɨ 9 ɹɛɥɭɤ. Ɂɚɦɿɫɬɶ ɧɢɯ ɧɚ ɹɛɥɭɧɿ ɜɿɞɪɚɡɭ ɜɢɪɨɫɬɚɸɬɶ ɜɿɞɩɨɜɿɞɧɨ 4, 5, 2 ɚɛɨ 7 ɧɨɜɢɯ ɹɛɥɭɤ. ɑɢ ɦɨɠɧɚ ɡ ɹɛɥɭɧɿ ɡɿɪɜɚɬɢ ɜɫɿ ɹɛɥɭɤɚ, ɹɤɳɨ ɩɿɫɥɹ ɡɪɢɜɚɧɧɹ ɨɫɬɚɧɧɶɨɝɨ ɜɨɧɢ ɛɿɥɶɲɟ ɧɟ ɜɢɪɨɫɬɚɸɬɶ?
  • 14. 2. Ɉɫɧɨɜɧɚ ɜɥɚɫɬɢɜɿɫɬɶ ɞɪɨɛɭ 13 1. Ɉɫɧɨɜɧɚ ɜɥɚɫɬɢɜɿɫɬɶ ɞɪɨɛɭ. ɉɪɢɝɚɞɚɣɦɨ ɨɫɧɨɜɧɭ ɜɥɚɫɬɢɜɿɫɬɶ ɡɜɢ- ɱɚɣɧɢɯ ɞɪɨɛɿɜ: ɹɤɳɨ ɱɢɫɟɥɶɧɢɤ ɿ ɡɧɚɦɟɧɧɢɤ ɞɪɨɛɭ ɩɨɦɧɨɠɢɬɢ ɚɛɨ ɩɨɞɿɥɢɬɢ ɧɚ ɬɟ ɫɚɦɟ ɧɚɬɭɪɚɥɶɧɟ ɱɢɫɥɨ, ɬɨ ɨɬɪɢɦɚɽɦɨ ɞɪɿɛ, ɹɤɢɣ ɞɨɪɿɜɧɸɽ ɞɚɧɨɦɭ. Ɉɬ- ɠɟ, ɹɤɳɨ a, b ɿ k — ɧɚɬɭɪɚɥɶɧɿ ɱɢɫɥɚ, ɬɨ a ak b bk = ɿ .ak a bk b = Ⱥɧɚɥɨɝɿɱɧɚ ɜɥɚɫɬɢɜɿɫɬɶ ɫɩɪɚɜɟɞɥɢɜɚ ɞɥɹ ɛɭɞɶ-ɹɤɢɯ ɞɪɨɛɿɜ. Ⱥ ɫɚɦɟ: Ⱦɥɹ ɛɭɞɶ-ɹɤɢɯ ɡɧɚɱɟɧɶ ɚ, b ɿ k, ɞɟ b ≠ 0 ɿ k ≠ 0, ɜɢɤɨɧɭɸɬɶɫɹ ɪɿɜɧɨɫɬɿ ; a ak b bk = . ak a bk b = Ⱦɚɧɿ ɪɿɜɧɨɫɬɿ ɽ ɬɨɬɨɠɧɨɫɬɹɦɢ ɿ ɜɢɪɚɠɚɸɬɶ ɨɫɧɨɜɧɭ ɜɥɚɫɬɢɜɿɫɬɶ ɞɪɨɛɭ, ɹɤɭ ɦɨɠɧɚ ɫɮɨɪɦɭɥɸɜɚɬɢ ɬɚɤ: əɤɳɨ ɱɢɫɟɥɶɧɢɤ ɿ ɡɧɚɦɟɧɧɢɤ ɞɪɨɛɭ ɩɨɦɧɨɠɢɬɢ ɚɛɨ ɩɨɞɿɥɢɬɢ ɧɚ ɜɢɪɚɡ, ɧɟ ɬɨɬɨɠɧɨ ɪɿɜɧɢɣ ɧɭɥɸ, ɬɨ ɨɞɟɪɠɢɦɨ ɞɪɿɛ, ɬɨɬɨɠɧɨ ɪɿɜɧɢɣ ɞɚɧɨɦɭ. 2. ɋɤɨɪɨɱɟɧɧɹ ɞɪɨɛɿɜ. Ɂɚ ɞɨɩɨɦɨɝɨɸ ɬɨɬɨɠɧɨɫɬɿ ak a bk b = ɞɪɿɛ ak bk ɦɨɠ- ɧɚ ɡɚɦɿɧɢɬɢ ɞɪɨɛɨɦ ,a b ɬɨɛɬɨ ɞɪɿɛ ak bk ɦɨɠɧɚ ɫɤɨɪɨɬɢɬɢ ɧɚ ɫɩɿɥɶɧɢɣ ɦɧɨɠ- ɧɢɤ k ɱɢɫɟɥɶɧɢɤɚ ɿ ɡɧɚɦɟɧɧɢɤɚ. ɇɚɩɪɢɤɥɚɞ, 2 4 2 2 2 ; 2 2 n n n n mn m n m ⋅= = ⋅ 2 2 2 ( )( ) . ( ) a b a ba b a b a a b aa ab + −− + = = −− Ɋɿɜɧɨɫɬɿ 2 4 2 2 n n mn m = ɿ 2 2 2 a b a b aa ab − + = − ɽ ɬɨɬɨɠɧɨɫɬɹɦɢ, ɬɨɛɬɨ ɜɨɧɢ ɽ ɩɪɚ- ɜɢɥɶɧɢɦɢ ɞɥɹ ɜɫɿɯ ɞɨɩɭɫɬɢɦɢɯ ɡɧɚɱɟɧɶ ɡɦɿɧɧɢɯ (ɩɟɪɲɚ — ɞɥɹ ɜɫɿɯ ɡɧɚɱɟɧɶ m ɿ n, ɞɟ m ≠ 0, n ≠ 0; ɞɪɭɝɚ — ɞɥɹ ɜɫɿɯ ɡɧɚɱɟɧɶ ɚ ɿ b, ɞɟ ɚ ≠ 0, a ≠ b). ɓɨɛ ɫɤɨɪɨɬɢɬɢ ɞɪɿɛ, ɩɨɬɪɿɛɧɨ: 1) ɜɢɞɿɥɢɬɢ ɫɩɿɥɶɧɢɣ ɦɧɨɠɧɢɤ ɱɢɫɟɥɶɧɢɤɚ ɿ ɡɧɚɦɟɧɧɢɤɚ ɞɪɨɛɭ; 2) ɜɢɤɨɧɚɬɢ ɫɤɨɪɨɱɟɧɧɹ ɧɚ ɫɩɿɥɶɧɢɣ ɦɧɨɠɧɢɤ.
  • 15. 14 § 1. Ɋɚɰɿɨɧɚɥɶɧɿ ɜɢɪɚɡɢ 3. Ɂɜɟɞɟɧɧɹ ɞɪɨɛɿɜ ɞɨ ɫɩɿɥɶɧɨɝɨ ɡɧɚɦɟɧɧɢɤɚ. Ɂɚ ɞɨɩɨɦɨɝɨɸ ɬɨɬɨɠɧɨɫɬɿ a ak b bk = ɞɪɿɛ a b ɦɨɠɧɚ ɡɜɨɞɢɬɢ ɞɨ ɧɨɜɨɝɨ ɡɧɚɦɟɧɧɢɤɚ. ɇɚɩɪɢɤɥɚɞ, 2 3 3 2 6 2 2 x x x x y y x xy ⋅= = ⋅ — ɡɜɟɥɢ ɞɪɿɛ 3x y ɞɨ ɡɧɚɦɟɧɧɢɤɚ 2ɯɭ. Ȼɭɞɶ-ɹɤɿ ɞɪɨɛɢ ɡ ɪɿɡɧɢɦɢ ɡɧɚɦɟɧɧɢɤɚɦɢ, ɹɤ ɿ ɡɜɢɱɚɣɧɿ ɞɪɨɛɢ, ɦɨɠɧɚ ɡɜɟɫ- ɬɢ ɞɨ ɫɩɿɥɶɧɨɝɨ ɡɧɚɦɟɧɧɢɤɚ. Ɋɨɡɝɥɹɧɟɦɨ ɩɪɢɤɥɚɞɢ. ɉɪɢɤɥɚɞ 1. Ɂɜɟɫɬɢ ɞɨ ɫɩɿɥɶɧɨɝɨ ɡɧɚɦɟɧɧɢɤɚ ɞɪɨɛɢ 2 3 y x ɬɚ 4 . y Ɣ ɋɩɿɥɶɧɢɦ ɡɧɚɦɟɧɧɢɤɨɦ ɞɚɧɢɯ ɞɪɨɛɿɜ ɽ ɞɨɛɭɬɨɤ ʀɯɧɿɯ ɡɧɚɦɟɧɧɢɤɿɜ, ɬɨɛɬɨ 3ɯ2 ɭ. Ⱦɨɞɚɬɤɨɜɢɦ ɦɧɨɠɧɢɤɨɦ ɞɥɹ ɩɟɪɲɨɝɨ ɞɪɨɛɭ ɽ ɭ, ɞɥɹ ɞɪɭɝɨɝɨ — 3ɯ2 . Ɍɨɞɿ: 2 2 2 2 ; 3 3 3 y y y y x x y x y ⋅ = = ⋅ 2 2 2 2 4 4 3 12 . 3 3 x x y y x x y ⋅= = ⋅ Ɣ ɉɪɢɤɥɚɞ 2. Ɂɜɟɫɬɢ ɞɨ ɫɩɿɥɶɧɨɝɨ ɡɧɚɦɟɧɧɢɤɚ ɞɪɨɛɢ 3 5 8a b ɿ 2 2 7 . 12a c Ɣ Ɂɧɚɦɟɧɧɢɤɢ ɨɛɨɯ ɞɪɨɛɿɜ ɽ ɨɞɧɨɱɥɟɧɚɦɢ, ɬɨɦɭ ɫɩɿɥɶɧɢɣ ɡɧɚɦɟɧɧɢɤ ɲɭɤɚɬɢɦɟɦɨ ɭ ɜɢɝɥɹɞɿ ɨɞɧɨɱɥɟɧɚ, ɞɨ ɬɨɝɨ ɠ ɹɤɨɦɨɝɚ ɦɟɧɲɨɝɨ ɫɬɟɩɟɧɹ. Ɂɚ ɤɨ- ɟɮɿɰɿɽɧɬ ɰɶɨɝɨ ɨɞɧɨɱɥɟɧɚ ɜɿɡɶɦɟɦɨ ɧɚɣɦɟɧɲɟ ɫɩɿɥɶɧɟ ɤɪɚɬɧɟ ɤɨɟɮɿɰɿɽɧɬɿɜ ɡɧɚɦɟɧɧɢɤɿɜ ɞɚɧɢɯ ɞɪɨɛɿɜ, ɬɨɛɬɨ 24, ɚ ɤɨɠɧɭ ɡɦɿɧɧɭ ɜɿɡɶɦɟɦɨ ɡ ɧɚɣɛɿɥɶɲɢɦ ɩɨɤɚɡɧɢɤɨɦ, ɡ ɹɤɢɦ ɜɨɧɚ ɜɯɨɞɢɬɶ ɭ ɡɧɚɦɟɧɧɢɤɢ ɞɪɨɛɿɜ, ɬɨɛɬɨ ɜɿɡɶɦɟɦɨ a3 , b ɿ c2 . Ɍɨɞɿ ɫɩɿɥɶɧɢɦ ɡɧɚɦɟɧɧɢɤɨɦ ɛɭɞɟ 24a3 bc2 . Ⱦɨɞɚɬɤɨɜɢɦ ɦɧɨɠɧɢɤɨɦ ɞɥɹ ɩɟɪɲɨɝɨ ɞɪɨɛɭ ɽ 3ɫ2 , ɛɨ 24a3 bc2 = 8a3 b ⋅ 3c2 ; ɞɥɹ ɞɪɭɝɨɝɨ — 2ab, ɛɨ 24a3 bc2 = = 12a2 c2 ⋅ 2ab. Ɇɚɬɢɦɟɦɨ: 2 2 3 3 2 3 2 5 5 3 15 ; 8 24 24 c c a b a bc a bc ⋅= = 2 2 3 2 3 2 7 7 2 14 . 12 24 24 ab ab a c a bc a bc ⋅= = Ɣ ɓɨɛ ɡɜɟɫɬɢ ɞɨ ɩɪɨɫɬɿɲɨɝɨ ɫɩɿɥɶɧɨɝɨ ɡɧɚɦɟɧɧɢɤɚ ɞɪɨɛɢ, ɡɧɚɦɟɧɧɢɤɚɦɢ ɹɤɢɯ ɽ ɨɞɧɨɱɥɟɧɢ, ɩɨɬɪɿɛɧɨ: 1) ɡɧɚɣɬɢ ɧɚɣɦɟɧɲɟ ɫɩɿɥɶɧɟ ɤɪɚɬɧɟ (ɇɋɄ) ɤɨɟɮɿɰɿɽɧɬɿɜ ɡɧɚɦɟɧɧɢɤɿɜ; 2) ɭɬɜɨɪɢɬɢ ɫɩɿɥɶɧɢɣ ɡɧɚɦɟɧɧɢɤ ɭ ɜɢɝɥɹɞɿ ɞɨɛɭɬɤɭ ɇɋɄ ɿ ɫɬɟɩɟɧɿɜ ɡɦɿɧ- ɧɢɯ ɡ ɧɚɣɛɿɥɶɲɢɦ ɩɨɤɚɡɧɢɤɨɦ, ɡ ɹɤɢɦ ɜɨɧɢ ɜɯɨɞɹɬɶ ɞɨ ɡɧɚɦɟɧɧɢɤɿɜ;
  • 16. 2. Ɉɫɧɨɜɧɚ ɜɥɚɫɬɢɜɿɫɬɶ ɞɪɨɛɭ 15 3) ɩɨɦɧɨɠɢɬɢ ɱɢɫɟɥɶɧɢɤ ɿ ɡɧɚɦɟɧɧɢɤ ɤɨɠɧɨɝɨ ɞɪɨɛɭ ɧɚ ɜɿɞɩɨɜɿɞɧɢɣ ɞɨ- ɞɚɬɤɨɜɢɣ ɦɧɨɠɧɢɤ. (ɓɨɛ ɡɧɚɣɬɢ ɞɨɞɚɬɤɨɜɢɣ ɦɧɨɠɧɢɤ ɞɥɹ ɞɪɨɛɭ, ɩɨɬɪɿ- ɛɧɨ ɡɚɩɢɫɚɬɢ ɫɩɿɥɶɧɢɣ ɡɧɚɦɟɧɧɢɤ ɭ ɜɢɝɥɹɞɿ ɞɨɛɭɬɤɭ ɞɜɨɯ ɨɞɧɨɱɥɟɧɿɜ, ɨɞɧɢɦ ɡ ɹɤɢɯ ɽ ɡɧɚɦɟɧɧɢɤ ɞɚɧɨɝɨ ɞɪɨɛɭ. Ɍɨɞɿ ɞɪɭɝɢɣ ɨɞɧɨɱɥɟɧ ɛɭɞɟ ɞɨ- ɞɚɬɤɨɜɢɦ ɦɧɨɠɧɢɤɨɦ.) ɉɪɢɤɥɚɞ 3. Ɂɜɟɫɬɢ ɞɨ ɫɩɿɥɶɧɨɝɨ ɡɧɚɦɟɧɧɢɤɚ ɞɪɨɛɢ 2 3 a ab− ɿ 2 2 . a ab+ Ɣ Ɋɨɡɤɥɚɞɟɦɨ ɧɚ ɦɧɨɠɧɢɤɢ ɡɧɚɦɟɧɧɢɤ ɤɨɠɧɨɝɨ ɞɪɨɛɭ: 2 3 3 ; ( )a a ba ab = −− 2 2 2 . ( )a a ba ab = ++ ɋɩɿɥɶɧɢɦ ɡɧɚɦɟɧɧɢɤɨɦ ɞɪɨɛɿɜ ɽ ɞɨɛɭɬɨɤ ɚ(ɚ – b)(ɚ + b) = ɚ(ɚ2 – b2 ). Ⱦɨɞɚɬ- ɤɨɜɢɦ ɦɧɨɠɧɢɤɨɦ ɞɥɹ ɩɟɪɲɨɝɨ ɞɪɨɛɭ ɽ ɜɢɪɚɡ ɚ + b, ɞɥɹ ɞɪɭɝɨɝɨ — ɜɢɪɚɡ ɚ – b. ɉɨɦɧɨɠɢɜɲɢ ɱɢɫɟɥɶɧɢɤ ɿ ɡɧɚɦɟɧɧɢɤ ɤɨɠɧɨɝɨ ɞɪɨɛɭ ɧɚ ɜɿɞɩɨɜɿɞɧɢɣ ɞɨɞɚɬɤɨ- ɜɢɣ ɦɧɨɠɧɢɤ, ɨɞɟɪɠɢɦɨ: 2 2 2 3( )3 ; ( ) a b a ab a a b + = − − 2 2 2 2( )2 . ( ) a b a ab a a b − = + − Ɣ ɓɨɛ ɡɜɟɫɬɢ ɞɨ ɩɪɨɫɬɿɲɨɝɨ ɫɩɿɥɶɧɨɝɨ ɡɧɚɦɟɧɧɢɤɚ ɞɪɨɛɢ, ɡɧɚɦɟɧɧɢɤɚɦɢ ɹɤɢɯ ɽ ɦɧɨɝɨɱɥɟɧɢ, ɩɨɬɪɿɛɧɨ: 1) ɪɨɡɤɥɚɫɬɢ ɧɚ ɦɧɨɠɧɢɤɢ ɡɧɚɦɟɧɧɢɤ ɤɨɠɧɨɝɨ ɞɪɨɛɭ; 2) ɭɬɜɨɪɢɬɢ ɫɩɿɥɶɧɢɣ ɡɧɚɦɟɧɧɢɤ ɭ ɜɢɝɥɹɞɿ ɞɨɛɭɬɤɭ ɨɞɟɪɠɚɧɢɯ ɦɧɨɠɧɢ- ɤɿɜ ɡ ɧɚɣɛɿɥɶɲɢɦ ɩɨɤɚɡɧɢɤɨɦ, ɡ ɹɤɢɦ ɜɨɧɢ ɜɯɨɞɹɬɶ ɞɨ ɡɧɚɦɟɧɧɢɤɿɜ; 3) ɩɨɦɧɨɠɢɬɢ ɱɢɫɟɥɶɧɢɤ ɿ ɡɧɚɦɟɧɧɢɤ ɤɨɠɧɨɝɨ ɞɪɨɛɭ ɧɚ ɜɿɞɩɨɜɿɞɧɢɣ ɞɨ- ɞɚɬɤɨɜɢɣ ɦɧɨɠɧɢɤ. 4. Ɂɦɿɧɚ ɡɧɚɤɚ ɱɢɫɟɥɶɧɢɤɚ ɚɛɨ ɡɧɚɦɟɧɧɢɤɚ ɞɪɨɛɭ. Ɋɨɡɝɥɹɧɟɦɨ ɩɪɚɜɢ- ɥɶɧɭ ɱɢɫɥɨɜɭ ɪɿɜɧɿɫɬɶ 1 1 . 2 2 − =− Ȳʀ ɦɨɠɧɚ ɩɪɨɤɨɦɟɧɬɭɜɚɬɢ ɬɚɤ: ɹɤɳɨ ɡɦɿɧɢɬɢ ɡɧɚɤ ɭ ɱɢɫɟɥɶɧɢɤɭ ɞɪɨɛɭ ɿ ɡɧɚɤ ɩɟɪɟɞ ɞɪɨɛɨɦ, ɬɨ ɨɞɟɪɠɢɦɨ ɞɪɿɛ, ɹɤɢɣ ɞɨɪɿɜ- ɧɸɽ ɞɚɧɨɦɭ.
  • 17. 16 § 1. Ɋɚɰɿɨɧɚɥɶɧɿ ɜɢɪɚɡɢ ɍ ɬɚɤɢɣ ɠɟ ɫɩɨɫɿɛ ɡɦɿɧɸɸɬɶ ɡɧɚɤ ɱɢɫɟɥɶɧɢɤɚ ɚɛɨ ɡɧɚɦɟɧɧɢɤɚ ɛɭɞɶ- ɹɤɨɝɨ ɞɪɨɛɭ, ɜɢɤɨɪɢɫɬɨɜɭɸɱɢ ɬɨɬɨɠɧɨɫɬɿ: ,a a b b − =− .a a b b =− − əɤɳɨ ɡɦɿɧɢɬɢ ɡɧɚɤ ɭ ɱɢɫɟɥɶɧɢɤɭ ɚɛɨ ɡɧɚɦɟɧɧɢɤɭ ɞɪɨɛɭ ɿ ɡɧɚɤ ɩɟɪɟɞ ɞɪɨ- ɛɨɦ, ɬɨ ɨɞɟɪɠɢɦɨ ɞɪɿɛ, ɬɨɬɨɠɧɨ ɪɿɜɧɢɣ ɞɚɧɨɦɭ. Ⱦɨɜɟɞɟɦɨ ɨɫɧɨɜɧɭ ɜɥɚɫɬɢɜɿɫɬɶ ɞɪɨɛɿɜ. ɉɨɤɚɠɟɦɨ, ɳɨ ɪɿɜɧɿɫɬɶ a ak b bk = ɽ ɬɨɬɨɠ- ɧɿɫɬɸ, ɬɨɛɬɨ ɳɨ ɜɨɧɚ ɜɢɤɨɧɭɽɬɶɫɹ ɞɥɹ ɛɭɞɶ-ɹɤɢɯ ɡɧɚɱɟɧɶ ɚ, b ɿ k, ɞɟ b ≠ 0 ɿ k ≠ 0. ɇɟɯɚɣ .a m b = Ɂɚ ɨɡɧɚɱɟɧɧɹɦ ɱɚɫɬɤɢ ɦɚɽɦɨ: ɚ = bm. ɉɨɦɧɨɠɢɜɲɢ ɨɛɢɞɜɿ ɱɚɫɬɢ- ɧɢ ɨɞɟɪɠɚɧɨʀ ɪɿɜɧɨɫɬɿ ɧɚ k, ɦɚɬɢɦɟɦɨ ɩɪɚɜɢɥɶɧɭ ɪɿɜɧɿɫɬɶ ɚk = (bm)k ɚɛɨ ɚk = (bk)m. Ɉɫɤɿɥɶɤɢ b ≠ 0 ɿ k ≠ 0, ɬɨ bk ≠ 0. ɍ ɬɚɤɨɦɭ ɜɢɩɚɞɤɭ ɡ ɪɿɜɧɨɫɬɿ ɚk = (bk)m ɡɧɨɜɭ ɠ ɬɚɤɢ ɡɚ ɨɡɧɚɱɟɧɧɹɦ ɱɚɫɬɤɢ ɦɚɽɦɨ: .akm bk = Ɉɬɠɟ, .a akm b bk = = ȼɩɪɚɜɚ 1. ȼɢɞɿɥɢɬɢ ɫɩɿɥɶɧɢɣ ɦɧɨɠɧɢɤ ɱɢɫɟɥɶɧɢɤɚ ɬɚ ɡɧɚɦɟɧɧɢɤɚ ɞɪɨɛɭ ɣ ɫɤɨɪɨɬɢɬɢ ɞɪɿɛ: ɚ) 12 ; 8 a ab ɛ) 3 2 2 18 . 6 xy x y − − Ɣ ɚ) 12 8 a ab = 4 3 4 2 a a b ⋅ ⋅ = 3 . 2b ɛ) 3 2 2 18 6 xy x y − − = 2 2 6 3 6 xy y xy x − ⋅ − ⋅ = 3 . y x Ɣ ȼɩɪɚɜɚ 2. ɋɤɨɪɨɬɢɬɢ ɞɪɿɛ: ɚ) 2 2 10 5 ; 4 b a a b − − ɛ) 2 2 3 3 . x xy y x y + + − Ɣ ɚ) 2 2 10 5 4 b a a b − − = 5( 2 ) ( 2 )( 2 ) a b a b a b − − − + = 5 2a b − + = 5 . 2a b − +
  • 18. 2. Ɉɫɧɨɜɧɚ ɜɥɚɫɬɢɜɿɫɬɶ ɞɪɨɛɭ 17 ɛ) 2 2 3 3 x xy y x y + + − = 2 2 2 2 ( )( ) x xy y x y x xy y + + − + + = 1 . x y− Ɣ ȼɩɪɚɜɚ 3. Ɂɜɟɫɬɢ ɞɪɿɛ 3 7 a b ɞɨ ɡɧɚɦɟɧɧɢɤɚ 42ɚ2 b. Ɣ Ɉɫɤɿɥɶɤɢ 42ɚ2 b = 7b ⋅ 6ɚ2 , ɬɨ, ɩɨɦɧɨɠɢɜɲɢ ɱɢɫɟɥɶɧɢɤ ɿ ɡɧɚɦɟɧɧɢɤ ɞɚ- ɧɨɝɨ ɞɪɨɛɭ ɧɚ 6ɚ2 , ɦɚɬɢɦɟɦɨ: 3 7 a b = 2 2 3 6 7 6 a a b a ⋅ ⋅ = 3 2 18 . 42 a a b Ɣ ȼɩɪɚɜɚ 4. Ɂɜɟɫɬɢ ɞɨ ɫɩɿɥɶɧɨɝɨ ɡɧɚɦɟɧɧɢɤɚ ɞɪɨɛɢ 2 2 3 m n− ɿ 9 . n m− • 2 2 3 3 ; ( )( )m n m nm n = − +− 9 9 . n m m n = − − − ɋɩɿɥɶɧɢɦ ɡɧɚɦɟɧɧɢɤɨɦ ɞɪɨɛɿɜ ɽ ɞɨɛɭɬɨɤ (m – n)(m + n). Ⱦɨɞɚɬɤɨɜɢɦ ɦɧɨɠɧɢɤɨɦ ɞɥɹ ɩɟɪɲɨɝɨ ɞɪɨɛɭ ɽ 1, ɞɥɹ ɞɪɭɝɨɝɨ — m + n. Ɍɨɦɭ ɩɟɪɲɢɣ ɞɪɿɛ ɡɚɥɢɲɚɽɦɨ ɛɟɡ ɡɦɿɧɢ, ɚ ɞɥɹ ɞɪɭɝɨ- ɝɨ ɞɪɨɛɭ ɦɚɬɢɦɟɦɨ: 2 2 9( ) 9( )9 . ( )( ) m n m n n m m n m n m n + + = − = − − − + − • 32. ɋɤɨɪɨɬɿɬɶ ɞɪɿɛ: ɚ) 5 ; 15 x y ɛ) ; 4 ab b ɜ) ( 2) ; ( 2) m n n n − − ɝ) 2 3 18 .a a 33. Ɂɜɟɞɿɬɶ ɞɪɿɛ: ɚ) 11 b ɞɨ ɡɧɚɦɟɧɧɢɤɚ b2 ; ɛ) 3 2 x y ɞɨ ɡɧɚɦɟɧɧɢɤɚ 4ɯɭ. 34. Ɂɜɟɞɿɬɶ ɞɨ ɫɩɿɥɶɧɨɝɨ ɡɧɚɦɟɧɧɢɤɚ ɞɪɨɛɢ: ɚ) a b ɿ 3 b ; ɛ) 1 mn ɿ 2 1 n . 35. Ɂɦɿɧɿɬɶ ɡɧɚɤ ɡɧɚɦɟɧɧɢɤɚ ɞɪɨɛɭ: ɚ) 2 ( )x y− − ; ɛ) 2 x y− .
  • 19. 18 § 1. Ɋɚɰɿɨɧɚɥɶɧɿ ɜɢɪɚɡɢ 36. ȼɢɞɿɥɿɬɶ ɫɩɿɥɶɧɢɣ ɦɧɨɠɧɢɤ ɱɢɫɟɥɶɧɢɤɚ ɬɚ ɡɧɚɦɟɧɧɢɤɚ ɞɪɨɛɭ ɣ ɫɤɨɪɨɬɿɬɶ ɞɪɿɛ: ɚ) 3 ; 5 x x ɛ) 4 ; 6 a a ɜ) 9 ; 6 ab b ɝ) 2 2 10 . 15 x y xy − ɋɤɨɪɨɬɿɬɶ ɞɪɿɛ: 37. ɚ) 2 2 2 3 28 ; 35 x y x y ɛ) 2 2 24 ; 36 b c bc ɜ) 2 2 2 15 ; 40 mn m n − ɝ) 2 4 4 3 8 . 12 k m k m− 38. ɚ) 2 2 3 18 ; 12 c n n ɛ) 2 36 ; 28 xy xy ɜ) 2 2 3 40 ; 24 ab a b− ɝ) 3 3 14 . 42 ac bc − − 39. ɚ) ( ) ; a m n m n − − ɛ) ( ) ; 3 ( ) b c d b c d + + ɜ) 5 ; 15 20 k k + ɝ) 2 .m mn mn − 40. ɚ) ( ) ; ( ) ab a b c a b + + ɛ) ( 2 ) ; ( ) m x y m x y − − ɜ) 3 9 ; 3 x x − − ɝ) 7 . 5 xy xy y− 41. ɉɨɞɚɣɬɟ ɱɚɫɬɤɭ ɭ ɜɢɝɥɹɞɿ ɞɪɨɛɭ ɣ ɫɤɨɪɨɬɿɬɶ ɞɪɿɛ: ɚ) 10ɚ2 b2 : (5ɚ3 b); ɛ) 24m2 n : (–6mn); ɜ) (–28ab3 ) : (–21b4 ). Ɂɧɚɣɞɿɬɶ ɡɧɚɱɟɧɧɹ ɜɢɪɚɡɭ: 42. ɚ) 3 2 2 20 4 a b a b , ɹɤɳɨ ɚ = 48; b = 16; ɚ = –4,2; b = 2,1. ɛ) 15ɯ2 ɭ3 : (30ɯɭ2 ), ɹɤɳɨ ɯ = 300; ɭ = 0,06. 43. ɚ) 3 2 2 18 2 bc b c , ɹɤɳɨ b = 3; c = 4,5; b = –1,4, c = 2,8; ɛ) 64m2 n4 : (16mn2 ), ɹɤɳɨ m = 0,25; n = 25. ɋɤɨɪɨɬɿɬɶ ɞɪɿɛ: 44. ɚ) 6 3 ; 8 4 a b a b − − ɛ) 2 2 12 16 ; 3 4 a a a a − − ɜ) 2 2 ; xy x y xy xy + − ɝ) 2 2 ;a b a b − − ɞ) 2 9 ; 7 21 a a − + ɟ) 2 10 20 ; 7 28 x x − − ɽ) 2 4 8 ; 4 4 y y y − − + ɠ) 2 2 6 9. 9 x x x + + −
  • 20. 2. Ɉɫɧɨɜɧɚ ɜɥɚɫɬɢɜɿɫɬɶ ɞɪɨɛɭ 19 45. ɚ) 9 6 ; 15 10 x y x y − − ɛ) 2 2 2 ; 2 c c c c + − ɜ) 2 10 10 ; x y xy y − − ɝ) 2 2 ;ab a ab b + + ɞ) 2 2 ;m n n m − + ɟ) 2 2 5 5 ; x y x y + − ɽ) 2 2 1; 3 3 m m m + + + ɠ) 2 2 25 . 10 25 a a a − − + 46. Ɂɜɟɞɿɬɶ ɞɪɿɛ: ɚ) 4 k p ɞɨ ɡɧɚɦɟɧɧɢɤɚ: 12p; 16p2 ; ɛ) 2 5 2a ɞɨ ɡɧɚɦɟɧɧɢɤɚ: 4ɚ4 ; 10ɚ2 b. 47. Ɂɜɟɞɿɬɶ ɞɪɿɛ 4 3xy ɞɨ ɡɧɚɦɟɧɧɢɤɚ: 15xy; 3xy2 ; 9ɯ3 y. Ɂɜɟɞɿɬɶ ɞɨ ɫɩɿɥɶɧɨɝɨ ɡɧɚɦɟɧɧɢɤɚ ɞɪɨɛɢ: 48. ɚ) x y ɿ 2 ; x ɛ) m ab ɿ 4 ; b ɜ) 2 d a ɿ 3 1 ; a ɝ) 3 2c ɿ 2 9 ; c ɞ) 1 3c ɿ 2 ; 5c ɟ) 3 8a ɿ 1 ; 12a ɽ) 2 18 x a ɿ 4 ; 27 y a ɠ) 5 6ab ɿ 5 ; 4b ɡ) 2 3 p a ɿ . 6 q ab 49. ɚ) 3 2a ɿ 2 ; b ɛ) 7 8a ɿ 5 ; a ɜ) 2 k b ɿ ; 3 n b ɝ) 8 15ab ɿ 7 ; 20ab ɞ) 2 5 6a ɿ 7 ; 18a ɟ) 3 3 4y ɿ 2 7 . 20y 50. ɚ) 5 1a + ɿ 4 ; 2a + ɛ) 3 2( 1)a − ɿ 2 ; 3( 1)a − ɜ) 1 ab b+ ɿ 1 . 1a + 51. ɚ) 1 3c + ɿ 2 ; 1c − ɛ) 3 8( 2)b + ɿ 1 ; 4( 2)b + ɜ) 8 xy x− ɿ 7 . 1y − ɋɤɨɪɨɬɿɬɶ ɞɪɿɛ: 52. ɚ) 2 2 2 6 9 ; 4 9 ab b a b − − ɛ) 2 2 2 2 4 25 ; 4 20 25 c x c cx x − + + ɜ) 3 3 2 2 2 8 ; 2 x y xy xy x y − −
  • 21. 20 § 1. Ɋɚɰɿɨɧɚɥɶɧɿ ɜɢɪɚɡɢ ɝ) 3 8; 2 x x + + ɞ) 2 3 3 9 ; 27 z z z + + − ɟ) 6 2 1 ; 1 y y − − ɽ) ; ax cx ay cy cx cy + − − − ɠ) 2 2 2 2 ;b ab a a ab ax bx + + + − − ɡ) 2 8 4 . 2 2 a b ab b ad bd + + − − 53. ɚ) 2 2 14 63 ; 4 81 b c b c − − ɛ) 2 3 12 ; 8 16 kn n k k − − + ɜ) 2 2 3 6 2 ; 9 mn m mn m + − ɝ) 3 15 5 ; 27 c c − − ɞ) 2 2 2 ; x y xy x y y − − + − ɟ) 2 2 .a ac bc ab a b abc + + + + Ɂɧɚɣɞɿɬɶ ɡɧɚɱɟɧɧɹ ɜɢɪɚɡɭ: 54. ɚ) 2 2 3 9 9 a a a + − , ɹɤɳɨ ɚ = 4; ɚ = 1 3 − ; ɛ) 2 2 2 ( ) m n m n m n − − + − , ɹɤɳɨ m = 9,51; n = –0,49. 55. ɚ) 2 4 5 10 x x − + , ɹɤɳɨ ɯ = –1; ɯ = 2 9 ; ɛ) 2 2 2 ( ) , a b a b ab + + ɹɤɳɨ a = 2,5; b = 4. Ⱦɨɜɟɞɿɬɶ ɬɨɬɨɠɧɿɫɬɶ: 56. ɚ) 2 2 2 2 12 36 6 ; 636 a ab b a b a ba b + + += −− ɛ) 4 2 2 8 2 4. 2 m m m m m m − = + + − 57. ɚ) 2 2 2 2 2 ; 4( )4 4 x xy y x y x yx y − + − = +− ɛ) 3 2 8 1 2 1. 4 2 1 x x x x + = + − + 58. Ɂɜɟɞɿɬɶ ɞɪɿɛ: ɚ) 7 x y+ ɞɨ ɡɧɚɦɟɧɧɢɤɚ ɯ2 + ɯɭ; ɛ) 2 x y+ ɞɨ ɡɧɚɦɟɧɧɢɤɚ ɯ2 + 2ɯɭ + ɭ2 ; ɜ) c a b− ɞɨ ɡɧɚɦɟɧɧɢɤɚ ɚ2 – b2 ; ɝ) n m n− ɞɨ ɡɧɚɦɟɧɧɢɤɚ m3 – n3 . 59. Ɂɜɟɞɿɬɶ ɞɪɿɛ: ɚ) 2a x y+ ɞɨ ɡɧɚɦɟɧɧɢɤɚ ɯ2 – ɭ2 ; ɛ) 1 a c+ ɞɨ ɡɧɚɦɟɧɧɢɤɚ ɚ3 + ɫ3 . Ɂɜɟɞɿɬɶ ɞɨ ɫɩɿɥɶɧɨɝɨ ɡɧɚɦɟɧɧɢɤɚ ɞɪɨɛɢ: 60. ɚ) 3 9 14a b ɿ 2 5 ; 21ab ɛ) 3 3 1 18x y ɿ 4 1 ; 27xy ɜ) 2 9 a m n ɿ 5 3 . 15 b m n
  • 22. 2. Ɉɫɧɨɜɧɚ ɜɥɚɫɬɢɜɿɫɬɶ ɞɪɨɛɭ 21 61. ɚ) 3 3 8 9x y ɿ 5 5 ; 24xy ɛ) 3 2 16 a m n ɿ 4 ; 24 b m n ɜ) 4 2 15 c x y ɿ 2 2 . 25x y 62. ɚ) 2 3 x xy+ ɿ 2 2 ; xy y+ ɛ) 2 2 x x y− ɿ ; y x y+ ɜ) 2 2 2 m m mn n+ + ɿ ;n m n+ ɝ) 2 4 1 c c − ɿ 2 ; 1 2 c c− ɞ) 3 1 1 x− ɿ 2 ; 1x − ɟ) 3 8 y y − ɿ 2 2 . 2 4y y+ + 63. ɚ) 2 2 x x x y+ ɿ 2 ; y y y+ ɛ) 2 4 4 m m m− + ɿ 3 ; 2 m− ɜ) 2 2 4 a a b− ɿ . 2 b b a− 64. ɋɤɨɪɨɬɿɬɶ ɞɪɿɛ, ɞɟ n — ɧɚɬɭɪɚɥɶɧɟ ɱɢɫɥɨ: ɚ) 2 13824 ; 15552 n n x x + ɛ) 2 2045 ; 1755 n n x x ɜ) 2 2 2 2 3 2 ; 2 x xy y x xy y + + − − ɝ) 3 2 2 2 2 . 2 y y y y y + − − + − 65. ɉɨɛɭɞɭɣɬɟ ɝɪɚɮɿɤ ɮɭɧɤɰɿʀ, ɡɚɞɚɧɨʀ ɮɨɪɦɭɥɨɸ: ɚ) 2 1; 1 xy x −= − ɛ) . x y x = 66. Ⱦɨɜɟɞɿɬɶ ɬɨɬɨɠɧɿɫɬɶ: ɚ) 2 2 2 3 5 15 10 25 ; 3 3 5 5 ab a b a a ab a b b a ab a b + + + + += + + + + + + ɛ) 2 3 2 3 3 2 3 2 . 2 3 4 6 3 2 6 4 xy y x xy y x xz z x xz z x + + + + + + = + + + + + + 67. Ɉɛɱɢɫɥɿɬɶ: ɚ) 4 2 ; 9 9 + ɛ) 5 2; 7 7 − ɜ) 1 53 2 ; 6 6 + ɝ) 1 54 1 . 8 8 −
  • 23. 22 § 1. Ɋɚɰɿɨɧɚɥɶɧɿ ɜɢɪɚɡɢ 68. Ɋɨɡɜ’ɹɠɿɬɶ ɪɿɜɧɹɧɧɹ: ɚ) 5 1 3 5; 7 9 7 9 x x− = + ɛ) ( )( ) ( ) 2 1 1 1 1 . 2 2 2 2 x x x− + − − = 69. ɋɶɨɝɨɞɧɿ ɜ ɦɚɝɚɡɢɧɿ 2 ɤɝ ɩɨɦɿɞɨɪɿɜ ɿ 3 ɤɝ ɨɝɿɪɤɿɜ ɤɨɲɬɭɸɬɶ 28 ɝɪɧ. Ɍɢɠɞɟɧɶ ɬɨɦɭ, ɤɨɥɢ ɩɨɦɿɞɨɪɢ ɣ ɨɝɿɪɤɢ ɛɭɥɢ ɞɨɪɨɠɱɢɦɢ ɧɚ 25 %, 1 ɤɝ ɩɨɦɿɞɨɪɿɜ ɿ 2 ɤɝ ɨɝɿɪɤɿɜ ɤɨɲɬɭɜɚɥɢ 20 ɝɪɧ. ɋɤɿɥɶɤɢ ɤɨɲɬɭɸɬɶ ɫɶɨɝɨɞɧɿ 1 ɤɝ ɩɨɦɿɞɨɪɿɜ ɿ ɫɤɿɥɶɤɢ 1 ɤɝ ɨɝɿɪɤɿɜ? 70. ȯ ɞɜɚ ɫɩɥɚɜɢ ɦɿɞɿ ɣ ɰɢɧɤɭ. ȼ ɨɞɢɧ ɫɩɥɚɜ ɦɿɞɶ ɿ ɰɢɧɤ ɜɯɨɞɹɬɶ ɭ ɜɿɞɧɨɲɟɧɧɿ 5 : 2, ɚ ɜ ɿɧɲɢɣ — ɭ ɜɿɞɧɨɲɟɧɧɿ 3 : 4. ɋɤɿɥɶɤɢ ɩɨɬɪɿɛɧɨ ɜɡɹɬɢ ɤɿɥɨɝɪɚɦɿɜ ɤɨɠɧɨɝɨ ɫɩɥɚɜɭ, ɳɨɛ ɨɞɟɪɠɚɬɢ 28 ɤɝ ɧɨɜɨɝɨ ɡ ɨɞɧɚɤɨɜɢɦ ɭɦɿɫɬɨɦ ɦɿɞɿ ɣ ɰɢɧɤɭ? 71. ɍ ɫɶɨɦɢɯ ɿ ɜɨɫɶɦɢɯ ɤɥɚɫɚɯ ɲɤɨɥɢ ɪɚɡɨɦ ɧɚɜɱɚɸɬɶɫɹ 180 ɭɱɧɿɜ. Ʉɨɠɧɢɣ ɜɨɫɶɦɢɤɥɚɫɧɢɤ ɞɪɭɠɢɬɶ ɿɡ 7 ɫɟɦɢɤɥɚɫɧɢɤɚɦɢ, ɚ ɤɨɠɧɢɣ ɫɟɦɢɤɥɚɫɧɢɤ — ɿɡ 8 ɜɨɫɶɦɢɤɥɚɫɧɢɤɚɦɢ. ɋɤɿɥɶɤɢ ɜɫɶɨɝɨ ɜɨɫɶɦɢɤɥɚɫɧɢɤɿɜ ɽ ɭ ɲɤɨɥɿ? 1. Ⱦɨɞɚɜɚɧɧɹ ɞɪɨɛɿɜ ɡ ɨɞɧɚɤɨɜɢɦɢ ɡɧɚɦɟɧɧɢɤɚɦɢ. Ⱦɪɨɛɢ ɡ ɨɞɧɚɤɨɜɢɦɢ ɡɧɚɦɟɧɧɢɤɚɦɢ ɞɨɞɚɸɬɶ ɬɚɤ ɫɚɦɨ, ɹɤ ɿ ɡɜɢɱɚɣɧɿ ɞɪɨɛɢ ɡ ɨɞɧɚɤɨɜɢɦɢ ɡɧɚɦɟɧɧɢ- ɤɚɦɢ, ɬɨɛɬɨ ɞɨɞɚɸɬɶ ʀɯɧɿ ɱɢɫɟɥɶɧɢɤɢ, ɚ ɡɧɚɦɟɧɧɢɤ ɡɚɥɢɲɚɸɬɶ ɬɨɣ ɫɚɦɢɣ: .a c a c b b b ++ = (1) Ɋɿɜɧɿɫɬɶ (1) ɽ ɬɨɬɨɠɧɿɫɬɸ, ɬɨɛɬɨ ɜɨɧɚ ɽ ɩɪɚɜɢɥɶɧɨɸ ɞɥɹ ɛɭɞɶ-ɹɤɢɯ ɡɧɚ- ɱɟɧɶ ɚ, b ɿ ɫ, ɞɟ b ≠ 0. Ɂ ɬɨɬɨɠɧɨɫɬɿ (1) ɜɢɩɥɢɜɚɽ ɬɚɤɟ ɩɪɚɜɢɥɨ ɞɨɞɚɜɚɧɧɹ ɞɪɨɛɿɜ ɡ ɨɞɧɚɤɨɜɢɦɢ ɡɧɚɦɟɧɧɢɤɚɦɢ: ɓɨɛ ɞɨɞɚɬɢ ɞɪɨɛɢ ɡ ɨɞɧɚɤɨɜɢɦɢ ɡɧɚɦɟɧɧɢɤɚɦɢ, ɩɨɬɪɿɛɧɨ ɞɨɞɚɬɢ ʀɯɧɿ ɱɢɫɟɥɶɧɢɤɢ, ɚ ɡɧɚɦɟɧɧɢɤ ɡɚɥɢɲɢɬɢ ɬɨɣ ɫɚɦɢɣ.
  • 24. 3. Ⱦɨɞɚɜɚɧɧɹ ɿ ɜɿɞɧɿɦɚɧɧɹ ɞɪɨɛɿɜ ɡ ɨɞɧɚɤɨɜɢɦɢ ɡɧɚɦɟɧɧɢɤɚɦɢ 23 2. ȼɿɞɧɿɦɚɧɧɹ ɞɪɨɛɿɜ ɡ ɨɞɧɚɤɨɜɢɦɢ ɡɧɚɦɟɧɧɢɤɚɦɢ. ȼɿɞɧɿɦɚɧɧɹ ɞɪɨɛɿɜ ɡ ɨɞɧɚɤɨɜɢɦɢ ɡɧɚɦɟɧɧɢɤɚɦɢ ɜɢɤɨɧɭɸɬɶ ɧɚ ɨɫɧɨɜɿ ɬɨɬɨɠɧɨɫɬɿ . a c a c b b b − − = (2) Ɂ ɬɨɬɨɠɧɨɫɬɿ (2) ɜɢɩɥɢɜɚɽ ɩɪɚɜɢɥɨ ɜɿɞɧɿɦɚɧɧɹ ɞɪɨɛɿɜ ɡ ɨɞɧɚɤɨɜɢɦɢ ɡɧɚ- ɦɟɧɧɢɤɚɦɢ: ɓɨɛ ɜɿɞɧɹɬɢ ɞɪɨɛɢ ɡ ɨɞɧɚɤɨɜɢɦɢ ɡɧɚɦɟɧɧɢɤɚɦɢ, ɩɨɬɪɿɛɧɨ ɜɿɞ ɱɢɫɟ- ɥɶɧɢɤɚ ɡɦɟɧɲɭɜɚɧɨɝɨ ɜɿɞɧɹɬɢ ɱɢɫɟɥɶɧɢɤ ɜɿɞ’ɽɦɧɢɤɚ, ɚ ɡɧɚɦɟɧɧɢɤ ɡɚɥɢɲɢɬɢ ɬɨɣ ɫɚɦɢɣ. 3. Ɂɚɩɢɫ ɞɪɨɛɭ ɭ ɜɢɝɥɹɞɿ ɫɭɦɢ ɚɛɨ ɪɿɡɧɢɰɿ ɞɪɨɛɿɜ. ɍ ɤɨɠɧɿɣ ɡ ɬɨɬɨɠɧɨ- ɫɬɟɣ (1) ɿ (2) ɩɟɪɟɫɬɚɜɢɦɨ ɦɿɫɰɹɦɢ ɥɿɜɭ ɿ ɩɪɚɜɭ ɱɚɫɬɢɧɢ: ;a c a c b b b + = + .a c a c b b b − = − Ɉɞɟɪɠɚɧɿ ɬɨɬɨɠɧɨɫɬɿ ɦɨɠɧɚ ɜɢɤɨɪɢɫɬɨɜɭɜɚɬɢ, ɹɤɳɨ ɩɨɬɪɿɛɧɨ ɡɚɩɢɫɚɬɢ ɞɪɿɛ ɭ ɜɢɝɥɹɞɿ ɫɭɦɢ ɚɛɨ ɪɿɡɧɢɰɿ ɞɪɨɛɿɜ. ȼɩɪɚɜɚ 1. Ⱦɨɞɚɬɢ ɞɪɨɛɢ: ɚ) 7 5 ; a a + ɛ) 2 1 1 3 ;a b ab ab ab + −+ + ɜ) 5 2 . 2 2 x y x y x y x y − − + − − Ɣ ɚ) 7 5 a a + = 7 5 a + = 12. a ɛ) 2 1 1 3a b ab ab ab + −+ + = 2 1 1 3a b ab + + − + = 2 3.a b ab + + ɜ) 5 2 2 2 x y x y x y x y − − + − − = 5 2 2 x y x y x y − + − − = 6 3 2 x y x y − − = 3(2 ) 2 x y x y − − = 3. Ɣ ȼɩɪɚɜɚ 2. ȼɿɞɧɹɬɢ ɞɪɨɛɢ: ɚ) 2 2 4 2 3 ; 2 3 2 3 n n n n n n +− − − ɛ) 2 3 .a a x y y x − − − Ɣ ɚ) 2 2 4 2 3 2 3 2 3 n n n n n n +− − − = 2 4 (2 3) 2 3 n n n n − + − = 2 3 (2 3) n n n − − = 1 . n
  • 25. 24 § 1. Ɋɚɰɿɨɧɚɥɶɧɿ ɜɢɪɚɡɢ ɛ) Ɂɦɿɧɢɜɲɢ ɡɧɚɤ ɡɧɚɦɟɧɧɢɤɚ ɞɪɭɝɨɝɨ ɞɪɨɛɭ, ɦɚɬɢɦɟɦɨ: 2 3a a x y y x − − − = 2 3 ( ) a a x y x y − − − − = 2 3a a x y x y + − − = 2 3a a x y + − = 5 .a x y− Ɣ ȼɩɪɚɜɚ 3. Ɂɚɩɢɫɚɬɢ ɞɪɿɛ ɭ ɜɢɝɥɹɞɿ ɫɭɦɢ ɚɛɨ ɪɿɡɧɢɰɿ ɰɿɥɨɝɨ ɱɢɫɥɚ ɿ ɞɪɨɛɭ: ɚ) 3 5;a a + ɛ) 2 2 1;a b a b + + + ɜ) 3 1. 1 n n + + Ɣ ɚ) 3 5 3 5 53 .a a a a a a + = + = + ɛ) 2( )2 2 1 1 12 . a ba b a b a b a b a b ++ + = + = + + + + + ɜ) 3( 1) 2 3( 1)3 1 3 3 2 2 23 . 1 1 1 1 1 1 n nn n n n n n n n + − ++ + −= = = − = − + + + + + + Ɣ 72. Ɂɧɚɣɞɿɬɶ ɫɭɦɭ ɞɪɨɛɿɜ: ɚ) ; 4 4 a b+ ɛ) 9 3 ; 11 11 b b+ ɜ) 3 ;a a x x + ɝ) .x a a d d + + 73. Ɂɧɚɣɞɿɬɶ ɪɿɡɧɢɰɸ ɞɪɨɛɿɜ: ɚ) ; 7 7 yx − ɛ) 8 3 ; 9 9 n n− ɜ) ; a y y x x + − ɝ) 2 . x y x c c + − ȼɢɤɨɧɚɣɬɟ ɞɨɞɚɜɚɧɧɹ (ɜɿɞɧɿɦɚɧɧɹ) ɞɪɨɛɿɜ: 74. ɚ) 2 3 ;b b a a + ɛ) 5 3 7 1; 3 1 3 1 n n n n + −+ + + ɜ) 2 3 4 3 ;a a xy xy xy − + + ɝ) 6 3 ; 5 5 a a p p − ɞ) 3 3 ; 9 9 a a a a + −− ɟ) 9 2. 7 7 b b b +− − − 75. ɚ) 3 5 2 7 ; 5 5 n n n n + −+ ɛ) 2 1 1 2 ; 1 1 x x x x + −+ − − ɜ) 2 2 ; 2 2 2 n n n m m m + −+ + ɝ) 3 ;c c b b + − ɞ) 7 3 3 2 ; 4 1 4 1 b b b b − −− − − ɟ) 4 1 4 5 . 2 2 2 a a a a a − ++ −
  • 26. 3. Ⱦɨɞɚɜɚɧɧɹ ɿ ɜɿɞɧɿɦɚɧɧɹ ɞɪɨɛɿɜ ɡ ɨɞɧɚɤɨɜɢɦɢ ɡɧɚɦɟɧɧɢɤɚɦɢ 25 ɋɩɪɨɫɬɿɬɶ ɜɢɪɚɡ: 76. ɚ) 7 2 5 2 ; 4 1 4 1 x x x x − ++ + + ɛ) 3 3 ;a b a b a b a b − −− + + ɜ) 3 5 1; 3 1 3 1 a a a a − ++ − − ɝ) 6 12 ; 2 2 p p p − − − ɞ) 5 5 ;a b a b b a + ++ − − ɟ) 4 3 4 . 2 2 x x x y y x + −− − − 77. ɚ) 6 1 4 ; 1 1 m m m m − +− − − ɛ) 3 7 5 ; x y x y x y x y + + + + + ɜ) 8 8 ;m n m n n m + − − ɝ) 9 9. 3 3 c c c c − +− − − Ɂɧɚɣɞɿɬɶ ɡɧɚɱɟɧɧɹ ɜɢɪɚɡɭ: 78. ɚ) 3 3 1 4 4 b b b b − −+ , ɹɤɳɨ b = –3; ɛ) 2 2 7 1 1 3 3 a a a a − −− , ɹɤɳɨ ɚ = 0,28. 79. ɚ) 2 2 3 3a a a a − ++ , ɹɤɳɨ ɚ = 5; ɛ) 4 5 2 1 2 2 c c c c + −− , ɹɤɳɨ ɫ = 0,4. ɋɩɪɨɫɬɿɬɶ ɜɢɪɚɡ: 80. ɚ) 2 2 1 ; 1 1 c c c c − − − − ɛ) 5 3 ; 1 1 a a a a − − − ɜ) 2 2 2 9 6 ; 9 9 b b b b + + − − ɝ) 4 2 3 2 4 2 4 12 4 ; 3 3 a a b a b b a b b − − − ɞ) 2 4 5 1 2 ; 2 1 2 1 2 1 x x x x x x − −+ − + + + ɟ) 2 2 2 2 2 2 2 3 . xy x xy y y x y x y x y − − − − − − − 81. ɚ) 2 2 5 14 10 ; 2 2 y y y y − − − − − ɛ) 2 2 3 3 2 ; ( ) ( ) x y xy x y x y + − − − ɜ) 2 3 3 1 ; 1 1 a a a a + + − − ɝ) 2 4 2 2 18 2 ; 3 3 a b a b a b − − − ɞ) 2 2 21 3 ; 3 3 3 n n n n n −− + − − − ɟ) 2 2 2 8 2 3 . (5 ) (5 ) (5 ) a b a b a b a b a b a b − − +− − + + +
  • 27. 26 § 1. Ɋɚɰɿɨɧɚɥɶɧɿ ɜɢɪɚɡɢ Ⱦɨɜɟɞɿɬɶ ɬɨɬɨɠɧɿɫɬɶ: 82. ɚ) 2 2 ( ) ( ) 4; a b a b ab ab + − − = ɛ) 3 2 2 2 2 4 4 . ( 2) ( 2) ( 2) x x x x x x x − + = − − − 83. ɚ) 2 2 2 2 2 2 ( ) ( ) 2; m n m n m n m n + − + = + + ɛ) 2 2 2 2 2 4 2 2 . 24 4 a ab a a ba b a b − = +− − ɉɨɞɚɣɬɟ ɞɪɿɛ ɭ ɜɢɝɥɹɞɿ ɫɭɦɢ ɚɛɨ ɪɿɡɧɢɰɿ ɰɿɥɨɝɨ ɱɢɫɥɚ ɿ ɞɪɨɛɭ: 84. ɚ) 3 8 ; 2 x x + ɛ) 5 5 2 ;m n m n − + − ɜ) 4 5 ; 2 y y + + ɝ) 2 . x y x y − + 85. ɚ) 14 5; 7 b + ɛ) 3 3 ;b c a b c + − + ɜ) 6 1; 2 1 c c + + ɝ) 4 3 . x y x y − − 86. ɋɩɪɨɫɬɿɬɶ ɜɢɪɚɡ: ɚ) 2 4 2 2 1 ; 1 1 x x x x x x + + + + + + ɛ) 4 4 2 2 3 3 3 3 .a x a x a x a x + + + + ȼɤɚɡɿɜɤɚ. ɚ) Ɇɧɨɝɨɱɥɟɧ ɯ4 + ɯ2 + 1 ɦɨɠɧɚ ɪɨɡɤɥɚɫɬɢ ɧɚ ɦɧɨɠɧɢɤɢ, ɡɚɩɢɫɚɜɲɢ ɣɨɝɨ ɭ ɜɢɝɥɹɞɿ (ɯ4 + 2ɯ2 + 1) – ɯ2 . 87. Ɉɛɱɢɫɥɿɬɶ: ɚ) 5 3; 6 8 + ɛ) 3 4 ; 14 21 − ɜ) 2 5 7 ; 9 6 12 − + ɝ) 1 2 57 4 . 3 11 6 ⋅ − 88. ɉɨɞɚɣɬɟ ɨɞɧɨɱɥɟɧ 24ɚ7 b8 ɭ ɜɢɝɥɹɞɿ ɞɨɛɭɬɤɭ ɞɜɨɯ ɨɞɧɨɱɥɟɧɿɜ, ɨɞɧɢɦ ɡ ɹɤɢɯ ɽ: ɚ) 6ɚ4 b7 ; ɛ) 4ɚ2 b5 ; ɜ) 24ɚ7 b; ɝ) 8ɚb7 . 89. Ɂ Ʉɢɽɜɚ ɿ ɑɟɪɤɚɫ, ɜɿɞɫɬɚɧɶ ɦɿɠ ɹɤɢɦɢ ɞɨɪɿɜɧɸɽ 190 ɤɦ, ɨɞɧɨɱɚɫɧɨ ɧɚɡɭ- ɫɬɪɿɱ ɨɞɢɧ ɨɞɧɨɦɭ ɜɢʀɯɚɥɢ ɚɜɬɨɛɭɫ ɬɚ ɥɟɝɤɨɜɢɣ ɚɜɬɨɦɨɛɿɥɶ ɿ ɡɭɫɬɪɿɥɢɫɹ ɱɟɪɟɡ 1 ɝɨɞ 15 ɯɜ. Ɂɧɚɣɞɿɬɶ ɲɜɢɞɤɿɫɬɶ ɚɜɬɨɦɨɛɿɥɹ, ɹɤɳɨ ɜɨɧɚ ɜ 11 9 ɪɚɡɭ ɛɿɥɶɲɚ ɜɿɞ ɲɜɢɞɤɨɫɬɿ ɚɜɬɨɛɭɫɚ.
  • 28. 4. Ⱦɨɞɚɜɚɧɧɹ ɿ ɜɿɞɧɿɦɚɧɧɹ ɞɪɨɛɿɜ ɡ ɪɿɡɧɢɦɢ ɡɧɚɦɟɧɧɢɤɚɦɢ 27 90. ȼɿɞɩɨɜɿɞɧɨ ɞɨ ɧɨɪɦ ɚɝɪɨɬɟɯɧɿɤɢ ɡɟɪɧɨ ɩɨɬɪɿɛɧɨ ɡɚɫɢɩɚɬɢ ɧɚ ɬɪɢɜɚɥɟ ɡɛɟɪɿɝɚɧɧɹ ɡɚ ɜɨɥɨɝɨɫɬɿ 14% (ɤɨɧɞɢɰɿɣɧɢɣ ɫɬɚɧ). Ɂɿɛɪɚɧɟ ɡɟɪɧɨ, ɦɚɫɚ ɹɤɨɝɨ ɞɨɪɿɜɧɸɽ 43 ɬ, ɦɚɽ ɜɨɥɨɝɿɫɬɶ 24%. ɇɚ ɫɤɿɥɶɤɢ ɬɨɧɧ ɡɦɟɧɲɢɬɶɫɹ ɦɚɫɚ ɰɶɨɝɨ ɡɟɪɧɚ ɩɪɢ ɞɨɜɟɞɟɧɧɿ ɣɨɝɨ ɞɨ ɤɨɧɞɢɰɿɣɧɨɝɨ ɫɬɚɧɭ? 91. ɇɚ ɤɥɿɬɱɚɫɬɨɦɭ ɩɚɩɟɪɿ ɫɢɞɹɬɶ 100 ɩɚɜɭɤɿɜ, ɩɨ ɨɞɧɨɦɭ ɭ ɤɥɿɬɢɧɰɿ. ɉɚɜɭɤɢ ɜɨɪɨɝɭɸɬɶ, ɹɤɳɨ ɤɥɿɬɢɧɤɢ, ɭ ɹɤɢɯ ɜɨɧɢ ɫɢɞɹɬɶ, ɦɚɸɬɶ ɯɨɱɚ ɛ ɨɞɧɭ ɫɩɿɥɶɧɭ ɜɟɪɲɢɧɭ. Ⱦɨɜɟɞɿɬɶ, ɳɨ ɫɟɪɟɞ ɧɢɯ ɡɚɜɠɞɢ ɦɨɠɧɚ ɜɢɛɪɚɬɢ ɧɟ ɦɟɧɲɟ ɧɿɠ 25 ɩɚɜɭɤɿɜ, ɛɭɞɶ-ɹɤɿ ɞɜɚ ɡ ɹɤɢɯ ɧɟ ɜɨɪɨɝɭɸɬɶ. ɓɨɛ ɞɨɞɚɬɢ ɚɛɨ ɜɿɞɧɹɬɢ ɡɜɢɱɚɣɧɿ ɞɪɨɛɢ ɡ ɪɿɡɧɢɦɢ ɡɧɚɦɟɧɧɢɤɚɦɢ, ɩɨɬɪɿ- ɛɧɨ ɡɜɟɫɬɢ ɞɪɨɛɢ ɞɨ ɫɩɿɥɶɧɨɝɨ ɡɧɚɦɟɧɧɢɤɚ ɿ ɞɨɞɚɬɢ ɚɛɨ ɜɿɞɧɹɬɢ ɨɞɟɪɠɚɧɿ ɞɪɨɛɢ ɡ ɨɞɧɚɤɨɜɢɦɢ ɡɧɚɦɟɧɧɢɤɚɦɢ. ɍ ɬɚɤɢɣ ɠɟ ɫɩɨɫɿɛ ɞɨɞɚɸɬɶ ɿ ɜɿɞɧɿɦɚɸɬɶ ɛɭɞɶ-ɹɤɿ ɞɪɨɛɢ ɡ ɪɿɡɧɢɦɢ ɡɧɚ- ɦɟɧɧɢɤɚɦɢ. ɇɟɯɚɣ ɩɨɬɪɿɛɧɨ ɞɨɞɚɬɢ ɞɪɨɛɢ a b ɿ ,c d ɹɤɿ ɦɚɸɬɶ ɪɿɡɧɿ ɡɧɚɦɟɧɧɢɤɢ. Ɂɜɟ- ɞɟɦɨ ɰɿ ɞɪɨɛɢ ɞɨ ɫɩɿɥɶɧɨɝɨ ɡɧɚɦɟɧɧɢɤɚ bd. Ⱦɥɹ ɰɶɨɝɨ ɱɢɫɟɥɶɧɢɤ ɿ ɡɧɚɦɟɧɧɢɤ ɩɟɪɲɨɝɨ ɞɪɨɛɭ ɩɨɦɧɨɠɢɦɨ ɧɚ d, ɚ ɞɪɭɝɨɝɨ ɞɪɨɛɭ — ɧɚ b. Ɉɞɟɪɠɢɦɨ: ;=a ad b bd .c bc d bd = Ɂɧɚɸɱɢ, ɹɤ ɞɨɞɚɬɢ ɞɪɨɛɢ ɡ ɨɞɧɚɤɨɜɢɦɢ ɡɧɚɦɟɧɧɢɤɚɦɢ, ɦɚɬɢɦɟɦɨ: .++ = + =a c ad bc ad bc b d bd bd bd Ɉɬɠɟ, .a c ad bc b d bd ++ = ȼɿɞɧɿɦɚɸɬɶ ɞɪɨɛɢ ɡ ɪɿɡɧɢɦɢ ɡɧɚɦɟɧɧɢɤɚɦɢ ɚɧɚɥɨɝɿɱɧɨ, ɚ ɫɚɦɟ: .a c ad bc b d bd −− =
  • 29. 28 § 1. Ɋɚɰɿɨɧɚɥɶɧɿ ɜɢɪɚɡɢ ɓɨɛ ɞɨɞɚɬɢ ɚɛɨ ɜɿɞɧɹɬɢ ɞɪɨɛɢ ɡ ɪɿɡɧɢɦɢ ɡɧɚɦɟɧɧɢɤɚɦɢ, ɩɨɬɪɿɛɧɨ: 1) ɡɜɟɫɬɢ ɞɪɨɛɢ ɞɨ ɫɩɿɥɶɧɨɝɨ ɡɧɚɦɟɧɧɢɤɚ; 2) ɞɨɞɚɬɢ ɚɛɨ ɜɿɞɧɹɬɢ ɨɞɟɪɠɚɧɿ ɞɪɨɛɢ ɡ ɨɞɧɚɤɨɜɢɦɢ ɡɧɚɦɟɧɧɢɤɚɦɢ. ȼɩɪɚɜɚ 1. ȼɢɤɨɧɚɬɢ ɞɨɞɚɜɚɧɧɹ (ɜɿɞɧɿɦɚɧɧɹ) ɞɪɨɛɿɜ: ɚ) 5 4 ;b c ac b + ɛ) 2 2 3 4 7 ; 9 12x y xy + ɜ) 2 2 2 2 . xy y x xy − − − • ɚ) ɋɩɿɥɶɧɢɦ ɡɧɚɦɟɧɧɢɤɨɦ ɞɪɨɛɿɜ ɽ ɞɨɛɭɬɨɤ ʀɯɧɿɯ ɡɧɚɦɟɧɧɢɤɿɜ. Ɍɨɦɭ ɞɨɞɚɬɤɨɜɢɣ ɦɧɨɠɧɢɤ ɞɥɹ ɩɟɪɲɨɝɨ ɞɪɨɛɭ — b, ɚ ɞɥɹ ɞɪɭɝɨɝɨ — ɚɫ. 5b ac 4c b + b ac = 2 2 5 4 .b ac abc + ɛ) ɋɩɿɥɶɧɢɦ ɡɧɚɦɟɧɧɢɤɨɦ ɞɪɨɛɿɜ ɽ 36ɯ2 ɭ3 . Ⱦɨɞɚɬɤɨɜɢɦ ɦɧɨɠɧɢɤɨɦ ɞɥɹ ɩɟɪɲɨɝɨ ɞɪɨɛɭ ɽ 4ɭ, ɞɥɹ ɞɪɭɝɨɝɨ — 3ɯ. 4 7+ 4y 3x 9x y 12xy32 2 = 2 3 16 21 . 36 y x x y + ɜ) Ɋɨɡɤɥɚɜɲɢ ɧɚ ɦɧɨɠɧɢɤɢ ɡɧɚɦɟɧɧɢɤɢ ɞɪɨɛɿɜ, ɦɚɬɢɦɟɦɨ: 2 2 2 2 xy y x xy − − − = 2 y x y( ) 2 x( )x y x y = 2 2 ( ) x y xy x y − − = 2( ) ( ) x y xy x y − − = 2 . xy • ȼɩɪɚɜɚ 2. ɉɨɞɚɬɢ ɭ ɜɢɝɥɹɞɿ ɞɪɨɛɭ ɜɢɪɚɡ 2 23 . 1 mm m +− + − • ȼɢɪɚɡ ɬ – 3 ɡɚɩɢɲɟɦɨ ɭ ɜɢɝɥɹɞɿ ɞɪɨɛɭ 3. 1 m − Ɍɨɞɿ: 2 23 1 mm m +− + − = m 3 1 2 + 1 m m 1 m + 2 = 2 ( 3)(1 ) 2 1 m m m m − − + + − = = 2 2 3 3 2 1 m m m m m − − + + + − = 4 1. 1 m m − − •
  • 30. 4. Ⱦɨɞɚɜɚɧɧɹ ɿ ɜɿɞɧɿɦɚɧɧɹ ɞɪɨɛɿɜ ɡ ɪɿɡɧɢɦɢ ɡɧɚɦɟɧɧɢɤɚɦɢ 29 ȼɩɪɚɜɚ 3. Ⱦɨɜɟɫɬɢ ɬɨɬɨɠɧɿɫɬɶ 2 1 . ( ) a b b a b a a a b + − = + + • ɉɟɪɟɬɜɨɪɢɦɨ ɥɿɜɭ ɱɚɫɬɢɧɭ ɪɿɜɧɨɫɬɿ: 1a b a b a + − + = a ba+ b a 1 1 a a b( + )a a b+ + = 2 ( ) ( ) ( ) a b a b a a b a a b + + − + + = = 2 2 2 ( ) a ab b a ab a a b + + − − + = 2 . ( )+ b a a b ɒɥɹɯɨɦ ɬɨɬɨɠɧɢɯ ɩɟɪɟɬɜɨɪɟɧɶ ɥɿɜɭ ɱɚɫɬɢɧɭ ɪɿɜɧɨɫɬɿ ɡɜɟɥɢ ɞɨ ɩɪɚɜɨʀ ɱɚɫɬɢɧɢ. Ɍɨɦɭ ɰɹ ɪɿɜɧɿɫɬɶ ɽ ɬɨɬɨɠɧɿɫɬɸ. • ɉɪɢɦɿɬɤɚ. ɇɚɝɚɞɚɽɦɨ, ɳɨ ɞɥɹ ɞɨɜɟɞɟɧɧɹ ɬɨɬɨɠɧɨɫɬɟɣ ɨɞɧɭ ɱɚɫɬɢɧɭ ɬɨɬɨɠɧɨɫɬɿ ɡɜɨɞɹɬɶ ɞɨ ɿɧɲɨʀ ɱɚɫɬɢɧɢ ɚɛɨ ɨɛɢɞɜɿ ɱɚɫɬɢɧɢ ɡɜɨɞɹɬɶ ɞɨ ɬɨɝɨ ɫɚɦɨɝɨ ɜɢɪɚɡɭ, ɚɛɨ ɭɬɜɨɪɸɸɬɶ ɪɿɡɧɢɰɸ ɥɿɜɨʀ ɬɚ ɩɪɚɜɨʀ ɱɚɫɬɢɧ ɿ ɞɨɜɨɞɹɬɶ, ɳɨ ɜɨɧɚ ɞɨɪɿɜɧɸɽ ɧɭɥɸ. ȼɢɤɨɧɚɣɬɟ ɞɨɞɚɜɚɧɧɹ (ɜɿɞɧɿɦɚɧɧɹ) ɞɪɨɛɿɜ: 92. ɚ) ;a m c n − ɛ) ; 3 12 a b+ ɜ) 5 3 ; 4 5 a b x x − ɝ) 7 ; 9 6 c c y y − ɞ) 5 7 ; 12 18 b b x x + ɟ) 4 6 . 15 25 b a a b − 93. ɚ) ; 6 18 c ad+ ɛ) 3 2 ; 5 3 k k a a + ɜ) 5 . 24 36 n n x x − 94. ɚ) 2 3 ; 4 a a + − ɛ) 10 37 3 ; yx x y −+ + ɜ) 2 2 ;a b a b b a + −− ɝ) 2 2 ; 1 1z z + − + ɞ) ;a a a c c − + ɟ) 1 2 . 2 1 y y y y − + − − + 95. ɚ) 2 1 1; 3 m m − + ɛ) 4 2 3 2 ;a b a b − ++ ɜ) 2 2 ; x y y x x y − − − ɝ) 2 1 ; 2k k + + ɞ) 2 3 ; 2 1 3 2 a a a a − + + ɟ) 1 . 1 x x x x + − −
  • 31. 30 § 1. Ɋɚɰɿɨɧɚɥɶɧɿ ɜɢɪɚɡɢ ɉɨɞɚɣɬɟ ɭ ɜɢɝɥɹɞɿ ɞɪɨɛɭ: 96. ɚ) 22 ; 2 1n + − ɛ) 3 2 3 ; x y x − − ɜ) 2 . 2 y y y − − 97. ɚ) 5 2 2; 1 y y − + + ɛ) 21 ; 2 3 x x − + ɜ) 2 2 3 3 . 1 c c c + − − ɋɩɪɨɫɬɿɬɶ ɜɢɪɚɡ: 98. ɚ) 4 1 5 6 ; 8 12 x x x x − −+ ɛ) 6 18; 6 18 a a a a − +− ɜ) 5 4 ; 2( ) 3( ) a a a b a b − + + ɝ) 2 2 2 ;a b a b a ab + −+ ɞ) 2 1 5;c c c −− ɟ) 2 2 ;m n m n m n mn + −+ ɽ) 2 1 2 3; 3 6 y x xy x − ++ ɠ) 77 ; ( ) y x x x y − + ɡ) 3 3 . ( ) a a b b a b + + + 99. ɚ) 3 1 2 1; 6 4 b b b b − −− ɛ) 4 4 ; 5( ) 3( ) b c b c b c b c + −+ − − ɜ) 3 1; 3 3 x y x xy x + −+ ɝ) 2 2 1 ;a aa + − ɞ) 2 2 ; 2 a b a abab + −+ ɟ) 2 1 2 . ( 1) 1 m m m m + − − − 100. ɚ) 3 3 ; 2 2 4 4 a b a b a b a b − ++ − − ɛ) 1 3 ; 3 12 2 8 x x x x − −− − − ɜ) 2 4 2 ; 2 cc c + − ɝ) 2 3 2 3 ; 3 3 b b b b −− + + ɞ) 2 2 ; 2 4 k k k k − − − ɟ) 2 2 4 4 . 2 m n m nm mn n − + ++ + 101. ɚ) 5 ; 2 2 m n m n m n m n + −+ − − ɛ) 2 3 ; 3 3 5 5 − + + a a a a ɜ) 2 1 2 ; 1 n n n n −+ − − ɝ) 2 15 3 ; 5 aa a − + ɞ) 2 16 4 ; 416 xx + +− ɟ) 2 2 1 . 2 x x yx xy y − −− + Ⱦɨɜɟɞɿɬɶ ɬɨɬɨɠɧɿɫɬɶ: 102. ɚ) 2 2 1 1 2 ;m m n m n m n + = − + − ɛ) 2 2 1 1 . 11 1 b bb b − = +− − 103. ɚ) 2 3 3 9 ; 3 3a a a a − = − − ɛ) 2 2 1 . ( ) ( ) a b a ba b a b − = −− −
  • 32. 4. Ⱦɨɞɚɜɚɧɧɹ ɿ ɜɿɞɧɿɦɚɧɧɹ ɞɪɨɛɿɜ ɡ ɪɿɡɧɢɦɢ ɡɧɚɦɟɧɧɢɤɚɦɢ 31 ɉɟɪɟɬɜɨɪɿɬɶ ɭ ɞɪɿɛ ɜɢɪɚɡ: 104. ɚ) 2 2 2 1 1 ;a bc a bc ab c + −+ ɛ) 4 2 3 3 3 14 1 ; 12 9 yx x y x y −+ − ɜ) 1 1 ;c d cm cn dm dn + +− + + ɝ) ; 12 12 18 18 x x x y x y + − + ɞ) ; 4 4 4 4 a b a b a b b a − ++ + − ɟ) 2 2 2 1 1 . ( )b a a b − − − 105. ɚ) 3 2 2 3 2 1 3 1 ; 16 24 a b a b a b + +− ɛ) 7 5 ; ax ay by bx − − − ɜ) 2 2 2 ;b b a b a ab − − + ɝ) 2 1 . 2 2( ) x y x yx y − + ++ ɋɩɪɨɫɬɿɬɶ ɜɢɪɚɡ: 106. ɚ) 2 2 5 3 1 ; 3 1 6 2 c c c c c + −− + + ɛ) 2 2 ;b b y bby y + −− ɜ) 2 ; 6 9 4 9 n n n n + + − ɝ) 2 5 2 1 ; 10 225 x xx − − −− ɞ) 2 2 2 8 ; 2 4 a a a a − + + − ɟ) 2 2 2 8 ; 8 2 4 a b a b a ab b ab + ++ − − ɽ) 2 5 15 ; 2 12 12 36m m m − + + + ɠ) 2 2 2 4 ; 9 24 16 3 4 x x x xa a xa a + − + − ɡ) 3 2 9 3 ; 27 3 9 a a a a −+ + − + ɢ) 2 3 2 2 4 . 2 8 x x x x +− − − 107. ɚ) 2 1 10 ; 10 − +− − a a a a a ɛ) 2 2 1 1 ;b a ab b ab a + ++ − − ɜ) 2 9 9 ; 2 6 9 x x x − + − ɝ) 2 5 20 ; 4 32 64b b + − − ɞ) 2 5 45 ; 9 18 81 + − − + a a a a a ɟ) 2 2 2 ; 4 12 9 4 6 y y x xy y x xy − + + + ɽ) 3 2 1 ; 1 1 x x x x − − + + ɠ) 3 1 2 . 2 8 a a a + + +
  • 33. 32 § 1. Ɋɚɰɿɨɧɚɥɶɧɿ ɜɢɪɚɡɢ ɉɨɞɚɣɬɟ ɭ ɜɢɝɥɹɞɿ ɞɪɨɛɭ: 108. ɚ) 5 6 1 1; 8 4 2 a a a a a a + − ++ − ɛ) 1 1 ; x y xy x y − − − ɜ) 2 2 ;a b a b a b aba b + + −− − ɝ) 2 2 2 5 ;m n mn n m − − + ɞ) 2 3 3 ; 2 2 a b b ab b a + − + ɟ) 1 2 1; 2 1 x x x x − −− + + + ɽ) 2 2 ; x y x y x y + + − + ɠ) 4 2 2 1 1. 1 aa a +− + − 109. ɚ) 3 1 1; 3 3 a a a + + − ɛ) 2 2 2 2 1 1 ;a b a b ab a b +− − ɜ) 2 2 1 1 ;nk k kn −− − ɝ) 2 1 3 1 ; 3 3 x x xy yx − −− + ɞ) 2 ( ) ; m n m n m n − − + + ɟ) 2 11 . 1 1 x x x − + − − + ɋɩɪɨɫɬɿɬɶ ɜɢɪɚɡ: 110. ɚ) 2 2 4 1 ; 5 25 5 2525a aa − − − +− ɛ) 2 2 2 3 3 1 . 2 4 4 4 x x x x x x + +− + + − − + 111. ɚ) 2 2 1 1 3 ; 2 2 4 a a b b a a b − − + − − ɛ) 2 3 1 2 . 3 36 9 m m mm m − + − − ++ + Ɂɧɚɣɞɿɬɶ ɡɧɚɱɟɧɧɹ ɜɢɪɚɡɭ: 112. ɚ) 2 2 2 1 2 2 x x x xx x − −− + + + , ɹɤɳɨ ɯ = 3 ; 11 − ɛ) 2 2 2 2 3 3a a a ab a b a b a b +− − − + − , ɹɤɳɨ ɚ = –1,5; b = 11,5. 113. ɚ) 2 2 3 2 3 3 3 a a a a a a − − − − , ɹɤɳɨ a = 2 ; 17 ɛ) 2 2 2 2 6 4 3 2 9 4 x y xy y x y x y + + − − − , ɹɤɳɨ ɯ = 4,2; ɭ = 1,3.
  • 34. 4. Ⱦɨɞɚɜɚɧɧɹ ɿ ɜɿɞɧɿɦɚɧɧɹ ɞɪɨɛɿɜ ɡ ɪɿɡɧɢɦɢ ɡɧɚɦɟɧɧɢɤɚɦɢ 33 Ⱦɨɜɟɞɿɬɶ ɬɨɬɨɠɧɿɫɬɶ: 114. ɚ) 2 2 2 2(2 1)2 ; 2 1 1 ( 1)( 1) xx x x x x x x ++= − + + − − + ɛ) 2 2 2 3 1 5 3 3 1 . 1 2 2 2 ( 1) b b bb b b b b − ++ − = − + − 115. ɚ) 2 2 2 2 2 ; 2 ( ) n m n m n mn m n mn n n m += + − + − − ɛ) 2 2 1 3 4 15 21 . 3 3 9 9 a a a a a −− − = + − − − 116. Ɂɧɚɣɞɿɬɶ ɬɚɤɿ ɱɢɫɥɚ ɚ ɿ b, ɳɨɛ ɪɿɜɧɿɫɬɶ ɜɢɤɨɧɭɜɚɥɚɫɹ ɞɥɹ ɜɫɿɯ ɞɨɩɭɫɬɢ- ɦɢɯ ɡɧɚɱɟɧɶ ɯ: ɚ) 1 ( 1)( 2)x x+ + = ; 1 2 a b x x + + + ɛ) 2 1 ( 3)( 4) x x x − − + = . 3 4 a b x x + − + 117. ɋɩɪɨɫɬɿɬɶ ɜɢɪɚɡ: ɚ) 1 1 1 ; ( )( ) ( )( ) ( )( )a b a c b c b a c a c b + + − − − − − − ɛ) 2 4 8 1 1 2 4 8 ; 1 1 1 1 1x x x x x + + + + − + + + + ɜ) 3 7 2 4 8 1 1 2 4 8 ; 1 1 1 1 1 x x x x x x x x − − − − − + + + + ɝ) 1 1 1 1 ; ( 1) ( 1)( 2) ( 2)( 3) ( 3)( 4)a a a a a a a a + + + + + + + + + + ɞ) 2 2 3 2 3 2 3 2 4 1 1 2 . 1 1 1 1 b b b b b b b b b b b b b b + − − −+ + + − − + − + + + − 118. Ⱦɨɜɟɞɿɬɶ ɬɨɬɨɠɧɿɫɬɶ: ɚ) ( )( ) ( )( ) ( )( ) a b b c c a b c c a c a a b a b b c + + ++ + − − − − − − = 0; ɛ) ( )( ) ( )( ) ( )( ) b c c a a b a b a c b c b a c a c b − − −+ + − − − − − − = 2 2 2 . a b b c c a + + − − − 2* ȼ. Ʉɪɚɜɱɭɤ. Ⱥɥɝɟɛɪɚ. 8 ɤɥ.
  • 35. 34 § 1. Ɋɚɰɿɨɧɚɥɶɧɿ ɜɢɪɚɡɢ 119. Ɉɛɱɢɫɥɿɬɶ: ɚ) 9 24 5 ; 16 25 27 ⋅ ⋅ ɛ) 4 2 16 8: : ; 9 27 17 51 − ɜ) ( )5 10,4 8 5 0,8 5: 2 ; 8 2 + ⋅ − ⋅ − ɝ) ( )7 2 21 8 8,9 2,6 : 34 . 8 3 5 § ·⋅ − − ⋅¨ ¸ © ¹ 120. Ɋɨɡɜ’ɹɠɿɬɶ ɫɢɫɬɟɦɭ ɪɿɜɧɹɧɶ: ɚ) 2 3 12; 2 4; x y x y + =­ ® − =¯ ɛ) 3 5 22; 7 2 24. x y x y − =­ ® + =¯ 121. ɉɪɨɬɹɝɨɦ ɪɨɤɭ ɜɤɥɚɞɧɢɤ ɡɧɹɜ ɡɿ ɫɜɨɝɨ ɪɚɯɭɧɤɭ 3 5 ɭɫɿɯ ɝɪɨɲɟɣ ɿ ɧɟ ɪɨɛɢɜ ɧɨɜɢɯ ɜɧɟɫɤɿɜ. ɍ ɤɿɧɰɿ ɪɨɤɭ ɛɚɧɤ ɧɚɪɚɯɭɜɚɜ 12% ɪɿɱɧɢɯ, ɿ ɧɚ ɪɚɯɭɧɤɭ ɜɤɥɚɞɧɢɤɚ ɫɬɚɥɨ 896 ɝɪɧ. ɋɤɿɥɶɤɢ ɝɪɨɲɟɣ ɛɭɥɨ ɧɚ ɪɚɯɭɧɤɭ ɜɤɥɚɞɧɢɤɚ ɧɚ ɩɨɱɚɬɤɭ ɪɨɤɭ? 122. Ʉɨɦɩ’ɸɬɟɪɧɢɣ ɤɥɭɛ ɩɥɚɧɭɽ ɩɪɚɰɸɜɚɬɢ 9 ɝɨɞ ɧɚ ɞɟɧɶ ɣ ɨɛɫɥɭɝɨɜɭɜɚɬɢ ɤɨɠɧɨɝɨ ɱɥɟɧɚ ɤɥɭɛɭ ɳɨɞɟɧɧɨ ɡɚ ɨɤɪɟɦɢɦ ɤɨɦɩ’ɸɬɟɪɨɦ ɩɪɨɬɹɝɨɦ 1,5 ɝɨɞ. əɤɭ ɧɚɣɦɟɧɲɭ ɤɿɥɶɤɿɫɬɶ ɤɨɦɩ’ɸɬɟɪɿɜ ɞɥɹ ɰɶɨɝɨ ɩɨɬɪɿɛɧɨ, ɹɤɳɨ ɤɿɥɶɤɿɫɬɶ ɱɥɟɧɿɜ ɤɥɭɛɭ ɞɨɪɿɜɧɸɽ 50? 123. ɍ ɤɥɚɫɿ ɧɚɜɱɚɸɬɶɫɹ 29 ɭɱɧɿɜ. ȼɿɞɨɦɨ, ɳɨ ɫɟɪɟɞ ɛɭɞɶ-ɹɤɢɯ ɬɪɶɨɯ ɡ ɧɢɯ ɽ ɩɪɢɧɚɣɦɧɿ ɞɜɨɽ ɞɪɭɡɿɜ. Ⱦɨɜɟɞɿɬɶ, ɳɨ ɭ ɤɥɚɫɿ ɡɧɚɣɞɟɬɶɫɹ ɭɱɟɧɶ, ɹɤɢɣ ɦɚɽ ɧɟ ɦɟɧɲɟ ɧɿɠ 14 ɞɪɭɡɿɜ.
  • 36. Ɂɚɜɞɚɧɧɹ ɞɥɹ ɫɚɦɨɩɟɪɟɜɿɪɤɢ ʋ 1 35 Ɂɚɜɞɚɧɧɹ ɞɥɹ ɫɚɦɨɩɟɪɟɜɿɪɤɢ ʋ 1 Ɋɿɜɟɧɶ 1 1. ɑɨɦɭ ɞɨɪɿɜɧɸɽ ɡɧɚɱɟɧɧɹ ɜɢɪɚɡɭ 2 1 5 x x − − , ɹɤɳɨ ɯ = –4? ɚ) 1; ɛ) –1; ɜ) 7 ; 9 ɝ) ɧɟ ɿɫɧɭɽ. 2. Ⱦɥɹ ɹɤɢɯ ɡɧɚɱɟɧɶ ɡɦɿɧɧɨʀ ɧɟ ɦɚɽ ɡɦɿɫɬɭ ɜɢɪɚɡ 8 ? 2 5 x x + − ɚ) ɯ = 0; ɛ) ɯ = 2; ɜ) ɯ = 2,5; ɝ) ɯ = 5. 3. ɋɤɨɪɨɬɿɬɶ ɞɪɿɛ 2 3 18 . 3 a a ɚ) 18; a ɛ) 3 1 ; 6a ɜ) 6 ; a ɝ) 3 6 . a 4. Ɂɜɟɞɿɬɶ ɞɪɿɛ 3 b ɞɨ ɡɧɚɦɟɧɧɢɤɚ b2 . ɚ) 2 3 ; b ɛ) 2 3 ;b b ɜ) 2 2 3 ;b b ɝ) 2 3 .b b 5. Ⱦɨɞɚɣɬɟ ɞɪɨɛɢ: 2 2 3 1 5 8 . y y y y − − + ɚ) 2 4 11 ; y y + ɛ) 2 4 5 ; 2 y y − ɜ) 4 – 5ɭ; ɝ) 2 4 5 . y y − 6. ɋɩɪɨɫɬɿɬɶ ɜɢɪɚɡ . 2 a b a b a a + −− ɚ) ; 2 a b a + ɛ) ;b a ɜ) 3 ; 2 a b a + ɝ) 3 . 2 a b a +
  • 37. 36 § 1. Ɋɚɰɿɨɧɚɥɶɧɿ ɜɢɪɚɡɢ Ɋɿɜɟɧɶ 2 7. ɍɫɬɚɧɨɜɿɬɶ ɜɿɞɩɨɜɿɞɧɿɫɬɶ ɦɿɠ ɜɢɪɚɡɚɦɢ (1–4) ɬɚ ɡɧɚɱɟɧɧɹɦɢ ɡɦɿɧɧɨʀ (Ⱥ–Ⱦ), ɞɥɹ ɹɤɢɯ ɜɢɪɚɡ ɧɟ ɦɚɽ ɡɦɿɫɬɭ. 1) 3 ; 2 3 a a − − Ⱥ) 0; –1; 2) 3 2 4 5;c c + Ȼ) 3; 3) 2 11 ; 2 10 x x x− ȼ) 1,5; 4) 2 1 1. 1 2 z z z z − +− + Ƚ) 0; 5; Ⱦ) 0. 8. ɋɤɨɪɨɬɿɬɶ ɞɪɿɛ: ɚ) 3 2 4 27 ; 36 a b a b ɛ) 5 10 . 3 6 a b a b − − 9. Ɂɧɚɣɞɿɬɶ ɡɧɚɱɟɧɧɹ ɜɢɪɚɡɭ 2 2 6 3 a a a − − , ɹɤɳɨ ɚ = 1,7. 10. ɉɨɞɚɣɬɟ ɭ ɜɢɝɥɹɞɿ ɞɪɨɛɭ: ɚ) 3 2 35 ; yx a b ab − ɛ) . 4 4 7 7 a b x y x y + + + 11. ɋɩɪɨɫɬɿɬɶ ɜɢɪɚɡ: ɚ) 2 2 ; x y x y + + − ɛ) 2 2 8 16 .− − − n m n m n Ɋɿɜɟɧɶ 3 12. Ɂɧɚɣɞɿɬɶ ɞɨɩɭɫɬɢɦɿ ɡɧɚɱɟɧɧɹ ɡɦɿɧɧɨʀ ɭ ɜɢɪɚɡɿ 2 14 . ( 2) 4− − k k 13. ɋɤɨɪɨɬɿɬɶ ɞɪɿɛ: ɚ) 2 5 3 196 ; 35 x y x y ɛ) 3 2 2 3 2 3 . 9 a a ab a b − − 14. ɉɨɞɚɣɬɟ ɭ ɜɢɝɥɹɞɿ ɞɪɨɛɭ: ɚ) 2 2 1 1 ;a a a a a a + −− − + ɛ) 2 . ( ) mn n n m n m nm n + + − ++
  • 38. Ɂɚɜɞɚɧɧɹ ɞɥɹ ɫɚɦɨɩɟɪɟɜɿɪɤɢ ʋ 1 37 15. Ɂɧɚɣɞɿɬɶ ɡɧɚɱɟɧɧɹ ɜɢɪɚɡɭ 2 2 1 1 m mn mn n − − − , ɹɤɳɨ ɬ = 0,7; ɩ = 1. 3 16. ɋɩɪɨɫɬɿɬɶ ɜɢɪɚɡ: ɚ) 2 ;aba b a b + − + ɛ) 2 2 2 . 2 b a b a ab b b ab +− − + − Ɋɿɜɟɧɶ 4 17. Ⱦɥɹ ɹɤɢɯ ɡɧɚɱɟɧɶ ɡɦɿɧɧɨʀ ɧɟ ɦɚɽ ɡɦɿɫɬɭ ɜɢɪɚɡ? ɚ) 2 15 ; 2 15a a+ − ɛ) 3 | 7 | | |− +x x . 18. ɋɤɨɪɨɬɿɬɶ ɞɪɿɛ: ɚ) 3 2 10 4 40 ; 10 x x x x − − + − ɛ) 2 2 16 64. 8 x x a x a − − + + − 19. ɉɨɞɚɣɬɟ ɭ ɜɢɝɥɹɞɿ ɞɪɨɛɭ: ɚ) 2 2 5 1 6 1 ; 5 5 6 12 6 x x x x x x − −− + + + ɛ) 2 2 2 2 2 4 . 2 2 4 4 a b a b a b a b a ab b ++ − − + + + 20. ɋɩɪɨɫɬɿɬɶ ɜɢɪɚɡ 2 2 3 1 1 4 . 2 2 4 8 2 a a a a a a a ++ − + − + − − 21. Ⱦɨɜɟɞɿɬɶ ɬɨɬɨɠɧɿɫɬɶ 2 4 8 1 1 2 4 8 . 1 1 1 1 1 + + + = − + + + −x x x x x
  • 39. 38 § 1. Ɋɚɰɿɨɧɚɥɶɧɿ ɜɢɪɚɡɢ 1. Ɇɧɨɠɟɧɧɹ ɞɪɨɛɿɜ. Ʉɨɥɢ ɦɧɨɠɚɬɶ ɡɜɢɱɚɣɧɿ ɞɪɨɛɢ, ɬɨ ɨɤɪɟɦɨ ɦɧɨɠɚɬɶ ʀɯɧɿ ɱɢɫɟɥɶɧɢɤɢ ɬɚ ɡɧɚɦɟɧɧɢɤɢ ɿ ɩɟɪɲɢɣ ɞɨɛɭɬɨɤ ɡɚɩɢɫɭɸɬɶ ɱɢɫɟɥɶɧɢɤɨɦ ɞɪɨɛɭ, ɚ ɞɪɭɝɢɣ — ɡɧɚɦɟɧɧɢɤɨɦ. ɇɚɩɪɢɤɥɚɞ, 3 5 3 5 15 . 4 7 4 7 28 ⋅⋅ = = ⋅ Ɍɚɤ ɫɚɦɨ ɦɧɨɠɚɬɶ ɛɭɞɶ-ɹɤɿ ɞɪɨɛɢ a b ɿ c d : .a c ac b d bd ⋅ = (1) Ɋɿɜɧɿɫɬɶ (1) ɽ ɬɨɬɨɠɧɿɫɬɸ, ɬɨɛɬɨ ɜɨɧɚ ɽ ɩɪɚɜɢɥɶɧɨɸ ɞɥɹ ɛɭɞɶ-ɹɤɢɯ ɡɧɚ- ɱɟɧɶ ɚ, b, c ɿ d, ɞɟ b ≠ 0 ɿ d ≠ 0. Ɂ ɬɨɬɨɠɧɨɫɬɿ (1) ɜɢɩɥɢɜɚɽ ɩɪɚɜɢɥɨ ɦɧɨɠɟɧɧɹ ɞɪɨɛɿɜ: ɓɨɛ ɩɨɦɧɨɠɢɬɢ ɞɪɿɛ ɧɚ ɞɪɿɛ, ɩɨɬɪɿɛɧɨ ɩɟɪɟɦɧɨɠɢɬɢ ɨɤɪɟɦɨ ʀɯɧɿ ɱɢɫɟɥɶɧɢɤɢ ɬɚ ɡɧɚɦɟɧɧɢɤɢ ɿ ɩɟɪɲɢɣ ɞɨɛɭɬɨɤ ɡɚɩɢɫɚɬɢ ɱɢɫɟɥɶɧɢɤɨɦ, ɚ ɞɪɭɝɢɣ — ɡɧɚɦɟɧɧɢɤɨɦ ɞɪɨɛɭ. ɐɟ ɩɪɚɜɢɥɨ ɩɨɲɢɪɸɽɬɶɫɹ ɧɚ ɜɢɩɚɞɨɤ ɦɧɨɠɟɧɧɹ ɬɪɶɨɯ ɿ ɛɿɥɶɲɟ ɞɪɨɛɿɜ. 2. ɉɿɞɧɟɫɟɧɧɹ ɞɪɨɛɭ ɞɨ ɫɬɟɩɟɧɹ. ȼɢɤɨɪɢɫɬɨɜɭɸɱɢ ɩɪɚɜɢɥɨ ɦɧɨɠɟɧɧɹ ɞɪɨɛɿɜ, ɩɿɞɧɟɫɟɦɨ ɞɪɿɛ ɞɨ n-ɝɨ ɫɬɟɩɟɧɹ: ( ) P N ɪɚɡɿɜ ɪɚɡɿɜɪɚɡɿɜ ...... . ... n n n n nn a a a a aa a a b b b b bb b b = ⋅ ⋅ ⋅ = = Ɉɬɠɟ, ( ) . n n n a a b b = (2) Ɂ ɬɨɬɨɠɧɨɫɬɿ (2) ɜɢɩɥɢɜɚɽ ɩɪɚɜɢɥɨ ɩɿɞɧɟɫɟɧɧɹ ɞɪɨɛɭ ɞɨ ɫɬɟɩɟɧɹ: ɓɨɛ ɩɿɞɧɟɫɬɢ ɞɪɿɛ ɞɨ ɫɬɟɩɟɧɹ, ɩɨɬɪɿɛɧɨ ɩɿɞɧɟɫɬɢ ɞɨ ɰɶɨɝɨ ɫɬɟɩɟɧɹ ɱɢɫɟɥɶɧɢɤ ɬɚ ɡɧɚɦɟɧɧɢɤ ɿ ɩɟɪɲɢɣ ɪɟɡɭɥɶɬɚɬ ɡɚɩɢɫɚɬɢ ɱɢɫɟɥɶɧɢ- ɤɨɦ, ɚ ɞɪɭɝɢɣ — ɡɧɚɦɟɧɧɢɤɨɦ ɞɪɨɛɭ.
  • 40. 5. Ɇɧɨɠɟɧɧɹ ɞɪɨɛɿɜ. ɉɿɞɧɟɫɟɧɧɹ ɞɪɨɛɭ ɞɨ ɫɬɟɩɟɧɹ 39 ȼɩɪɚɜɚ 1. ȼɢɤɨɧɚɬɢ ɦɧɨɠɟɧɧɹ: ɚ) 4 3 2 3 6 ; 8 a b c c a ⋅ ɛ) 2 2 2 2 .ab b b a a b + ⋅ − • ɚ) 4 3 2 3 6 8 a b c c a ⋅ = 4 3 2 3 6 8 a b c c a ⋅ ⋅ = 3 ; 4 abc ɛ) 2 2 2 2 ab b b a a b + ⋅ − = 2 ( ) ( )( ) b a b b a a b a b + ⋅ ⋅ − + = 2 2 ( ) b a a b ⋅ − • ȼɩɪɚɜɚ 2. ɉɨɦɧɨɠɢɬɢ ɞɪɿɛ 3 3 x x + − ɧɚ ɦɧɨɝɨɱɥɟɧ ɯ – 3. • Ɂɚɩɢɫɚɜɲɢ ɦɧɨɝɨɱɥɟɧ ɯ – 3 ɭ ɜɢɝɥɹɞɿ ɞɪɨɛɭ 3 1 x − , ɦɚɬɢɦɟɦɨ: 3 ( 3) 3 + ⋅ − − x x x = 3 3 3 1 x x x + −⋅ − = ( 3) ( 3) ( 3) 1 x x x + ⋅ − − ⋅ = 3x + . ɋɤɨɪɨɱɟɧɢɣ ɡɚɩɢɫ: 3 ( 3) 3 + ⋅ − − x x x = ( 3)( 3) 3 x x x + − − = 3x + .• ȼɩɪɚɜɚ 3. ɉɿɞɧɟɫɬɢ ɞɨ ɤɜɚɞɪɚɬɚ ɞɪɿɛ 3 2 2 5 a b m n − . • 23 2 2 5 a b m n § · −¨ ¸ © ¹ = 23 2 2 5 § · ¨ ¸ © ¹ a b m n = 3 2 2 2 (2 ) (5 ) a b m n = 2 3 2 2 2 2 2 2 2 ( ) 5 ( ) a b m n ⋅ ⋅ ⋅ ⋅ = 6 2 4 2 4 . 25 a b m n ɋɤɨɪɨɱɟɧɢɣ ɡɚɩɢɫ: 23 2 2 5 a b m n § · −¨ ¸ © ¹ = 6 2 4 2 4 . 25 a b m n • 124. ȼɢɤɨɧɚɣɬɟ ɦɧɨɠɟɧɧɹ: ɚ) ;a m b n ⋅ ɛ) 5 2 ; 3 5 b a ⋅ ɜ) 2 ;m k ⋅ ɝ) 3 1 . 4 x x ⋅
  • 41. 40 § 1. Ɋɚɰɿɨɧɚɥɶɧɿ ɜɢɪɚɡɢ 125. ɉɿɞɧɟɫɿɬɶ ɞɨ ɫɬɟɩɟɧɹ: ɚ) ( ) 2 ;a c ɛ) ( ) 2 2 ; 3 a c ɜ) 42 ;a c § · ¨ ¸ © ¹ ɝ) 32 3 3 .a c § · −¨ ¸ © ¹ ȼɢɤɨɧɚɣɬɟ ɦɧɨɠɟɧɧɹ: 126. ɚ) 4 5 ; 3 16 b a ⋅ ɛ) 3 2 ; 5 9 k k ⋅ ɜ) 2 8 1 ; 11 b b ⋅ ɝ) 4 2 14 ; 7 § · ⋅ −¨ ¸ © ¹ y y ɞ) 2 4 25 ; 5 c d d c ⋅ ɟ) 3 2 1012 ; 5 yx y x ⋅ ɽ) 3 34 ; 2 ax x ⋅ ɠ) ( ) 25 3 . 6 m n m n − ⋅ 127. ɚ) 4 2 3 2 ; 94 b b ⋅ ɛ) 5 6 21; 7 k k ⋅ ɜ) 3 2 6 ; 9 § · ⋅ −¨ ¸ © ¹ x a a x ɝ) 2 5 2 . 4 a ay y ⋅ ɉɨɞɚɣɬɟ ɭ ɜɢɝɥɹɞɿ ɞɪɨɛɭ: 128. ɚ) 2 2 3 5 25 10 ; 4 15 a b b a ⋅ ɛ) 3 4 5 3 ; 9 a ab b − ⋅ ɜ) 2 5 17 . 34 § · − ⋅ −¨ ¸ © ¹ y x y x 129. ɚ) 22 2 3 272 ; 9 4 yx y x ⋅ ɛ) 2 3 2 ( 10 ); 5 a ab b − ⋅ − ɜ) 2 3 312 . 16 § ·⋅ −¨ ¸ © ¹ m mn ȼɢɤɨɧɚɣɬɟ ɦɧɨɠɟɧɧɹ: 130. ɚ) 3 2 2 ; ( ) x y a b a x y − ⋅ − ɛ) 33 ( ) ; m nx m n x + ⋅ + ɜ) 2 3 3 3 ; x y b x yb + ⋅ + ɝ) 2 2 1 7 ; 2 17 x x xx x − −⋅ −− ɞ) 2 9 2 ; 2 3 m m m m − +⋅ + − ɟ) 2 2 4 4 4 . 4 4 a a a a a − + +⋅ + − 131. ɚ) 4 2 ; ya b a by + ⋅ + ɛ) 4 3 2 ; ( ) x y a a x y + ⋅ + ɜ) 2 ;ab ac k b ck + ⋅ + ɝ) 2 2 1 1; 1 1 b b x x b − + −⋅ − − ɞ) 2 2 16 ; 4 y b ab y − ⋅ − ɟ) 2 5 3 2 2 1 . 1 c c c c c + + ⋅ − ɉɿɞɧɟɫɿɬɶ ɞɨ ɫɬɟɩɟɧɹ: 132. ɚ) 22 2 ;x y § · ¨ ¸ © ¹ ɛ) 43 2 ; 3 a b § · −¨ ¸ © ¹ ɜ) 32 3 ; 5 n k m § · −¨ ¸ © ¹ ɝ) 32 4 3 3 . 4 a b c § · ¨ ¸ © ¹
  • 42. 5. Ɇɧɨɠɟɧɧɹ ɞɪɨɛɿɜ. ɉɿɞɧɟɫɟɧɧɹ ɞɪɨɛɭ ɞɨ ɫɬɟɩɟɧɹ 41 133. ɚ) 2 2 3 ;m n § · ¨ ¸ © ¹ ɛ) 3 2 2 2 ;x y z § · ¨ ¸ © ¹ ɜ) 33 2 3 ; 5 a b c § · −¨ ¸ © ¹ ɝ) 23 4 3 9 . 5 x y a § · −¨ ¸ © ¹ ȼɢɤɨɧɚɣɬɟ ɦɧɨɠɟɧɧɹ: 134. ɚ) 23 3 2 2 512 ; 18 x ya b x a a ⋅ ⋅ ɛ) 2 3 2 3 2 4 ; 4 3 3 m n n n m m § ·⋅ − ⋅¨ ¸ © ¹ ɜ) 2 3 5 ; 10 bx byab xy y a + ⋅ + ɝ) 2 2 3 2 3 ; xy a a a a x y +⋅ + ɞ) 2 2 18 4 2 ; 272 a x axx x −⋅ − ɟ) 2 2 3 2 . 9 6 a b ab a a bab b + +⋅ ++ 135. ɚ) 2 3 2 3 2 6 ; 3 4 ab m mn mn a ab ⋅ ⋅ ɛ) 2 3 2 5 5 ; 15 15 x y x x x yx x + +⋅ ++ ɜ) 2 3 6 316 . 2 20 ymn y y m n + ⋅ + ɋɩɪɨɫɬɿɬɶ ɜɢɪɚɡ: 136. ɚ) 2 2 3 1 1 ; 1 9 1 a a a a − +⋅ − − ɛ) 22 2 ; 9 9 x xyb a x y a b −− ⋅ − − ɜ) 2 2 2 4 ; ( ) 32 xy x xy x y x + ⋅ + ɝ) 2 2 2 5 ; 10 y a y a y y −§ · ⋅¨ ¸−© ¹ ɞ) 2 2 2 (2 1) ; 7 7 4 2 x a b b a x + −⋅ − + ɟ) 2 2 3 2 2 4 4 2 ; ( ) 2 2 m n m mn n m n m n − + +⋅ + − ɽ) 2 23 3 2 2 ; 2 2 x ym n y x m mn n −− ⋅ − + + ɠ) 2 2 3 3 4 3 3 . 2 c a a b a ca b − +⋅ ++ 137. ɚ) 2 2 5 5 ; 2 x ya b a bx xy y −− ⋅ −− + ɛ) 2 2 2 ( )3 3 ; 5 5 a ba b a b b a +− ⋅ + − ɜ) 2 2 2 2 (4 ) ; 12 3 x x y xxy y − − ⋅ −+ ɝ) 2 2 2 3 3 1 ; 3 32 2 y a ab b ya b − − +⋅ −+ ɞ) ( ) 2 2 15 ; 52 2 a a b aab b +⋅ + ɟ) 2 2 3 2 1 1 . 1 2 1 a x x x a a − + +⋅ − + +
  • 43. 42 § 1. Ɋɚɰɿɨɧɚɥɶɧɿ ɜɢɪɚɡɢ Ɂɧɚɣɞɿɬɶ ɡɧɚɱɟɧɧɹ ɜɢɪɚɡɭ: 138. ɚ) 22 5 3 2 3 98 , 7 94 a b a b a § ·⋅ − ⋅¨ ¸ © ¹ ɹɤɳɨ ɚ = –1,25; b = 8; ɛ) 2 3 2 2 2 8 16 3 12 16 4 x x x x x x x + + −⋅ − + , ɹɤɳɨ ɯ = –1; ɯ = 0,8; ɯ = 24 . 3 139. ɚ) 3 9 2 4 2 16 3 , 2 3 yx y x y § · § · ⋅ ⋅ −¨ ¸ ¨ ¸ © ¹ © ¹ ɹɤɳɨ ɯ = 1 ; 7 ɭ = 0,5; ɛ) 3 3 3 2 27 4 0,2 3 9 a a a a a + ⋅ − + , ɹɤɳɨ ɚ = –4; ɚ = 5; ɚ = 1 . 4 − 140. ɋɩɪɨɫɬɿɬɶ ɜɢɪɚɡ: ɚ) 2 2 2 2 2 2 ; 2 2 x xy xz yz x xy x y x xy xz yz x xy x y + + + − − + ⋅ − + − + − − ɛ) 23 34 2 2 2 2 2 2 . 2 a a b b a b a ab b a b § ·§ ·− + −§ ·⋅¨ ¸¨ ¸¨ ¸ − + −© ¹© ¹ © ¹ 141. Ɂɧɚɣɞɿɬɶ ɡɧɚɱɟɧɧɹ ɜɢɪɚɡɭ 21 1 16 4 1616 64 4 64 n n n n n + + § ·+ ⋅⋅¨ ¸ − ⋅© ¹ , ɹɤɳɨ n = 74; n = 1000. 142. Ⱦɨɜɟɞɿɬɶ ɬɨɬɨɠɧɿɫɬɶ 2 2 ( 2) 2 2 2( 2) 2 x a x a x a x x a xx a x a + + + − + ⋅ = + −− + + . 143. Ɂɧɚɣɞɿɬɶ ɱɢɫɥɚ, ɨɛɟɪɧɟɧɿ ɞɨ ɞɚɧɢɯ: 2 ; 7 4; 51 ; 6 0,2; 1,6. 144. Ɉɛɱɢɫɥɿɬɶ: ɚ) 18 4: ; 25 15 ɛ) 2 14 : 42 ; 3 6 − ɜ) 1 20,125:3 1 : 7. 8 5 −
  • 44. 6. Ⱦɿɥɟɧɧɹ ɞɪɨɛɿɜ 43 145. ɍ ɩɟɪɲɨɦɭ ɪɟɡɟɪɜɭɚɪɿ ɛɭɥɨ 480 ɥ ɜɨɞɢ, ɚ ɜ ɞɪɭɝɨɦɭ — 282 ɥ. Ɂ ɩɟɪɲɨɝɨ ɪɟɡɟɪɜɭɚɪɚ ɛɟɪɭɬɶ ɳɨɞɟɧɧɨ 25 ɥ ɜɨɞɢ, ɚ ɡ ɞɪɭɝɨɝɨ — 16 ɥ. ɑɟɪɟɡ ɫɤɿɥɶɤɢ ɞɧɿɜ ɭ ɩɟɪɲɨɦɭ ɪɟɡɟɪɜɭɚɪɿ ɜɨɞɢ ɛɭɞɟ ɭɞɜɿɱɿ ɛɿɥɶɲɟ, ɧɿɠ ɭ ɞɪɭɝɨɦɭ? 146*. ȼɿɞ ɩɪɢɫɬɚɧɿ A ɞɨ ɩɪɢɫɬɚɧɿ B ɡɚ ɬɟɱɿɽɸ ɪɿɱɤɢ ɨɞɧɨɱɚɫɧɨ ɜɿɞɩɥɢɜɥɢ ɤɚɬɟɪ ɿ ɩɥɿɬ. Ʉɨɥɢ ɱɟɪɟɡ 1,5 ɝɨɞ ɤɚɬɟɪ ɩɪɢɛɭɜ ɞɨ ɩɪɢɫɬɚɧɿ B, ɩɥɨɬɭ ɡɚɥɢɲɚɥɨɫɹ ɩɪɨɩɥɢɫɬɢ ɞɨ ɰɿɽʀ ɩɪɢɫɬɚɧɿ ɳɟ 27 ɤɦ. ɇɟ ɡɚɬɪɢɦɭɸɱɢɫɶ ɧɚ ɩɪɢɫɬɚɧɿ B, ɤɚɬɟɪ ɜɢɪɭɲɢɜ ɭ ɡɜɨɪɨɬɧɢɣ ɲɥɹɯ. ɑɟɪɟɡ ɹɤɢɣ ɱɚɫ ɩɿɫɥɹ ɜɿɞɩɪɚɜɤɢ ɜɿɞ ɩɪɢɫɬɚɧɿ B ɤɚɬɟɪ ɡɭɫɬɪɿɧɟ ɩɥɿɬ? ɑɨɦɭ ɞɨɪɿɜɧɸɽ ɲɜɢɞɤɿɫɬɶ ɤɚɬɟɪɚ ɭ ɫɬɨɹɱɿɣ ɜɨɞɿ? 147. ɍ ɬɪɢɞɟɜ’ɹɬɨɦɭ ɤɨɪɨɥɿɜɫɬɜɿ ɤɨɠɧɿ ɞɜɚ ɦɿɫɬɚ ɡ’ɽɞɧɚɧɿ ɞɨɪɨɝɨɸ ɡ ɨɞɧɨ- ɫɬɨɪɨɧɧɿɦ ɪɭɯɨɦ. Ⱦɨɜɟɞɿɬɶ, ɳɨ ɿɫɧɭɽ ɦɿɫɬɨ, ɡ ɹɤɨɝɨ ɜ ɛɭɞɶ-ɹɤɟ ɿɧɲɟ ɦɿɫɬɨ ɦɨɠɧɚ ɩɪɨʀɯɚɬɢ ɥɢɲɟ ɨɞɧɿɽɸ ɚɛɨ ɞɜɨɦɚ ɞɨɪɨɝɚɦɢ. Ʉɨɥɢ ɞɿɥɹɬɶ ɡɜɢɱɚɣɧɿ ɞɪɨɛɢ, ɬɨ ɩɟɪɲɢɣ ɞɪɿɛ ɦɧɨɠɚɬɶ ɧɚ ɞɪɿɛ, ɨɛɟɪɧɟɧɢɣ ɞɨ ɞɪɭɝɨɝɨ. ɇɚɩɪɢɤɥɚɞ, 2 3 2 5 10: . 7 5 7 3 21 = ⋅ = ɍ ɬɚɤɢɣ ɠɟ ɫɩɨɫɿɛ ɞɿɥɹɬɶ ɛɭɞɶ-ɹɤɿ ɞɪɨɛɢ a b ɿ c d : : .a c a d b d b c = ⋅ Ɉɫɬɚɧɧɹ ɪɿɜɧɿɫɬɶ ɽ ɬɨɬɨɠɧɿɫɬɸ, ɬɨɛɬɨ ɜɨɧɚ ɽ ɩɪɚɜɢɥɶɧɨɸ ɞɥɹ ɜɫɿɯ ɡɧɚ- ɱɟɧɶ ɚ, b, c ɿ d, ɞɟ b ≠ 0, ɫ ≠ 0 ɿ d ≠ 0. Ɂ ɰɿɽʀ ɬɨɬɨɠɧɨɫɬɿ ɜɢɩɥɢɜɚɽ ɩɪɚɜɢɥɨ ɞɿ- ɥɟɧɧɹ ɞɪɨɛɿɜ: ɓɨɛ ɩɨɞɿɥɢɬɢ ɨɞɢɧ ɞɪɿɛ ɧɚ ɞɪɭɝɢɣ, ɩɨɬɪɿɛɧɨ ɩɟɪɲɢɣ ɞɪɿɛ ɩɨɦɧɨ- ɠɢɬɢ ɧɚ ɞɪɿɛ, ɨɛɟɪɧɟɧɢɣ ɞɨ ɞɪɭɝɨɝɨ. ɇɚɩɪɢɤɥɚɞ, 2 2 2 2 2 2 2 2 4: . 2 a a a b a b b a bb b b a ⋅= ⋅ = = ⋅
  • 45. 44 § 1. Ɋɚɰɿɨɧɚɥɶɧɿ ɜɢɪɚɡɢ ȼɩɪɚɜɚ 1. ȼɢɤɨɧɚɬɢ ɞɿɥɟɧɧɹ: ɚ) 2 3 3 15 : ; 714 a a cc ɛ) 2 2 3: ; 1 ab b a a a− − ɜ) 2 2 4 : (2 ). 2 x y x y x − − • ɚ) 2 3 3 15 : 714 a a cc = 2 3 3 15 7 14 a c c a ⋅ = 2 3 3 15 7 14 a c c a ⋅ ⋅ = 2 15 . 2c a ɛ) 2 2 3: 1 ab b a a a− − = ( 1) ( 1)( 1) 3 a aab a a b − ⋅ − + = ( 1) ( 1)( 1) 3 ab a a a a b ⋅ − − + ⋅ = 2 3( 1) a a ⋅ + ɜ) 2 2 4 :(2 ) 2 x y x y x − − = (2 )(2 ) 1 2 2 x y x y x x y − + ⋅ − = (2 )(2 ) 2 (2 ) x y x y x x y − + − = 2 2 x y x + ⋅ • 148. ȼɢɤɨɧɚɣɬɟ ɞɿɥɟɧɧɹ: ɚ) : ;x m y n ɛ) 1 1: ; a b ɜ) : 2; 4 a ɝ) 33: . x ȼɢɤɨɧɚɣɬɟ ɞɿɥɟɧɧɹ: 149. ɚ) 2: ; 9 3 a a ɛ) 6 2: ; 5 15 ab b c ɜ) 2 1: ;x x ɝ) 9 :3; d ɞ) 3 2 3819 : ; 5 nn p ɟ) ( ) 3 33 : 11 ; 12 c c m ɽ) 3 5 3 : ; 2 4 c c a a ɠ) 2 3 3 4 12 3: . 25 5 ab b x x 150. ɚ) 4 3 3 1: ; 10 5 x y y ɛ) 3 4 2 1827 : ; 7 aa b ɜ) 2 2 3 6 3: ; 5 x x y y ɝ) ( ) 2 2 3 5 : 10 . 7 mn n k
  • 46. 6. Ⱦɿɥɟɧɧɹ ɞɪɨɛɿɜ 45 151. ɚ) 3 4 4 18 6: ; 155 a a bb − − ɛ) 3 4 2 3 9 36: ; 20 5 b b n n − ɜ) 2 3 2 28 14 : . 5 x y xy z § · −¨ ¸ © ¹ 152. ɚ) 2 2 2 3 8 4: ; 9 3 x x y y − ɛ) 3 4 2 15 9: ; 82 m m nn − ɜ) 3 2 4 6 3 : . 5 10 xy x y§ · − −¨ ¸ © ¹ ɋɩɪɨɫɬɿɬɶ ɜɢɪɚɡ: 153. ɚ) 4 2 6 6 : ;a b a b c c + + ɛ) 2 3 : ;mn n m n aa − − ɜ) 2 2 2 3 : ;c d c d k k − + ɝ) 6 4 2 : ; 416 x x xx −− ɞ) 3 2 : ; 36 9 b b bb b −− + ɟ) 2 2 4 4 4 : . 1 1 y y y y y − + − + + 154. ɚ) 3 : ;x x ab ac b c+ + ɛ) 2 4 3 4 2: ;a a c c − − ɜ) 2 5 1: ;m n mn m yy + + ɝ) 2 2 25 5: ;k k k k − + ɞ) 2 2 2 : ; 10 25 x xy y x y− + − ɟ) 2 2 2 2: . 2 1 1 a a a a a − − + + − ȼɢɤɨɧɚɣɬɟ ɞɿɥɟɧɧɹ: 155. ɚ) 3 2 3 4 5 : ; 4 8 x x x a a − ɛ) 2 2 3 2 2 1 : ;b b b ac a c − + ɜ) 2 2 5 10 2: ; ( ) a a a a ba b − − −− ɝ) 2 2 3 3 : (2 2); 1 x x x − + + ɞ) 2 2 2 2 18 24: ; 1 (1 ) ab a b x x− − ɟ) 2 2 2 : .a ab b a b a b ac bc + + + − − 156. ɚ) 3 2 2 3 3 7 : ; 11 22 c c c ab a b − ɛ) 2 2 2 4 2: ; 1 x x x xx x − − ++ ɜ) 2 2 2 ( ) 6 6: . 1 m n m n m m m − − + − ɋɩɪɨɫɬɿɬɶ ɜɢɪɚɡ: 157. ɚ) 32 6 2 2 36 3: ; 25 5 a x ax b b § · −¨ ¸ © ¹ ɛ) ( ) 24 2 3 2 10: ; 3 9 m m n n § · − ¨ ¸ © ¹
  • 47. 46 § 1. Ɋɚɰɿɨɧɚɥɶɧɿ ɜɢɪɚɡɢ ɜ) 2 2 2 5 15: ; 7 142 a b a a b bcc § · ⋅¨ ¸ © ¹ ɝ) 33 4 4 39 3: : . 5 45 yx x b y xy § · ¨ ¸ © ¹ 158. ɚ) 52 3 7 5: ; 2 64 mn m n§ · −¨ ¸ © ¹ ɛ) 3 22 3 3 9: ; 2 4 a b a c c § · § · ¨ ¸ ¨ ¸ © ¹ © ¹ ɜ) 2 2 3 4 3 65: : ; 65 5 xy x yz yz z § · ¨ ¸ © ¹ ɝ) 3 2 2 2 4 2 2 27 2 3: . 4 3 4 a b a c b c b ac § · ⋅¨ ¸ © ¹ 159. ɚ) 2 2 2 2 3 6 7 14: ; 2 a b a b a b a ab b + + − − + ɛ) 2 2 2 2 4 4 1 1 4: ; 3 3 c c c x y x y + + − − − ɜ) 2 2 3 3 2 2 4: ;mn m n m n m mn n+ − + ɝ) ( ) 2 2 2 2 3 : ;a a ab a b a b + − − ɞ) 2 2 3 2 2 2 2 2 : ; ( ) 2 x y x xy y x y x xy y − − + + + + ɟ) 3 2 2 2 27 3 9: . 2 4 1 a a a a a a − + + − − 160. ɚ) 2 2 2 2 2 3 ( ) : ; a b a b ab b ab b − − + + ɛ) 2 2 2 2 2( 2 ) : ; 3 3 + + − − x y x y x y x y ɜ) 2 3 2 4 2: ; 1 1 c d ac ad b b b + + + + − ɝ) 2 2 2 3 : . 4 2 8 ab ac c b a a a − − − + + Ɂɧɚɣɞɿɬɶ ɡɧɚɱɟɧɧɹ ɜɢɪɚɡɭ: 161. ɚ) 4 22 3 3 2 : , 7 49 a a b b § · § · −¨ ¸ ¨ ¸ © ¹ © ¹ ɹɤɳɨ ɚ = –0,25; b = 4; ɛ) 2 2 2 4 4 1 2: 10 54 1 m m m m mm + + + −− , ɹɤɳɨ m = –5; m = 0,5; m = 1 . 15 162. ( ) 2 2 2 2 2: 2 4 a a ab a b a b − + − , ɹɤɳɨ ɚ = –3, b = 4; ɚ = 78, b = 11. Ⱦɨɜɟɞɿɬɶ ɬɨɬɨɠɧɿɫɬɶ: 163. ɚ) 2 2 2 2 ( )5 5: ; 4 58 b a ba b a b ab ab −− + = ɛ) ( ) 2 2 2 1 : . ( ) m m m n m n m n m n m n m n − +− − =− + − + −
  • 48. 6. Ⱦɿɥɟɧɧɹ ɞɪɨɛɿɜ 47 164. ɚ) ( ) 2 2 16 63 12 : ; 2 4 x y xx y x x y − − = + ɛ) 3 2 2 2 1 1 1 2 2: : . 2 42 2 1 a a a aa aa a a − − + −− = + − 165. ɋɩɪɨɫɬɿɬɶ ɜɢɪɚɡ: ɚ) 22 6 4 2 2 0,25 0,5 : ; 1 1 x x x x x x − −§ · ¨ ¸ − − + −© ¹ ɛ) 2 2 2 2 6 5 4 5: .a a a a a ab a b a ab a b + + + − − + − − − + 166. Ⱦɨɜɟɞɿɬɶ, ɳɨ ɜɢɪɚɡ 2 2 2 2 3 2 3 : 22 x y x xy y x yx xy y + + − ++ − ɧɚɛɭɜɚɽ ɥɢɲɟ ɞɨɞɚɬɧɢɯ ɡɧɚɱɟɧɶ. 167. ɇɚ ɩɪɢɱɚɥɿ Ⱥ ɫɬɨɹɬɶ ɩɿɞɧɿɦɚɥɶɧɿ ɤɪɚɧɢ ʋ 1 ɿ ʋ 2, ɚ ɧɚ ɩɪɢɱɚɥɿ Ȼ — ɩɿɞ- ɧɿɦɚɥɶɧɿ ɤɪɚɧɢ ʋ 3 ɿ ʋ 4. Ɂɚ ɞɨɩɨɦɨɝɨɸ ɤɪɚɧɚ ʋ 1 ɦɨɠɧɚ ɪɨɡɜɚɧ- ɬɚɠɢɬɢ ɛɚɪɠɭ ɧɚ 3 ɝɨɞ, 2 ɝɨɞ ɿ 1 ɝɨɞ ɲɜɢɞɲɟ, ɧɿɠ ɡɚ ɞɨɩɨɦɨɝɨɸ ɜɿɞ- ɩɨɜɿɞɧɨ ɤɪɚɧɿɜ ʋ 2, ʋ 3 ɿ ʋ 4. ɇɚ ɹɤɨɦɭ ɩɪɢɱɚɥɿ ɡɚ ɞɨɩɨɦɨɝɨɸ ɨɛɨɯ ɣɨ- ɝɨ ɤɪɚɧɿɜ ɦɨɠɧɚ ɲɜɢɞɲɟ ɪɨɡɜɚɧɬɚɠɢɬɢ ɛɚɪɠɭ? 168. Ɋɨɡɜ’ɹɠɿɬɶ ɪɿɜɧɹɧɧɹ: ɚ) 3(ɯ + 4) = 4(ɯ + 3); ɛ) 2ɯ(ɯ – 1) + ɯ(ɯ – 2) = 3ɯ2 – 2. 169. Ɏɭɧɤɰɿɹ ɡɚɞɚɧɚ ɮɨɪɦɭɥɨɸ ɭ = 5ɯ – 8. ɚ) Ɂɧɚɣɞɿɬɶ ɡɧɚɱɟɧɧɹ ɮɭɧɤɰɿʀ, ɹɤɳɨ ɯ = –1; ɯ = 3. ɛ) Ɂɧɚɣɞɿɬɶ ɡɧɚɱɟɧɧɹ ɚɪɝɭɦɟɧɬɭ, ɹɤɳɨ ɭ = –3; ɭ = 6. 170. Ɏɭɬɛɨɥɤɚ ɤɨɲɬɭɽ n ɝɪɧ. əɤɳɨ ɤɭɩɭɜɚɬɢ ɞɜɿ ɮɭɬɛɨɥɤɢ, ɬɨ ɦɚɝɚɡɢɧ ɧɚ ɞɪɭɝɭ ɮɭɬɛɨɥɤɭ ɞɚɽ ɡɧɢɠɤɭ 30%. ɋɤɿɥɶɤɢ ɝɪɢɜɟɧɶ ɞɨɜɟɞɟɬɶɫɹ ɡɚɩɥɚɬɢɬɢ, ɹɤɳɨ ɤɭɩɭɜɚɬɢ ɞɜɿ ɮɭɬɛɨɥɤɢ? 171. ȯ ɫɬɚɥɶ ɞɜɨɯ ɫɨɪɬɿɜ ɡ ɭɦɿɫɬɨɦ ɧɿɤɟɥɸ 10% ɿ 40%. ɋɤɿɥɶɤɢ ɬɨɧɧ ɫɬɚɥɿ ɤɨɠɧɨɝɨ ɫɨɪɬɭ ɩɨɬɪɿɛɧɨ ɜɡɹɬɢ, ɳɨɛ ɩɿɫɥɹ ɩɟɪɟɩɥɚɜɤɢ ɨɞɟɪɠɚɬɢ 12 ɬ ɫɬɚ- ɥɿ, ɹɤɚ ɦɿɫɬɢɥɚ ɛ 30% ɧɿɤɟɥɸ?
  • 49. 48 § 1. Ɋɚɰɿɨɧɚɥɶɧɿ ɜɢɪɚɡɢ 172. ɍ ɤɥɿɬɢɧɤɚɯ ɬɚɛɥɢɰɿ ɪɨɡɦɿɪɭ 3 × 3 ɡɚɩɢɫɚɧɨ ɰɿɥɿ ɱɢɫɥɚ ɬɚɤ, ɳɨ ɛɭɞɶ-ɹɤɿ ɞɜɚ ɱɢɫɥɚ, ɡɚɩɢɫɚɧɿ ɭ ɫɭɫɿɞɧɿɯ ɩɨ ɫɬɨɪɨɧɿ ɤɥɿɬɢɧɤɚɯ, ɜɿɞɪɿɡɧɹɸɬɶɫɹ ɧɟ ɛɿɥɶɲɟ ɧɿɠ ɧɚ 1. Ⱦɨɜɟɞɿɬɶ, ɳɨ ɿɫɧɭɸɬɶ ɞɜɿ ɤɥɿɬɢɧɤɢ, ɭ ɹɤɢɯ ɡɚɩɢɫɚɧɨ ɬɟ ɫɚɦɟ ɱɢɫɥɨ. ɍ ɤɭɪɫɿ ɚɥɝɟɛɪɢ ɧɚɦ ɭɠɟ ɬɪɚɩɥɹɥɨɫɹ ɱɢɦɚɥɨ ɡɚɜɞɚɧɶ, ɞɥɹ ɪɨɡɜ’ɹɡɚɧɧɹ ɹɤɢɯ ɩɨɬɪɿɛɧɨ ɛɭɥɨ ɩɟɪɟɬɜɨɪɸɜɚɬɢ ɬɨɣ ɱɢ ɿɧɲɢɣ ɜɢɪɚɡ. Ɂɨɤɪɟɦɚ, ɩɟɪɟɬɜɨɪɟɧ- ɧɹ ɰɿɥɢɯ ɪɚɰɿɨɧɚɥɶɧɢɯ ɜɢɪɚɡɿɜ ɦɢ ɜɢɤɨɪɢɫɬɨɜɭɜɚɥɢ ɞɥɹ ɪɨɡɜ’ɹɡɭɜɚɧɧɹ ɪɿɜɧɹɧɶ, ɞɨɜɟɞɟɧɧɹ ɬɨɬɨɠɧɨɫɬɟɣ, ɡɧɚɯɨɞɠɟɧɧɹ ɡɧɚɱɟɧɶ ɜɢɪɚɡɿɜ. Ɋɨɡɝɥɹɧɟɦɨ ɞɟɹɤɿ ɡɚ- ɞɚɱɿ, ɩɨɜ’ɹɡɚɧɿ ɡ ɬɨɬɨɠɧɢɦɢ ɩɟɪɟɬɜɨɪɟɧɧɹɦɢ ɞɪɨɛɨɜɢɯ ɪɚɰɿɨɧɚɥɶɧɢɯ ɜɢɪɚɡɿɜ. ɉɪɢɤɥɚɞ 1. ɋɩɪɨɫɬɢɬɢ ɜɢɪɚɡ ( ) 2 2 31 : 1 . 1 1 a a a a § · + −¨ ¸+ −© ¹ • ɋɩɨɱɚɬɤɭ ɩɨɞɚɦɨ ɜɢɪɚɡɢ ɜ ɤɨɠɧɿɣ ɞɭɠɰɿ ɭ ɜɢɝɥɹɞɿ ɞɪɨɛɿɜ, ɚ ɩɨɬɿɦ ɡɧɚɣɞɟɦɨ ʀɯ ɱɚɫɬɤɭ: 1) 1 1 a a + + = 1 1 a a a + + + = 2 1; 1 a a + + 2) 2 2 31 1 a a − − = 2 2 2 1 3 1 a a a − − − = 2 2 1 4 ; 1 a a − − 3) 2 2 2 1 1 4: 1 1 a a a a + − + − = 2 2 2 1 1 1 1 4 a a a a + −⋅ + − = (2 1) (1 )(1 ) ( 1) (1 2 )(1 2 ) a a a a a a + ⋅ − + + ⋅ − + = 1 . 1 2 − − a a ɉɪɨɜɟɞɟɧɿ ɩɟɪɟɬɜɨɪɟɧɧɹ ɦɨɠɧɚ ɡɚɩɢɫɭɜɚɬɢ ɜ ɪɹɞɨɤ: ( ) 2 2 31 : 1 1 1 a a a a § · + −¨ ¸+ −© ¹ = 2 2 2 1 1 3: 1 1 a a a a a a + + − − + − = 2 2 2 1 1 4: 1 1 a a a a + − + − = = 2 2 2 1 1 1 1 4 a a a a + −⋅ + − = (2 1) (1 )(1 ) ( 1) (1 2 )(1 2 ) a a a a a a + ⋅ − + + ⋅ − + = 1 . 1 2 − − a a •
  • 50. 7. Ɍɨɬɨɠɧɿ ɩɟɪɟɬɜɨɪɟɧɧɹ ɪɚɰɿɨɧɚɥɶɧɢɯ ɜɢɪɚɡɿɜ 49 Ɋɚɰɿɨɧɚɥɶɧɢɣ ɜɢɪɚɡ ɭ ɩɪɢɤɥɚɞɿ 1 ɦɢ ɡɜɟɥɢ ɞɨ ɪɚɰɿɨɧɚɥɶɧɨɝɨ ɞɪɨɛɭ 1 . 1 2 − − a a ȼɡɚɝɚɥɿ, ɛɭɞɶ-ɹɤɢɣ ɪɚɰɿɨɧɚɥɶɧɢɣ ɜɢɪɚɡ ɦɨɠɧɚ ɩɨɞɚɬɢ ɭ ɜɢɝɥɹɞɿ ɪɚɰɿɨ- ɧɚɥɶɧɨɝɨ ɞɪɨɛɭ. ɉɪɢɤɥɚɞ 2. Ⱦɨɜɟɫɬɢ, ɳɨ ɞɥɹ ɜɫɿɯ ɞɨɩɭɫɬɢɦɢɯ ɡɧɚɱɟɧɶ ɡɦɿɧɧɢɯ ɜɢɪɚɡ 2 2 2 2 2 3 : 2 xy y x y x y x y x yx y − + + + + +− ɧɚɛɭɜɚɽ ɬɨɝɨ ɫɚɦɨɝɨ ɡɧɚɱɟɧɧɹ. • ɋɩɪɨɫɬɢɦɨ ɞɚɧɢɣ ɜɢɪɚɡ: 2 2 2 2 2 3 : 2 xy y x y x y x y x yx y − + + + + +− = ( ) 2 3 ( )( ) 2 2 y x y x y x y x y x y x y x y − + + ⋅ + − + + + = = 2 3 2 2 y x y x y x y + + + + = 2 4 2 x y x y + + = 2( 2 ) 2 x y x y + + = 2. Ɉɬɠɟ, ɞɥɹ ɜɫɿɯ ɞɨɩɭɫɬɢɦɢɯ ɡɧɚɱɟɧɶ ɡɦɿɧɧɢɯ ɡɧɚɱɟɧɧɹ ɜɢɪɚɡɭ ɞɨɪɿɜɧɸɽ ɬɨɦɭ ɫɚɦɨɦɭ ɱɢɫɥɭ (ɱɢɫɥɭ 2). • ɉɪɢɤɥɚɞ 3. Ⱦɨɜɟɫɬɢ ɬɨɬɨɠɧɿɫɬɶ 1 1 . 1 1 b aa b b a a b + += −− • ɋɩɪɨɫɬɢɦɨ ɥɿɜɭ ɱɚɫɬɢɧɭ ɪɿɜɧɨɫɬɿ: 1 1 1 1 a b a b + − = ( ) ( )1 1 1 1: a b a b + − = :b a b a ab ab + − = b a ab ab b a + ⋅ − = .b a b a + − ɒɥɹɯɨɦ ɬɨɬɨɠɧɢɯ ɩɟɪɟɬɜɨɪɟɧɶ ɥɿɜɭ ɱɚɫɬɢɧɭ ɪɿɜɧɨɫɬɿ ɡɜɟɥɢ ɞɨ ɩɪɚɜɨʀ ɱɚɫɬɢɧɢ. Ɍɨɦɭ ɰɹ ɪɿɜɧɿɫɬɶ ɽ ɬɨɬɨɠɧɿɫɬɸ. • ɋɩɪɨɫɬɿɬɶ ɜɢɪɚɡ: 173. ɚ) ( ) 2 1 11 : ; 3 −+ a a ɛ) ( )1 1 5: ; 5 5 5a a a − + − +
  • 51. 50 § 1. Ɋɚɰɿɨɧɚɥɶɧɿ ɜɢɪɚɡɢ ɜ) 2 2 49 1 1 ; 7 a a aa − ⋅ − + ɝ) ( )2 1 6: ; 2 2 1 2 b b b b − − − − ɞ) 4 3 2 : ; 2 8 48 16 a a a a aa a − − −− + ɟ) . x yx x x y x y xy +§ · − ⋅¨ ¸− +© ¹ 174. ɚ) ( ) 2 1 41 ; 2 1b b b − ⋅ − + ɛ) 2 5 100 1 ; 5 10 a a a a a − −− ⋅ − − ɜ) 2 2 2 9 3 ; 3 2 4 x x x x x + ⋅ − + − ɝ) ( )4 2 8: . 2 1 2 c c c c − + + + 175. ɋɩɪɨɫɬɿɬɶ ɜɢɪɚɡ 2 2 12 6 2 1 3 x x x x x + ⋅ − + + ɿ ɡɧɚɣɞɿɬɶ ɣɨɝɨ ɡɧɚɱɟɧɧɹ, ɹɤɳɨ ɯ = 6. 176. ɋɩɪɨɫɬɿɬɶ ɜɢɪɚɡ ( ) 9: 9 9 9 c c c c c c − − + + ɿ ɡɧɚɣɞɿɬɶ ɣɨɝɨ ɡɧɚɱɟɧɧɹ, ɹɤɳɨ ɫ = 11. Ⱦɨɜɟɞɿɬɶ ɬɨɬɨɠɧɿɫɬɶ: 177. ɚ) 2 : 1; 9 81 918 81 a a a a aa a + = + ++ + ɛ) 6 4 2 2 2 1: 1. 3 3 12 1 b b b b b bb b +− = + − −− + 178. ɚ) 4 1 1: 2 2 ;x x y x y x y x y § · + = −¨ ¸+ − +© ¹ ɛ) 2 2 5 10 3 15 5. 2 a a a a aa + +⋅ − = + ɋɩɪɨɫɬɿɬɶ ɜɢɪɚɡ: 179. ɚ) 3 2 2 2 2 2 2 ; ( ) § ·−− ⋅ −¨ ¸− + − −© ¹ m m mn m n n m m n m n m n ɛ) 2 2 2 3 3 ; 3 2 3 3 9 a a a a a a a a a a + +§ ·+ ⋅ −¨ ¸− + − −© ¹
  • 52. 7. Ɍɨɬɨɠɧɿ ɩɟɪɟɬɜɨɪɟɧɧɹ ɪɚɰɿɨɧɚɥɶɧɢɯ ɜɢɪɚɡɿɜ 51 ɜ) 2 2 2 2 2 2 4 2 1: ; 2 24 4 4 a a a a b b aa ab b a b § · § ·− +¨ ¸¨ ¸+ −+ + −© ¹© ¹ ɝ) ( ) 2 2 2 ( )1 1 2 1 1 : ; a b a b a b aba b +§ ·+ + ⋅ +¨ ¸+© ¹ ɞ) ( ) 2 3 2 2 1 1 51 . 11 1 § ·+ + + +− ⋅ + −¨ ¸ +− + +© ¹ x x x x x x xx x x 180. ɚ) 22 2 2 2 2 : 1 ; xy yx x x yx y x y − § · − −¨ ¸++ − © ¹ ɛ) 2 2 2 2 2 3 3 1 3 2: ; 2 4 4 4 4 x x x x x x x x + + +§ ·+ +¨ ¸+ − − + −© ¹ ɜ) 2 3 3 2 6 13 15 5: ; 2 4 8 2 16 x x x x x x x − − −§ ·−¨ ¸ − + + +© ¹ ɝ) 4 2 3 2 2 49 3 27 40 . 427 3 9 16 a a a a aa a a a + + −§ ·− ⋅ +¨ ¸ ++ − + −© ¹ 181. ɚ) 1 ; 11 a b ab + + ɛ) 96 ; 31 x x x − + − ɜ) 1 1 ; 1 1 m n m n m n m n + + − − + − ɝ) 2 2 . 1 m a ma m a a m + + − 182. ɚ) ; 1 1 a b b a a b − − ɛ) 1 1 1 1. 1 1 1 1 c c c c − − + + − + 183. Ⱦɨɜɟɞɿɬɶ, ɳɨ ɞɥɹ ɜɫɿɯ ɞɨɩɭɫɬɢɦɢɯ ɡɧɚɱɟɧɶ ɡɦɿɧɧɢɯ ɡɧɚɱɟɧɧɹ ɜɢɪɚɡɭ a b a a b a a b b a b b + ++ − ⋅ − − ɞɨɪɿɜɧɸɽ ɬɨɦɭ ɫɚɦɨɦɭ ɱɢɫɥɭ. 184. Ⱦɨɜɟɞɿɬɶ, ɳɨ ɡɧɚɱɟɧɧɹ ɜɢɪɚɡɭ 2 2 2 2 2 2 :m n m nm n mn m n m n § · § · − −¨ ¸ ¨ ¸+ +© ¹ © ¹ ɽ ɞɨɞɚɬɧɢɦ ɱɢɫɥɨɦ ɞɥɹ ɛɭɞɶ-ɹɤɢɯ ɞɨɩɭɫɬɢɦɢɯ ɡɧɚɱɟɧɶ ɡɦɿɧɧɢɯ.
  • 53. 52 § 1. Ɋɚɰɿɨɧɚɥɶɧɿ ɜɢɪɚɡɢ 185. Ⱦɨɜɟɞɿɬɶ, ɳɨ ɞɥɹ ɜɫɿɯ ɞɨɩɭɫɬɢɦɢɯ ɡɧɚɱɟɧɶ ɡɦɿɧɧɢɯ ɡɧɚɱɟɧɧɹ ɜɢɪɚɡɭ ( ) 2 2 2 2 1 1 4: 1 a b a b a b a b § ·−+ −¨ ¸− + −© ¹ ɧɟ ɡɚɥɟɠɚɬɶ ɜɿɞ ɡɧɚɱɟɧɶ b. Ⱦɨɜɟɞɿɬɶ ɬɨɬɨɠɧɿɫɬɶ: 186. ɚ) 2 2 2 2 1 · : ;m n m n m n m mn m n n m m m n § ·§ ·− −− − =¨ ¸¨ ¸+ +© ¹© ¹ ɛ) 2 22 2 2 2 2 ( 2)2 1 3 1· ; 3 4 4 a aa a a a a a a a a § ·+ −− + −− =¨ ¸ − − −© ¹ ɜ) ( )2 2 2 1: 2 .b a b a a b a b a ba ab b ab § ·− + − + =¨ ¸+ ++ +© ¹ 187. ɚ) ( )3 2 1 3 3 2 1 1; 1 11 1 aa a aa a a −§ ·− + ⋅ − =¨ ¸+ ++ − +© ¹ ɛ) 2 2 2 2 : . 2 a a a a a n a n a n a na n an a n −§ · § ·− − =¨ ¸ ¨ ¸+ − ++ + −© ¹ © ¹ 188. ɋɩɪɨɫɬɿɬɶ ɜɢɪɚɡ: ɚ) 2 2 3 1 3 3: : ; 9 3 39 3 9 27 b a a a b a aba a a b −§ ·−¨ ¸− − +− + −© ¹ ɛ) 2 2 2 10 3 211 2 . 53 ( 5 ) y xy y x yx xy x y § · § ·− + ⋅ ⋅ +¨ ¸ ¨ ¸−− −© ¹ © ¹ 189. Ɂɧɚɣɞɿɬɶ ɡɧɚɱɟɧɧɹ ɜɢɪɚɡɭ: ɚ) 2 2 1 ,x x + ɹɤɳɨ 1 2,5;x x + = ɛ) 2 2 1 , 4 x x + ɹɤɳɨ 1 0,5. 2 x x − = − 190. Ⱦɨɜɟɞɿɬɶ ɬɨɬɨɠɧɿɫɬɶ: ɚ) 2 2 3 2 ( 3)3 4 1 1 ; 1 12 1 1 3 3 2 1 x xx x xx x x x x x x +§ ·−§ ·+ ⋅ + =¨ ¸ ¨ ¸+ −+ + − + − − +© ¹ © ¹
  • 54. 7. Ɍɨɬɨɠɧɿ ɩɟɪɟɬɜɨɪɟɧɧɹ ɪɚɰɿɨɧɚɥɶɧɢɯ ɜɢɪɚɡɿɜ 53 ɛ) ( ) 22 2 3 2 2 1 3 21 . 2 312 4 3 6 5 a a a a a a aa a a a a + − − −− = ⋅ − − −− − + − + 191. ɉɨɞɚɣɬɟ ɭ ɜɢɝɥɹɞɿ ɪɚɰɿɨɧɚɥɶɧɨɝɨ ɞɪɨɛɭ: 1 . 11 11 1 x + − − 192. Ɋɨɡɜ’ɹɠɿɬɶ ɪɿɜɧɹɧɧɹ: ɚ) (ɯ – 1)(ɯ2 + ɯ + 1) – ɯ3 – ɯ2 = 2ɯ; ɛ) (ɯ + 2)2 – 4 = 0; ɜ) 4 1; 2 3 x x +− = ɝ) 3 3 6. 5 4 − + + = y y 193. Ⱦɥɹ ɹɤɢɯ ɡɧɚɱɟɧɶ ɚ ɪɿɜɧɹɧɧɹ (ɚ2 – 3ɚ)ɯ = 2ɚ – 6 ɧɟ ɦɚɽ ɤɨɪɟɧɿɜ? ɦɚɽ ɨɞɢɧ ɤɨɪɿɧɶ? 194. Ɂ ɩɭɧɤɬɭ A ɞɨ ɩɭɧɤɬɭ B, ɜɿɞɫɬɚɧɶ ɦɿɠ ɹɤɢɦɢ ɞɨɪɿɜɧɸɽ 360 ɤɦ, ɜɢɣɲɨɜ ɬɨɜɚɪɧɢɣ ɩɨʀɡɞ ɿ ɪɭɯɚɜɫɹ ɡɿ ɲɜɢɞɤɿɫɬɸ 50 ɤɦ/ɝɨɞ. ɑɟɪɟɡ 40 ɯɜ ɧɚɡɭɫɬɪɿɱ ɣɨɦɭ ɡ ɩɭɧɤɬɭ B ɜɢɣɲɨɜ ɩɚɫɚɠɢɪɫɶɤɢɣ ɩɨʀɡɞ ɿ ɪɭɯɚɜɫɹ ɡɿ ɲɜɢɞɤɿɫɬɸ 90 ɤɦ/ɝɨɞ. ɇɚ ɹɤɿɣ ɜɿɞɫɬɚɧɿ ɜɿɞ ɩɭɧɤɬɭ A ɩɨʀɡɞɢ ɡɭɫɬɪɿɥɢɫɹ? 195*. ɉɨ ɤɪɭɝɨɜɿɣ ɞɨɪɿɠɰɿ ɜɟɥɨɬɪɟɤɭ ʀɞɭɬɶ ɞɜɚ ɜɟɥɨɫɢɩɟɞɢɫɬɢ ɡɿ ɫɬɚɥɢɦɢ ɲɜɢɞɤɨɫɬɹɦɢ. Ʉɨɥɢ ɜɨɧɢ ʀɞɭɬɶ ɭ ɩɪɨɬɢɥɟɠɧɢɯ ɧɚɩɪɹɦɤɚɯ, ɬɨ ɡɭɫɬɪɿɱɚ- ɸɬɶɫɹ ɱɟɪɟɡ ɤɨɠɧɿ 10 ɫ; ɤɨɥɢ ɠ ʀɞɭɬɶ ɜ ɨɞɧɨɦɭ ɧɚɩɪɹɦɤɭ, ɬɨ ɨɞɢɧ ɧɚɡɞɨ- ɝɚɧɹɽ ɿɧɲɨɝɨ ɱɟɪɟɡ ɤɨɠɧɿ 100 ɫ. əɤɚ ɲɜɢɞɤɿɫɬɶ ɤɨɠɧɨɝɨ ɜɟɥɨɫɢɩɟɞɢɫɬɚ, ɹɤɳɨ ɞɨɜɠɢɧɚ ɞɨɪɿɠɤɢ ɞɨɪɿɜɧɸɽ 200 ɦ? 196. ɇɚ ɞɨɲɰɿ ɧɚɩɢɫɚɧɿ ɱɢɫɥɚ 1, 2, 3, …, 25. Ⱦɨɡɜɨɥɹɽɬɶɫɹ ɫɬɟɪɬɢ ɛɭɞɶ-ɹɤɿ ɞɜɚ ɱɢɫɥɚ ɿ ɧɚɩɢɫɚɬɢ ʀɯ ɞɨɛɭɬɨɤ. ɉɨɜɬɨɪɢɜɲɢ ɬɚɤɭ ɨɩɟɪɚɰɿɸ 24 ɪɚɡɢ, ɨɞɟɪ- ɠɢɦɨ ɨɞɧɟ ɱɢɫɥɨ. Ⱦɨɜɟɞɿɬɶ, ɳɨ ɰɟ ɱɢɫɥɨ ɞɿɥɢɬɶɫɹ ɧɚ 1 000 000.
  • 55. 54 § 1. Ɋɚɰɿɨɧɚɥɶɧɿ ɜɢɪɚɡɢ 1. ɐɿɥɿ ɬɚ ɞɪɨɛɨɜɿ ɪɚɰɿɨɧɚɥɶɧɿ ɪɿɜɧɹɧɧɹ. Ɋɨɡɝɥɹɧɟɦɨ ɪɿɜɧɹɧɧɹ: 2(ɯ – 7) = 3ɯ – 9; 6 4; 9 x x = − 5 3 1 4 . x x = − − Ʌɿɜɚ ɿ ɩɪɚɜɚ ɱɚɫɬɢɧɢ ɤɨɠɧɨɝɨ ɡ ɰɢɯ ɪɿɜɧɹɧɶ ɽ ɪɚɰɿɨɧɚɥɶɧɢɦɢ ɜɢɪɚɡɚɦɢ. Ɍɚɤɿ ɪɿɜɧɹɧɧɹ ɧɚɡɢɜɚɸɬɶ ɪɚɰɿɨɧɚɥɶɧɢɦɢ ɪɿɜɧɹɧɧɹɦɢ. Ɉɡɧɚɱɟɧɧɹ Ɋɿɜɧɹɧɧɹ, ɥɿɜɚ ɿ ɩɪɚɜɚ ɱɚɫɬɢɧɢ ɹɤɢɯ ɽ ɪɚɰɿɨɧɚɥɶɧɢɦɢ ɜɢ- ɪɚɡɚɦɢ, ɧɚɡɢɜɚɸɬɶ ɪɚɰɿɨɧɚɥɶɧɢɦɢ ɪɿɜɧɹɧɧɹɦɢ. Ɋɚɰɿɨɧɚɥɶɧɿ ɪɿɜɧɹɧɧɹ ɩɨɞɿɥɹɸɬɶ ɧɚ ɰɿɥɿ ɣ ɞɪɨɛɨɜɿ. əɤɳɨ ɨɛɢɞɜɿ ɱɚɫɬɢɧɢ ɪɚɰɿɨɧɚɥɶɧɨɝɨ ɪɿɜɧɹɧɧɹ ɽ ɰɿɥɢɦɢ ɜɢɪɚɡɚɦɢ, ɬɨ ɬɚɤɟ ɪɿɜɧɹɧɧɹ ɧɚɡɢɜɚɸɬɶ ɰɿɥɢɦ ɪɚɰɿɨɧɚɥɶɧɢɦ ɪɿɜɧɹɧɧɹɦ. Ɋɚɰɿɨɧɚɥɶɧɟ ɪɿɜɧɹɧɧɹ, ɭ ɹɤɨɝɨ ɯɨɱɚ ɛ ɨɞɧɚ ɱɚɫɬɢɧɚ ɽ ɞɪɨɛɨɜɢɦ ɜɢɪɚɡɨɦ, ɧɚɡɢɜɚɸɬɶ ɞɪɨɛɨɜɢɦ ɪɚɰɿɨɧɚɥɶɧɢɦ ɪɿɜɧɹɧɧɹɦ. 2(ɯ – 7) = 3ɯ – 9 — ɰɿɥɟ ɪɚɰɿɨɧɚɥɶɧɟ ɪɿɜɧɹɧɧɹ; 3( 2) 2( 1) 5 3 y y− + = — ɰɿɥɟ ɪɚɰɿɨɧɚɥɶɧɟ ɪɿɜɧɹɧɧɹ; 6 4 9 x x = − — ɞɪɨɛɨɜɟ ɪɚɰɿɨɧɚɥɶɧɟ ɪɿɜɧɹɧɧɹ; 5 3 1 4x x = − − — ɞɪɨɛɨɜɟ ɪɚɰɿɨɧɚɥɶɧɟ ɪɿɜɧɹɧɧɹ. 2. Ɋɨɡɜ’ɹɡɭɜɚɧɧɹ ɞɪɨɛɨɜɢɯ ɪɚɰɿɨɧɚɥɶɧɢɯ ɪɿɜɧɹɧɶ ɧɚ ɨɫɧɨɜɿ ɭɦɨɜɢ ɪɿɜɧɨɫɬɿ ɞɪɨɛɭ ɧɭɥɸ. ɉɪɢɝɚɞɚɣɦɨ: ɞɪɿɛ ɞɨɪɿɜɧɸɽ ɧɭɥɸ ɬɨɞɿ ɣ ɬɿɥɶɤɢ ɬɨɞɿ, ɤɨɥɢ ɣɨɝɨ ɱɢɫɟɥɶɧɢɤ ɞɨɪɿɜɧɸɽ ɧɭɥɸ, ɚ ɡɧɚɦɟɧɧɢɤ ɜɿɞɦɿɧɧɢɣ ɜɿɞ ɧɭɥɹ. 0=a b ɬɨɞɿ ɣ ɬɿɥɶɤɢ ɬɨɞɿ, ɤɨɥɢ ɚ = 0 ɿ b ≠ 0. Ⱦɚɧɟ ɬɜɟɪɞɠɟɧɧɹ ɦɨɠɧɚ ɜɢɤɨɪɢɫɬɨɜɭɜɚɬɢ ɞɥɹ ɪɨɡɜ’ɹɡɭɜɚɧɧɹ ɞɪɨɛɨɜɢɯ ɪɚɰɿɨɧɚɥɶɧɢɯ ɪɿɜɧɹɧɶ. Ɋɨɡɝɥɹɧɟɦɨ ɩɪɢɤɥɚɞɢ. ɉɪɢɤɥɚɞ 1. Ɋɨɡɜ’ɹɡɚɬɢ ɪɿɜɧɹɧɧɹ 2 2 0. 2 x x x − = − • ȼɢɤɨɪɢɫɬɚɽɦɨ ɭɦɨɜɭ, ɡɚ ɹɤɨʀ ɞɪɿɛ ɞɨɪɿɜɧɸɽ ɧɭɥɸ. ɉɪɢɪɿɜɧɹɽɦɨ ɱɢ- ɫɟɥɶɧɢɤ ɞɪɨɛɭ ɞɨ ɧɭɥɹ: ɯ2 – 2ɯ = 0; ɯ(ɯ – 2) = 0; ɯ = 0 ɚɛɨ ɯ = 2.
  • 56. 8. Ɋɚɰɿɨɧɚɥɶɧɿ ɪɿɜɧɹɧɧɹ 55 ɉɟɪɟɜɿɪɢɦɨ, ɱɢ ɞɥɹ ɡɧɚɣɞɟɧɢɯ ɡɧɚɱɟɧɶ ɯ ɡɧɚɦɟɧɧɢɤ ɯ – 2 ɜɿɞɦɿɧɧɢɣ ɜɿɞ ɧɭɥɹ. əɤɳɨ ɯ = 0, ɬɨ ɯ – 2 = 0 – 2 = –2 ≠ 0. Ɍɨɦɭ ɯ = 0 — ɤɨɪɿɧɶ ɪɿɜɧɹɧɧɹ. əɤɳɨ ɯ = 2, ɬɨ ɯ – 2 = 2 – 2 = 0. Ɍɨɦɭ ɯ = 2 — ɧɟ ɽ ɤɨɪɟɧɟɦ ɪɿɜɧɹɧɧɹ. ȼɿɞɩɨɜɿɞɶ. 0. • ɉɪɢɤɥɚɞ 2. Ɋɨɡɜ’ɹɡɚɬɢ ɪɿɜɧɹɧɧɹ 6 4 9 = − x . x • Ɂɜɟɞɟɦɨ ɞɚɧɟ ɪɿɜɧɹɧɧɹ ɞɨ ɪɿɜɧɹɧɧɹ, ɥɿɜɚ ɱɚɫɬɢɧɚ ɹɤɨɝɨ ɽ ɞɪɨɛɨɦ, ɚ ɩɪɚɜɚ — ɧɭɥɟɦ: 6 4; 9 x x = − 6 4 0; 9 x x − = − 6 4( 9) 0; 9 x x x − − = − 2 36 0 9 x . x + = − ɉɪɢɪɿɜɧɹɽɦɨ ɱɢɫɟɥɶɧɢɤ ɞɪɨɛɭ 2 36 9 x x + − ɞɨ ɧɭɥɹ: 2ɯ + 36 = 0; 2ɯ = –36; ɯ = –18. əɤɳɨ ɯ = –18, ɬɨ ɡɧɚɦɟɧɧɢɤ ɯ – 9 ɞɪɨɛɭ ɜɿɞɦɿɧɧɢɣ ɜɿɞ ɧɭɥɹ. ɋɩɪɚɜɞɿ: ɯ – 9 = –18 – 9 = –27 ≠ 0. Ɉɬɠɟ, ɯ = –18 — ɤɨɪɿɧɶ ɞɚɧɨɝɨ ɪɿɜɧɹɧɧɹ. ȼɿɞɩɨɜɿɞɶ. –18. • ɓɨɛ ɪɨɡɜ’ɹɡɚɬɢ ɞɪɨɛɨɜɟ ɪɚɰɿɨɧɚɥɶɧɟ ɪɿɜɧɹɧɧɹ ɧɚ ɨɫɧɨɜɿ ɭɦɨɜɢ ɪɿɜɧɨɫɬɿ ɞɪɨɛɭ ɧɭɥɸ, ɩɨɬɪɿɛɧɨ: 1) ɡɜɟɫɬɢ ɣɨɝɨ ɞɨ ɜɢɝɥɹɞɭ ( ) 0, ( ) f x g x = ɞɟ f(x) ɿ g(x) — ɰɿɥɿ ɪɚɰɿɨɧɚɥɶɧɿ ɜɢɪɚɡɢ; 2) ɩɪɢɪɿɜɧɹɬɢ ɞɨ ɧɭɥɹ ɱɢɫɟɥɶɧɢɤ ɞɪɨɛɭ ɣ ɪɨɡɜ’ɹɡɚɬɢ ɨɞɟɪɠɚɧɟ ɰɿɥɟ ɪɚ- ɰɿɨɧɚɥɶɧɟ ɪɿɜɧɹɧɧɹ f(x) = 0; 3) ɜɢɤɥɸɱɢɬɢ ɡ ɣɨɝɨ ɤɨɪɟɧɿɜ ɬɿ, ɞɥɹ ɹɤɢɯ ɡɧɚɦɟɧɧɢɤ ɞɪɨɛɭ ɞɨɪɿɜɧɸɽ ɧɭɥɸ. 3. Ɋɿɜɧɨɫɢɥɶɧɿɫɬɶ ɪɿɜɧɹɧɶ. Ɋɨɡɜ’ɹɡɭɸɱɢ ɩɪɢɤɥɚɞ 1, ɦɢ ɦɚɥɢ ɥɚɧɰɸɠɨɤ ɪɿɜɧɹɧɶ Ɋɿɜɧɹɧɧɹ 2 2 0 2 x x x − = − ɯ2 – 2ɯ = 0 ɯ(ɯ – 2) = 0 Ʉɨɪɟɧɿ 0 0; 2 0; 2 ɉɟɪɲɟ ɡ ɰɢɯ ɪɿɜɧɹɧɶ ɦɚɽ ɨɞɢɧ ɤɨɪɿɧɶ — ɱɢɫɥɨ 0, ɞɪɭɝɟ ɬɚ ɬɪɟɬɽ ɪɿɜɧɹɧ- ɧɹ ɦɚɸɬɶ ɞɜɚ ɬɿ ɫɚɦɿ ɤɨɪɟɧɿ — ɱɢɫɥɚ 0 ɿ 2.
  • 57. 56 § 1. Ɋɚɰɿɨɧɚɥɶɧɿ ɜɢɪɚɡɢ Ɉɡɧɚɱɟɧɧɹ Ⱦɜɚ ɪɿɜɧɹɧɧɹ, ɹɤɿ ɦɚɸɬɶ ɬɿ ɫɚɦɿ ɤɨɪɟɧɿ, ɧɚɡɢɜɚɸɬɶ ɪɿɜɧɨɫɢ- ɥɶɧɢɦɢ. Ⱦɜɚ ɪɿɜɧɹɧɧɹ, ɹɤɿ ɧɟ ɦɚɸɬɶ ɤɨɪɟɧɿɜ, ɬɟɠ ɜɜɚɠɚɸɬɶ ɪɿɜɧɨɫɢɥɶɧɢɦɢ. Ɉɬɠɟ, ɪɿɜɧɹɧɧɹ ɯ2 – 2ɯ = 0 ɿ ɯ(ɯ – 2) = 0 ɪɿɜɧɨɫɢɥɶɧɿ; ɪɿɜɧɹɧɧɹ 2 2 0 2 x x x − = − ɿ ɯ2 – 2ɯ = 0 ɧɟ ɪɿɜɧɨɫɢɥɶɧɿ. Ɋɿɜɧɹɧɧɹ ɯ + 6 = ɯ ɿ 0ɯ = 1 ɪɿɜɧɨɫɢɥɶɧɿ, ɛɨ ɤɨɠɧɟ ɡ ɧɢɯ ɧɟ ɦɚɽ ɤɨɪɟɧɿɜ. Ɉɫɤɿɥɶɤɢ ɪɨɡɜ’ɹɡɭɜɚɧɧɹ ɪɿɜɧɹɧɧɹ 2 2 0 2 x x x − = − ɡɜɨɞɢɬɶɫɹ ɞɨ ɪɨɡɜ’ɹɡɭɜɚɧ- ɧɹ ɪɿɜɧɹɧɧɹ ɯ2 – 2ɯ = 0 ɿ ɩɟɪɟɜɿɪɤɢ ɭɦɨɜɢ ɯ – 2 ≠ 0, ɬɨ ɤɚɠɭɬɶ, ɳɨ ɪɿɜɧɹɧɧɹ 2 2 0 2 x x x − = − ɪɿɜɧɨɫɢɥɶɧɟ ɫɢɫɬɟɦɿ 2 2 0; 2 0. x x x ­ − = ® − ≠¯ Ɋɨɡɜ’ɹɡɤɨɦ ɰɿɽʀ ɫɢɫɬɟɦɢ, ɹɤ ɦɢ ɜɠɟ ɡ’ɹɫɭɜɚɥɢ, ɽ ɱɢɫɥɨ ɯ = 0. Ɋɿɜɧɹɧɧɹ ( ) 0 ( ) f x g x = ɪɿɜɧɨɫɢɥɶɧɟ ɫɢɫɬɟɦɿ ( ) 0; ( ) 0. f x g x =­ ® ≠¯ ɍ 7 ɤɥɚɫɿ ɦɢ ɪɨɡɝɥɹɞɚɥɢ ɩɟɪɟɬɜɨɪɟɧɧɹ ɪɿɜɧɹɧɶ, ɜɢɤɨɧɭɸɱɢ ɹɤɿ, ɨɞɟɪɠɭ- ɸɬɶ ɪɿɜɧɹɧɧɹ ɡ ɬɢɦɢ ɫɚɦɢɦɢ ɤɨɪɟɧɹɦɢ. Ɉɬɠɟ, ɰɿ ɩɟɪɟɬɜɨɪɟɧɧɹ ɩɟɪɟɜɨɞɹɬɶ ɪɿ- ɜɧɹɧɧɹ ɜ ɪɿɜɧɨɫɢɥɶɧɟ ɣɨɦɭ ɪɿɜɧɹɧɧɹ. Ɂ ɧɢɦɢ ɩɨɜ’ɹɡɚɧɿ ɬɚɤɿ ɨɫɧɨɜɧɿ ɜɥɚɫɬɢɜɨ- ɫɬɿ ɪɿɜɧɹɧɶ: ȼɥɚɫɬɢɜɿɫɬɶ 1. əɤɳɨ ɜ ɞɟɹɤɿɣ ɱɚɫɬɢɧɿ ɪɿɜɧɹɧɧɹ ɜɢɤɨɧɚɬɢ ɬɨɬɨɠɧɟ ɩɟ- ɪɟɬɜɨɪɟɧɧɹ, ɹɤɟ ɧɟ ɡɦɿɧɸɽ ɞɨɩɭɫɬɢɦɿ ɡɧɚɱɟɧɧɹ ɡɦɿɧɧɨʀ, ɬɨ ɨɞɟɪɠɢɦɨ ɪɿɜɧɹɧɧɹ, ɪɿɜɧɨɫɢɥɶɧɟ ɞɚɧɨɦɭ. ȼɥɚɫɬɢɜɿɫɬɶ 2. əɤɳɨ ɞɟɹɤɢɣ ɞɨɞɚɧɨɤ ɩɟɪɟɧɟɫɬɢ ɡ ɨɞɧɿɽʀ ɱɚɫɬɢɧɢ ɪɿɜ- ɧɹɧɧɹ ɜ ɿɧɲɭ, ɡɦɿɧɢɜɲɢ ɣɨɝɨ ɡɧɚɤ ɧɚ ɩɪɨɬɢɥɟɠɧɢɣ, ɬɨ ɨɞɟɪɠɢɦɨ ɪɿɜɧɹɧɧɹ, ɪɿɜɧɨɫɢɥɶɧɟ ɞɚɧɨɦɭ. ȼɥɚɫɬɢɜɿɫɬɶ 3. əɤɳɨ ɨɛɢɞɜɿ ɱɚɫɬɢɧɢ ɪɿɜɧɹɧɧɹ ɩɨɦɧɨɠɢɬɢ ɚɛɨ ɩɨɞɿɥɢ- ɬɢ ɬɟ ɫɚɦɟ, ɜɿɞɦɿɧɧɟ ɜɿɞ ɧɭɥɹ ɱɢɫɥɨ, ɬɨ ɨɞɟɪɠɢɦɨ ɪɿɜɧɹɧɧɹ, ɪɿɜɧɨɫɢɥɶɧɟ ɞɚ- ɧɨɦɭ.
  • 58. 8. Ɋɚɰɿɨɧɚɥɶɧɿ ɪɿɜɧɹɧɧɹ 57 4. Ɇɧɨɠɟɧɧɹ ɨɛɨɯ ɱɚɫɬɢɧ ɪɿɜɧɹɧɧɹ ɧɚ ɜɢɪɚɡ ɡɿ ɡɦɿɧɧɨɸ. Ɋɨɡɝɥɹɧɟɦɨ ɩɪɢɤɥɚɞ. ɉɪɢɤɥɚɞ 3. Ɋɨɡɜ’ɹɡɚɬɢ ɪɿɜɧɹɧɧɹ 2 1 1 6 . 3 3 9 y y y y + − = − + − • Ɉɫɤɿɥɶɤɢ ɭ2 – 9 = (ɭ – 3)(ɭ + 3), ɬɨ ɫɩɿɥɶɧɢɦ ɡɧɚɦɟɧɧɢɤɨɦ ɭɫɿɯ ɞɪɨɛɿɜ, ɹɤɿ ɜɯɨɞɹɬɶ ɭ ɪɿɜɧɹɧɧɹ, ɽ (ɭ – 3)(ɭ + 3). ɉɨɦɧɨɠɢɜɲɢ ɨɛɢɞɜɿ ɱɚɫɬɢɧɢ ɪɿɜɧɹɧɧɹ ɧɚ ɫɩɿɥɶɧɢɣ ɡɧɚɦɟɧɧɢɤ, ɡɚ ɭɦɨɜɢ, ɳɨ (ɭ – 3)(ɭ + 3) ≠ 0, ɦɚɬɢɦɟɦɨ: 2 1 1 6 ( 3)( 3); 3 3 9 y y y y y y + − = ⋅ − + − + − (ɭ + 3)(ɭ + 1) – (ɭ – 3) = 6; ɭ2 + ɭ + 3ɭ + 3 – ɭ + 3 – 6 = 0; ɭ2 + 3ɭ = 0; ɭ(ɭ + 3) = 0; ɭ = 0 ɚɛɨ ɭ = –3. əɤɳɨ ɭ = 0, ɬɨ (ɭ – 3)(ɭ + 3) = –3 ⋅ 3 ≠ 0. Ɍɨɦɭ ɭ = 0 — ɤɨɪɿɧɶ ɪɿɜɧɹɧɧɹ. əɤɳɨ ɭ = –3, ɬɨ (ɭ – 3)(ɭ + 3) = –6 ⋅ 0 = 0. Ɍɨɦɭ ɭ = –3 — ɧɟ ɽ ɤɨɪɟɧɟɦ ɪɿɜ- ɧɹɧɧɹ. ȼɿɞɩɨɜɿɞɶ. 0. • Ɂɜɟɪɧɟɦɨ ɭɜɚɝɭ, ɳɨ ɪɿɜɧɹɧɧɹ 2 1 1 6 3 3 9 y y y y + − = − + − ɦɚɽ ɨɞɢɧ ɤɨɪɿɧɶ ɭ = 0, ɚ ɨɞɟɪɠɚɧɟ ɭ ɪɨɡɜ’ɹɡɚɧɧɿ ɪɿɜɧɹɧɧɹ (ɭ + 3)(ɭ + 1) – (ɭ – 3) = 6 — ɞɜɚ ɤɨɪɟɧɿ ɭ = 0 ɬɚ ɭ = –3. Ɉɬɠɟ, ɩɨɦɧɨɠɢɜɲɢ ɨɛɢɞɜɿ ɱɚɫɬɢɧɢ ɞɪɨɛɨɜɨɝɨ ɪɿɜɧɹɧɧɹ ɧɚ ɫɩɿ- ɥɶɧɢɣ ɡɧɚɦɟɧɧɢɤ, ɦɢ ɧɟ ɜɬɪɚɬɢɥɢ ɣɨɝɨ ɤɨɪɿɧɶ, ɩɪɨɬɟ ɨɞɟɪɠɚɥɢ ɫɬɨɪɨɧɧɿɣ ɳɨɞɨ ɰɶɨɝɨ ɪɿɜɧɹɧɧɹ ɤɨɪɿɧɶ ɭ = –3. ɉɪɚɜɢɥɶɧɢɦ ɽ ɬɚɤɟ ɬɜɟɪɞɠɟɧɧɹ: əɤɳɨ ɨɛɢɞɜɿ ɱɚɫɬɢɧɢ ɞɟɹɤɨɝɨ ɪɿɜɧɹɧɧɹ ɩɨɦɧɨɠɢɬɢ ɧɚ ɰɿɥɢɣ ɜɢɪɚɡ ɡɿ ɡɦɿɧɧɨɸ, ɬɨ ɦɨɠɧɚ ɨɞɟɪɠɚɬɢ ɪɿɜɧɹɧɧɹ, ɧɟ ɪɿɜɧɨɫɢɥɶɧɟ ɞɚɧɨɦɭ. Ɉɞɟɪɠɚɧɟ ɪɿɜɧɹɧɧɹ ɦɚɽ ɬɚɤɿ ɜɥɚɫɬɢɜɨɫɬɿ: 1) ɣɨɝɨ ɤɨɪɟɧɹɦɢ ɽ ɜɫɿ ɤɨɪɟɧɿ ɞɚɧɨɝɨ ɪɿɜɧɹɧ- ɧɹ; 2) ɜɨɧɨ ɦɨɠɟ ɦɚɬɢ ɫɬɨɪɨɧɧɿ ɤɨɪɟɧɿ ɳɨɞɨ ɞɚɧɨɝɨ ɪɿɜɧɹɧɧɹ. ɋɬɨɪɨɧɧɿɦɢ ɤɨɪɟɧɹɦɢ ɦɨɠɭɬɶ ɛɭɬɢ ɡɧɚɱɟɧɧɹ ɡɦɿɧɧɨʀ, ɞɥɹ ɹɤɢɯ ɰɿɥɢɣ ɜɢɪɚɡ, ɧɚ ɹɤɢɣ ɦɢ ɦɧɨɠɢɦɨ ɨɛɢɞɜɿ ɱɚɫɬɢɧɢ ɪɿɜɧɹɧɧɹ, ɧɚɛɭɜɚɽ ɡɧɚɱɟɧɧɹ 0. ɐɿ ɫɬɨɪɨɧɧɿ ɤɨɪɟɧɿ ɦɨɠɧɚ ɜɿɞɤɢɧɭɬɢ, ɡɪɨɛɢɜɲɢ ɩɟɪɟɜɿɪɤɭ.
  • 59. 58 § 1. Ɋɚɰɿɨɧɚɥɶɧɿ ɜɢɪɚɡɢ ɓɨɛ ɪɨɡɜ’ɹɡɚɬɢ ɞɪɨɛɨɜɟ ɪɚɰɿɨɧɚɥɶɧɟ ɪɿɜɧɹɧɧɹ, ɦɨɠɧɚ: 1) ɩɨɦɧɨɠɢɬɢ ɨɛɢɞɜɿ ɱɚɫɬɢɧɢ ɪɿɜɧɹɧɧɹ ɧɚ ɫɩɿɥɶɧɢɣ ɡɧɚɦɟɧɧɢɤ ɞɪɨɛɿɜ, ɹɤɿ ɜɯɨɞɹɬɶ ɞɨ ɪɿɜɧɹɧɧɹ, ɿ ɡɚɦɿɧɢɬɢ ɣɨɝɨ ɰɿɥɢɦ ɪɚɰɿɨɧɚɥɶɧɢɦ ɪɿɜɧɹɧɧɹɦ; 2) ɪɨɡɜ’ɹɡɚɬɢ ɨɞɟɪɠɚɧɟ ɰɿɥɟ ɪɚɰɿɨɧɚɥɶɧɟ ɪɿɜɧɹɧɧɹ; 3) ɜɢɤɥɸɱɢɬɢ ɡ ɣɨɝɨ ɤɨɪɟɧɿɜ ɬɿ, ɞɥɹ ɹɤɢɯ ɫɩɿɥɶɧɢɣ ɡɧɚɦɟɧɧɢɤ ɞɪɨɛɿɜ ɞɨ- ɪɿɜɧɸɽ ɧɭɥɸ. ȼɩɪɚɜɚ 1. Ɋɨɡɜ’ɹɡɚɬɢ ɪɿɜɧɹɧɧɹ 2 2 2 2 1 22 x x . x xx x − + = ++ • 2 2 2 1 0; ( 2) 2 x x x x x x − + − = + + 2 2 2 2 ( 2) 0; ( 2) x x x x x − + + − = + 2 0; ( 2) x x x x + = + 2 0; ( 2) 0; x x x x ­ + = ® + ≠¯ ( 1) 0; ( 2) 0; x x x x + =­ ® + ≠¯ 0 ɚɛɨ 1; 0; 2; x x x x = = −­ ° ≠® ° ≠ −¯ x = –1. ȼɿɞɩɨɜɿɞɶ. –1. • ȼɩɪɚɜɚ 2. Ɋɨɡɜ’ɹɡɚɬɢ ɪɿɜɧɹɧɧɹ 2 2 2 5. 2 4 x x x += − − • Ɋɨɡɝɥɹɞɚɬɢɦɟɦɨ ɪɿɜɧɿɫɬɶ 2 2 2 5 2 4 x x x += − − ɹɤ ɩɪɨɩɨɪɰɿɸ. Ɂɚ ɨɫɧɨɜɧɨɸ ɜɥɚɫɬɢɜɿɫɬɸ ɩɪɨɩɨɪɰɿʀ ɦɚɽɦɨ: 2(ɯ2 – 4) = (ɯ – 2)(2ɯ + 5), ɡɚ ɭɦɨɜɢ, ɳɨ ɯ – 2 ≠ 0 ɿ ɯ2 – 4 ≠ 0. Ɋɨɡɜ’ɹɠɟɦɨ ɨɞɟɪɠɚɧɟ ɪɿɜɧɹɧɧɹ: 2ɯ2 – 8 = 2ɯ2 + 5ɯ – 4ɯ – 10; –8 = ɯ – 10; ɯ = 2. əɤɳɨ ɯ = 2, ɬɨ ɯ – 2 = 2 – 2 = 0, ɬɨɛɬɨ ɞɥɹ ɯ = 2 ɭɦɨɜɚ ɯ – 2 ≠ 0 ɧɟ ɜɢɤɨ- ɧɭɽɬɶɫɹ. Ɍɨɦɭ ɯ = 2 — ɧɟ ɤɨɪɿɧɶ ɪɿɜɧɹɧɧɹ. ȼɿɞɩɨɜɿɞɶ. Ʉɨɪɟɧɿɜ ɧɟɦɚɽ. • ȼɩɪɚɜɚ 3. Ɂ ɦɿɫɬɚ A ɞɨ ɦɿɫɬɚ B, ɜɿɞɫɬɚɧɶ ɦɿɠ ɹɤɢɦɢ ɞɨɪɿɜɧɸɽ 21 ɤɦ, ɜɢʀɯɚɜ ɜɟɥɨɫɢɩɟɞɢɫɬ, ɚ ɱɟɪɟɡ 20 ɯɜ ɭɫɥɿɞ ɡɚ ɧɢɦ — ɦɨɬɨɰɢɤɥɿɫɬ, ɲɜɢɞɤɿɫɬɶ ɹɤɨɝɨ ɭɬɪɢɱɿ ɛɿɥɶɲɚ ɜɿɞ ɲɜɢɞɤɨɫɬɿ ɜɟɥɨɫɢɩɟɞɢɫɬɚ. Ɂɧɚɣɬɢ ɲɜɢɞɤɿɫɬɶ ɜɟɥɨɫɢɩɟɞɢɫɬɚ, ɹɤɳɨ ɜɿɞɨɦɨ, ɳɨ ɦɨɬɨɰɢɤɥɿɫɬ ɩɪɢʀɯɚɜ ɭ ɦɿɫɬɨ B ɧɚ 40 ɯɜ ɪɚɧɿɲɟ, ɧɿɠ ɜɟɥɨɫɢɩɟɞɢɫɬ.
  • 60. 8. Ɋɚɰɿɨɧɚɥɶɧɿ ɪɿɜɧɹɧɧɹ 59 • ɇɟɯɚɣ ɲɜɢɞɤɿɫɬɶ ɜɟɥɨɫɢɩɟɞɢɫɬɚ ɞɨɪɿɜɧɸɽ ɯ ɤɦ/ɝɨɞ, ɬɨɞɿ ɲɜɢɞɤɿɫɬɶ ɦɨ- ɬɨɰɢɤɥɿɫɬɚ — 3ɯ ɤɦ/ɝɨɞ. ɒɥɹɯ ɡɚɜɞɨɜɠɤɢ 21 ɤɦ ɜɟɥɨɫɢɩɟɞɢɫɬ ɩɨɞɨɥɚɜ ɡɚ 21 x ɝɨɞ, ɚ ɦɨɬɨɰɢɤɥɿɫɬ — ɡɚ 21 7 3x x = (ɝɨɞ). Ɉɫɤɿɥɶɤɢ ɜɟɥɨɫɢɩɟɞɢɫɬ ɛɭɜ ɭ ɞɨɪɨɡɿ ɧɚ 20 ɯɜ + 40 ɯɜ = 60 ɯɜ = 1 ɝɨɞ ɞɨɜɲɟ, ɧɿɠ ɦɨɬɨɰɢɤɥɿɫɬ, ɬɨ ɦɚɽɦɨ ɪɿɜɧɹɧɧɹ 21 7 1− = . x x Ɋɨɡɜ’ɹɠɟɦɨ ɰɟ ɪɿɜɧɹɧɧɹ: 21 7 1 0; x x − − = 21 7 0; x x − − = 14 0; x x − = 14 0; 0; x x − =­ ® ≠¯ ɯ = 14. ȼɿɞɩɨɜɿɞɶ. 14 ɤɦ/ɝɨɞ. • 197. ɇɚɡɜɿɬɶ ɰɿɥɿ ɪɚɰɿɨɧɚɥɶɧɿ ɪɿɜɧɹɧɧɹ; ɞɪɨɛɨɜɿ ɪɚɰɿɨɧɚɥɶɧɿ ɪɿɜɧɹɧɧɹ: ɚ) 5 1; 7x = − ɛ) 3( 11) 0;x − = ɜ) 1 3; 3 x− = ɝ) 2 2 0. x = 198. ɑɢ ɽ ɱɢɫɥɨ 1 ɤɨɪɟɧɟɦ ɪɿɜɧɹɧɧɹ? ɚ) 1 0; 5 x x − = − ɛ) 1 0; 4 4 x x − = − ɜ) 2 1 0; 1 x x − = + ɝ) 2 3 1 1 . x x = − + 199. ɑɢ ɪɿɜɧɨɫɢɥɶɧɿ ɪɿɜɧɹɧɧɹ? ɚ) 4 8 1 x x = − ɿ 2 1 x ; x = − ɛ) 1 3 2 1x x = + ɿ 1 3 0; 2 1x x − = + ɜ) 2 ( 3) 0 x x x + = ɿ ( 3) 0;x x + = ɝ) 2 4 2 2 x x x = − − ɿ 2ɯ = 4. Ɋɨɡɜ’ɹɠɿɬɶ ɪɿɜɧɹɧɧɹ: 200. ɚ) 8 0; 1 x x + = − ɛ) 1 0; 8 x x − = + ɜ) 2 2 8 0. 16 x x − = − 201. ɚ) 2 6 0; 3 3 x x x x −+ = − − ɛ) 3 1 2 2 1 1 x x ; x x + −= + + ɜ) 5 2 . 6 6 x x x x − = − − 202. ɚ) 2 1 0;x x + + = ɛ) 2; 1 x x = + ɜ) 10 3.x x − =
  • 61. 60 § 1. Ɋɚɰɿɨɧɚɥɶɧɿ ɜɢɪɚɡɢ 203. ɚ) 1 1 1; 3 4x x + = ɛ) 1 3 2; 5 4x x + = ɜ) 4 1 = 1. 3 2x x − 204. ɚ) 2 4 0; 4 x x + = + ɛ) 2 2 10 0; 25 x x − = − ɜ) 4 12 ; 2 2 x x x = − − ɝ) 3 4 6 ; 2 2 x x x x − −= ɞ) 1 2 0; 5 x x − + = + ɟ) 5 10 4;x x − = ɽ) 5 4; 2 1 x x − = − − ɠ) 2 1 5; 2x x + = ɡ) 1 1 = 1. 2 3x x − 205. ɚ) 2 2 4 0; 2 − = − x x x ɛ) 2 3 0; 2 2x x − = + − ɜ) 3 2 1 2 3 x x . x x + = − + 206. ɚ) 2 3 0; 2 1 x x x − = + ɛ) 2 0; 2 + = + x x x ɜ) 5 3 2 3 1 x x . x x − = + − 207. əɤɟ ɱɢɫɥɨ ɩɨɬɪɿɛɧɨ ɞɨɞɚɬɢ ɞɨ ɡɧɚɦɟɧɧɢɤɚ ɞɪɨɛɭ 19 41 , ɳɨɛ ɨɞɟɪɠɚɬɢ ɞɪɿɛ, ɹɤɢɣ ɞɨɪɿɜɧɸɽ 1 3 ? 208. əɤɟ ɱɢɫɥɨ ɩɨɬɪɿɛɧɨ ɜɿɞɧɹɬɢ ɜɿɞ ɡɧɚɦɟɧɧɢɤɚ ɞɪɨɛɭ 3 47 , ɳɨɛ ɨɞɟɪɠɚɬɢ ɞɪɿɛ, ɹɤɢɣ ɞɨɪɿɜɧɸɽ 1 4 ? 209. əɤɟ ɬɟ ɫɚɦɟ ɱɢɫɥɨ ɩɨɬɪɿɛɧɨ ɞɨɞɚɬɢ ɞɨ ɱɢɫɟɥɶɧɢɤɚ ɞɪɨɛɭ 1 2 ɣ ɩɨɦɧɨɠɢɬɢ ɧɚ ɧɶɨɝɨ ɡɧɚɦɟɧɧɢɤ ɰɶɨɝɨ ɞɪɨɛɭ, ɳɨɛ ɨɞɟɪɠɚɬɢ ɞɪɿɛ, ɹɤɢɣ ɞɨɪɿɜɧɸɽ 2 3 ? 210. ɇɚ ɹɤɟ ɬɟ ɫɚɦɟ ɱɢɫɥɨ ɩɨɬɪɿɛɧɨ ɩɨɦɧɨɠɢɬɢ ɱɢɫɟɥɶɧɢɤ ɞɪɨɛɭ 1 5 ɣ ɞɨɞɚɬɢ ɣɨɝɨ ɞɨ ɡɧɚɦɟɧɧɢɤɚ ɰɶɨɝɨ ɞɪɨɛɭ, ɳɨɛ ɨɞɟɪɠɚɬɢ ɞɪɿɛ, ɹɤɢɣ ɞɨɪɿɜɧɸɽ 1 2 ? 211. ɑɢ ɪɿɜɧɨɫɢɥɶɧɿ ɪɿɜɧɹɧɧɹ (2 3) 0 x x x − = ɬɚ ɯ(2ɯ – 3) = 0? ȼɿɞɩɨɜɿɞɶ ɨɛʉɪɭɧ- ɬɭɣɬɟ. 212. Ⱦɨɜɟɞɿɬɶ, ɳɨ ɪɿɜɧɹɧɧɹ 2 3 1 1 x x x + = − − ɬɚ ɯ + 2 = 3 ɧɟ ɪɿɜɧɨɫɢɥɶɧɿ.